contents 1 introduction 3 1.1 syllabus . . . . . . . . . . . . . . . . . . . . . . . . . . . . . . ....

Post on 29-Dec-2020

3 Views

Category:

Documents

0 Downloads

Preview:

Click to see full reader

TRANSCRIPT

My Physics 501 pageSpecial Topics: Mathematical Models of Physical

Problems IFall 2018

University of Wisconsin, Milwaukee

Nasser M. Abbasi

Fall 2018 Compiled on November 3, 2019 at 11:35am [public]

Contents

1 Introduction 31.1 syllabus . . . . . . . . . . . . . . . . . . . . . . . . . . . . . . . . . . . . . . . . 31.2 Topics covered . . . . . . . . . . . . . . . . . . . . . . . . . . . . . . . . . . . 3

2 exams 112.1 Practice exam from 2017 . . . . . . . . . . . . . . . . . . . . . . . . . . . . . 11

2.1.1 Problem 1 . . . . . . . . . . . . . . . . . . . . . . . . . . . . . . . . . . 112.1.2 Problem 2 . . . . . . . . . . . . . . . . . . . . . . . . . . . . . . . . . . 132.1.3 Problem 3 . . . . . . . . . . . . . . . . . . . . . . . . . . . . . . . . . . 152.1.4 Problem 4 . . . . . . . . . . . . . . . . . . . . . . . . . . . . . . . . . . 162.1.5 Problem 5 . . . . . . . . . . . . . . . . . . . . . . . . . . . . . . . . . . 18

2.2 exam 1 . . . . . . . . . . . . . . . . . . . . . . . . . . . . . . . . . . . . . . . . 212.2.1 exam 1 prep sheet . . . . . . . . . . . . . . . . . . . . . . . . . . . . . 212.2.2 exam 1 writeup . . . . . . . . . . . . . . . . . . . . . . . . . . . . . . . 22

2.3 final exam . . . . . . . . . . . . . . . . . . . . . . . . . . . . . . . . . . . . . . 302.3.1 exam 1 prep sheet . . . . . . . . . . . . . . . . . . . . . . . . . . . . . 302.3.2 my final exam cheat sheet . . . . . . . . . . . . . . . . . . . . . . . . 31

3 HWs 333.1 HW 1 . . . . . . . . . . . . . . . . . . . . . . . . . . . . . . . . . . . . . . . . . 33

3.1.1 Problem 1 . . . . . . . . . . . . . . . . . . . . . . . . . . . . . . . . . . 333.1.2 Problem 2 . . . . . . . . . . . . . . . . . . . . . . . . . . . . . . . . . . 353.1.3 Problem 3 . . . . . . . . . . . . . . . . . . . . . . . . . . . . . . . . . . 363.1.4 key solution to HW 1 . . . . . . . . . . . . . . . . . . . . . . . . . . . 39

3.2 HW 2 . . . . . . . . . . . . . . . . . . . . . . . . . . . . . . . . . . . . . . . . . 413.2.1 Problem 1 . . . . . . . . . . . . . . . . . . . . . . . . . . . . . . . . . . 413.2.2 Problem 2 . . . . . . . . . . . . . . . . . . . . . . . . . . . . . . . . . . 433.2.3 Problem 3 . . . . . . . . . . . . . . . . . . . . . . . . . . . . . . . . . . 453.2.4 Problem 4 . . . . . . . . . . . . . . . . . . . . . . . . . . . . . . . . . . 483.2.5 Problem 5 . . . . . . . . . . . . . . . . . . . . . . . . . . . . . . . . . . 493.2.6 key solution to HW 2 . . . . . . . . . . . . . . . . . . . . . . . . . . . 50

3.3 HW 3 . . . . . . . . . . . . . . . . . . . . . . . . . . . . . . . . . . . . . . . . . 543.3.1 Problem 1 . . . . . . . . . . . . . . . . . . . . . . . . . . . . . . . . . . 543.3.2 Problem 2 . . . . . . . . . . . . . . . . . . . . . . . . . . . . . . . . . . 593.3.3 Problem 3 . . . . . . . . . . . . . . . . . . . . . . . . . . . . . . . . . . 643.3.4 key solution to HW 3 . . . . . . . . . . . . . . . . . . . . . . . . . . . 66

3.4 HW 4 . . . . . . . . . . . . . . . . . . . . . . . . . . . . . . . . . . . . . . . . . 703.4.1 Problem 1 . . . . . . . . . . . . . . . . . . . . . . . . . . . . . . . . . . 703.4.2 Problem 2 . . . . . . . . . . . . . . . . . . . . . . . . . . . . . . . . . . 733.4.3 Problem 3 . . . . . . . . . . . . . . . . . . . . . . . . . . . . . . . . . . 743.4.4 Problem 4 . . . . . . . . . . . . . . . . . . . . . . . . . . . . . . . . . . 783.4.5 key solution to HW 4 . . . . . . . . . . . . . . . . . . . . . . . . . . . 82

3.5 HW 5 . . . . . . . . . . . . . . . . . . . . . . . . . . . . . . . . . . . . . . . . . 863.5.1 Problem 1 . . . . . . . . . . . . . . . . . . . . . . . . . . . . . . . . . . 863.5.2 Problem 2 . . . . . . . . . . . . . . . . . . . . . . . . . . . . . . . . . . 963.5.3 Problem 3 . . . . . . . . . . . . . . . . . . . . . . . . . . . . . . . . . . 993.5.4 key solution to HW 5 . . . . . . . . . . . . . . . . . . . . . . . . . . . 102

3.6 HW 6 . . . . . . . . . . . . . . . . . . . . . . . . . . . . . . . . . . . . . . . . . 1083.6.1 Problem 1 . . . . . . . . . . . . . . . . . . . . . . . . . . . . . . . . . . 1083.6.2 Problem 2 . . . . . . . . . . . . . . . . . . . . . . . . . . . . . . . . . . 1123.6.3 Problem 3 . . . . . . . . . . . . . . . . . . . . . . . . . . . . . . . . . . 114

2

3

3.6.4 key solution to HW 6 . . . . . . . . . . . . . . . . . . . . . . . . . . . 1193.7 HW 7 . . . . . . . . . . . . . . . . . . . . . . . . . . . . . . . . . . . . . . . . . 124

3.7.1 Problem 1 . . . . . . . . . . . . . . . . . . . . . . . . . . . . . . . . . . 1243.7.2 Problem 2 . . . . . . . . . . . . . . . . . . . . . . . . . . . . . . . . . . 1293.7.3 key solution to HW 7 . . . . . . . . . . . . . . . . . . . . . . . . . . . 133

4

Chapter 1

Introduction

1.1 syllabus

1.2 Topics covered

This is list of lectures and topics covered in each

5

6

Table 1.1: Topics covered

# Date Topics

1 Wed. Sept 4, 20181. Infinite series, geometric series.

2. conditions for convergence,

3. harmonic series, alternating series.

4. tests for convergence such as ratio test, integral test.

2 Monday Sept 10, 2018

1. Talked about series solution to (1โˆ’๐‘ฅ2)๐‘ฆโ€ณโˆ’2๐‘ฅ๐‘ฆโ€ฒ+๐‘›(๐‘›+1)๐‘ฆ =0.

2. Binomial series (1 + ๐‘ฅ)๐‘Ÿ = 1 + ๐‘Ÿ๐‘ฅ + ๐‘Ÿ(๐‘Ÿโˆ’1)๐‘ฅ2

2! + โ€ฆ

3. series of ๐‘’, sin(๐‘ฅ), cos(๐‘ฅ).4. Introducing Bernoulli numbers.

5. Show that alternating series is convergent but not abso-lutely.

6. Leibniz condition for convergence and its proof.

7. Showed that sum of 1 โˆ’ 1/2 + 1/3 โˆ’ 1/4 + โ€ฆ is ln 2.8. Working with absolutely convergent series.

3 Wed Sept 12, 2018

1. Familiar series, ๐‘’, sin ๐‘ฅ, cos ๐‘ฅ, ln(1 + ๐‘ฅ), arctan(๐‘ฅ)2. How to get Bernulli numbers. More on Bernulli numbers

but I really did not understand these well and how touse them. hopefully they will not be on the exam.

3. Started Complex analysis. Basic introduction. Propertiesof complex numbers and mapping.

4 Monday Sept 17, 2018

1. complex functions ๐‘ข(๐‘ฅ, ๐‘ฆ) + ๐‘–๐‘ฃ(๐‘ฅ, ๐‘ฆ)

2. continuity in complex domain.

3. Derivative in complex domain and how direction is im-portant.

4. Cauchy-Riemman equation to test for analytical function.

5. Harmonic functions. Exponential function in complexdomain.

6. Multivalued functions, such as log ๐‘ง.7. How to obtain inverse trig function and solve ๐‘ค =

arcsin(๐‘ง)

Continued on next page

7

Table 1.1 โ€“ continued from previous page

Lecture # Date Topics

5 Wed Sept 19, 2018

1. derivative in complex plane. Definition of analytic func-tion.

2. log(๐‘ง) and โˆš(๐‘ง) in complex plane and multivalued.Branch points and branch cuts.

3. Integration over contour. Parameterization โˆซ๐ถ๐‘“(๐‘ง) ๐‘‘๐‘ง =

โˆซ๐‘๐‘Ž๐‘“(๐‘ง(๐‘ก))๐‘งโ€ฒ(๐‘ก) ๐‘‘๐‘ก

4. Cauchy-Goursat theorem: โˆฎ๐‘“(๐‘ง) = 0 for analyticalfunctions. Proof using Cauchy-Riemman equations andGreen theorem.

5. Cauchy integral formula 2๐œ‹๐‘–๐‘“(๐‘ง0) = โˆฎ๐‘“(๐‘ง)๐‘งโˆ’๐‘ง0

๐‘‘๐‘ง

6. Like in real, in complex domain, Continuity Does NotImply Di๏ฟฝerentiability.

7. More on analytic functions and multivalued functions.Principal value.

8. Power functions ๐‘ง๐‘ = ๐‘’๐‘ ln ๐‘ง

9. Complex integration.

6 Monday Sept 24, 2018

1. Proof of Cauchy integral formula.

2. Maximum moduli of analytic functions. If ๐‘“(๐‘ง) is analyticin ๐ท and not constant, then it has no maximum valueinside ๐ท. The maximum of ๐‘“(๐‘ง) is on the boundary.

3. Taylor series for complex functions and Laurent series.

7 Wed Sept 26, 2018

1. If number of terms in principal part of Laurent series isinfinite, then it is essential singularity.

2. Proof of Laurent theorem.

3. properties of power series. Uniqueness.

4. Residues, types of singularities. How to find residues andexamples

8 Monday Oct 1, 2018

1. Residue theorem โˆฎ๐ถ๐‘“(๐‘ง) ๐‘‘๐‘ง = 2๐œ‹๐‘–โˆ‘ residues inside C

2. examples using Residue theorem.

3. Analytic continuation. Examples.

4. ฮ“(๐‘ง) function. Defined for โ„œ(๐‘ง) > 0. Using analytical con-tinuation to extend it to negative complex plane. Eulerrepresentation and Weistrass represenation.

Continued on next page

8

Table 1.1 โ€“ continued from previous page

Lecture # Date Topics

9 Wed Oct 3, 2018

1. More on Euler represenation of ฮ“(๐‘ง) and how to use itfor extending definition ฮ“(๐‘ง) = โˆซ

โˆž

0๐‘’โˆ’๐‘ก๐‘ก๐‘งโˆ’1 ๐‘‘๐‘ก for negative ๐‘ง

using ฮ“(๐‘ง) = ฮ“(๐‘ง+1)๐‘ง for โˆ’1 < ๐‘ง.

2. Euler reflection formula

ฮ“(๐‘ฅ)ฮ“(1 โˆ’ ๐‘ฅ) = ๏ฟฝโˆž

0

๐‘ก๐‘ฅโˆ’1

1 + ๐‘ก๐‘‘๐‘ก =

๐œ‹sin(๐œ‹๐‘ฅ)

3. proof of Euler reflection formula using contour integra-tion.

4. Some useful formulas for ฮ“(๐‘ง)

5. Method for integrations, some tricks to obtain definiteintegrations.

10 Monday Oct 8, 2018 No class.

11 Wed Oct 10, 2018 No class.

12 Monday Oct 15, 2018

1. More on method of integration. Starting Contour inte-gration.

2. How to decide that โˆซ๐ถ๐‘…๐‘“(๐‘ง) = 0 on the upper half plane.

Using Jordan inquality.

3. More examples of integrals on real line using contourintegration.

13 Wed Oct 17, 2018

1. More contour integrations.

2. Starting approximation expansion of integrals. Exampleusing error function erf(๐‘ฅ) = 2

โˆš๐œ‹โˆซ๐‘ฅ0๐‘’โˆ’๐‘ก2 ๐‘‘๐‘ก by applying

Taylor series.

3. Large ๐‘ฅ expansion by repeated integration by parts.

4. Starting Asymprtotic series. Definition. Example on find-ing ๐‘†(๐‘ฅ) for erf(๐‘ฅ) for large ๐‘ฅ. When to truncate.

5. Saddle point methods of integration to approximate in-tegral for large ๐‘ฅ.

14 Monday Oct 22, 2018

1. More saddle point integration.

2. Saddle point methods of integration to approximate in-tegral for large ๐‘ฅ. Method of steepest decsent. Exampleto find ฮ“(๐‘ฅ + 1) = โˆซ

โˆž

0๐‘ก๐‘ฅ๐‘’โˆ’๐‘ก ๐‘‘๐‘ก = โˆš2๐œ‹๐‘ฅ๐‘ฅ๐‘ฅ๐‘’โˆ’๐‘ฅ

3. extend saddle point method to complex plane. Findingcorrect angle. Long example.

Continued on next page

9

Table 1.1 โ€“ continued from previous page

Lecture # Date Topics

15 Wed Oct 24, 2018

1. More on saddle point in complex plane. Angles. Exampleapplied on โˆซฮ“(1 + ๐‘ง) = โˆซ

โˆž

0expโˆ’๐‘ก + ๐‘ง ln ๐‘ก ๐‘‘๐‘ก

2. how to determine coe๏ฟฝcients of asymptotic series expan-sion.

3. Starting new topic. Fourier series. Definitions.

4. proprties of Fourier series. Examples how to find ๐ด๐‘›, ๐ต๐‘›.

16 Friday Oct 26, 2018 Make up lecture.

1. More on Fourier series. Examples. Fourier series usingthe complex formula.

2. Parseval identity.

3. Fourier Transform derivation.

17 Monday Oct 29, 2018

1. Fourier transform pairs.

2. How to find inverse fourier transform. Generalization tohigher dimensions.

3. Properties of Fourier transform.

4. convolution.

5. Example on driven harmonic oscillator.

6. Statring ODEโ€™s. Order and degree of ODE.

18 Wed Oct 31, 2018

1. More on first order ODEโ€™s. Separable, exact. How to findintegrating factor.

2. Bernulli ODE ๐‘ฆโ€ฒ + ๐‘“(๐‘ฅ)๐‘ฆ = ๐‘”(๐‘ฅ)๐‘ฆ๐‘›

3. Homogeneous functions. defintion. order of.

4. isobaric ODEโ€™s.

19 Monday Nov 5, 2018

1. How to find integating factor for exact ODE.

2. Finished example on isobaric first order ODE.

๐‘ฅ๐‘ฆ2(3๐‘ฆ ๐‘‘๐‘ฅ + ๐‘ฅ ๐‘‘๐‘ฆ) โˆ’ (2๐‘ฆ ๐‘‘๐‘ฅ โˆ’ ๐‘ฅ ๐‘‘๐‘ฆ) = 0

3. Higher order ODEโ€™s. How to solve. How to find partic-ular solution. Undetermined coe๏ฟฝcients. What to do ifforcing function has same form as one of the solutionsto homogeneous solutions.

4. How to use power series to solve nonlinear ode ๐‘ฆโ€ณ = ๐‘ฅโˆ’๐‘ฆ2

20 Wed Nov 7, 2018 First exam

Continued on next page

10

Table 1.1 โ€“ continued from previous page

Lecture # Date Topics

21 Monday Nov 12, 2018

1. More on higher order ODEโ€™s. Series solutions.

2. ordinary point. Regular singular point. Example Legen-dre ODE (1 โˆ’ ๐‘ฅ2)๐‘ฆโ€ณ โˆ’ 2๐‘ฅ๐‘ฆโ€ฒ + ๐‘›(๐‘› + 1)๐‘ฆ = 0.

3. Example for regular singular point, Bessel ODE ๐‘ฅ2๐‘ฆโ€ณ +๐‘ฅ๐‘ฆโ€ฒ + (๐‘ฅ2 โˆ’ ๐‘š2)๐‘ฆ = 0 Use ๐‘ฆ = ๐‘ฅ2โˆ‘โˆž

๐‘›=0 ๐‘๐‘›๐‘ฅ๐‘›

22 Wed Nov 14, 2018

1. Continue Bessel ODE ๐‘ฅ2๐‘ฆโ€ณ + ๐‘ฅ๐‘ฆโ€ฒ + (๐‘ฅ2 โˆ’ ๐‘š2)๐‘ฆ = 0 solvingusing ๐‘ฆ = ๐‘ฅ2โˆ‘โˆž

๐‘›=0 ๐‘๐‘›๐‘ฅ๐‘›. How to find second independent

solution.

23 Monday Nov 19, 2018

1. Started on Sturm Lioville, Hermetian operators

2. setting Bessel ODE in Sturm Lioville form

3. more on Hermitian operator.

4. Wronskian to check for linear independece of solutions.

24 Wed Nov 21, 2018 Thanks Giving.

25 Monday Nov 26, 2018

1. finding second solution to Bessel ODE for ๐‘š integer us-ing the Wronskian. ๐‘Š(๐‘ฅ) = ๐ถ

๐‘(๐‘ฅ) =โˆ’2 sin๐œ‹๐‘š

๐œ‹

2. Generating functions to find way to generate Besself func-tions.

26 Wed Nov 28, 2018

1. Using Generating functions

2. Bessel functions of half integer order, spherical Besselfunctions

3. Legendre polynomials, recusrive relations.

4. orthonomalization.

5. physical applications

27 Monday December 3, 2018

1. Second solution to Legendre using Wronskian

2. Spherical harmonics

3. Normalization of eigenfunctions

4. Degenerncy, using Gram-Schmidt to find other L.I. solu-tions.

5. Expanding function using complete set of basis functions,example using Fourier series

6. Inhomogeneous problems, starting Green function

Continued on next page

11

Table 1.1 โ€“ continued from previous page

Lecture # Date Topics

28 Wed December 5, 2018

1. Green function. Solution to the ODE with point source.

2. Example using vibrating string ๐‘ฆโ€ณ + ๐‘˜2๐‘ฆ = 0. Find Greenfunction. Two Methods. Use second method.

3. Started on PDE.

29 Friday December 7, 2018 (Make up lecture)

1. more PDEโ€™s. Solve wave PDE in 1D

2. separation of variables. Solve Wave PDE in 3D in spher-ical coordiates.

30 monday December 10, 2018

1. Solving wave PDE in 3D in spherical coordinates. Nor-mal modes.

2. Solving wave PDE in 3D in cylindrical coordinates.

3. Inhomogeneous B.C. on heat PDE. Break it into 2 parts.

31 Wed December 12, 2018 Last lecture.

1. Finish Inhomogeneous B.C. on heat PDE. Break it into2 parts. Final solution, using Fourier series.

2. Last problem. INtegral transform method. Solving heatpde on infinite line using Fourier transform.

12

Chapter 2

exams

2.1 Practice exam from 2017

2.1.1 Problem 1

i) Find Laurent series for ๐‘“ (๐‘ง) = 1

๏ฟฝ๐‘ง2+1๏ฟฝ3 around isolated singular pole ๐‘ง = ๐‘–. What is the

order of the pole? ii) Use residues to evaluate the integral โˆซโˆž

0๐‘‘๐‘ฅ

๏ฟฝ๐‘ฅ2+1๏ฟฝ3

solution

๐‘ง2 + 1 = 0 gives ๐‘ง = ยฑ๐‘–. Hence there is a pole at ๐‘ง = ๐‘– of order 3 and also a pole at ๐‘ง = โˆ’๐‘–of order 3. Hence ๐‘” (๐‘ง) = (๐‘ง โˆ’ ๐‘–)3 ๐‘“ (๐‘ง) is analytic at ๐‘ง = ๐‘– and therefore it has a Taylor seriesexpansion around ๐‘ง = ๐‘– given by

๐‘” (๐‘ง) =โˆž๏ฟฝ๐‘›=0

๐‘Ž๐‘› (๐‘ง โˆ’ ๐‘–)๐‘›

(๐‘ง โˆ’ ๐‘–)3 ๐‘“ (๐‘ง) =โˆž๏ฟฝ๐‘›=0

๐‘Ž๐‘› (๐‘ง โˆ’ ๐‘–)๐‘› (1)

Where ๐‘Ž๐‘› =๐‘‘๐‘›๐‘‘๐‘ง๐‘› ๐‘”(๐‘ง)

๐‘›! ๏ฟฝ๐‘ง=๐‘–

. But

๐‘” (๐‘ง) = (๐‘ง โˆ’ ๐‘–)3 ๐‘“ (๐‘ง)

= (๐‘ง โˆ’ ๐‘–)31

๏ฟฝ๐‘ง2 + 1๏ฟฝ3

= (๐‘ง โˆ’ ๐‘–)31

((๐‘ง โˆ’ ๐‘–) (๐‘ง + ๐‘–))3

= (๐‘ง โˆ’ ๐‘–)31

(๐‘ง โˆ’ ๐‘–)3 (๐‘ง + ๐‘–)3

=1

(๐‘ง + ๐‘–)3

To find ๐‘Ž๐‘› then ๐‘Ž๐‘› =1๐‘›!

๐‘‘๐‘›

๐‘‘๐‘ง๐‘›1

(๐‘ง+๐‘–)3is evaluated for few ๐‘› terms. Since order is 3, at least 5 terms

are needed to see the residue and the first term in the analytical part of the series (๐‘› > 0).Starting with ๐‘› = 0

๐‘Ž0 =1

(๐‘ง + ๐‘–)3๏ฟฝ๐‘ง=๐‘–

=1

(2๐‘–)3=

1โˆ’8๐‘–

=18๐‘–

For ๐‘› = 1

๐‘Ž1 =๐‘‘๐‘‘๐‘ง

1(๐‘ง + ๐‘–)3

๏ฟฝ๐‘ง=๐‘–

=โˆ’3

(๐‘ง + ๐‘–)4๏ฟฝ๐‘ง=๐‘–

=โˆ’3(2๐‘–)4

=โˆ’316

For ๐‘› = 2

๐‘Ž2 =12๐‘‘๐‘‘๐‘ง

โˆ’3(๐‘ง + ๐‘–)4

๏ฟฝ๐‘ง=๐‘–

=12โˆ’3 (โˆ’4)(๐‘ง + ๐‘–)5

๏ฟฝ๐‘ง=๐‘–

=12โˆ’3 (โˆ’4)(2๐‘–)5

=12โˆ’3 (โˆ’4)25๐‘–

=632๐‘–

=316๐‘–

= โˆ’3๐‘–16

13

14

For ๐‘› = 3

๐‘Ž3 =13!

๐‘‘๐‘‘๐‘งโˆ’3 (โˆ’4)(๐‘ง + ๐‘–)5

๏ฟฝ๐‘ง=๐‘–

=16โˆ’3 (โˆ’4) (โˆ’5)(๐‘ง + ๐‘–)6

๏ฟฝ๐‘ง=๐‘–

=16โˆ’3 (โˆ’4) (โˆ’5)

(2๐‘–)6=16โˆ’3 (โˆ’4) (โˆ’5)

โˆ’26=532

For ๐‘› = 4

๐‘Ž4 =14!

๐‘‘๐‘‘๐‘งโˆ’3 (โˆ’4) (โˆ’5)(๐‘ง + ๐‘–)6

๏ฟฝ๐‘ง=๐‘–

=124

โˆ’3 (โˆ’4) (โˆ’5) (โˆ’6)(๐‘ง + ๐‘–)7

๏ฟฝ๐‘ง=๐‘–

=124โˆ’3 (โˆ’4) (โˆ’5) (โˆ’6)

(2๐‘–)7=1243 (4) (5) (6)

โˆ’๐‘–27=15128

๐‘–

Substituting all these back into (1) gives

(๐‘ง โˆ’ ๐‘–)3 ๐‘“ (๐‘ง) =โˆž๏ฟฝ๐‘›=0

๐‘Ž๐‘› (๐‘ง โˆ’ ๐‘–)๐‘›

= ๐‘Ž0 + ๐‘Ž1 (๐‘ง โˆ’ ๐‘–) + ๐‘Ž2 (๐‘ง โˆ’ ๐‘–)2 + ๐‘Ž3 (๐‘ง โˆ’ ๐‘–)

3 + ๐‘Ž4 (๐‘ง โˆ’ ๐‘–)4 +โ‹ฏ

Therefore

๐‘“ (๐‘ง) =1

(๐‘ง โˆ’ ๐‘–)3๏ฟฝ๐‘Ž0 + ๐‘Ž1 (๐‘ง โˆ’ ๐‘–) + ๐‘Ž2 (๐‘ง โˆ’ ๐‘–)

2 + ๐‘Ž3 (๐‘ง โˆ’ ๐‘–)3 + ๐‘Ž4 (๐‘ง โˆ’ ๐‘–)

4 +โ‹ฏ๏ฟฝ

=1

(๐‘ง โˆ’ ๐‘–)3๏ฟฝ18๐‘– +

โˆ’316(๐‘ง โˆ’ ๐‘–) โˆ’

3๐‘–16(๐‘ง โˆ’ ๐‘–)2 +

532(๐‘ง โˆ’ ๐‘–)3 +

15128

๐‘– (๐‘ง โˆ’ ๐‘–)4 +โ‹ฏ๏ฟฝ

=18

๐‘–(๐‘ง โˆ’ ๐‘–)3

โˆ’316

1(๐‘ง โˆ’ ๐‘–)2

โˆ’316

๐‘–(๐‘ง โˆ’ ๐‘–)

+532+15128

๐‘– (๐‘ง โˆ’ ๐‘–) โˆ’โ‹ฏ (1A)

The residue is the coe๏ฟฝcient of the term with 1๐‘งโˆ’๐‘– factor. Hence residue is โˆ’ 3๐‘–

16 . The order

is 3 since that is the highest power in 1๐‘งโˆ’๐‘– .

The above method always works, but it means having to evaluate derivatives a number oftimes. For a pole of high order, it means evaluating the derivative for as many times asthe pole order and more to reach the analytical part. Another method is to expand thefunction using binomial expansion

(1 + ๐‘ฅ)๐‘ = 1 + ๐‘๐‘ง +๐‘ ๏ฟฝ๐‘ โˆ’ 1๏ฟฝ

2!๐‘ฅ2 +

๐‘ ๏ฟฝ๐‘ โˆ’ 1๏ฟฝ ๏ฟฝ๐‘ โˆ’ 2๏ฟฝ3!

๐‘ฅ3 +โ‹ฏ (2)

The above is valid for real ๐‘ , which can be negative or positive, but only for |๐‘ฅ| < 1. Thisis now applied to expand

๐‘“ (๐‘ง) =1

๏ฟฝ๐‘ง2 + 1๏ฟฝ3

=1

(๐‘ง โˆ’ ๐‘–)3 (๐‘ง + ๐‘–)3

Let ๐‘ง โˆ’ ๐‘– = ๐œ‰, or ๐‘ง = ๐œ‰ + ๐‘– and the above becomes

๐‘“ (๐‘ง) =1

๐œ‰3 (๐œ‰ + 2๐‘–)3

=1๐œ‰3

1

(2๐‘–)3 ๏ฟฝ1 + ๐œ‰2๐‘–๏ฟฝ3

= ๏ฟฝ๐‘–๐œ‰318๏ฟฝ

1

๏ฟฝ1 + ๐œ‰2๐‘–๏ฟฝ3 (3)

Now the binomial expansion can be used on 1

๏ฟฝ1+ ๐œ‰2๐‘– ๏ฟฝ

3 term above, which is valid for ๏ฟฝ ๐œ‰2๐‘– ๏ฟฝ < 1,

which gives

1

๏ฟฝ1 + ๐œ‰2๐‘–๏ฟฝ3 = 1 โˆ’ (3)

๐œ‰2๐‘–+(โˆ’3) (โˆ’4)

2! ๏ฟฝ๐œ‰2๐‘–๏ฟฝ

2

โˆ’(โˆ’3) (โˆ’4) (โˆ’5)

3! ๏ฟฝ๐œ‰2๐‘–๏ฟฝ

3

+(โˆ’3) (โˆ’4) (โˆ’5) (โˆ’6)

4! ๏ฟฝ๐œ‰2๐‘–๏ฟฝ

4

+โ‹ฏ

= 1 +32๐‘–๐œ‰ + 6

๐œ‰2

4๐‘–2+ 10

๐œ‰3

23๐‘–3+ 15

๐œ‰4

24๐‘–4+โ‹ฏ

= 1 +32๐‘–๐œ‰ โˆ’

32๐œ‰2 โˆ’

108๐‘–๐œ‰3 +

1516๐œ‰4 +โ‹ฏ

15

Therefore (3) becomes

๐‘“ (๐‘ง) =๐‘–8๐œ‰3 ๏ฟฝ

1 +32๐‘–๐œ‰ โˆ’

32๐œ‰2 โˆ’

108๐‘–๐œ‰3 +

1516๐œ‰4 +โ‹ฏ๏ฟฝ

= ๏ฟฝ๐‘–8๐œ‰3

โˆ’316

1๐œ‰2โˆ’316๐‘–๐œ‰+1064+

15(16) (8)

๐‘–๐œ‰ +โ‹ฏ๏ฟฝ

But ๐‘ง = ๐œ‰ + ๐‘– or ๐œ‰ = ๐‘ง โˆ’ ๐‘–, and the above becomes

๐‘“ (๐‘ง) = ๏ฟฝ๐‘–

8 (๐‘ง โˆ’ ๐‘–)3โˆ’316

1(๐‘ง โˆ’ ๐‘–)2

โˆ’316

๐‘–(๐‘ง โˆ’ ๐‘–)

+532+15128

๐‘– (๐‘ง โˆ’ ๐‘–) +โ‹ฏ๏ฟฝ (4)

Which is valid for |๐‘ง โˆ’ ๐‘–| < 1. In other words, inside a disk of radius 2, centered around๐‘ง = ๐‘–.

Comparing (4) with (1A), shows they are the same as expected. Which is the bettermethod? After working both, I think the second method is faster, but requires carefultransformation, the first method is more direct but requires more computations.

ii) Let โˆซโˆž

0๐‘‘๐‘ฅ

๏ฟฝ๐‘ฅ2+1๏ฟฝ3 = ๐ผ, hence, because

1

๏ฟฝ๐‘ฅ2+1๏ฟฝ3 is even, then

๐ผ =12 ๏ฟฝ

โˆž

โˆ’โˆž

๐‘‘๐‘ฅ

๏ฟฝ๐‘ฅ2 + 1๏ฟฝ3

=12

lim๐‘…โ†’โˆž

๏ฟฝ๐‘…

โˆ’๐‘…

๐‘‘๐‘ฅ

๏ฟฝ๐‘ฅ2 + 1๏ฟฝ3

=12

lim๐‘…โ†’โˆž

โŽ›โŽœโŽœโŽœโŽœโŽœโŽœโŽ๏ฟฝ

๐‘…

โˆ’๐‘…

๐‘‘๐‘ฅ

๏ฟฝ๐‘ฅ2 + 1๏ฟฝ3 +โˆฎ

๐ถ

๐‘‘๐‘ง

๏ฟฝ๐‘ง2 + 1๏ฟฝ3

โŽžโŽŸโŽŸโŽŸโŽŸโŽŸโŽŸโŽ 

The above is valid as long as one can show โˆฎ๐ถ

๐‘‘๐‘ง

๏ฟฝ๐‘ง2+1๏ฟฝ3 โ†’ 0 as ๐‘… โ†’ โˆž. The contour ๐ถ is

from ๐‘… to โˆ’๐‘… over semicircle, going anticlock wise. The radius of the circle is ๐‘…. Since theabove integration now includes ๐‘ง = ๐‘–, then by residual theorem, the above is just โˆ’ 3๐‘–

16 . Theresidue was found in the first part. In other words

12

2๐œ‹๐‘–๏ฟฝโˆ’ 3๐‘–16 ๏ฟฝ

๏ฟฝ๏ฟฝ๏ฟฝ๏ฟฝ๏ฟฝ๏ฟฝ๏ฟฝ๏ฟฝ๏ฟฝ๏ฟฝ๏ฟฝ๏ฟฝ๏ฟฝ๏ฟฝ๏ฟฝ๏ฟฝ๏ฟฝ๏ฟฝ๏ฟฝ๏ฟฝ๏ฟฝ๏ฟฝ๏ฟฝ๏ฟฝ๏ฟฝ๏ฟฝ๏ฟฝ๏ฟฝ๏ฟฝ๏ฟฝ๏ฟฝ๏ฟฝ๏ฟฝ๏ฟฝ๏ฟฝ๏ฟฝ๏ฟฝ๏ฟฝ๏ฟฝ๏ฟฝ๏ฟฝ๏ฟฝ๏ฟฝ๏ฟฝ๏ฟฝ๏ฟฝ๏ฟฝ๏ฟฝ๏ฟฝlim๐‘…โ†’โˆž

โŽ›โŽœโŽœโŽœโŽœโŽœโŽœโŽ๏ฟฝ

๐‘…

โˆ’๐‘…

๐‘‘๐‘ฅ

๏ฟฝ๐‘ฅ2 + 1๏ฟฝ3 +โˆฎ

๐ถ

๐‘‘๐‘ง

๏ฟฝ๐‘ง2 + 1๏ฟฝ3

โŽžโŽŸโŽŸโŽŸโŽŸโŽŸโŽŸโŽ  =

12 ๏ฟฝ2๐œ‹๐‘– ๏ฟฝโˆ’

3๐‘–16๏ฟฝ๏ฟฝ

Letting ๐‘ง = ๐‘… ๐‘’๐‘–๐œƒ and taking ๐‘… โ†’ โˆž, then โˆฎ๐ถ

๐‘‘๐‘ง

๏ฟฝ๐‘ง2+1๏ฟฝ3 โ†’ 0 and the above simplifies to

12

lim๐‘…โ†’โˆž

๏ฟฝ๐‘…

โˆ’๐‘…

๐‘‘๐‘ฅ

๏ฟฝ๐‘ฅ2 + 1๏ฟฝ3 =

12 ๏ฟฝ2๐œ‹

316๏ฟฝ

12

lim๐‘…โ†’โˆž

๏ฟฝ๐‘…

โˆ’๐‘…

๐‘‘๐‘ฅ

๏ฟฝ๐‘ฅ2 + 1๏ฟฝ3 = ๐œ‹

316

lim๐‘…โ†’โˆž

๏ฟฝ๐‘…

0

๐‘‘๐‘ฅ

๏ฟฝ๐‘ฅ2 + 1๏ฟฝ3 =

3๐œ‹16

Therefore

๏ฟฝโˆž

0

๐‘‘๐‘ฅ

๏ฟฝ๐‘ฅ2 + 1๏ฟฝ3 =

3๐œ‹16

2.1.2 Problem 2

Expand ๐‘“ (๐‘ฅ) = ๐‘ฅ๐ฟ as Fourier series for 0 < ๐‘ฅ < ๐œ‹

๐ฟ and ๐‘“ (๐‘ฅ) = โˆ’ ๐‘ฅ๐ฟ for โˆ’๐œ‹๐ฟ < ๐‘ฅ < 0.

solution:

This function is even. For example, for ๐ฟ = 2, it looks like this

16

Hence the Fourier series will not have sin terms.

๐‘“ (๐‘ฅ) =๐‘Ž02+

โˆž๏ฟฝ๐‘›=1

๐‘Ž๐‘› cos ๏ฟฝ2๐‘›๐œ‹๐‘‡๐‘ฅ๏ฟฝ +

โˆž๏ฟฝ๐‘›=1

๐‘๐‘› sin ๏ฟฝ2๐‘›๐œ‹๐‘‡๐‘ฅ๏ฟฝ

=๐‘Ž02+

โˆž๏ฟฝ๐‘›=1

๐‘Ž๐‘› cos ๏ฟฝ2๐‘›๐œ‹๐‘‡๐‘ฅ๏ฟฝ

Where in the above ๐‘‡ is the period of the function. In this problem ๐‘‡ = 2๐œ‹๐ฟ , hence the above

becomes

๐‘“ (๐‘ฅ) =๐‘Ž02+

โˆž๏ฟฝ๐‘›=1

๐‘Ž๐‘› cos

โŽ›โŽœโŽœโŽœโŽœโŽœโŽ2๐‘›๐œ‹2๐œ‹๐ฟ

๐‘ฅ

โŽžโŽŸโŽŸโŽŸโŽŸโŽŸโŽ 

=๐‘Ž02+

โˆž๏ฟฝ๐‘›=1

๐‘Ž๐‘› cos (๐‘›๐ฟ๐‘ฅ) (1)

Where

๐‘Ž0 =1๐‘‡2

๏ฟฝ๐‘‡2

โˆ’๐‘‡2

๐‘“ (๐‘ฅ) ๐‘‘๐‘ฅ =22๐œ‹๐ฟ

๏ฟฝ๐œ‹๐ฟ

โˆ’๐œ‹๐ฟ

๐‘“ (๐‘ฅ) ๐‘‘๐‘ฅ =๐ฟ๐œ‹ ๏ฟฝ

๐œ‹๐ฟ

โˆ’๐œ‹๐ฟ

๐‘“ (๐‘ฅ) ๐‘‘๐‘ฅ

=๐ฟ๐œ‹(2)๏ฟฝ

๐œ‹๐ฟ

0

๐‘ฅ๐ฟ๐‘‘๐‘ฅ

=2๐ฟ๐œ‹1๐ฟ ๏ฟฝ

๐‘ฅ2

2 ๏ฟฝ

๐œ‹๐ฟ

0

=1๐œ‹ ๏ฟฝ

๐œ‹2

๐ฟ2 ๏ฟฝ

=๐œ‹๐ฟ2

And

๐‘Ž๐‘› =1๐‘‡2

๏ฟฝ๐œ‹๐ฟ

โˆ’๐œ‹๐ฟ

๐‘“ (๐‘ฅ) cos (๐‘›๐ฟ๐‘ฅ) ๐‘‘๐‘ฅ

=22๐œ‹๐ฟ

(2)๏ฟฝ๐œ‹๐ฟ

0๐‘“ (๐‘ฅ) cos (๐‘›๐ฟ๐‘ฅ) ๐‘‘๐‘ฅ

=2๐ฟ๐œ‹ ๏ฟฝ

๐œ‹๐ฟ

0

๐‘ฅ๐ฟ

cos (๐‘›๐ฟ๐‘ฅ) ๐‘‘๐‘ฅ

=2๐œ‹ ๏ฟฝ

๐œ‹๐ฟ

0๐‘ฅ cos (๐‘›๐ฟ๐‘ฅ) ๐‘‘๐‘ฅ

Using integration by parts โˆซ๐‘ข๐‘‘๐‘ฃ = ๐‘ข๐‘ฃ โˆ’ โˆซ๐‘ฃ๐‘‘๐‘ข. Let ๐‘ข = ๐‘ฅ, ๐‘‘๐‘ข = 1 and let ๐‘‘๐‘ฃ = cos (๐‘›๐ฟ๐‘ฅ) , ๐‘ฃ =

17

sin(๐‘›๐ฟ๐‘ฅ)๐‘›๐ฟ , therefore the above becomes

๐‘Ž๐‘› =2๐œ‹

โŽ›โŽœโŽœโŽœโŽœโŽœโŽ๏ฟฝ๐‘ฅ

sin (๐‘›๐ฟ๐‘ฅ)๐‘›๐ฟ ๏ฟฝ

๐œ‹๐ฟ

0โˆ’๏ฟฝ

๐œ‹๐ฟ

0

sin (๐‘›๐ฟ๐‘ฅ)๐‘›๐ฟ

๐‘‘๐‘ฅ

โŽžโŽŸโŽŸโŽŸโŽŸโŽŸโŽ 

=2๐œ‹

โŽ›โŽœโŽœโŽœโŽœโŽœโŽ

โŽกโŽขโŽขโŽขโŽขโŽขโŽฃ๐œ‹๐ฟ

sin ๏ฟฝ๐‘›๐ฟ๐œ‹๐ฟ ๏ฟฝ๐‘›๐ฟ

โˆ’ 0

โŽคโŽฅโŽฅโŽฅโŽฅโŽฅโŽฆ โˆ’

1๐‘›๐ฟ ๏ฟฝ

๐œ‹๐ฟ

0sin (๐‘›๐ฟ๐‘ฅ) ๐‘‘๐‘ฅ

โŽžโŽŸโŽŸโŽŸโŽŸโŽŸโŽ 

=2๐œ‹

โŽ›โŽœโŽœโŽœโŽโˆ’

1๐‘›๐ฟ ๏ฟฝ

๐œ‹๐ฟ

0sin (๐‘›๐ฟ๐‘ฅ) ๐‘‘๐‘ฅ

โŽžโŽŸโŽŸโŽŸโŽ 

=โˆ’2๐œ‹๐‘›๐ฟ

โŽ›โŽœโŽœโŽœโŽ๏ฟฝ

๐œ‹๐ฟ

0sin (๐‘›๐ฟ๐‘ฅ) ๐‘‘๐‘ฅ

โŽžโŽŸโŽŸโŽŸโŽ 

=โˆ’2๐œ‹๐‘›๐ฟ ๏ฟฝ

โˆ’cos (๐‘›๐ฟ๐‘ฅ)

๐‘›๐ฟ ๏ฟฝ

๐œ‹๐ฟ

0

=2

๐œ‹๐‘›2๐ฟ2(cos (๐‘›๐ฟ๐‘ฅ))

๐œ‹๐ฟ0

=2

๐œ‹๐‘›2๐ฟ2๏ฟฝcos ๏ฟฝ๐‘›๐ฟ๐œ‹

๐ฟ๏ฟฝ โˆ’ 1๏ฟฝ

=2

๐œ‹๐‘›2๐ฟ2(cos (๐‘›๐œ‹) โˆ’ 1)

=2

๐œ‹๐‘›2๐ฟ2๏ฟฝ(โˆ’1)๐‘› โˆ’ 1๏ฟฝ

=โˆ’2 + 2 (โˆ’1)๐‘›

๐œ‹๐‘›2๐ฟ2Therefore from (1) the Fourier series is

๐‘“ (๐‘ฅ) =๐‘Ž02+

โˆž๏ฟฝ๐‘›=1

๐‘Ž๐‘› cos (๐‘›๐ฟ๐‘ฅ)

=๐œ‹2๐ฟ2

+โˆž๏ฟฝ๐‘›=1

2๐œ‹๐‘›2๐ฟ2

๏ฟฝ(โˆ’1)๐‘› โˆ’ 1๏ฟฝ cos (๐‘›๐ฟ๐‘ฅ)

The convergence is of order ๐‘›2, so it is fast. Only few terms are needed to obtain very goodapproximation.

2.1.3 Problem 3

(i) Solve ๐‘ฅ๐‘ฆโ€ฒ + 3๐‘ฅ + ๐‘ฆ = 0. (ii) Solve ๐‘ฆโ€ฒโ€ฒ โˆ’ 2๐‘ฆโ€ฒ + ๐‘ฆ = ๐‘’๐‘ฅ

Solution

(i). This is linear first order ODE.

๐‘ฆโ€ฒ + 3 +๐‘ฆ๐‘ฅ= 0 ๐‘ฅ โ‰  0

๐‘ฆโ€ฒ +๐‘ฆ๐‘ฅ= โˆ’3

Integrating factor is ๐œ‡ = ๐‘’โˆซ1๐‘ฅ๐‘‘๐‘ฅ = ๐‘’ln ๐‘ฅ = ๐‘ฅ. Multiplying both sides of the above by ๐œ‡, the left

side becomes complete di๏ฟฝerential and it simplifies to๐‘‘๐‘‘๐‘ฅ๏ฟฝ๐œ‡๐‘ฆ๏ฟฝ = โˆ’3๐œ‡

๐‘‘๐‘‘๐‘ฅ๏ฟฝ๐‘ฅ๐‘ฆ๏ฟฝ = โˆ’3๐‘ฅ

๐‘‘ ๏ฟฝ๐‘ฅ๐‘ฆ๏ฟฝ = โˆ’3๐‘ฅ๐‘‘๐‘ฅ

Integrating gives

๐‘ฅ๐‘ฆ = โˆ’32๐‘ฅ2 + ๐ถ

Hence the solution is

๐‘ฆ = โˆ’32๐‘ฅ +๐ถ๐‘ฅ ๐‘ฅ โ‰  0

(ii) ๐‘ฆโ€ฒโ€ฒ โˆ’ 2๐‘ฆโ€ฒ + ๐‘ฆ = ๐‘’๐‘ฅ is linear second order with constant coe๏ฟฝcients. The solution to the

18

homogeneous part ๐‘ฆโ€ฒโ€ฒโˆ’2๐‘ฆโ€ฒ+๐‘ฆ = 0 can be found by first finding the roots of the characteristicequation ๐‘ 2 โˆ’ 2๐‘  + 1 = 0 , hence ๐‘  = โˆ’๐‘

2๐‘Ž ยฑ12๐‘Žโˆš๐‘

2 โˆ’ 4๐‘Ž๐‘ or, ๐‘  = 22 ยฑ

12โˆš4 โˆ’ 4 = 1. One double root.

Therefore

๐‘ฆโ„Ž (๐‘ฅ) = ๐ถ1๐‘’๐‘ฅ + ๐ถ2๐‘ฅ๐‘’๐‘ฅ

To find the particular solution, the method of undetermined coe๏ฟฝcients is used. Since theforcing function is ๐‘’๐‘ฅ, then a guess ๐‘ฆ๐‘ = ๐‘˜๐‘’๐‘ฅ. But ๐‘’๐‘ฅ is a basis solution. Hence ๐‘ฆ๐‘ = ๐‘˜๐‘ฅ๐‘’๐‘ฅ isnow selected. But also ๐‘ฅ๐‘’๐‘ฅ is basis solution. Then ๐‘ฆ๐‘ = ๐‘˜๐‘ฅ2๐‘’๐‘ฅ is finally selected. Substitutingthis into the original ODE in order to solve for ๐‘˜, gives

๐‘ฆโ€ฒโ€ฒ๐‘ โˆ’ 2๐‘ฆโ€ฒ๐‘ + ๐‘ฆ๐‘ = ๐‘’๐‘ฅ

But ๐‘ฆโ€ฒ๐‘ = 2๐‘˜๐‘ฅ๐‘’๐‘ฅ + ๐‘˜๐‘ฅ2๐‘’๐‘ฅ and ๐‘ฆโ€ฒโ€ฒ๐‘ = 2๐‘˜๐‘’๐‘ฅ + 2๐‘˜๐‘ฅ๐‘’๐‘ฅ + 2๐‘˜๐‘ฅ๐‘’๐‘ฅ + ๐‘˜๐‘ฅ2๐‘’๐‘ฅ. Hence the above becomes

๏ฟฝ2๐‘˜๐‘’๐‘ฅ + 2๐‘˜๐‘ฅ๐‘’๐‘ฅ + 2๐‘˜๐‘ฅ๐‘’๐‘ฅ + ๐‘˜๐‘ฅ2๐‘’๐‘ฅ๏ฟฝ โˆ’ 2 ๏ฟฝ2๐‘˜๐‘ฅ๐‘’๐‘ฅ + ๐‘˜๐‘ฅ2๐‘’๐‘ฅ๏ฟฝ + ๐‘˜๐‘ฅ2๐‘’๐‘ฅ = ๐‘’๐‘ฅ

๏ฟฝ2๐‘˜ + 2๐‘˜๐‘ฅ + 2๐‘˜๐‘ฅ + ๐‘˜๐‘ฅ2๏ฟฝ โˆ’ 2 ๏ฟฝ2๐‘˜๐‘ฅ + ๐‘˜๐‘ฅ2๏ฟฝ + ๐‘˜๐‘ฅ2 = 12๐‘˜ + 4๐‘˜๐‘ฅ + ๐‘˜๐‘ฅ2 โˆ’ 4๐‘˜๐‘ฅ โˆ’ 2๐‘˜๐‘ฅ2 + ๐‘˜๐‘ฅ2 = 1

2๐‘˜ = 1

๐‘˜ =12

Therefore ๐‘ฆ๐‘ =12๐‘ฅ2๐‘’๐‘ฅ, and the complete general solution is

๐‘ฆ (๐‘ฅ) = ๐‘ฆโ„Ž (๐‘ฅ) + ๐‘ฆ๐‘ (๐‘ฅ)

Therefore

๐‘ฆ (๐‘ฅ) = ๐ถ1๐‘’๐‘ฅ + ๐ถ2๐‘ฅ๐‘’๐‘ฅ +12๐‘ฅ2๐‘’๐‘ฅ

2.1.4 Problem 4

Figure 2.1: Problem 4 Statement

solution

Part (1)

๐‘ฅ2๐‘ฆโ€ฒโ€ฒ + 2๐‘ฅ๐‘ฆโ€ฒ + ๐‘ฅ2๐‘ฆ = 0

๐‘ฆโ€ฒโ€ฒ +2๐‘ฅ๐‘ฆโ€ฒ + ๐‘ฆ = 0

๐‘ฅ is a regular singular point. Becuase lim๐‘ฅโ†’0 (๐‘ฅ โˆ’ 0)2๐‘ฅ = 2. Since limit exist, then regular

singular point.

19

Part (2)

Let

๐‘ฆ =โˆž๏ฟฝ๐‘›=0

๐‘Ž๐‘›๐‘ฅ๐‘›

๐‘ฆโ€ฒ =โˆž๏ฟฝ๐‘›=0

๐‘›๐‘Ž๐‘›๐‘ฅ๐‘›โˆ’1

๐‘ฆโ€ฒโ€ฒ =โˆž๏ฟฝ๐‘›=0

๐‘› (๐‘› โˆ’ 1) ๐‘Ž๐‘›๐‘ฅ๐‘›โˆ’2

The ODE becomes

๐‘ฅ2โˆž๏ฟฝ๐‘›=0

๐‘› (๐‘› โˆ’ 1) ๐‘Ž๐‘›๐‘ฅ๐‘›โˆ’2 + 2๐‘ฅโˆž๏ฟฝ๐‘›=0

๐‘›๐‘Ž๐‘›๐‘ฅ๐‘›โˆ’1 + ๐‘ฅ2โˆž๏ฟฝ๐‘›=0

๐‘Ž๐‘›๐‘ฅ๐‘› = 0

โˆž๏ฟฝ๐‘›=0

๐‘› (๐‘› โˆ’ 1) ๐‘Ž๐‘›๐‘ฅ๐‘› +โˆž๏ฟฝ๐‘›=0

2๐‘›๐‘Ž๐‘›๐‘ฅ๐‘› +โˆž๏ฟฝ๐‘›=0

๐‘Ž๐‘›๐‘ฅ๐‘›+2 = 0

โˆž๏ฟฝ๐‘›=0

๐‘› (๐‘› โˆ’ 1) ๐‘Ž๐‘›๐‘ฅ๐‘› +โˆž๏ฟฝ๐‘›=0

2๐‘›๐‘Ž๐‘›๐‘ฅ๐‘› +โˆž๏ฟฝ๐‘›=2

๐‘Ž๐‘›โˆ’2๐‘ฅ๐‘› = 0

Thereforeโˆž๏ฟฝ๐‘›=0

(๐‘› (๐‘› โˆ’ 1) + 2๐‘›) ๐‘Ž๐‘›๐‘ฅ๐‘› +โˆž๏ฟฝ๐‘›=2

๐‘Ž๐‘›โˆ’2๐‘ฅ๐‘› = 0

We start from ๐‘› = 1 since ๐‘› = 0 is used to find the indicial equation. For ๐‘› = 1

2๐‘Ž1 = 0๐‘Ž1 = 0

For ๐‘› โ‰ฅ 2(๐‘› (๐‘› โˆ’ 1) + 2๐‘›) ๐‘Ž๐‘› + ๐‘Ž๐‘›โˆ’2 = 0

๐‘Ž๐‘› (๐‘› (๐‘› โˆ’ 1) + 2๐‘›) = โˆ’๐‘Ž๐‘›โˆ’2

๐‘Ž๐‘› =โˆ’๐‘Ž๐‘›โˆ’2

๐‘› (๐‘› โˆ’ 1) + 2๐‘›For ๐‘› = 2

๐‘Ž2 =โˆ’๐‘Ž0

2 (2 โˆ’ 1) + 4

= โˆ’16๐‘Ž0

For ๐‘› = 3

๐‘Ž3 =โˆ’๐‘Ž1

3 (2) + 6Since ๐‘Ž1 = 0 then ๐‘Ž3 = 0. All odd terms are therefore zero.

For ๐‘› = 4

๐‘Ž๐‘› =โˆ’๐‘Ž2

(4) (3) + 8= โˆ’

120๐‘Ž2 = โˆ’

120 ๏ฟฝ

โˆ’16๐‘Ž0๏ฟฝ =

1120

๐‘Ž0

Therefore

๐‘ฆ1 (๐‘ฅ) =โˆž๏ฟฝ๐‘›=0

๐‘Ž๐‘›๐‘ฅ๐‘›

= ๐‘Ž0 + ๐‘Ž2๐‘ฅ2 + ๐‘Ž4๐‘ฅ4 +โ‹ฏ

= ๐‘Ž0 โˆ’16๐‘Ž0๐‘ฅ2 +

160๐‘Ž0๐‘ฅ4 โˆ’โ‹ฏ

= ๐‘Ž0 ๏ฟฝ1 โˆ’16๐‘ฅ2 +

11200

๐‘ฅ4 โˆ’โ‹ฏ๏ฟฝ

Part (3)

setting ๐‘Ž0 = 1 as problem says, and since sin ๐‘ฅ = ๐‘ฅ โˆ’ ๐‘ฅ3

6 +๐‘ฅ5

120 โˆ’โ‹ฏ then the above is

๐‘ฆ1 (๐‘ฅ) =sin ๐‘ฅ๐‘ฅ

20

Part (4)

๐‘Š(๐‘ฅ) = ๏ฟฝ๐‘ฆ1 ๐‘ฆ2๐‘ฆโ€ฒ1 ๐‘ฆโ€ฒ2

๏ฟฝ = ๏ฟฝsin ๐‘ฅ๐‘ฅ ๐‘ฆ2

๐‘ฅ cos ๐‘ฅโˆ’sin ๐‘ฅ๐‘ฅ2 ๐‘ฆโ€ฒ2

๏ฟฝ = ๐‘ฆโ€ฒ2sin ๐‘ฅ๐‘ฅ

โˆ’ ๐‘ฆ2๐‘ฅ cos ๐‘ฅ โˆ’ sin ๐‘ฅ

๐‘ฅ2

But from Abelโ€™s theorem, ๐‘Š(๐‘ฅ) = ๐ถ๐‘’โˆซโˆ’๐‘(๐‘ฅ)๐‘‘๐‘ฅ where ๐‘ (๐‘ฅ) is the coe๏ฟฝcient in the ODE

๐‘ฆโ€ฒโ€ฒ + ๐‘ (๐‘ฅ) ๐‘ฆโ€ฒ + ๐‘ž (๐‘ฅ) ๐‘ฆ = 0. Since the ODE is ๐‘ฆโ€ฒโ€ฒ โˆ’ 2๐‘ฅ + ๐‘ฆ = 0 then ๐‘ =

โˆ’2๐‘ฅ and ๐‘Š(๐‘ฅ) = ๐‘’โˆซ

โˆ’2๐‘ฅ ๐‘‘๐‘ฅ =

๐‘’โˆ’2 ln ๐‘ฅ = ๐‘ฅโˆ’2. Hence

๐‘ฆโ€ฒ2sin ๐‘ฅ๐‘ฅ

โˆ’ ๐‘ฆ2๐‘ฅ cos ๐‘ฅ โˆ’ sin ๐‘ฅ

๐‘ฅ2=๐ถ๐‘ฅ2

๐‘ฅ sin (๐‘ฅ) ๐‘ฆโ€ฒ2 โˆ’ ๐‘ฆ2 (๐‘ฅ cos ๐‘ฅ โˆ’ sin ๐‘ฅ) = ๐ถ

๐‘ฆโ€ฒ2 โˆ’ ๐‘ฆ2 ๏ฟฝcos ๐‘ฅsin ๐‘ฅ โˆ’

1๐‘ฅ๏ฟฝ=

๐ถ๐‘ฅ sin ๐‘ฅ

Integrating factor is ๐œ‡ = ๐‘’โˆซโˆ’cos ๐‘ฅsin ๐‘ฅ +

1๐‘ฅ๐‘‘๐‘ฅ = ๐‘’โˆซ

โˆ’ ๐‘‘๐‘‘๐‘ฅ (sin ๐‘ฅ)

sin ๐‘ฅ ๐‘‘๐‘ฅ๐‘’โˆซ1๐‘ฅ๐‘‘๐‘ฅ = ๐‘’โˆซ

โˆ’1sin ๐‘ฅ๐‘‘(sin ๐‘ฅ) + ๐‘’ln ๐‘ฅ = ๐‘’โˆ’ ln(sin ๐‘ฅ)๐‘ฅ =

๐‘ฅsin ๐‘ฅ . Multiplying both sides by this integrating factor gives

๐‘‘๐‘‘๐‘ฅ๏ฟฝ๐‘ฆ2

๐‘ฅsin ๐‘ฅ

๏ฟฝ =๐ถ

sin2 ๐‘ฅIntegrating gives

๐‘ฆ2๐‘ฅ

sin ๐‘ฅ =๐ถ cos (๐‘ฅ)โˆ’ sin (๐‘ฅ) + ๐ถ2

๐‘ฆ2 (๐‘ฅ) = โˆ’๐ถcos ๐‘ฅ๐‘ฅ

+ ๐ถ2sin ๐‘ฅ๐‘ฅ

= ๐ถ1cos ๐‘ฅ๐‘ฅ

+ ๐ถ2sin ๐‘ฅ๐‘ฅ

But ๐ถ2 sin ๐‘ฅ is the second solution, so we only keep ๐‘ฆ2 (๐‘ฅ) = ๐ถ1cos ๐‘ฅ๐‘ฅ . Hence the final solution

is

๐‘ฆ (๐‘ฅ) = ๐ถ1cos ๐‘ฅ๐‘ฅ

+ ๐ถ2sin ๐‘ฅ๐‘ฅ

2.1.5 Problem 5

Find solution to ๐‘‘2

๐‘‘๐‘ฅ2๐บ (๐‘ฅ, ๐‘ฅ0)+๐‘˜2๐บ (๐‘ฅ, ๐‘ฅ0) = ๐›ฟ (๐‘ฅ โˆ’ ๐‘ฅ0) subject to boundary conditions ๐บ (0, ๐‘ฅ0) =

๐บ (๐ฟ, ๐‘ฅ0) = 0

solution

First we obtain the solution to the homogeneous ODE ๐‘ฆโ€ฒโ€ฒ + ๐‘˜2๐‘ฆ = 0 with ๐‘ฆ (0) = 0, ๐‘ฆ (๐‘) = 0which has the two solutions ๐‘ฆ1 (๐‘ฅ) = cos ๐‘˜๐‘ฅ, ๐‘ฆ2 (๐‘ฅ) = sin (๐‘˜๐‘ฅ).

Therefore the solution to the Green function is

๐บ (๐‘ฅ, ๐‘ฅ0) =

โŽงโŽชโŽชโŽจโŽชโŽชโŽฉ๐ด1๐‘ฆ1 (๐‘ฅ) + ๐ด2๐‘ฆ2 (๐‘ฅ) 0 < ๐‘ฅ < ๐‘ฅ0๐ต1๐‘ฆ1 (๐‘ฅ) + ๐ต2๐‘ฆ2 (๐‘ฅ) ๐‘ฅ < ๐‘ฅ0 < ๐‘

=

โŽงโŽชโŽชโŽจโŽชโŽชโŽฉ๐ด1 cos (๐‘˜๐‘ฅ) + ๐ด2 sin (๐‘˜๐‘ฅ) 0 < ๐‘ฅ < ๐‘ฅ0๐ต1 cos (๐‘˜๐‘ฅ) + ๐ต2 sin (๐‘˜๐‘ฅ) ๐‘ฅ < ๐‘ฅ0 < ๐‘

Where ๐ด๐‘–, ๐ต๐‘– are constants to be found. From the condition ๐บ (0, ๐‘ฅ0) = 0 then the firstsolution above gives ๐ด1 cos (0) = 0 โ†’ ๐ด1 = 0. And from ๐บ (๐ฟ, ๐‘ฅ0) = 0 the second solutionabove gives ๐ต1 cos (๐‘˜๐ฟ) + ๐ต2 sin (๐‘˜๐ฟ๐‘) = 0 or ๐ต1 = โˆ’

๐ต2 sin(๐‘˜๐ฟ)cos(๐‘˜๐ฟ๐‘) , hence the solution now becomes

21

๐บ (๐‘ฅ, ๐‘ฅ0) =

โŽงโŽชโŽชโŽจโŽชโŽชโŽฉ

๐ด2 sin (๐‘˜๐‘ฅ) 0 < ๐‘ฅ < ๐‘ฅ0๐ต1 cos (๐‘˜๐‘ฅ) + ๐ต2 sin (๐‘˜๐‘ฅ) ๐‘ฅ < ๐‘ฅ0 < ๐ฟ

=

โŽงโŽชโŽชโŽจโŽชโŽชโŽฉ

๐ด2 sin (๐‘˜๐‘ฅ) 0 < ๐‘ฅ < ๐‘ฅ0โˆ’๐ต2 sin(๐‘˜๐ฟ)

cos(๐‘˜๐ฟ) cos (๐‘˜๐‘ฅ) + ๐ต2 sin (๐‘˜๐‘ฅ) ๐‘ฅ < ๐‘ฅ0 < ๐ฟ

=

โŽงโŽชโŽชโŽจโŽชโŽชโŽฉ

๐ด2 sin (๐‘˜๐‘ฅ) 0 < ๐‘ฅ < ๐‘ฅ0๐ต2

cos(๐‘˜๐ฟ) (sin (๐‘˜๐‘ฅ) cos (๐‘˜๐ฟ) โˆ’ sin (๐‘˜๐ฟ) cos (๐‘˜๐‘ฅ)) ๐‘ฅ < ๐‘ฅ0 < ๐ฟ

Using sin (๐‘Ž โˆ’ ๐‘) = sin ๐‘Ž cos ๐‘โˆ’cos ๐‘Ž sin ๐‘, then sin (๐‘˜๐‘ฅ) cos (๐‘˜๐ฟ)โˆ’sin (๐‘˜๐ฟ) cos (๐‘˜๐‘ฅ) = sin (๐‘˜๐‘ฅ โˆ’ ๐‘˜๐ฟ) =sin (๐‘˜ (๐‘ฅ โˆ’ ๐ฟ)) and the above becomes

๐บ (๐‘ฅ, ๐‘ฅ0) =

โŽงโŽชโŽชโŽจโŽชโŽชโŽฉ

๐ด2 sin (๐‘˜๐‘ฅ) 0 < ๐‘ฅ < ๐‘ฅ0๐ต2

cos(๐‘˜๐ฟ) sin (๐‘˜ (๐‘ฅ โˆ’ ๐ฟ)) ๐‘ฅ < ๐‘ฅ0 < ๐ฟ(1A)

Now from continuity condition ๐บ (๐‘ฅ, ๐‘ฅ0)๐‘ฅ=๐‘ฅ0โˆ’๐œ€ = ๐บ (๐‘ฅ, ๐‘ฅ0)๐‘ฅ=๐‘ฅ0+๐œ€ i.e. at ๐‘ฅ = ๐‘ฅ0, then (from nowon, we switch to ๐‘ฅ0).

๐ด2 sin (๐‘˜๐‘ฅ0) =๐ต2

cos (๐‘˜๐ฟ) sin (๐‘˜ (๐‘ฅ0 โˆ’ ๐ฟ)) (1)

Now we find the derivative of ๐บ at ๐‘ฅ = ๐‘ฅ0 gives

๐‘‘๐‘‘๐‘ฅ๐บ (๐‘ฅ, ๐‘ฅ0)๏ฟฝ

๐‘ฅ=๐‘ฅ0=

โŽงโŽชโŽชโŽจโŽชโŽชโŽฉ

๐‘˜๐ด2 cos (๐‘˜๐‘ฅ0) 0 < ๐‘ฅ < ๐‘ฅ0๐‘˜๐ต2

cos(๐‘˜๐ฟ) cos (๐‘˜ (๐‘ฅ0 โˆ’ ๐ฟ)) ๐‘ฅ < ๐‘ฅ0 < ๐ฟ

And from jump discontinuty in derivative of ๐บ at ๐‘ฅ = ๐‘ฅ0 would obtain, since ๐บโ€ฒ๐‘ฅ>๐‘ฅ0+๐œ€ โˆ’๐บโ€ฒ๐‘ฅ<๐‘ฅ0โˆ’๐œ€ =

โˆ’1๐‘(๐‘ฅ) , then

๐‘˜๐ต2cos (๐‘˜๐‘) cos (๐‘˜ (๐‘ฅ0 โˆ’ ๐ฟ)) โˆ’ ๐‘˜๐ด2 cos (๐‘˜๐‘ฅ0) =

โˆ’1๐‘ (๐‘ฅ)

But since the ODE is ๐‘ฆโ€ฒโ€ฒ + ๐‘˜2๐‘ฆ = 0 then in SL form this is โˆ’ ๏ฟฝ๐‘ฆโ€ฒ๏ฟฝโ€ฒ+ ๐‘˜2๐‘ฆ = 0, and comparing

to โˆ’ ๏ฟฝ๐‘๐‘ฆโ€ฒ๏ฟฝโ€ฒ+ ๐‘˜2๐‘ฆ = 0 we see that ๐‘ = โˆ’1. Then above becomes

๐‘˜๐ต2cos (๐‘˜๐ฟ) cos (๐‘˜ (๐‘ฅ0 โˆ’ ๐ฟ)) โˆ’ ๐‘˜๐ด2 cos (๐‘˜๐‘ฅ0) = 1 (2)

From (1,2) we solve for ๐ด1, ๐ต2. From (1)

๐ด2 =๐ต2

cos (๐‘˜๐‘) sin (๐‘˜๐‘ฅ0)sin (๐‘˜ (๐‘ฅ0 โˆ’ ๐ฟ)) (3)

Substituting into (2) gives

๐‘˜๐ต2cos (๐‘˜๐ฟ) cos (๐‘˜ (๐‘ฅ0 โˆ’ ๐ฟ)) โˆ’ ๐‘˜ ๏ฟฝ

๐ต2cos (๐‘˜๐ฟ) sin (๐‘˜๐‘ฅ0)

sin (๐‘˜ (๐‘ฅ0 โˆ’ ๐ฟ))๏ฟฝ cos (๐‘˜๐‘ฅ0) = 1

๐‘˜๐ต2 cos (๐‘˜ (๐‘ฅ0 โˆ’ ๐ฟ)) โˆ’ ๐‘˜๐ต2 sin (๐‘˜ (๐‘ฅ0 โˆ’ ๐ฟ))cos (๐‘˜๐‘ฅ0)sin (๐‘˜๐‘ฅ0)

= cos (๐‘˜๐ฟ)

๐‘˜๐ต2 (sin (๐‘˜๐‘ฅ0) cos (๐‘˜ (๐‘ฅ0 โˆ’ ๐ฟ)) โˆ’ sin (๐‘˜ (๐‘ฅ0 โˆ’ ๐ฟ)) cos (๐‘˜๐‘ฅ0)) = cos (๐‘˜๐ฟ) sin (๐‘˜๐‘ฅ0) (4)

Using sin (๐‘Ž โˆ’ ๐‘) = sin ๐‘Ž cos ๐‘ โˆ’ cos ๐‘Ž sin ๐‘, thensin (๐‘˜๐‘ฅ0) cos (๐‘˜ (๐‘ฅ0 โˆ’ ๐ฟ)) โˆ’ sin (๐‘˜ (๐‘ฅ0 โˆ’ ๐ฟ)) cos (๐‘˜๐‘ฅ0) = sin (๐‘˜๐‘ฅ0 โˆ’ ๐‘˜ (๐‘ฅ0 โˆ’ ๐ฟ))

= sin (๐‘˜๐ฟ)

22

Then (3) becomes

๐‘˜๐ต2 sin (๐‘˜๐ฟ) = cos (๐‘˜๐ฟ) sin (๐‘˜๐‘ฅ0)

๐ต2 =cos (๐‘˜๐ฟ) sin (๐‘˜๐‘ฅ0)

๐‘˜ sin (๐‘˜๐ฟ)

Substituing the above in (3) gives

๐ด2 =cos (๐‘˜๐ฟ) sin (๐‘˜๐‘ฅ0)

๐‘˜ sin (๐‘˜๐ฟ) cos (๐‘˜๐ฟ) sin (๐‘˜๐‘ฅ0)sin (๐‘˜ (๐‘ฅ0 โˆ’ ๐ฟ))

=sin (๐‘˜ (๐‘ฅ0 โˆ’ ๐ฟ))๐‘˜ sin (๐‘˜๐ฟ)

Using ๐ด2, ๐ต2 found above in (1A) gives

๐บ (๐‘ฅ, ๐‘ฅ0) =

โŽงโŽชโŽชโŽจโŽชโŽชโŽฉ

sin(๐‘˜(๐‘ฅ0โˆ’๐ฟ))๐‘˜ sin(๐‘˜๐ฟ) sin (๐‘˜๐‘ฅ) 0 < ๐‘ฅ < ๐‘ฅ0

cos(๐‘˜๐ฟ) sin(๐‘˜๐‘ฅ0)๐‘˜ sin(๐‘˜๐ฟ) cos(๐‘˜๐‘) sin (๐‘˜ (๐‘ฅ โˆ’ ๐ฟ)) ๐‘ฅ < ๐‘ฅ0 < ๐ฟ

=1

๐‘˜ sin (๐‘˜๐ฟ)

โŽงโŽชโŽชโŽจโŽชโŽชโŽฉ

sin (๐‘˜ (๐‘ฅ0 โˆ’ ๐ฟ)) sin (๐‘˜๐‘ฅ) 0 < ๐‘ฅ < ๐‘ฅ0sin (๐‘˜๐‘ฅ0) sin (๐‘˜ (๐‘ฅ โˆ’ ๐ฟ)) ๐‘ฅ < ๐‘ฅ0 < ๐ฟ

The following approach seems faster.

second solution

Instead of starting from

๐บ (๐‘ฅ, ๐‘ฅ0) =

โŽงโŽชโŽชโŽจโŽชโŽชโŽฉ๐ด1๐‘ฆ1 (๐‘ฅ) + ๐ด2๐‘ฆ2 (๐‘ฅ) 0 < ๐‘ฅ < ๐‘ฅ0๐ต1๐‘ฆ1 (๐‘ฅ) + ๐ต2๐‘ฆ2 (๐‘ฅ) ๐‘ฅ < ๐‘ฅ0 < ๐‘

We first find the eigenfunction ฮฆ๐‘› (๐‘ฅ) that solves ๐‘ฆโ€ฒโ€ฒ + ๐‘˜2๐‘ฆ = 0 which satisfies the boundaryconditions ๐‘ฆ (0) = 0, ๐‘ฆ (๐ฟ) = 0. Then write

๐บ (๐‘ฅ, ๐‘ฅ0) =

โŽงโŽชโŽชโŽจโŽชโŽชโŽฉ

๐ดฮฆ๐‘› (๐‘ฅ) 0 < ๐‘ฅ < ๐‘ฅ0๐ตฮฆ๐‘› (๐‘ฅ โˆ’ ๐ฟ) ๐‘ฅ < ๐‘ฅ0 < ๐‘

So now we have only 2 unknowns to find, ๐ด,๐ต using the continutity and jump conditionson ๐บ. Let see how this works on this same problem. The solution to ๐‘ฆโ€ฒโ€ฒ + ๐‘˜2๐‘ฆ = 0 is๐‘ฆ (๐‘ฅ) = ๐ด cos ๐‘˜๐‘ฅ + ๐ต sin ๐‘˜๐‘ฅ. At ๐‘ฆ (0) = 0 implies ๐ด = 0, so the solution becomes ๐‘ฆ (๐‘ฅ) = ๐ต sin ๐‘˜๐‘ฅand at ๐‘ฅ (๐ฟ) = 0 this gives 0 = ๐ต sin (๐‘˜๐ฟ), which implies ๐‘˜๐ฟ = ๐‘›๐œ‹ or ๐‘˜ = ๐‘›๐œ‹

๐ฟ . Hence the solution

is ฮฆ๐‘› (๐‘ฅ) = sin ๏ฟฝ๐‘›๐œ‹๐ฟ ๐‘ฅ๏ฟฝ. Therefore we set up the Green function as

๐บ (๐‘ฅ, ๐‘ฅ0) =

โŽงโŽชโŽชโŽจโŽชโŽชโŽฉ

๐ด sin ๏ฟฝ๐‘›๐œ‹๐ฟ ๐‘ฅ๏ฟฝ 0 < ๐‘ฅ < ๐‘ฅ0๐ต sin ๏ฟฝ๐‘›๐œ‹๐ฟ (๐‘ฅ โˆ’ ๐ฟ)๏ฟฝ ๐‘ฅ < ๐‘ฅ0 < ๐‘

Or by letting ๐‘˜๐‘› =๐‘›๐œ‹๐ฟ

๐บ (๐‘ฅ, ๐‘ฅ0) =

โŽงโŽชโŽชโŽจโŽชโŽชโŽฉ

๐ด sin (๐‘˜๐‘›๐‘ฅ) 0 < ๐‘ฅ < ๐‘ฅ0๐ต sin (๐‘˜๐‘› (๐‘ฅ โˆ’ ๐ฟ)) ๐‘ฅ < ๐‘ฅ0 < ๐ฟ

(1)

Now continutity says

๐ด sin (๐‘˜๐‘›๐‘ฅ0) = ๐ต sin (๐‘˜๐‘› (๐‘ฅ0 โˆ’ ๐ฟ)) (2)

23

Taking derivative of (1) at ๐‘ฅ = ๐‘ฅ0 gives

๐บโ€ฒ (๐‘ฅ, ๐‘ฅ0) =

โŽงโŽชโŽชโŽจโŽชโŽชโŽฉ

๐ด๐‘˜๐‘› cos (๐‘˜๐‘›๐‘ฅ0) 0 < ๐‘ฅ < ๐‘ฅ0๐ต๐‘˜๐‘› cos (๐‘˜๐‘› (๐‘ฅ0 โˆ’ ๐ฟ)) ๐‘ฅ < ๐‘ฅ0 < ๐ฟ

Then jump discontinutiy gives

๐ต๐‘˜๐‘› cos (๐‘˜๐‘› (๐‘ฅ0 โˆ’ ๐ฟ)) โˆ’ ๐ด๐‘˜๐‘› cos (๐‘˜๐‘›๐‘ฅ0) = 1 (3)

Solving (2,3) for ๐ด,๐ต gives, as we did earlier

๐ด =sin (๐‘˜ (๐‘ฅ0 โˆ’ ๐ฟ))๐‘˜ sin (๐‘˜๐ฟ)

๐ต =sin (๐‘˜๐‘ฅ0)๐‘˜ sin (๐‘˜๐ฟ)

Using these in (1) gives

๐บ (๐‘ฅ, ๐‘ฅ0) =1

๐‘˜ sin (๐‘˜๐ฟ)

โŽงโŽชโŽชโŽจโŽชโŽชโŽฉ

sin (๐‘˜ (๐‘ฅ0 โˆ’ ๐ฟ)) sin (๐‘˜๐‘›๐‘ฅ) 0 < ๐‘ฅ < ๐‘ฅ0sin (๐‘˜๐‘ฅ0) sin (๐‘˜๐‘› (๐‘ฅ โˆ’ ๐ฟ)) ๐‘ฅ < ๐‘ฅ0 < ๐ฟ๐‘

Which is the same result obtained earlier.

2.2 exam 1

2.2.1 exam 1 prep sheet

Preparation sheet for the test: November 7 in class

The test will cover notes and examples on infinite series, complex analysis, andevaluation of integrals (not including the saddle point integration) and problemsets 1-4.

1. Things to know without thought:

(a) Sum of a geometric series.(b) Definitions of convergence and absolute convergences of infinite series.(c) Ratio test for convergence.(d) Series expansions for ex, sin x, cos x, and ln(1 + x) (for |x| < 1).(e) Cauchy-Riemann conditions and definition of analytic functions.(f) The meaning of log z, Logz, and zc

(g) Cauchy-Goursat Theorem and Cauchy Integral formula.(h) The form of Taylor and Laurent series.(i) The meaning of analytic continuation.(j) The residue theorem.(k) The definition of Cauchy principle value.(l) The definition of an asymptotic series.

24

2.2.2 exam 1 writeup

Problem 1

Using a well known sum, find a closed for expression for the following series

๐‘“ (๐‘ง) = 1 + 2๐‘ง + 3๐‘ง2 + 4๐‘ง3 + 5๐‘ง4 +โ‹ฏ

Using the ratio test, find for what values of ๐‘ง this series converges.

Solution

Method 1

Assume that the closed form is

(1 โˆ’ ๐‘ง)๐‘Ž = 1 + 2๐‘ง + 3๐‘ง2 + 4๐‘ง3 + 5๐‘ง4 +โ‹ฏ

For some unknown ๐‘Ž. Now ๐‘Ž will be solved for. Using Binomial series definition (1 โˆ’ ๐‘ง)๐‘Ž =1 โˆ’ ๐‘Ž๐‘ง + (๐‘Ž)(๐‘Žโˆ’1)

2! ๐‘ง2 โˆ’ ๐‘Ž(๐‘Žโˆ’1)(๐‘Žโˆ’2)3! ๐‘ง3 +โ‹ฏ. in the LHS above gives

1 โˆ’ ๐‘Ž๐‘ง +๐‘Ž (๐‘Ž โˆ’ 1)2!

๐‘ง2 โˆ’๐‘Ž (๐‘Ž โˆ’ 1) (๐‘Ž โˆ’ 2)

3!๐‘ง3 +โ‹ฏ = 1 + 2๐‘ง + 3๐‘ง2 + 4๐‘ง3 + 5๐‘ง4 +โ‹ฏ

By comparing coe๏ฟฝcients of ๐‘ง in the left side and on the right side shows that ๐‘Ž = โˆ’2 fromthe coe๏ฟฝcient of ๐‘ง term. Verifying this on the coe๏ฟฝcient of ๐‘ง2 shows it is correct since itgives (โˆ’2)(โˆ’3)

2 = 3. Therefore

๐‘Ž = โˆ’2

The closed form is therefore1

(1 โˆ’ ๐‘ง)2= 1 + 2๐‘ง + 3๐‘ง2 + 4๐‘ง3 + 5๐‘ง4 +โ‹ฏ

Method 2

Starting with Binomial series expansion given by1

1 โˆ’ ๐‘ง= 1 + ๐‘ง + ๐‘ง2 + ๐‘ง3 + ๐‘ง4 +โ‹ฏ

Taking derivative w.r.t. ๐‘ง on both sides of the above results in

๐‘‘๐‘‘๐‘ง ๏ฟฝ

11 โˆ’ ๐‘ง๏ฟฝ

=๐‘‘๐‘‘๐‘ง๏ฟฝ1 + ๐‘ง + ๐‘ง2 + ๐‘ง3 + ๐‘ง4 +โ‹ฏ๏ฟฝ

โˆ’ (1 โˆ’ ๐‘ง)โˆ’2 (โˆ’1) = 0 + 1 + 2๐‘ง + 3๐‘ง2 + 4๐‘ง3 +โ‹ฏ1

(1 โˆ’ ๐‘ง)2= 1 + 2๐‘ง + 3๐‘ง2 + 4๐‘ง3 +โ‹ฏ

Therefore the closed form expression is1

(1 โˆ’ ๐‘ง)2= 1 + 2๐‘ง + 3๐‘ง2 + 4๐‘ง3 +โ‹ฏ

Which is the same as method 1.

The series general term of the series is

1 + 2๐‘ง + 3๐‘ง2 + 4๐‘ง3 +โ‹ฏ =โˆž๏ฟฝ๐‘›=0

(๐‘› + 1) ๐‘ง๐‘›

Applying the ratio test

๐ฟ = lim๐‘›โ†’โˆž

๏ฟฝ๐‘Ž๐‘›+1๐‘Ž๐‘›

๏ฟฝ

= lim๐‘›โ†’โˆž

๏ฟฝ(๐‘› + 2) ๐‘ง๐‘›+1

(๐‘› + 1) ๐‘ง๐‘› ๏ฟฝ

= lim๐‘›โ†’โˆž

๏ฟฝ(๐‘› + 2) ๐‘ง๐‘› + 1

๏ฟฝ

= ๐‘ง lim๐‘›โ†’โˆž

๏ฟฝ๐‘› + 2๐‘› + 1

๏ฟฝ

= ๐‘ง lim๐‘›โ†’โˆž

๏ฟฝ๏ฟฝ1 + 2

๐‘›

1 + 1๐‘›

๏ฟฝ๏ฟฝ

25

But lim๐‘›โ†’โˆž ๏ฟฝ1+ 2

๐‘›

1+ 1๐‘›

๏ฟฝ = 1 and the above limit becomes

๐ฟ = ๐‘ง

By the ratio test, the series converges when |๐ฟ| < 1. Therefore 1+2๐‘ง+3๐‘ง2+4๐‘ง3+โ‹ฏ convergesabsolutely when |๐‘ง| < 1. An absolutely convergent series is also a convergent series. Hencethe series converges for |๐‘ง| < 1.

Problem 2

Find the Laurent series for the function

๐‘“ (๐‘ง) =1

๏ฟฝ๐‘ง2 + 4๏ฟฝ3

About the isolated singular pole ๐‘ง = 2๐‘–. What is the order of this pole? What is the residueat this pole?

Solution

The poles are at ๐‘ง2 = 4 or ๐‘ง = ยฑ2๐‘–. The expansion of ๐‘“ (๐‘ง) is around the isolated pole at๐‘ง = 2๐‘–. This pole has order 3. The region where this expansion is valid is inside a diskcentered at 2๐‘– (but not including the point ๐‘ง = 2๐‘– itself) and up to the nearest pole whichis located at โˆ’2๐‘–. Therefore the disk will have radius 4.

2i

โˆ’2i

R = 4

Region where Laurent seriesexpansion around z = 2i is valid

<z

=z

Let

๐‘ข = ๐‘ง โˆ’ 2๐‘–๐‘ง = ๐‘ข + 2๐‘–

26

Substituting this expression for ๐‘ง back in ๐‘“ (๐‘ง) gives

๐‘“ (๐‘ง) =1

๏ฟฝ(๐‘ข + 2๐‘–)2 + 4๏ฟฝ3

=1

๏ฟฝ๐‘ข2 โˆ’ 4 + 4๐‘ข๐‘– + 4๏ฟฝ3

=1

๏ฟฝ๐‘ข2 + 4๐‘ข๐‘–๏ฟฝ3

=1

(๐‘ข (๐‘ข + 4๐‘–))3

=1๐‘ข3

1(๐‘ข + 4๐‘–)3

=1๐‘ข3

1

๏ฟฝ4๐‘– ๏ฟฝ ๐‘ข4๐‘– + 1๏ฟฝ๏ฟฝ3

=1๐‘ข3

1

(4๐‘–)3 ๏ฟฝ ๐‘ข4๐‘– + 1๏ฟฝ3

=1

โˆ’๐‘–64๐‘ข31

๏ฟฝ ๐‘ข4๐‘– + 1๏ฟฝ

3

= ๏ฟฝ๐‘–

64๐‘ข3 ๏ฟฝ1

๏ฟฝ ๐‘ข4๐‘– + 1๏ฟฝ

3 (1)

Expanding the term 1

๏ฟฝ1+ ๐‘ข4๐‘– ๏ฟฝ

3 using Binomial series, which is valid for ๏ฟฝ ๐‘ข4๐‘– ๏ฟฝ < 1 or |๐‘ข| < 4 gives

1

๏ฟฝ1 + ๐‘ข4๐‘–๏ฟฝ3 = 1 + (โˆ’3)

๐‘ข4๐‘–+(โˆ’3) (โˆ’4)

2!๏ฟฝ๐‘ข4๐‘–๏ฟฝ2+(โˆ’3) (โˆ’4) (โˆ’5)

3!๏ฟฝ๐‘ข4๐‘–๏ฟฝ3+(โˆ’3) (โˆ’4) (โˆ’5) (โˆ’6)

4!๏ฟฝ๐‘ข4๐‘–๏ฟฝ4+โ‹ฏ

= 1 โˆ’ 3๐‘ข4๐‘–+3 โ‹… 42!

๐‘ข2

16๐‘–2โˆ’3 โ‹… 4 โ‹… 53!

๐‘ข3

64๐‘–3+3 โ‹… 4 โ‹… 5 โ‹… 6

4!๐‘ข4

256๐‘–4+โ‹ฏ

= 1 + 3๐‘–๐‘ข4โˆ’3 โ‹… 42!

๐‘ข2

16โˆ’3 โ‹… 4 โ‹… 53!

๐‘ข3

64 (โˆ’๐‘–)+3 โ‹… 4 โ‹… 5 โ‹… 6

4!๐‘ข4

256+โ‹ฏ

= 1 + 3๐‘–๐‘ข4โˆ’3 โ‹… 42!

๐‘ข2

16โˆ’ ๐‘–3 โ‹… 4 โ‹… 53!

๐‘ข3

64+3 โ‹… 4 โ‹… 5 โ‹… 6

4!๐‘ข4

256+โ‹ฏ (2)

Substituting (2) into (1) and simplifying gives

๐‘“ (๐‘ง) = ๏ฟฝ๐‘–

64๐‘ข3 ๏ฟฝ ๏ฟฝ1 + 3๐‘–

๐‘ข4โˆ’3 โ‹… 42!

๐‘ข2

16โˆ’ ๐‘–3 โ‹… 4 โ‹… 53!

๐‘ข3

64+3 โ‹… 4 โ‹… 5 โ‹… 6

4!๐‘ข4

256+โ‹ฏ๏ฟฝ

=๐‘–

64๐‘ข3+

๐‘–64๐‘ข3

๏ฟฝ3๐‘–๐‘ข4๏ฟฝ โˆ’

๐‘–64๐‘ข3 ๏ฟฝ

3 โ‹… 42!

๐‘ข2

16๏ฟฝโˆ’

๐‘–64๐‘ข3 ๏ฟฝ

๐‘–3 โ‹… 4 โ‹… 53!

๐‘ข3

64๏ฟฝ+

๐‘–64๐‘ข3 ๏ฟฝ

3 โ‹… 4 โ‹… 5 โ‹… 64!

๐‘ข4

256๏ฟฝ+โ‹ฏ

=๐‘–

64๐‘ข3โˆ’

164๐‘ข2

34โˆ’

๐‘–64๐‘ข ๏ฟฝ

3 โ‹… 42!

116๏ฟฝ

+164 ๏ฟฝ

3 โ‹… 4 โ‹… 53!

164๏ฟฝ

+๐‘–64 ๏ฟฝ

3 โ‹… 4 โ‹… 5 โ‹… 64!

๐‘ข256๏ฟฝ

+โ‹ฏ

=๐‘–

64๐‘ข3โˆ’

3256๐‘ข2

โˆ’ ๐‘–3512

1๐‘ข+

52048

+ ๐‘–15

16 384๐‘ข +โ‹ฏ

Replacing ๐‘ข back by ๐‘ง โˆ’ 2๐‘– in the above results in

๐‘“ (๐‘ง) =๐‘–64

1(๐‘ง โˆ’ 2๐‘–)3

โˆ’3256

1(๐‘ง โˆ’ 2๐‘–)2

โˆ’3๐‘–512

1(๐‘ง โˆ’ 2๐‘–)

+5

2048+

15๐‘–16 384

(๐‘ง โˆ’ 2๐‘–) +โ‹ฏ (3)

This expansion is valid for |๐‘ง โˆ’ 2๐‘–| < 4. The above shows that the residue is

โˆ’ 3๐‘–512

Which is the coe๏ฟฝcient of the 1(๐‘งโˆ’2๐‘–) term in (3).

27

Problem 3

Use residues to evaluate the following integral

๐ผ = ๏ฟฝโˆž

0

๐‘‘๐‘ฅ๐‘ฅ4 + 6๐‘ฅ2 + 9

Solution

The integrand is an even function. Therefore the integral โˆซโˆž

โˆ’โˆž๐‘‘๐‘ฅ

๐‘ฅ4+6๐‘ฅ2+9is evaluated instead

and then the required integral ๐ผ will be half the value obtained. The poles of 1๐‘ฅ4+6๐‘ฅ2+9

are

the zeros of the denominator. Factoring the denominator as ๏ฟฝ๐‘ฅ2 + 3๏ฟฝ ๏ฟฝ๐‘ฅ2 + 3๏ฟฝ = 0, shows theroots are ๐‘ฅ = ยฑ๐‘–โˆš3 from the first factor and ๐‘ฅ = ยฑ๐‘–โˆš3 from the second factor.

Since the upper half plane will be used, the pole located there is +๐‘–โˆš3 and it is of order two.

Now that pole locations are known, the following contour is used to evaluate โˆซโˆž

โˆ’โˆž๐‘‘๐‘ฅ

๐‘ฅ4+6๐‘ฅ2+9as shown in the plot below

โˆ’R +R

R

CR

<z

=z

iโˆš3

pole of order 2

โˆฎ๐ถ

๐‘“ (๐‘ง) ๐‘‘๐‘ง = lim๐‘…โ†’โˆž

๏ฟฝ๐ถ๐‘“ (๐‘ง) ๐‘‘๐‘ง + lim

๐‘…โ†’โˆž๏ฟฝ

+๐‘…

โˆ’๐‘…๐‘“ (๐‘ฅ) ๐‘‘๐‘ฅ

= lim๐‘…โ†’โˆž

๏ฟฝ๐ถ

๐‘‘๐‘ง๐‘ง4 + 6๐‘ง2 + 9

+ lim๐‘…โ†’โˆž

๏ฟฝ+๐‘…

โˆ’๐‘…

๐‘‘๐‘ฅ๐‘ฅ4 + 6๐‘ฅ2 + 9

๐‘‘๐‘ฅ (2)

Where the integral โˆซ+๐‘…

โˆ’๐‘…is Cauchy principal integral. Since the contour ๐ถ is closed and

because ๐‘“ (๐‘ง) is analytic on and inside ๐ถ except for the isolated singularity inside at ๐‘ง = ๐‘–โˆš3,then by Cauchy integral formula โˆฎ

๐ถ

๐‘“ (๐‘ง) ๐‘‘๐‘ง = 2๐œ‹๐‘–โˆ‘Residue. Where the sum of residues is

over all poles inside ๐ถ. Therefore (2) can becomes

๏ฟฝ+โˆž

โˆ’โˆž

๐‘‘๐‘ฅ๐‘ฅ4 + 6๐‘ฅ2 + 9

๐‘‘๐‘ฅ = 2๐œ‹๐‘–๏ฟฝResidueโˆ’ lim๐‘…โ†’โˆž

๏ฟฝ๐ถ๐‘“ (๐‘ง) ๐‘‘๐‘ง (3)

But

๏ฟฝ๏ฟฝ๐ถ๐‘“ (๐‘ง) ๐‘‘๐‘ง๏ฟฝ

maxโ‰ค ๐‘€๐ฟ

= ๏ฟฝ๐‘“ (๐‘ง)๏ฟฝmax

๐œ‹๐‘… (4)

Using

๏ฟฝ๐‘“ (๐‘ง)๏ฟฝmax

โ‰ค1

๏ฟฝ๐‘ง2 + 3๏ฟฝmin

๏ฟฝ๐‘ง2 + 3๏ฟฝmin

By inverse triangle inequality ๏ฟฝ๐‘ง2 + 3๏ฟฝ โ‰ฅ |๐‘ง|2 โˆ’ 3. But |๐‘ง| = ๐‘… on ๐ถ, therefore ๏ฟฝ๐‘ง2 + 3๏ฟฝ โ‰ฅ ๐‘…2 โˆ’ 3and the above can now be written as

๏ฟฝ๐‘“ (๐‘ง)๏ฟฝmax

โ‰ค1

๏ฟฝ๐‘…2 โˆ’ 3๏ฟฝ ๏ฟฝ๐‘…2 โˆ’ 3๏ฟฝ

28

Using the above in (4) gives

๏ฟฝ๏ฟฝ๐ถ๐‘“ (๐‘ง) ๐‘‘๐‘ง๏ฟฝ

maxโ‰ค

๐œ‹๐‘…๏ฟฝ๐‘…2 โˆ’ 3๏ฟฝ ๏ฟฝ๐‘…2 โˆ’ 3๏ฟฝ

=๐œ‹๐‘…

๐‘…4 โˆ’ 6๐‘…2 + 9

=๐œ‹๐‘…

๐‘…2 โˆ’ 6 + 9๐‘…2

In the limit as ๐‘… โ†’ โˆž then ๏ฟฝโˆซ๐ถ๐‘“ (๐‘ง) ๐‘‘๐‘ง๏ฟฝ

maxโ†’ 0. Using this result in (3) it simplifies to

๏ฟฝ+โˆž

โˆ’โˆž

๐‘‘๐‘ฅ๐‘ฅ4 + 6๐‘ฅ2 + 9

๐‘‘๐‘ฅ = 2๐œ‹๐‘–๏ฟฝResidue (5)

What is left now is to determine the residue at pole ๐‘ง0 = ๐‘–โˆš3 which is of order 2. This isdone using

Residue (๐‘ง0) = lim๐‘งโ†’๐‘ง0

๐‘‘๐‘‘๐‘ง๏ฟฝ(๐‘ง โˆ’ ๐‘ง0)

2 ๐‘“ (๐‘ง)๏ฟฝ

But ๐‘ง0 = ๐‘–โˆš3 and the above becomes

Residue ๏ฟฝ๐‘–โˆš3๏ฟฝ = lim๐‘งโ†’๐‘–โˆš3

๐‘‘๐‘‘๐‘ง

โŽ›โŽœโŽœโŽœโŽœโŽœโŽœโŽ๏ฟฝ๐‘ง โˆ’ ๐‘–โˆš3๏ฟฝ

2 1

๏ฟฝ๐‘ง โˆ’ ๐‘–โˆš3๏ฟฝ2๏ฟฝ๐‘ง + ๐‘–โˆš3๏ฟฝ

2

โŽžโŽŸโŽŸโŽŸโŽŸโŽŸโŽŸโŽ 

= lim๐‘งโ†’๐‘–โˆš3

๐‘‘๐‘‘๐‘ง

1

๏ฟฝ๐‘ง + ๐‘–โˆš3๏ฟฝ2

= lim๐‘งโ†’๐‘–โˆš3

โˆ’2

๏ฟฝ๐‘ง + ๐‘–โˆš3๏ฟฝ3

=โˆ’2

๏ฟฝ๐‘–โˆš3 + ๐‘–โˆš3๏ฟฝ3

=โˆ’2

๏ฟฝ2๐‘–โˆš3๏ฟฝ3

=โˆ’2

โˆ’ (8) (3) ๐‘–โˆš3

=1

12๐‘–โˆš3Using the above value of the residue in (5) gives

๏ฟฝ+โˆž

โˆ’โˆž

๐‘‘๐‘ฅ๐‘ฅ4 + 6๐‘ฅ2 + 9

๐‘‘๐‘ฅ = 2๐œ‹๐‘– ๏ฟฝ1

12๐‘–โˆš3๏ฟฝ

=๐œ‹6โˆš3

Therefore the integral โˆซโˆž

0๐‘‘๐‘ฅ

๐‘ฅ4+6๐‘ฅ3+9is half of the above result which is

๏ฟฝโˆž

0

๐‘‘๐‘ฅ๐‘ฅ4 + 6๐‘ฅ2 + 9

=๐œ‹

12โˆš3

Problem 4

Find two approximations for the integral ๐‘ฅ > 0

๐ผ (๐‘ฅ) =12๐œ‹ ๏ฟฝ

๐œ‹2

โˆ’๐œ‹2

๐‘’๐‘ฅ cos2 ๐œƒ๐‘‘๐œƒ

One for small ๐‘ฅ (keeping up to linear order in ๐‘ฅ) and one for large values of ๐‘ฅ (keepingonly the leading order term).

Solution

The integrand has the form ๐‘’๐‘ง. This has a known Taylor series expansion around zero

29

given by

๐‘’๐‘ง = 1 + ๐‘ง +๐‘ง2

2!+โ‹ฏ

Replacing ๐‘ง by ๐‘ฅ cos2 ๐œƒ in the above gives

๐‘’๐‘ฅ cos2 ๐œƒ = 1 + ๐‘ฅ cos2 ๐œƒ +๏ฟฝ๐‘ฅ cos2 ๐œƒ๏ฟฝ

2

2+โ‹ฏ

The problem is asking to keep linear terms in ๐‘ฅ. Therefore

๐‘’๐‘ฅ cos2 ๐œƒ โ‰ˆ 1 + ๐‘ฅ cos2 ๐œƒReplacing the integrand in the original integral by the above approximation gives

๐ผ (๐‘ฅ) โ‰ˆ12๐œ‹ ๏ฟฝ

๐œ‹2

โˆ’๐œ‹2

๏ฟฝ1 + ๐‘ฅ cos2 ๐œƒ๏ฟฝ ๐‘‘๐œƒ

โ‰ˆ12๐œ‹

โŽ›โŽœโŽœโŽœโŽœโŽ๏ฟฝ

๐œ‹2

โˆ’๐œ‹2

๐‘‘๐œƒ + ๐‘ฅ๏ฟฝ๐œ‹2

โˆ’๐œ‹2

cos2 ๐œƒ๐‘‘๐œƒโŽžโŽŸโŽŸโŽŸโŽŸโŽ 

โ‰ˆ12๐œ‹

โŽ›โŽœโŽœโŽœโŽœโŽ๏ฟฝ

๐œ‹2

โˆ’๐œ‹2

๐‘‘๐œƒ + ๐‘ฅ๏ฟฝ๐œ‹2

โˆ’๐œ‹2

12+12

cos 2๐œƒ๐‘‘๐œƒโŽžโŽŸโŽŸโŽŸโŽŸโŽ 

โ‰ˆ12๐œ‹

โŽ›โŽœโŽœโŽœโŽœโŽœโŽœโŽ๐œ‹ + ๐‘ฅ ๏ฟฝ

12๐œƒ +

12

sin 2๐œƒ2 ๏ฟฝ

๐œ‹2

โˆ’๐œ‹2

โŽžโŽŸโŽŸโŽŸโŽŸโŽŸโŽŸโŽ 

โ‰ˆ12๐œ‹ ๏ฟฝ

๐œ‹ +๐‘ฅ4(2๐œƒ + sin 2๐œƒ)

๐œ‹2โˆ’๐œ‹2๏ฟฝ

โ‰ˆ12๐œ‹

๏ฟฝ๐œ‹ +๐‘ฅ4(2๐œ‹ + 0)๏ฟฝ

โ‰ˆ12๐œ‹

๏ฟฝ๐œ‹ +๐‘ฅ2๐œ‹๏ฟฝ

โ‰ˆ12๏ฟฝ1 +

๐‘ฅ2๏ฟฝ

For large value of ๐‘ฅ, The integrand is written as ๐‘’๐‘“(๐œƒ) where ๐‘“ (๐œƒ) = ๐‘ฅ cos2 ๐œƒ. The value of ๐œƒwhere ๐‘“ (๐œƒ) is maximum is first found. Then solving for ๐œƒ in

๐‘‘๐‘‘๐œƒ๐‘ฅ cos2 ๐œƒ = 0

โˆ’2๐‘ฅ cos๐œƒ sin๐œƒ = 0Hence solving for ๐œƒ in

cos๐œƒ sin๐œƒ = 0There are two solutions to this. Either ๐œƒ = ๐œ‹

2 or ๐œƒ = 0. To find which is the correct choice,

the sign of ๐‘‘2

๐‘‘๐œƒ2๐‘“ (๐œƒ) is checked for each choice.

๐‘‘2

๐‘‘๐œƒ2๐‘ฅ cos2 ๐œƒ = ๐‘‘

๐‘‘๐œƒ(โˆ’2๐‘ฅ cos๐œƒ sin๐œƒ)

= โˆ’2๐‘ฅ๐‘‘๐‘‘๐œƒ

(cos๐œƒ sin๐œƒ)

= โˆ’2๐‘ฅ (โˆ’ sin๐œƒ sin๐œƒ + cos๐œƒ cos๐œƒ)= โˆ’2๐‘ฅ ๏ฟฝโˆ’ sin2 ๐œƒ + cos2 ๐œƒ๏ฟฝ (1)

Substituting ๐œƒ = ๐œ‹2 in (1) and using cos ๏ฟฝ๐œ‹2 ๏ฟฝ = 0 and sin ๏ฟฝ๐œ‹2 ๏ฟฝ = 1 gives

๐‘‘2

๐‘‘๐œƒ2๐‘ฅ cos2 ๐œƒ๏ฟฝ

๐œƒ=๐œ‹2

= โˆ’2๐‘ฅ (โˆ’1)

= 2๐‘ฅ

Since the problem says that ๐‘ฅ > 0 then ๐‘‘2

๐‘‘๐œƒ2๐‘ฅ cos2 ๐œƒ๏ฟฝ๐œƒ=๐œ‹

2

> 0. Therefore this is a minimum.

30

Using the second choice ๐œƒ = 0, then (1) becomes (after using cos (0) = 1 and sin (0) = 0)๐‘‘2

๐‘‘๐œƒ2๐‘ฅ cos2 ๐œƒ๏ฟฝ

๐œƒ=0

= โˆ’2๐‘ฅ

And because ๐‘ฅ > 0 then ๐‘‘2

๐‘‘๐œƒ2๐‘ฅ cos2 ๐œƒ๏ฟฝ๐œƒ=0

< 0. Therefore the integrand is maximum at

๐œƒ๐‘๐‘’๐‘Ž๐‘˜ = 0

Now that peak ๐œƒ is found, then ๐‘“ (๐œƒ) is expanded in Taylor series around ๐œƒ๐‘๐‘’๐‘Ž๐‘˜ = 0. Since๐‘“ (๐œƒ) = ๐‘ฅ cos2 ๐œƒ, then

๐‘“ ๏ฟฝ๐œƒ๐‘๐‘’๐‘Ž๐‘˜๏ฟฝ = ๐‘ฅ

And ๐‘“โ€ฒ (๐œƒ) = โˆ’2๐‘ฅ cos๐œƒ sin๐œƒ. At ๐œƒ๐‘๐‘’๐‘Ž๐‘˜ this becomes ๐‘“โ€ฒ ๏ฟฝ๐œƒ๐‘๐‘’๐‘Ž๐‘˜๏ฟฝ = 0. The next term is thequadratic one, given by

๐‘“โ€ฒโ€ฒ (๐œƒ) = โˆ’2๐‘ฅ๐‘‘๐‘‘๐œƒ

(cos๐œƒ sin๐œƒ)

= โˆ’2๐‘ฅ ๏ฟฝโˆ’ sin2 ๐œƒ + cos2 ๐œƒ๏ฟฝEvaluating the above at ๐œƒ๐‘๐‘’๐‘Ž๐‘˜ = 0 gives

๐‘“โ€ฒโ€ฒ ๏ฟฝ๐œƒ๐‘๐‘’๐‘Ž๐‘˜๏ฟฝ = โˆ’2๐‘ฅ

The problem says to keep leading term, so no need for more terms. Therefore the Taylorseries expansion of ๐‘“ (๐œƒ) = ๐‘ฅ cos2 ๐œƒ around ๐œƒ = ๐œƒ๐‘๐‘’๐‘Ž๐‘˜ is

๐‘ฅ cos2 ๐œƒ โ‰ˆ ๐‘“ ๏ฟฝ๐œƒ๐‘๐‘’๐‘Ž๐‘˜๏ฟฝ + ๐‘“โ€ฒ ๏ฟฝ๐œƒ๐‘๐‘’๐‘Ž๐‘˜๏ฟฝ ๐œƒ +12!๐‘“โ€ฒโ€ฒ ๏ฟฝ๐œƒ๐‘๐‘’๐‘Ž๐‘˜๏ฟฝ ๐œƒ2

= ๐‘ฅ + 0 โˆ’2๐‘ฅ2!๐œƒ2

= ๐‘ฅ โˆ’ ๐‘ฅ๐œƒ2

= ๐‘ฅ ๏ฟฝ1 โˆ’ ๐œƒ2๏ฟฝ

The integral now becomes

๐ผ (๐‘ฅ) =12๐œ‹ ๏ฟฝ

๐œ‹2

โˆ’๐œ‹2

๐‘’๐‘ฅ๏ฟฝ1โˆ’๐œƒ2๏ฟฝ๐‘‘๐œƒ

โ‰ˆ12๐œ‹ ๏ฟฝ

๐œ‹2

โˆ’๐œ‹2

๐‘’๐‘ฅ๐‘’โˆ’๐‘ฅ๐œƒ2๐‘‘๐œƒ

=12๐œ‹

โŽ›โŽœโŽœโŽœโŽœโŽ๐‘’๐‘ฅ๏ฟฝ

๐œ‹2

โˆ’๐œ‹2

๐‘’โˆ’๐‘ฅ๐œƒ2๐‘‘๐œƒโŽžโŽŸโŽŸโŽŸโŽŸโŽ 

Comparing โˆซ๐œ‹2

โˆ’๐œ‹2๐‘’โˆ’๐‘ฅ๐œƒ2๐‘‘๐œƒ to the Gaussian integral โˆซ

โˆž

โˆ’โˆž๐‘’โˆ’๐‘Ž๐œƒ2๐‘‘๐œƒ = ๏ฟฝ

๐œ‹๐‘ฅ , then the above can be

approximated as

๐ผ (๐‘ฅ) =๐‘’๐‘ฅ

2๐œ‹๏ฟฝ๐œ‹๐‘ฅ

Summary of result

Small ๐‘ฅ approximation 12๏ฟฝ1 + ๐‘ฅ

2๏ฟฝ

Large ๐‘ฅ approximation ๐‘’๐‘ฅ

2๐œ‹๏ฟฝ๐œ‹๐‘ฅ

Note that using the computer, the exact solution is

12๐œ‹ ๏ฟฝ

๐œ‹2

โˆ’๐œ‹2

๐‘’๐‘ฅ cos2 ๐œƒ๐‘‘๐œƒ =12๐‘’๐‘ฅ2 BesselI ๏ฟฝ0, ๐‘ฅ

2๏ฟฝ

Problem 5

Use the Cauchy-Riemann equations to determine where the function

๐‘“ (๐‘ง) = ๐‘ง + ๐‘ง2

31

Is analytic. Evaluate โˆฎ๐ถ

๐‘“ (๐‘ง) ๐‘‘๐‘ง where contour ๐ถ is on the unit circle |๐‘ง| = 1 in a counter-

clockwise sense.

Solution

Using ๐‘ง = ๐‘ฅ + ๐‘–๐‘ฆ, the function ๐‘“ (๐‘ง) becomes

๐‘“ (๐‘ง) = ๐‘ฅ + ๐‘–๐‘ฆ + ๏ฟฝ๐‘ฅ + ๐‘–๐‘ฆ๏ฟฝ2

= ๐‘ฅ + ๐‘–๐‘ฆ + ๏ฟฝ๐‘ฅ2 โˆ’ ๐‘ฆ2 + 2๐‘–๐‘ฅ๐‘ฆ๏ฟฝ

= ๐‘ฅ + ๐‘–๐‘ฆ + ๏ฟฝ๐‘ฅ2 โˆ’ ๐‘ฆ2 โˆ’ 2๐‘–๐‘ฅ๐‘ฆ๏ฟฝ

= ๏ฟฝ๐‘ฅ + ๐‘ฅ2 โˆ’ ๐‘ฆ2๏ฟฝ + ๐‘– ๏ฟฝ๐‘ฆ โˆ’ 2๐‘ฅ๐‘ฆ๏ฟฝ

Writing ๐‘“ (๐‘ง) = ๐‘ข + ๐‘–๐‘ฃ, and comparing this to the above result shows that

๐‘ข = ๐‘ฅ + ๐‘ฅ2 โˆ’ ๐‘ฆ2

๐‘ฃ = ๐‘ฆ โˆ’ 2๐‘ฅ๐‘ฆ (1)

Cauchy-Riemann are given by

๐œ•๐‘ข๐œ•๐‘ฅ

=๐œ•๐‘ฃ๐œ•๐‘ฆ

โˆ’๐œ•๐‘ข๐œ•๐‘ฆ

=๐œ•๐‘ฃ๐œ•๐‘ฅ

Using result in (1), Cauchy-Riemann are checked to see if they are satisfied or not. Thefirst equation above results in

๐œ•๐‘ข๐œ•๐‘ฅ

= 1 + 2๐‘ฅ

๐œ•๐‘ฃ๐œ•๐‘ฆ

= 1 โˆ’ 2๐‘ฅ

Therefore ๐œ•๐‘ข๐œ•๐‘ฅ โ‰ 

๐œ•๐‘ฃ๐œ•๐‘ฆ . This shows that ๐‘“ (๐‘ง) is not analytic for all ๐‘ฅ, ๐‘ฆ.

Since ๐‘“ (๐‘ง) is not analytic, Cauchy integral formula can not be used. Instead this can beintegrated using parameterization. Let ๐‘ง = ๐‘’๐‘–๐œƒ (No need to use ๐‘Ÿ๐‘’๐‘–๐œƒ since ๐‘Ÿ = 1 in this casebecause it is the unit circle). The function ๐‘“ (๐‘ง) becomes

๐‘“ (๐‘ง) = ๐‘’๐‘–๐œƒ + ๏ฟฝ๐‘’๐‘–๐œƒ๏ฟฝ2

= ๐‘’๐‘–๐œƒ + ๐‘’2๐‘–๐œƒ

= ๐‘’๐‘–๐œƒ + ๐‘’โˆ’2๐‘–๐œƒ

And because ๐‘ง = ๐‘’๐‘–๐œƒ then ๐‘‘๐‘ง = ๐‘‘๐œƒ๐‘’๐‘–๐œƒ. The integral now becomes

โˆฎ๐ถ

๐‘“ (๐‘ง) ๐‘‘๐‘ง = ๏ฟฝ2๐œ‹

0๏ฟฝ๐‘’๐‘–๐œƒ + ๐‘’โˆ’2๐‘–๐œƒ๏ฟฝ ๐‘’๐‘–๐œƒ๐‘‘๐œƒ

= ๏ฟฝ2๐œ‹

0๏ฟฝ๐‘’2๐‘–๐œƒ + ๐‘’โˆ’๐‘–๐œƒ๏ฟฝ ๐‘‘๐œƒ

= ๏ฟฝ๐‘’2๐‘–๐œƒ

2๐‘– ๏ฟฝ2๐œ‹

0+ ๏ฟฝ

๐‘’โˆ’๐‘–๐œƒ

โˆ’๐‘– ๏ฟฝ2๐œ‹

0

=12๐‘–[cos 2๐œƒ + ๐‘– sin 2๐œƒ]2๐œ‹0 + ๐‘– [cos๐œƒ โˆ’ ๐‘– sin๐œƒ]2๐œ‹0

=12๐‘–[(cos 4๐œ‹ + ๐‘– sin 4๐œ‹) โˆ’ (cos 0 + ๐‘– sin 0)] + ๐‘– [(cos 2๐œ‹ โˆ’ ๐‘– sin 2๐œ‹) โˆ’ (cos 0 โˆ’ ๐‘– sin 0)]

=12๐‘–[1 โˆ’ 1] + ๐‘– [1 โˆ’ 1]

Hence

โˆฎ๐ถ

๐‘“ (๐‘ง) ๐‘‘๐‘ง = 0

32

2.3 ๏ฟฝnal exam

2.3.1 exam 1 prep sheet

Preparation sheet for the exam: Dec 19 from 2:45 to 5:15 in class

The exam will cover notes and examples on infinite series, complex analysis, eval-uation of integrals, integral transforms, ordinary differential equations, eigenvalueproblems, and partial differential equations. The exam will also cover problem sets1-7. You are allowed to have a 8.5 by 11 sheet of sheet of paper with equations on it.

1. Things to know without thought:

(a) Sum of a geometric series.(b) Definitions of convergence and absolute convergences of infinite series.(c) Ratio test for convergence.(d) Series expansions for ex, sin x, cos x, and ln(1 + x) (for |x| < 1).(e) Cauchy-Riemann conditions and definition of analytic functions.(f) The meaning of log z, Logz, and zc

(g) Cauchy-Goursat Theorem and Cauchy Integral formula.(h) The form of Taylor and Laurent series.(i) The meaning of analytic continuation.(j) The residue theorem.(k) The definition of Cauchy principle value.(l) The definition of an asymptotic series.(m) The definition of Fourier series and transform The definition of a separablefirst order ODE.(n) The criterium to check if an ODE is exact.(o) Solution of linear first order differential equation using an integrating factor.(q) Definition and solution of a homogeneous ODE.(q) Use of the method of undetermined coefficients to solve inhomogeneous linearODEs.(r) Ordinary and regular singular points of a linear differential equation (and theform of the series solutions for these).(s) Definition of a Hermitian differential operator.(t) Sturm Lioville differential equation and related orthogonality of eigenfunctions.(u) Use of the Wronskian to check if two solutions are independent.(v) The definition of generating functions and how to use these to find recurrencerelations.(w) The completeness relation.(x) The definition of the Green function in solving inhomogeneous problems.(y) The eigenfunction expansion of the Green function.(z) The form of the wave, Laplace, and diffusion equations in 1,2, and 3 dimen-sions.(aa) The solution of the 1D wave equation.(bb) The use of separation of variables. Specifically in 3D spherical coordinatesand in 2D polar coordinates.

33

2.3.2 my ๏ฟฝnal exam cheat sheet

Solve for ฯˆ(r, ฮธ, ฯ†, t) where โˆ‚ฯˆ2

โˆ‚t2= c2โˆ‡ฯˆ (Wave PDE 3D spherical coordinates)

Let ฯˆ = T (t)X(r, ฮธ, ฯ†). Firstseperation gives the following twoequations

T โ€ฒโ€ฒ(t) + c2k2T (t) = 0

โˆ‡X + k2X = 0

k2 is firstseparation constant

T (t) = cos(ฯ‰t) + sin(ฯ‰t)solution

Helmholtz eqution

Let X = R(r)ฮ˜(ฮธ)ฮฆ(ฯ†) and apply separation again.

ฮฆโ€ฒโ€ฒ +m2ฮฆ = 0

solve first, m2 is separation constant

ฮฆ (ฯ†) =

{eimฯ†

eโˆ’imฯ†

}

(1โˆ’ x2

)ฮ˜โ€ฒโ€ฒ โˆ’ 2xฮ˜โ€ฒ +

(l (l + 1)โˆ’ m2

1+x2

)ฮ˜ = 0

angular equation.Second sepration. Usel(l + 1) as separationconstant.

Associated legendre equationNote: l is integer and โˆ’l โ‰ค m โ‰ค l

ฮ˜ (x) =

{Pml (x)Qml (x)

}associated Legendrepolynomial of first kind

associated Legendrepolynomial of second kind.not used. blows up x = ยฑ1

x = cos ฮธ

r2Rโ€ฒโ€ฒ + 2rRโ€ฒ + (r2k2 โˆ’ l(l + 1))R = 0

Radial equation(Bessel like)

Can be converted to standard Bessel ODEby some transformation (not shown)

R(r) =

{jl(kr)yl(kr)

}shperical besselfunction first kind

shperical bessel functionsecond kind. Not used as itblows up at r = 0

Final solution is summation of fundamental solutionโˆ‘jl (kr)P

ml (cos(mฯ†) + sin(mฯ†)) (cosฯ‰t+ sinฯ‰t)

โ‡“

ฯ‰ = ck

Solve for ฯˆ(r, ฯ†, t) where โˆ‚ฯˆ2

โˆ‚t2= c2โˆ‡ฯˆ (Wave PDE in 2D disk, polar coordinates).

Membrane is fixed on edge of disk. Radius a.

Let ฯˆ = T (t)X(r, ฯ†). Firstseperation gives the following twoequations

T โ€ฒโ€ฒ(t) + c2k2T (t) = 0

โˆ‡X + k2X = 0

k2 is firstseparation constant

T (t) = cos(ฯ‰t) + sin(ฯ‰t)solution

Helmholtz eqution

ฯ‰ = ckmn

Let X = R(r)ฮฆ(ฯ†) and apply separation again.

ฮฆโ€ฒโ€ฒ +m2ฮฆ = 0

solve first, m2 is separation constant

ฮฆ (ฯ†) =

{eimฯ†

eโˆ’imฯ†

}solution

Due to periodicity, m must be integer.

r2Rโ€ฒโ€ฒ + rRโ€ฒ + (r2k2 โˆ’m2)R = 0

Radial equation

Bessel ODE

solutionR(r) =

{Jm(kr)Ym(kr)

}Bessel function order m

Bessel function second kind.Not used as blow up at r = 0

Jm(ka) = 0 from boundary conditions. This fixes k. Let Zmn be the nnt zeroof the Bessel Jm function. Therefore kmn = Zmn

aare allowed values of k.

ฯˆ โˆ Jm(kmnr) (cos(mฯ†) + sin(mฯ†)) This gives rise to modal shapesฯˆ(r, ฯ†, t) =

โˆ‘Jm(kmnr) (cos(mฯ†) + sin(mฯ†)) (cos(ckmnt) + sin(ckmnt))

Nasser M. Abbasi December 14, 2018

wave 1D: โˆ‚ฯˆ2

โˆ‚t2 = c2 โˆ‚ฯˆ2

โˆ‚x2

ฯˆ(x, t) = 12 (f0(xโˆ’ ct) + f0(x+ ct)) + 1

2c

โˆซ x+ctxโˆ’ct g0(s) ds

Where f0(x) is initial position of string and g0(x) isinitial velocity

Laplacian in 2D polar

โˆ‡u = โˆ‚u2

โˆ‚r2 + 1rโˆ‚uโˆ‚r + 1

r2โˆ‚u2

โˆ‚ฯ†2

Bessel: x2yโ€ฒโ€ฒ + xyโ€ฒ + (x2 โˆ’m2)y = 0, W = Cx . When m

not intger, solutions Jm(x), Jโˆ’m(x). When m integer,solutions Jm(x), Ym(x)

Legendre: (1โˆ’ x2)yโ€ฒโ€ฒ โˆ’ 2xyโ€ฒ + n(n+ 1)y = 0,

W = C1โˆ’x2 . Solutions Pn(x), Qn(x). Where Q blows

at ยฑ1

Sturm-Lioville: (pyโ€ฒ)โ€ฒ โˆ’ qy + ฮปry = 0, operatorL[y] = โˆ’(pyโ€ฒ)โ€ฒ + q. When q = 0, equation becomesL[y] = ฮปry. Inner product < ui, uj >=

โˆซuiuj dx

Green function. For boundary value problem yโ€ฒโ€ฒ + y = f (x) write it as

G (x, x0) =

{Ay1 (x) 0 < x < x0

By2 (x) x0 < x < L

Where y1 (x) is one of the solutions to yโ€ฒโ€ฒ + y = 0 that satisfies left BC and y2 (x) is one which satifies right BC. Then solve for A,B fromcontuinity condition Ay1 (x0) = By2 (x0) and jump discontinuty Byโ€ฒ2 (x0)โˆ’Ayโ€ฒ1 (x0) = โˆ’1

p(x)where p (x) is from SL form. (โˆ’1) for the above.

Then flipe the x, x0 roles and then do y (x) =โˆซ x

0G (x0, x) f (x0) dx0 +

โˆซ LxG (x0, x) f (x0) dx0

Variation of parameters yp (x) = โˆ’y1 (x)โˆซ y2(x)f(x)

W (x)dx+ y2 (x)

โˆซ y1(x)f(x)W (x)

dx

Fourier series f (x) = A0

2 +โˆ‘โˆžn=0An cos

(2ฯ€T nx

)+B2 sin

(2ฯ€T nx

)where An = 2

T

โˆซ T2

โˆ’T2f (x) cos

(2ฯ€T nx

)and

Bn = 2T

โˆซ T2

โˆ’T2f (x) sin

(2ฯ€T nx

). Complex form is f (x) =

โˆ‘โˆžn=โˆ’โˆž Cne

i 2ฯ€T nx, Cn =โˆซ T

2

โˆ’T2f (x) eโˆ’i

2ฯ€T nxdx.

Fourier transform f (x) = 12ฯ€

โˆซโˆžโˆ’โˆž F (ฯ‰) eiฯ‰xdฯ‰ and F (ฯ‰) =

โˆซโˆžโˆ’โˆž f (x) eโˆ’iฯ‰xdx. Parsvales:

โˆซโˆžโˆ’โˆž |f (x)|2 dx = 1

2ฯ€

โˆซโˆžโˆ’โˆž |F (ฯ‰)|2 dฯ‰

Wronskian W (x) = det[[y1, y2], [yโ€ฒ1, yโ€ฒ2]] =

Ceโˆ’โˆซp(x) dx Where p is from

yโ€ฒโ€ฒ + pyโ€ฒ + qy = 0

Bernuli yโ€ฒ + f (x) y = g (x) yn, where n 6= 0, 1. Divideby yn โ†’ 1

yn yโ€ฒ + f (x) y1โˆ’n = g (x) and let

v = y1โˆ’n โ†’ vโ€ฒ = (1โˆ’ n) yโˆ’nyโ€ฒ. converts ODE toseparable in v (x)isobaric given y weight m and x weight 1. each term musthave the same weight if we can find m. Then let y = vxm.Find dy

dx. sub into ODE to get rid of y. Separable in v.

solve.Exact Write ode as Mdx+Ndx = 0 then check ifโˆ‚Mโˆ‚y

= โˆ‚Nโˆ‚x

. If so then set up 2 equations โˆ‚ฮฆโˆ‚x

= M, โˆ‚ฮฆโˆ‚y

= N .

Integrate the first to get ฮฆ =โˆซMdx+ f (y), differentiate

this w.r.t y, and compare to โˆ‚ฮฆโˆ‚y

= N and solve for f (y).Solution is ฮฆ (x, y) = c

Sturm-Liouville To convert ayโ€ฒโ€ฒ + byโ€ฒ + (c+ ฮปd)y = 0 to(p (x) dy

dx

)โ€ฒ โˆ’ q (x) y (x) + ฮปr (x) y (x) = 0 use:

p (x) = eโˆซ b(x)a(x)

dxand q (x) = โˆ’p (x) c(x)

a(x)and r (x) = p(x)

a(x)d

generating functions. Bessel g (x.t) = ex2 (tโˆ’ 1

t ), then writeg (x, t) =

โˆ‘โˆžn=โˆ’โˆž Jn (x) tn. Find

An (x) = 12ฯ€i

โˆฎex2 (tโˆ’ 1

t )tn+1 dt = 1

ฯ€

โˆซ ฯ€0

cos (nฮธ โˆ’ x sin ฮธ) dฮธ. To

find recusive relations, do โˆ‚gโˆ‚t, โˆ‚gโˆ‚x

add/subtract we getJ โ€ฒn (x) = Jnโˆ’1 (x)โˆ’ n

xJn (x) and

J โ€ฒn (x) = nxJn (x)โˆ’ Jnโˆ’1 (x) For Legendre use

g (x, t) = 1โˆš1โˆ’2xtโˆ’t2

=โˆ‘โˆžn=0 Pn (x) tn. Do โˆ‚g

โˆ‚t, โˆ‚gโˆ‚x

add/subtract we getP โ€ฒn+1 (x) = (n+ 1)Pn (x) + xP โ€ฒn (x)and P โ€ฒnโˆ’1 (x) = โˆ’nPn (x) + xP โ€ฒn (x)Pn (x) = 1

2nn!dn

dxn

(x2 โˆ’ 1

)n. Special values,

P0 (x) = 1, P1 (x) = x, P2 (x) = 12

(3x2 โˆ’ 1

)Laplace PDE on disk:y (x) = A0 +

โˆ‘rn (Cn cosnฮธ + kn sinnฮธ) where

A0 = 12ฯ€

โˆซ 2ฯ€

0f (ฮธ) dฮธ and

Cnan = 1

ฯ€

โˆซ 2ฯ€

0f (ฮธ) cosnฮธdฮธ and

knan = 1

ฯ€

โˆซ 2ฯ€

0f (ฮธ) sinnฮธdฮธ where a is radius and

f (ฮธ) is boundary condition.

log (z) = ln |z|+ i (ฮธ0 + 2nฯ€)

Euler ode r2yโ€ฒโ€ฒ+ ryโ€ฒ+ y = 0, let y = rฮฑ

Polynomial ODE. yโ€ฒ (x) = f (ax+ by + c). LetV = ax+ by+ c, then dV = adx+ bdy, converts it to

separable dydv = f(v)

a+bf(v)

34

Eigenfunction expansion. Startting from L [y]โˆ’ ky = f (x), assuming y (x) =โˆ‘n Cnฮฆn (x) where ฮฆn (x) eigenfunctions found by solving

L [y]โˆ’ ฮปy = 0 with homogenous B.C. Then f (x) =โˆ‘n dnฮฆn (x). Subtituting in the ode and use that L [y] = ฮปy results inโˆ‘

n (ฮปn โˆ’ k)Cnฮฆn (x) =โˆ‘n dnฮฆn (x). But dn = ใ€ˆฮฆn (x) , f (x)ใ€‰ = 2

L

โˆซ L0

ฮฆn (x) f (x) dx. The 2L due to normalization. This gives

Cn = dnฮปnโˆ’k . Now that Cn is found the solution is y (x) =

โˆ‘n

dnฮปnโˆ’kฮฆn (x) =

โˆ‘ ใ€ˆฮฆn(x),f(x)ใ€‰ฮปnโˆ’k ฮฆn (x) or y (x) =

โˆ‘ ฮฆn(x)ฮปnโˆ’k

โˆซฮฆn (xโ€ฒ) f (xโ€ฒ) dxโ€ฒ

or y (x) =โˆซ โˆ‘ ฮฆn(xโ€ฒ)ฮฆn(xโ€ฒ)

ฮปnโˆ’k f (xโ€ฒ) dxโ€ฒ. Compare to y (x) =โˆซ x

G (x, xโ€ฒ) f (xโ€ฒ) dxโ€ฒ shows that G (x, xโ€ฒ) =โˆ‘ ฮฆn(xโ€ฒ)ฮฆn(xโ€ฒ)

ฮปnโˆ’k . This is calledGreen function eigenfunction expansion. This assumes ฮฆn are normalized so weight is 1.

completeness relationโˆซuiujr (x) dx = ฮดij . f =

โˆ‘Cnฮฆn (x) and r (xโ€ฒ)

โˆ‘n ฮฆn (x) ฮฆn (xโ€ฒ) = ฮด (xโˆ’ xโ€ฒ). If f (x) = ฮด (xโˆ’ xโ€ฒ), then

L [G (x, xโ€ฒ)]โˆ’ kG (x, xโ€ฒ) = ฮด (xโˆ’ xโ€ฒ) Operator is Hermite ifโˆซuL [v] dx =

โˆซvL [u] dx

Asymptotic series S (z) = c0 + c1z + c2

z2 + ยท ยท ยท is series expansion of f (z) gives good approximation for large z. we truncate S (z) before

it becomes divergent. n is the number of terms in Sn (z). It is optimally truncated when n โ‰ˆ |z|2. S (x) has the following twoimportant properties

1. lim|z|โ†’โˆž zn (f (z)โˆ’ Sn (z)) = 0 for fixed n.

2. limnโ†’โˆž zn (f (z)โˆ’ Sn (z)) =โˆž for fixed z.

S (z) โˆผ f (z) when S (z) is the asymptotic series expansion of f (z) for large z. common method to find S (z) is by integration by parts

Cauchy theoreom. Cauchy-Goursat: If f (z) is analytic on and inside closed contour C then

โˆฎC

f (z) dz = 0. But remember that ifโˆฎC

f (z) dz = 0 then this does not necessarily imply f (z). If f (z) is analytic on and inside C then and z0 is a point in C then

2ฯ€if (z0) =

โˆฎC

f(z)zโˆ’z0 dz and 2ฯ€if โ€ฒ (z0) =

โˆฎC

f(z)

(zโˆ’z0)2dz and 2ฯ€i

n! f(n) (z0) =

โˆฎC

f(z)

(zโˆ’z0)n+1 dz

Laurent series f (z) =โˆ‘โˆžn=0 an (z โˆ’ z0)

n+โˆ‘โˆžn=1

bn(zโˆ’z0)n where an = 1

2ฯ€i

โˆฎf(z)

(zโˆ’z0)n+1 dz and bn = 12ฯ€i

โˆฎf(z)

(zโˆ’z0)โˆ’n+1 dz. Power series of

f (z) around z0 isโˆ‘โˆž

0 an (z โˆ’ z0)n

where an = 1n! f

(n) (z)โˆฃโˆฃz=z0

For Laurent series, lets say singularity is at z = 0 and z = 1. To expand

about z = 0, get f (z) to look like 11โˆ’z and use geometric series for |z| < 1. To expand about z = 1, there are two choices, to the inside

and to the outside. For the outside, i.e. |z| > 1, get f (z) to have 11โˆ’ 1

z

form, this now valid for |z| > 1.

Euler Gamma functionฮ“ (z) =

โˆซโˆž0tzโˆ’1eโˆ’tdt Re (z) > 0 To

extend ฮ“ (z) to the left half plane, i.e. fornegative values. Let us define, using theabove recursive formulaฮ“ (z) = ฮ“(z+1)

zRe (z) > โˆ’1

ฮ“ (z) = (z โˆ’ 1) ฮ“ (z โˆ’ 1) Re (z) > 1

ฮ“ (1) = 1

ฮ“ (n) = (nโˆ’ 1)!

ฮ“ (n+ 1) = n!

ฮ“

(1

2

)=โˆšฯ€

ฮ“ (z + 1) = zฮ“ (z) recursive formula

ฮ“ (z) = ฮ“ (z)

ฮ“

(n+

1

2

)=

1 ยท 3 ยท 5 ยท ยท ยท (2nโˆ’ 1)

2nโˆšฯ€

ฮ“ (x) ฮ“ (1โˆ’ x) =

โˆซ โˆž0

txโˆ’1

1 + tdt 0 < x < 1

=ฯ€

sin (ฯ€x)

If f (z) satisfies CReverywhere in that regionthen it is analytic. Letf (z) = u (x, y) + iv (x, y),then these two equationsin Cartesian coordinatesare โˆ‚u

โˆ‚x= โˆ‚v

โˆ‚yand

โˆ’ โˆ‚uโˆ‚y

= โˆ‚vโˆ‚x

Sometimes it iseasier to use the polarform of these. Letf (z) = r cos ฮธ + i sin ฮธ,then the equations becomeโˆ‚uโˆ‚r

= 1rโˆ‚vโˆ‚ฮธ

and โˆ’ 1rโˆ‚uโˆ‚ฮธ

= โˆ‚vโˆ‚r

Greenโ€™s Theorem saysโˆซC Pdx+Qdy =

โˆซD

(โˆ‚Qโˆ‚xโˆ’ โˆ‚P

โˆ‚y

)dA

ToโˆซC f (z) dz where C is some open

path, i.e. not closed, such as a straightline or a half circle arc. Useparameterization. This converts theintegral to line integration. If C isstraight line, use standard tparameterization,x (t) = (1โˆ’ t)x0 + tx1 andy (t) = (1โˆ’ t) y0 + ty1 where (x0, y0)in the line initial point and (x1, y1) isthe line end point. This works forstraight lines. Use the above andrewrite z = x+ iy asz (t) = x (t) + iy (t) and plug-in in thisz (t) in f (z) to obtain f (t), then theintegral becomesโˆซ

Cf (z) dz =

โˆซ t=1

t=0f (t) zโ€ฒ (t) dt

Now evaluate this integral.

If pole of order n. to find residue, use

Residue (z0) = limzโ†’z0

dnโˆ’1

dzn(z โˆ’ z0)n

(nโˆ’ 1)!f (z)

residue, useR (z0) = limzโ†’z0 (z โˆ’ z0) f (z)

ex = 1 + x+x2

2!+x3

3!+ ยท ยท ยท

ln (1 + x) = xโˆ’ x2

2+x3

3โˆ’ x4

4+ ยท ยท ยท |x| < 1

1

2ln

(1 + x

1โˆ’ x

)= x+

x3

3+x5

5+x7

7โˆ’ ยท ยท ยท |x| < 1

sinx = xโˆ’ x3

3!+ ยท ยท ยท

cosx = 1โˆ’ x2

2+x4

4!+ ยท ยท ยท

โˆซx cos ax =

cos ax

a2+x sin ax

aโˆซx sin ax =

sin ax

a2

x cos ax

aโˆซsin2 ax =

x

2โˆ’

sin 2ax

4aโˆซcos2 ax =

x

2+

sin 2ax

4aโˆซsin ax cos ax =

sin2 ax

2aโˆซ1

ax+ b=

1

aln (ax+ b)โˆซ

x

ax+ b=x

aโˆ’

b

a2ln (ax+ b)โˆซ

1โˆš

1โˆ’ x2= arcsin (x)โˆซ โˆ’1

โˆš1โˆ’ x2

= arccos (x)โˆซ1

1 + x2= arctan (x)

sinx =1

2

eix โˆ’ eโˆ’ix

2i

cosx =1

2

eix + eโˆ’ix

2

sin (AยฑB) = sinA cosB ยฑ cosA sinB

cos (AยฑB) = cosA cosB โˆ“ sinA sinB

sin 2A = 2 sinA cosA

cos 2A = cos2 Aโˆ’ sin2 A = 1โˆ’ 2 sin2 A

sinA sinB =1

2(cos (Aโˆ’B)โˆ’ cos (A+B))

cosA cosB =1

2(cos (A+B) + cos (A+B))

sinA cosB =1

2(sin (Aโˆ’B) + sin (A+B))

F.S. (period 2L).โˆซ L0 sin2

(ฯ€Lx)dx = L

2andโˆซ L

โˆ’L sin2(ฯ€Lx)dx = L

โˆซxeax =

eax

a

(xโˆ’ 1

a

)โˆซx2eax =

eax

a

(x2 โˆ’ 2

ax+

2

a2

)โˆซ

sin bxeax =eax

(a2 + b2)(a sin bxโˆ’ b cos bx)โˆซ

lnx = x lnxโˆ’ x

Geometric series:โˆ‘Nn=0 r

n = 1 + r + r2 + r3 + ยท ยท ยท+ rN = 1โˆ’rNโˆ’1

1โˆ’r andโˆ‘โˆžn=0 r

n = 1 + r + r2 + r3 + ยท ยท ยท = 11โˆ’r |r| < 1 andโˆ‘โˆž

n=0 (โˆ’1)n rn = 1โˆ’ r + r2 โˆ’ r3 + ยท ยท ยท = 11+r

|r| < 1

Binomial series: (x+ y)n =

xn +nxnโˆ’1y+n(nโˆ’1)

2!xnโˆ’2y2 +

n(nโˆ’1)(nโˆ’2)3!

xnโˆ’3y3 + ยท ยท ยทFrom this can generate all other special cases. For

1

(1 + x)= 1โˆ’ x+ x2 โˆ’ x3 + ยท ยท ยท |x| < 1

1

(1โˆ’ x)= 1 + x+ x2 + x3 + ยท ยท ยท |x| < 1

z = rei(ฮธ+2nฯ€), hence

z12 = r

12 e

iฮธ2 e2nฯ€ for

n = 0, 1. Hence 2roots. n = 0 givesprinciple part

log(z) =ln |z|+ i(ฮธ0 + 2nฯ€)n = 0 principle partzc = ec ln z

โˆซ โˆž0

eโˆ’ax cosฮปx =a

a2 + ฮป2โˆซ โˆž0

eโˆ’ax sinฮปx =ฮป

a2 + ฮป2

if eโˆ’y โ‰ค 1 does not work, use Jordan inqualityand use eโˆ’y โ‰ค ฯ€

Rfor contour integration

sin ฮธ =z โˆ’ zโˆ’1

2i

cos ฮธ =z + zโˆ’1

2

Gaussianโˆซโˆžโˆ’โˆž eโˆ’ax

2dx =

โˆšฯ€x

Chapter 3

HWs

3.1 HW 1

3.1.1 Problem 1

Part a

For what values of ๐‘ฅ does the following series converge. ๐‘“ (๐‘ฅ) = 1 + 9๐‘ฅ2 +

81๐‘ฅ2 +

729๐‘ฅ3 +โ‹ฏ

answer

The general term of the series is

๐‘“ (๐‘ฅ) =โˆž๏ฟฝ๐‘›=0

๏ฟฝ3๐‘ฅ๏ฟฝ

2๐‘›

The ratio test can be used to determine convergence. Since all the terms are positive(powers are even), then the absolute value is not needed.

๐ฟ = lim๐‘›โ†’โˆž

๏ฟฝ๐‘Ž๐‘›+1๐‘Ž๐‘›

๏ฟฝ

= lim๐‘›โ†’โˆž

๐‘Ž๐‘›+1๐‘Ž๐‘›

= lim๐‘›โ†’โˆž

๏ฟฝ3๐‘ฅ๏ฟฝ2(๐‘›+1)

๏ฟฝ3๐‘ฅ๏ฟฝ2๐‘›

= lim๐‘›โ†’โˆž

๏ฟฝ3๐‘ฅ๏ฟฝ2๐‘›

๏ฟฝ3๐‘ฅ๏ฟฝ2๐‘› ๏ฟฝ

3๐‘ฅ๏ฟฝ

2

= lim๐‘›โ†’โˆž

32

๐‘ฅ2

=9๐‘ฅ2

The series converges when ๐ฟ < 1, which means 9๐‘ฅ2 < 1 or ๐‘ฅ

2 > 9. Therefore it convergencefor

|๐‘ฅ| > 3

Part b

Does the following series converges or diverges? โˆ‘โˆž๐‘›=1 ln ๏ฟฝ1 + 1

๐‘›๏ฟฝ

answer

The terms are {ln (2) , ln ๏ฟฝ2 + 12๏ฟฝ , ln ๏ฟฝ2 + 1

3๏ฟฝ , ln ๏ฟฝ2 + 1

4๏ฟฝ ,โ‹ฏ}.

35

36

Since the terms become monotonically decreasing after some ๐‘› (after the second term inthis case), the integral test could be used. Let

๐ผ = ๏ฟฝ ln ๏ฟฝ1 +1๐‘ฅ๏ฟฝ๐‘‘๐‘ฅ

The above indefinite integral is first evaluated. Since ln ๏ฟฝ1 + 1๐‘ฅ๏ฟฝ = ln ๏ฟฝ1+๐‘ฅ๐‘ฅ ๏ฟฝ = ln (1 + ๐‘ฅ) โˆ’ ln (๐‘ฅ),

the above can be written as

๐ผ = ๏ฟฝ ln (1 + ๐‘ฅ) ๐‘‘๐‘ฅ โˆ’๏ฟฝ ln (๐‘ฅ) ๐‘‘๐‘ฅ (1)

To evaluate the first integral in (2) โˆซ ln (1 + ๐‘ฅ) ๐‘‘๐‘ฅ, let ๐‘ข = 1 + ๐‘ฅ, then ๐‘‘๐‘ข = ๐‘‘๐‘ฅ, therefore

๏ฟฝ ln (1 + ๐‘ฅ) ๐‘‘๐‘ฅ = ๏ฟฝ ln (๐‘ข) ๐‘‘๐‘ข

= ๐‘ข ln (๐‘ข) โˆ’ ๐‘ขHence

๏ฟฝ ln (1 + ๐‘ฅ) ๐‘‘๐‘ฅ = (1 + ๐‘ฅ) ln (1 + ๐‘ฅ) โˆ’ (1 + ๐‘ฅ) (2)

The second integral in (1) is

๏ฟฝ ln (๐‘ฅ) ๐‘‘๐‘ฅ = ๐‘ฅ ln (๐‘ฅ) โˆ’ ๐‘ฅ (3)

Using (2,3) back into (1) gives

๐ผ = ((1 + ๐‘ฅ) ln (1 + ๐‘ฅ) โˆ’ (1 + ๐‘ฅ)) โˆ’ (๐‘ฅ ln (๐‘ฅ) โˆ’ ๐‘ฅ)= (1 + ๐‘ฅ) ln (1 + ๐‘ฅ) โˆ’ 1 โˆ’ ๐‘ฅ โˆ’ ๐‘ฅ ln (๐‘ฅ) + ๐‘ฅ= (1 + ๐‘ฅ) ln (1 + ๐‘ฅ) โˆ’ ๐‘ฅ ln (๐‘ฅ) โˆ’ 1 (4)

Now that the indefinite integral is evaluated, the limit is taken using

๐‘… = lim๐‘โ†’โˆž

๏ฟฝ๐‘

ln ๏ฟฝ1 +1๐‘ฅ๏ฟฝ๐‘‘๐‘ฅ

Only upper limit is used following the book method1. Using the result found in (4), theabove becomes

๐‘… = lim๐‘โ†’โˆž

[(1 + ๐‘ฅ) ln (1 + ๐‘ฅ) โˆ’ ๐‘ฅ ln (๐‘ฅ) โˆ’ 1]๐‘

The above becomes

๐‘… = lim๐‘โ†’โˆž

[(1 + ๐‘ฅ) ln (1 + ๐‘ฅ) โˆ’ ๐‘ฅ ln (๐‘ฅ) โˆ’ 1]๐‘

= lim๐‘โ†’โˆž

[(1 + ๐‘) ln (1 + ๐‘) โˆ’ ๐‘ ln (๐‘) โˆ’ 1]

= lim๐‘โ†’โˆž

[ln (1 + ๐‘) + ๐‘ ln (1 + ๐‘) โˆ’ ๐‘ ln (๐‘) โˆ’ 1]

= lim๐‘โ†’โˆž

๏ฟฝln (1 + ๐‘) + ๐‘ ln ๏ฟฝ1 + ๐‘๐‘ ๏ฟฝ โˆ’ 1๏ฟฝ

= lim๐‘โ†’โˆž

ln (1 + ๐‘) + lim๐‘โ†’โˆž

๐‘ ln ๏ฟฝ1 + ๐‘๐‘ ๏ฟฝ โˆ’ 1 (5)

But

lim๐‘โ†’โˆž

๐‘ ln ๏ฟฝ1 + ๐‘๐‘ ๏ฟฝ = lim

๐‘โ†’โˆž

ln ๏ฟฝ1+๐‘๐‘ ๏ฟฝ1๐‘

This gives indeterminate form 1/0. So using Lโ€™Hospitalโ€™s rule, by taking derivatives ofnumerator and denominator gives

lim๐‘โ†’โˆž

ln ๏ฟฝ1+๐‘๐‘ ๏ฟฝ1๐‘

= lim๐‘โ†’โˆž

๏ฟฝ1โˆ’ 1+๐‘๐‘ ๏ฟฝ

1+๐‘

โˆ’ 1๐‘2

= lim๐‘โ†’โˆž

โˆ’๏ฟฝ1 โˆ’ 1+๐‘

๐‘๏ฟฝ๐‘2

1 + ๐‘= lim๐‘โ†’โˆž

โˆ’(๐‘ โˆ’ 1 โˆ’ ๐‘)๐‘

1 + ๐‘= lim๐‘โ†’โˆž

๐‘1 + ๐‘

= 1

1See page 131, second edition. Mathematical methods for physics and engineering. Riley, Hobson andBence.

37

Therefore lim๐‘โ†’โˆž๐‘ ln ๏ฟฝ1+๐‘๐‘ ๏ฟฝ = 1. Using this result in (5) results in

๐‘… = lim๐‘โ†’โˆž

ln (1 + ๐‘) + 1 โˆ’ 1

= lim๐‘โ†’โˆž

ln (1 + ๐‘)

= โˆž

Therefore, by the integral test the series diverges.

3.1.2 Problem 2

Find closed form for the series ๐‘“ (๐‘ฅ) = โˆ‘โˆž๐‘›=0 ๐‘›

2๐‘ฅ2๐‘› by taking derivatives of variant of 11โˆ’๐‘ฅ .

For what values of ๐‘ฅ does the series converge?

answer

๐‘“ (๐‘ฅ) = ๐‘ฅ2 + 4๐‘ฅ4 + 9๐‘ฅ6 + 16๐‘ฅ8 + 25๐‘ฅ10 +โ‹ฏ

Observing that

๐‘›2๐‘ฅ2๐‘› =๐‘ฅ2๐‘‘๐‘‘๐‘ฅ๏ฟฝ๐‘›๐‘ฅ2๐‘›๏ฟฝ

Therefore the sum can be written as

๐‘“ (๐‘ฅ) =๐‘ฅ2

โˆž๏ฟฝ๐‘›=0

๐‘‘๐‘‘๐‘ฅ๏ฟฝ๐‘›๐‘ฅ2๐‘›๏ฟฝ

=๐‘ฅ2๐‘‘๐‘‘๐‘ฅ

โˆž๏ฟฝ๐‘›=0

๐‘›๐‘ฅ2๐‘›

=๐‘ฅ2๐‘‘๐‘‘๐‘ฅ๏ฟฝ๐‘ฅ2 + 2๐‘ฅ4 + 3๐‘ฅ6 + 4๐‘ฅ8 + 5๐‘ฅ10 +โ‹ฏ๏ฟฝ

=๐‘ฅ2๐‘‘๐‘‘๐‘ฅ๏ฟฝ๐‘ฅ2 ๏ฟฝ1 + 2๐‘ฅ2 + 3๐‘ฅ4 + 4๐‘ฅ6 + 5๐‘ฅ8 +โ‹ฏ๏ฟฝ๏ฟฝ (1)

To find what 1 + 2๐‘ฅ2 + 3๐‘ฅ4 + 4๐‘ฅ6 +โ‹ฏ sums to, we compare it to the binomial series

(1 + ๐‘ง)๐›ผ = 1 + ๐›ผ๐‘ง +๐›ผ (๐›ผ โˆ’ 1) ๐‘ง2

2!+๐›ผ (๐›ผ โˆ’ 1) (๐›ผ โˆ’ 2) ๐‘ง3

3!+โ‹ฏ = 1 + 2๐‘ฅ2 + 3๐‘ฅ4 + 4๐‘ฅ6 + 5๐‘ฅ8 +โ‹ฏ

Hence, by setting

๐‘ง = โˆ’๐‘ฅ2

๐›ผ = โˆ’2

shows they are the same. Therefore

๏ฟฝ1 โˆ’ ๐‘ฅ2๏ฟฝโˆ’2= 1 + (โˆ’2) ๏ฟฝโˆ’๐‘ฅ2๏ฟฝ +

(โˆ’2) (โˆ’3) ๏ฟฝโˆ’๐‘ฅ2๏ฟฝ2

2!+(โˆ’2) (โˆ’3) (โˆ’4) ๏ฟฝโˆ’๐‘ฅ2๏ฟฝ

3

3!+โ‹ฏ

= 1 + 2๐‘ฅ2 + 3๐‘ฅ2 + 3๐‘ฅ4 +โ‹ฏ

The above is valid for |๐‘ง| < 1 which implies ๐‘ฅ2 < 1 or |๐‘ฅ| < 1. Hence

1 + 2๐‘ฅ2 + 3๐‘ฅ4 + 4๐‘ฅ6 +โ‹ฏ = ๏ฟฝ1 โˆ’ ๐‘ฅ2๏ฟฝโˆ’2

38

Using the above result in (1) gives

๐‘“ (๐‘ฅ) =๐‘ฅ2๐‘‘๐‘‘๐‘ฅ

โŽ›โŽœโŽœโŽœโŽœโŽœโŽœโŽ

๐‘ฅ2

๏ฟฝ1 โˆ’ ๐‘ฅ2๏ฟฝ2

โŽžโŽŸโŽŸโŽŸโŽŸโŽŸโŽŸโŽ 

=๐‘ฅ2

โŽ›โŽœโŽœโŽœโŽœโŽœโŽœโŽ

4๐‘ฅ3

๏ฟฝ1 โˆ’ ๐‘ฅ2๏ฟฝ3 +

2๐‘ฅ

๏ฟฝ1 โˆ’ ๐‘ฅ2๏ฟฝ2

โŽžโŽŸโŽŸโŽŸโŽŸโŽŸโŽŸโŽ 

=2๐‘ฅ4

๏ฟฝ1 โˆ’ ๐‘ฅ2๏ฟฝ3 +

๐‘ฅ2

๏ฟฝ1 โˆ’ ๐‘ฅ2๏ฟฝ2

=2๐‘ฅ4 + ๐‘ฅ2 ๏ฟฝ1 โˆ’ ๐‘ฅ2๏ฟฝ

๏ฟฝ1 โˆ’ ๐‘ฅ2๏ฟฝ3

=2๐‘ฅ4 + ๐‘ฅ2 โˆ’ ๐‘ฅ4

๏ฟฝ1 โˆ’ ๐‘ฅ2๏ฟฝ3

=๐‘ฅ4 + ๐‘ฅ2

๏ฟฝ1 โˆ’ ๐‘ฅ2๏ฟฝ3

Therefore

๐‘“ (๐‘ฅ) =๐‘ฅ2๏ฟฝ๐‘ฅ2+1๏ฟฝ

๏ฟฝ1โˆ’๐‘ฅ2๏ฟฝ3

Where the above converges for |๐‘ฅ| < 1, from above, where we used Binomial expansionwhich is valid for |๐‘ฅ| < 1. This result could also be obtained by using the ratio test.

lim๐‘›โ†’โˆž

๏ฟฝ๐‘Ž๐‘›+1๐‘Ž๐‘›

๏ฟฝ = lim๐‘›โ†’โˆž

๏ฟฝ(๐‘› + 1)2 ๐‘ฅ2(๐‘›+1)

๐‘›2๐‘ฅ2๐‘› ๏ฟฝ

Since all powers are even, the absolute value is not needed. The above becomes

lim๐‘›โ†’โˆž

๐‘Ž๐‘›+1๐‘Ž๐‘›

= lim๐‘›โ†’โˆž

(๐‘› + 1)2 ๐‘ฅ2

๐‘›2

= ๐‘ฅ2 lim๐‘›โ†’โˆž

(๐‘› + 1)2

๐‘›2= ๐‘ฅ2

Therefore for the series to converge, we know that ๐‘Ž๐‘›+1๐‘Ž๐‘›

must be less than 1. Hence ๐‘ฅ2 < 1or |๐‘ฅ| < 1, which is the same result as above.

3.1.3 Problem 3

Part a

Find the sum of 1 + 14 โˆ’

116 โˆ’

164 +

1256 +

11024 โˆ’โ‹ฏ

solution

We would like to combine each two consecutive negative terms and combine each twoconsecutive positive terms in the series in order to obtain an alternating series which iseasier to work with. but to do that, we first need to check that the series is absolutelyconvergent. The |๐‘Ž๐‘›| term is 1

4๐‘› , therefore

39

๐ฟ = lim๐‘›โ†’โˆž

๏ฟฝ๐‘Ž๐‘›+1๐‘Ž๐‘›

๏ฟฝ

= lim๐‘›โ†’โˆž

๏ฟฝ๏ฟฝ

14๐‘›+114๐‘›

๏ฟฝ๏ฟฝ

= lim๐‘›โ†’โˆž

๏ฟฝ4๐‘›

4๐‘›+1๏ฟฝ

= ๏ฟฝ14๏ฟฝ

Since |๐ฟ| < 1 then the series is absolutely convergent so we are allowed now to group (orrearrange) terms as follows

๐‘† = ๏ฟฝ1 +14๏ฟฝโˆ’ ๏ฟฝ

116+164๏ฟฝ

+ ๏ฟฝ1256

+1

1024๏ฟฝโˆ’ ๏ฟฝ

14096

+1

16384๏ฟฝ+โ‹ฏ

=54โˆ’564+

51024

โˆ’5

16 384+โ‹ฏ

=54 ๏ฟฝ1 โˆ’

116+

1256

โˆ’1

4096+โ‹ฏ๏ฟฝ

=54

โˆž๏ฟฝ๐‘›=0

(โˆ’1)๐‘›

42๐‘›

=54

โˆž๏ฟฝ๐‘›=0

(โˆ’1)๐‘› ๏ฟฝ116๏ฟฝ

๐‘›

(1)

But โˆ‘โˆž๐‘›=0 (โˆ’1)

๐‘› ๏ฟฝ 116๏ฟฝ๐‘›has the form โˆ‘โˆž

๐‘›=0 (โˆ’1)๐‘› ๐‘Ÿ๐‘› where ๐‘Ÿ = 1

16 and since |๐‘Ÿ| < 1 then by the

binomial seriesโˆž๏ฟฝ๐‘›=0

(โˆ’1)๐‘› ๐‘Ÿ๐‘› = 1 โˆ’ ๐‘Ÿ + ๐‘Ÿ2 โˆ’ ๐‘Ÿ3 +โ‹ฏ

=1

1 + ๐‘ŸTherefore the sum in (1) becomes, using ๐‘Ÿ = 1

16

๐‘† =54

โŽ›โŽœโŽœโŽœโŽœโŽœโŽ

11 + 1

16

โŽžโŽŸโŽŸโŽŸโŽŸโŽŸโŽ 

=54 ๏ฟฝ1617๏ฟฝ

Hence

๐‘† = 2017

Or

๐‘† โ‰ˆ 1.176

Part b

Find the sum of 11! +

82! +

163! +

644! +โ‹ฏ

solution

The sum can be written as

๐‘† =โˆž๏ฟฝ๐‘›=1

๐‘›3

๐‘›!

=โˆž๏ฟฝ๐‘›=1

๐‘›๐‘›2

๐‘›!

40

But ๐‘›๐‘›! =

๐‘›(๐‘›โˆ’1)!๐‘› =

1(๐‘›โˆ’1)! and the above reduces to

๐‘† =โˆž๏ฟฝ๐‘›=1

๐‘›2

(๐‘› โˆ’ 1)!Let ๐‘› โˆ’ 1 = ๐‘š or ๐‘› = ๐‘š + 1. The above becomes

๐‘† =โˆž๏ฟฝ๐‘š=0

(๐‘š + 1)2

๐‘š!

=โˆž๏ฟฝ๐‘›=0

(๐‘› + 1)2

๐‘›!

=โˆž๏ฟฝ๐‘›=0

๐‘›2 + 1 + 2๐‘›๐‘›!

=โˆž๏ฟฝ๐‘›=0

๐‘›2

๐‘›!+

โˆž๏ฟฝ๐‘›=0

1๐‘›!+ 2

โˆž๏ฟฝ๐‘›=0

๐‘›๐‘›!

(1)

Considering the first term โˆ‘โˆž๐‘›=0

๐‘›2

๐‘›! which can be written asโˆž๏ฟฝ๐‘›=0

๐‘›2

๐‘›!=

โˆž๏ฟฝ๐‘›=1

๐‘›2

๐‘›!

=โˆž๏ฟฝ๐‘›=1

๐‘›(๐‘› โˆ’ 1)!

(2)

Again, letting Let ๐‘› โˆ’ 1 = ๐‘š then โˆ‘โˆž๐‘›=1

๐‘›(๐‘›โˆ’1)! becomes โˆ‘โˆž

๐‘š=0๐‘š+1๐‘š! . Hence (2) becomes

โˆž๏ฟฝ๐‘›=0

๐‘›2

๐‘›!=

โˆž๏ฟฝ๐‘š=0

๐‘š + 1๐‘š!

=โˆž๏ฟฝ๐‘›=0

๐‘› + 1๐‘›!

=โˆž๏ฟฝ๐‘›=0

๐‘›๐‘›!+

โˆž๏ฟฝ๐‘›=0

1๐‘›!

=โˆž๏ฟฝ๐‘›=1

๐‘›๐‘›!+

โˆž๏ฟฝ๐‘›=0

1๐‘›!

But โˆ‘โˆž๐‘›=1

๐‘›๐‘›! = โˆ‘

โˆž๐‘›=1

1(๐‘›โˆ’1)! . Letting ๐‘› โˆ’ 1 = ๐‘š, this becomes โˆ‘โˆž

๐‘š=01๐‘š! = โˆ‘โˆž

๐‘›=01๐‘›!and the above

reduces toโˆž๏ฟฝ๐‘›=0

๐‘›2

๐‘›!=

โˆž๏ฟฝ๐‘›=0

1๐‘›!+

โˆž๏ฟฝ๐‘›=0

1๐‘›!

= ๐‘’ + ๐‘’= 2๐‘’ (3)

The above takes care of the first term in (1). Therefore (1) can now be written as

๐‘† =โˆž๏ฟฝ๐‘›=0

๐‘›2

๐‘›!+

โˆž๏ฟฝ๐‘›=0

1๐‘›!+ 2

โˆž๏ฟฝ๐‘›=0

๐‘›๐‘›!

= 2๐‘’ + ๐‘’ + 2โˆž๏ฟฝ๐‘›=0

๐‘›๐‘›!

= 3๐‘’ + 2โˆž๏ฟฝ๐‘›=0

๐‘›๐‘›!

But โˆ‘โˆž๐‘›=0

๐‘›๐‘›! = โˆ‘

โˆž๐‘›=1

๐‘›๐‘›! and โˆ‘

โˆž๐‘›=1

๐‘›๐‘›! was calculated above. It can be written as โˆ‘โˆž

๐‘›=01๐‘›! . The

above now becomes

๐‘† = 3๐‘’ + 2 ๏ฟฝโˆž๏ฟฝ๐‘›=0

1๐‘›!๏ฟฝ

= 3๐‘’ + 2๐‘’

Therefore

๐‘† = 5๐‘’

or

๐‘† โ‰ˆ 13.5914

41

3.1.4 key solution to HW 1

42

43

3.2 HW 2

3.2.1 Problem 1

Find all possible values for (put into ๐‘ฅ + ๐‘–๐‘ฆ form)

1. log ๏ฟฝ1 + โˆš3๐‘–๏ฟฝ

2. ๏ฟฝ1 + โˆš3๐‘–๏ฟฝ2๐‘–

Answer

44

Part 1

Let ๐‘ง = ๐‘ฅ + ๐‘–๐‘ฆ, where here ๐‘ฅ = 1, ๐‘ฆ = 3, then |๐‘ง| = ๏ฟฝ๐‘ฅ2 + ๐‘ฆ2 = โˆš1 + 3 = 2 and arg (๐‘ง) = ๐œƒ0 =

arctan ๏ฟฝ๐‘ฆ๐‘ฅ๏ฟฝ = arctan ๏ฟฝโˆš31 ๏ฟฝ =๐œ‹6 = 60

0. The function log (๐‘ง) is infinitely multi-valued, given by

log (๐‘ง) = ln |๐‘ง| + ๐‘– (๐œƒ0 + 2๐‘›๐œ‹) ๐‘› = 0, ยฑ1, ยฑ2,โ‹ฏ (1)

Where ๐œƒ0 is the principal argument, which is 600 in this example, which is when ๐‘› = 0.This is done to make log (๐‘ง) single valued. This makes the argument of ๐‘ง restricted toโˆ’๐œ‹ < ๐œƒ0 < ๐œ‹. This makes the negative real axis the branch cut, including the origin. Tofind all values, we simply use (1) for all possible ๐‘› values other than ๐‘› = 0. Each di๏ฟฝerent ๐‘›values gives di๏ฟฝerent branch cut. This gives, where ln |๐‘ง| = ln (2) in all cases, the following

log (๐‘ง) = ln (2) + ๐‘– ๏ฟฝ๐œ‹3๏ฟฝ ๐‘› = 0

= ln (2) + ๐‘– ๏ฟฝ๐œ‹3+ 2๐œ‹๏ฟฝ ๐‘› = 1

= ln (2) + ๐‘– ๏ฟฝ๐œ‹3โˆ’ 2๐œ‹๏ฟฝ ๐‘› = โˆ’1

= ln (2) + ๐‘– ๏ฟฝ๐œ‹3+ 4๐œ‹๏ฟฝ ๐‘› = 2

= ln (2) + ๐‘– ๏ฟฝ๐œ‹3โˆ’ 4๐œ‹๏ฟฝ ๐‘› = โˆ’2

โ‹ฎ

Or

log (๐‘ง) = 0.693 + 1.047๐‘–= 0.693 + 7.330๐‘–= 0.693 โˆ’ 5.236๐‘–= 0.693 + 13.614๐‘–= 0.693 โˆ’ 11.519๐‘–โ‹ฎ

These are in ๏ฟฝ๐‘ฅ + ๐‘–๐‘ฆ๏ฟฝ form. There are infinite number of values. Picking a specific branchcuts (i.e. specific ๐‘› value), picks one of these values. The principal value is one associatedwith ๐‘› = 0.

Part 2

Let ๐‘ง = 1 + ๐‘–โˆš3, hence

๐‘“ (๐‘ง) = ๐‘ง2๐‘–

= exp (2๐‘– log (๐‘ง))= exp (2๐‘– (ln |๐‘ง| + ๐‘– (๐œƒ0 + 2๐‘›๐œ‹))) ๐‘› = 0, ยฑ1, ยฑ2,โ‹ฏ

Where in this example, as in first part, ln |๐‘ง| = ln (2) = 0.693 and principal argument is๐œƒ0 =

๐œ‹3 = 60

0. Hence the above becomes

๐‘“ (๐‘ง) = exp ๏ฟฝ2๐‘– ๏ฟฝln (2) + ๐‘– ๏ฟฝ๐œ‹3+ 2๐‘›๐œ‹๏ฟฝ๏ฟฝ๏ฟฝ

= exp ๏ฟฝ2๐‘– ln (2) โˆ’ ๏ฟฝ2๐œ‹3+ 4๐‘›๐œ‹๏ฟฝ๏ฟฝ

= exp ๏ฟฝ๐‘– ln 4 โˆ’ ๏ฟฝ2๐œ‹3+ 4๐‘›๐œ‹๏ฟฝ๏ฟฝ

= exp (๐‘– ln 4) exp ๏ฟฝโˆ’ ๏ฟฝ2๐œ‹3+ 4๐‘›๐œ‹๏ฟฝ๏ฟฝ

= ๐‘’โˆ’๏ฟฝ 2๐œ‹3 +4๐‘›๐œ‹๏ฟฝ (cos (ln 4) + ๐‘– sin (ln 4))

= ๐‘’โˆ’๏ฟฝ 2๐œ‹3 +4๐‘›๐œ‹๏ฟฝ cos (ln 4) + ๐‘–๐‘’โˆ’๏ฟฝ

2๐œ‹3 +4๐‘›๐œ‹๏ฟฝ sin (ln 4)

45

Which is now in the form of ๐‘ฅ + ๐‘–๐‘ฆ. First few values are

๐‘“ (๐‘ง) = ๐‘’โˆ’๏ฟฝ 2๐œ‹3 ๏ฟฝ cos (ln 4) + ๐‘–๐‘’โˆ’๏ฟฝ

2๐œ‹3 ๏ฟฝ sin (ln 4) ๐‘› = 0

= ๐‘’โˆ’๏ฟฝ 2๐œ‹3 +4๐œ‹๏ฟฝ cos (ln 4) + ๐‘–๐‘’โˆ’๏ฟฝ

2๐œ‹3 +4๐œ‹๏ฟฝ sin (ln 4) ๐‘› = 1

= ๐‘’โˆ’๏ฟฝ 2๐œ‹3 โˆ’4๐œ‹๏ฟฝ cos (ln 4) + ๐‘–๐‘’โˆ’๏ฟฝ

2๐œ‹3 โˆ’4๐œ‹๏ฟฝ sin (ln 4) ๐‘› = โˆ’1

= ๐‘’โˆ’๏ฟฝ 2๐œ‹3 +8๐œ‹๏ฟฝ cos (ln 4) + ๐‘–๐‘’โˆ’๏ฟฝ

2๐œ‹3 +8๐œ‹๏ฟฝ sin (ln 4) ๐‘› = 2

= ๐‘’โˆ’๏ฟฝ 2๐œ‹3 โˆ’8๐œ‹๏ฟฝ cos (ln 4) + ๐‘–๐‘’โˆ’๏ฟฝ

2๐œ‹3 โˆ’8๐œ‹๏ฟฝ sin (ln 4) ๐‘› = โˆ’2

โ‹ฎ

Or

๐‘“ (๐‘ง) = 0.0226 + ๐‘–0.121= 7.878 ร— 10โˆ’8 + ๐‘–4.222 ร— 10โˆ’7

= 6478 + ๐‘–34713= 2.748 ร— 10โˆ’13 + ๐‘–1.472 ร— 10โˆ’12

= 1.858 ร— 109 + ๐‘–9.954 ร— 109

โ‹ฎ

3.2.2 Problem 2

Given that ๐‘ข ๏ฟฝ๐‘ฅ, ๐‘ฆ๏ฟฝ = 3๐‘ฅ2๐‘ฆ โˆ’ ๐‘ฆ3 find ๐‘ฃ ๏ฟฝ๐‘ฅ, ๐‘ฆ๏ฟฝ such that ๐‘“ (๐‘ง) is analytic. Do the same for

๐‘ข ๏ฟฝ๐‘ฅ, ๐‘ฆ๏ฟฝ = ๐‘ฆ๐‘ฅ2+๐‘ฆ2

Solution

Part (1)

๐‘ข ๏ฟฝ๐‘ฅ, ๐‘ฆ๏ฟฝ = 3๐‘ฅ2๐‘ฆ โˆ’ ๐‘ฆ3. The function ๐‘“ (๐‘ง) is analytic if it satisfies Cauchy-Riemann equations

๐œ•๐‘ข๐œ•๐‘ฅ

=๐œ•๐‘ฃ๐œ•๐‘ฆ

(1)

โˆ’๐œ•๐‘ข๐œ•๐‘ฆ

=๐œ•๐‘ฃ๐œ•๐‘ฅ

(2)

Applying the first equation gives

6๐‘ฅ๐‘ฆ =๐œ•๐‘ฃ๐œ•๐‘ฆ

Hence, solving for ๐‘ฃ by integrating, gives

๐‘ฃ ๏ฟฝ๐‘ฅ, ๐‘ฆ๏ฟฝ = 3๐‘ฅ๐‘ฆ2 + ๐‘“ (๐‘ฅ) (3)

Is the solution to (3) where ๐‘“ (๐‘ฅ) is the constant of integration since it is a partial di๏ฟฝerentialequation. We now use equation (2) to find ๐‘“ (๐‘ฅ). From (2)

โˆ’ ๏ฟฝ3๐‘ฅ2 โˆ’ 3๐‘ฆ2๏ฟฝ =๐œ•๐‘ฃ๐œ•๐‘ฅ

โˆ’3๐‘ฅ2 + 3๐‘ฆ2 =๐œ•๐‘ฃ๐œ•๐‘ฅ

But (3) gives ๐œ•๐‘ฃ๐œ•๐‘ฅ = 3๐‘ฆ

2 + ๐‘“โ€ฒ (๐‘ฅ), hence the above becomes

โˆ’3๐‘ฅ2 + 3๐‘ฆ2 = 3๐‘ฆ2 + ๐‘“โ€ฒ (๐‘ฅ)๐‘“โ€ฒ (๐‘ฅ) = โˆ’3๐‘ฅ2 + 3๐‘ฆ2 โˆ’ 3๐‘ฆ2

= โˆ’3๐‘ฅ2

Integrating gives

๐‘“ (๐‘ฅ) = ๏ฟฝโˆ’3๐‘ฅ2๐‘‘๐‘ฅ

= โˆ’๐‘ฅ3 + ๐ถ

46

Therefore, (3) becomes

๐‘ฃ ๏ฟฝ๐‘ฅ, ๐‘ฆ๏ฟฝ = 3๐‘ฅ๐‘ฆ2 + ๐‘“ (๐‘ฅ)

Or

๐‘ฃ ๏ฟฝ๐‘ฅ, ๐‘ฆ๏ฟฝ = 3๐‘ฅ๐‘ฆ2 โˆ’ ๐‘ฅ3 + ๐ถ

Where ๐ถ is arbitrary constant. To verify, we apply CR again. Equation (1) now gives

๐œ•๐‘ข๐œ•๐‘ฅ

=๐œ•๐‘ฃ๐œ•๐‘ฆ

6๐‘ฅ๐‘ฆ = 6๐‘ฆ๐‘ฅ

Verified. Equation (2) gives

โˆ’๐œ•๐‘ข๐œ•๐‘ฆ

=๐œ•๐‘ฃ๐œ•๐‘ฅ

โˆ’3๐‘ฅ2 + 3๐‘ฆ2 = โˆ’3๐‘ฅ2 + 3๐‘ฆ2

Verified.

Part (2)

๐‘ข ๏ฟฝ๐‘ฅ, ๐‘ฆ๏ฟฝ = ๐‘ฆ๐‘ฅ2+๐‘ฆ2 . The function ๐‘“ (๐‘ง) is analytic if it satisfies Cauchy-Riemann equations

๐œ•๐‘ข๐œ•๐‘ฅ

=๐œ•๐‘ฃ๐œ•๐‘ฆ

(1)

โˆ’๐œ•๐‘ข๐œ•๐‘ฆ

=๐œ•๐‘ฃ๐œ•๐‘ฅ

(2)

Applying the first equation gives

โˆ’2๐‘ฅ๐‘ฆ

๏ฟฝ๐‘ฅ2 + ๐‘ฆ2๏ฟฝ2 =

๐œ•๐‘ฃ๐œ•๐‘ฆ

Hence, solving for ๐‘ฃ by integrating, gives

๐‘ฃ = โˆ’2๐‘ฅ๏ฟฝ๐‘ฆ

๏ฟฝ๐‘ฅ2 + ๐‘ฆ2๏ฟฝ2๐‘‘๐‘ฆ

=๐‘ฅ

๐‘ฅ2 + ๐‘ฆ2+ ๐‘“ (๐‘ฅ) (3)

Is the solution to (3) where ๐‘“ (๐‘ฅ) is the constant of integration since it is a partial di๏ฟฝerentialequation. equation (2) gives

โˆ’1

๐‘ฅ2 + ๐‘ฆ2+

2๐‘ฆ2

๏ฟฝ๐‘ฅ2 + ๐‘ฆ2๏ฟฝ2 =

๐œ•๐‘ฃ๐œ•๐‘ฅ

But (3) gives ๐œ•๐‘ฃ๐œ•๐‘ฅ =

1๐‘ฅ2+๐‘ฆ2 โˆ’

2๐‘ฅ2

๏ฟฝ๐‘ฅ2+๐‘ฆ2๏ฟฝ2 + ๐‘“โ€ฒ (๐‘ฅ), hence the above becomes

โˆ’1

๐‘ฅ2 + ๐‘ฆ2+

2๐‘ฆ2

๏ฟฝ๐‘ฅ2 + ๐‘ฆ2๏ฟฝ2 =

1๐‘ฅ2 + ๐‘ฆ2

โˆ’2๐‘ฅ2

๏ฟฝ๐‘ฅ2 + ๐‘ฆ2๏ฟฝ2 + ๐‘“

โ€ฒ (๐‘ฅ)

๐‘“โ€ฒ (๐‘ฅ) = โˆ’2

๐‘ฅ2 + ๐‘ฆ2+2 ๏ฟฝ๐‘ฆ2 + ๐‘ฅ2๏ฟฝ

๏ฟฝ๐‘ฅ2 + ๐‘ฆ2๏ฟฝ2

= โˆ’2

๐‘ฅ2 + ๐‘ฆ2+

2๏ฟฝ๐‘ฅ2 + ๐‘ฆ2๏ฟฝ

= 0

Hence

๐‘“ (๐‘ฅ) = ๐ถ

where ๐ถ is arbitrary constant. Therefore, (3) becomes

๐‘ฃ ๏ฟฝ๐‘ฅ, ๐‘ฆ๏ฟฝ = ๐‘ฅ๐‘ฅ2+๐‘ฆ2 + ๐ถ

47

To verify, CR is applied again. Equation (1) now gives

๐œ•๐‘ข๐œ•๐‘ฅ

=๐œ•๐‘ฃ๐œ•๐‘ฆ

โˆ’2๐‘ฅ๐‘ฆ

๏ฟฝ๐‘ฅ2 + ๐‘ฆ2๏ฟฝ2 =

โˆ’2๐‘ฅ๐‘ฆ

๏ฟฝ๐‘ฅ2 + ๐‘ฆ2๏ฟฝ2

Hence verified. Equation (2) gives

โˆ’๐œ•๐‘ข๐œ•๐‘ฆ

=๐œ•๐‘ฃ๐œ•๐‘ฅ

โˆ’1

๐‘ฅ2 + ๐‘ฆ2+

2๐‘ฆ2

๏ฟฝ๐‘ฅ2 + ๐‘ฆ2๏ฟฝ2 =

1๐‘ฅ2 + ๐‘ฆ2

โˆ’2๐‘ฅ2

๏ฟฝ๐‘ฅ2 + ๐‘ฆ2๏ฟฝ2

โˆ’ ๏ฟฝ๐‘ฅ2 + ๐‘ฆ2๏ฟฝ + 2๐‘ฆ2

๏ฟฝ๐‘ฅ2 + ๐‘ฆ2๏ฟฝ2 =

๐‘ฅ2 + ๐‘ฆ2 โˆ’ 2๐‘ฅ2

๏ฟฝ๐‘ฅ2 + ๐‘ฆ2๏ฟฝ2

โˆ’๐‘ฅ2 + ๐‘ฆ2

๏ฟฝ๐‘ฅ2 + ๐‘ฆ2๏ฟฝ2 =

โˆ’๐‘ฅ2 + ๐‘ฆ2

๏ฟฝ๐‘ฅ2 + ๐‘ฆ2๏ฟฝ2

Verified.

3.2.3 Problem 3

Evaluate the integral (i) โˆฎ๐ถ

|๐‘ง|2 ๐‘‘๐‘ง and (ii) โˆฎ๐ถ

1๐‘ง2๐‘‘๐‘ง along two contours. These contours are

1. Line segment with initial point 1 and fixed point ๐‘–

2. Arc of unit circle with Im (๐‘ง) โ‰ฅ 0 with initial point 1 and final point ๐‘–

Solution

Part (1)

y

x

C

1

i

(x0, y0)

(x1, y1)

Figure 3.1: Integration path

First integral We start by finding the parameterization. For line segments that starts at

๏ฟฝ๐‘ฅ0, ๐‘ฆ0๏ฟฝ and ends at ๏ฟฝ๐‘ฅ1, ๐‘ฆ1๏ฟฝ, the parametrization is given by

๐‘ฅ (๐‘ก) = (1 โˆ’ ๐‘ก) ๐‘ฅ0 + ๐‘ก๐‘ฅ1๐‘ฆ (๐‘ก) = (1 โˆ’ ๐‘ก) ๐‘ฆ0 + ๐‘ก๐‘ฆ1

For 0 โ‰ค ๐‘ก โ‰ค 1. Hence for ๐‘ง = ๐‘ฅ + ๐‘–๐‘ฆ, it becomes ๐‘ง (๐‘ก) = ๐‘ฅ (๐‘ก) + ๐‘–๐‘ฆ (๐‘ก). In this case, ๐‘ฅ0 = 1, ๐‘ฆ0 =0, ๐‘ฅ1 = 0, ๐‘ฆ1 = 1, therefore

๐‘ฅ (๐‘ก) = (1 โˆ’ ๐‘ก)๐‘ฆ (๐‘ก) = ๐‘ก

48

Using these, ๐‘ง (๐‘ก) is found from

๐‘ง (๐‘ก) = ๐‘ฅ (๐‘ก) + ๐‘–๐‘ฆ (๐‘ก)= (1 โˆ’ ๐‘ก) + ๐‘–๐‘ก

And

๐‘งโ€ฒ (๐‘ก) = โˆ’1 + ๐‘–

Since |๐‘ง|2 = ๐‘ฅ2 + ๐‘ฆ2, then in terms of ๐‘ก it becomes

|๐‘ง (๐‘ก)|2 = (1 โˆ’ ๐‘ก)2 + ๐‘ก2

Hence the line integral now becomes

๏ฟฝ๐ถ|๐‘ง|2 ๐‘‘๐‘ง = ๏ฟฝ

1

0|๐‘ง (๐‘ก)|2 ๐‘งโ€ฒ (๐‘ก) ๐‘‘๐‘ก

= ๏ฟฝ1

0๏ฟฝ(1 โˆ’ ๐‘ก)2 + ๐‘ก2๏ฟฝ (โˆ’1 + ๐‘–) ๐‘‘๐‘ก

= (โˆ’1 + ๐‘–)๏ฟฝ1

0(1 โˆ’ ๐‘ก)2 + ๐‘ก2๐‘‘๐‘ก

= (โˆ’1 + ๐‘–)๏ฟฝ1

01 + ๐‘ก2 โˆ’ 2๐‘ก + ๐‘ก2๐‘‘๐‘ก

= (โˆ’1 + ๐‘–)๏ฟฝ1

01 + 2๐‘ก2 โˆ’ 2๐‘ก ๐‘‘๐‘ก

= (โˆ’1 + ๐‘–) ๏ฟฝ๏ฟฝ1

0๐‘‘๐‘ก +๏ฟฝ

1

02๐‘ก2๐‘‘๐‘ก โˆ’๏ฟฝ

1

02๐‘ก ๐‘‘๐‘ก๏ฟฝ

= (โˆ’1 + ๐‘–)โŽ›โŽœโŽœโŽœโŽœโŽ(๐‘ก)

10 + 2 ๏ฟฝ

๐‘ก3

3 ๏ฟฝ1

0โˆ’ 2 ๏ฟฝ

๐‘ก2

2 ๏ฟฝ1

0

โŽžโŽŸโŽŸโŽŸโŽŸโŽ 

= (โˆ’1 + ๐‘–) ๏ฟฝ1 +23โˆ’ 2 ๏ฟฝ

12๏ฟฝ๏ฟฝ

Hence

โˆซ๐ถ|๐‘ง|2 ๐‘‘๐‘ง = 2

3(๐‘– โˆ’ 1)

second integral

Using the same parameterization above. But here the integrand is1๐‘ง2=

1((1 โˆ’ ๐‘ก) + ๐‘–๐‘ก)2

Hence the integral becomes

๏ฟฝ๐ถ

1๐‘ง2๐‘‘๐‘ง = ๏ฟฝ

1

0

1((1 โˆ’ ๐‘ก) + ๐‘–๐‘ก)2

๐‘งโ€ฒ (๐‘ก) ๐‘‘๐‘ก

= (๐‘– โˆ’ 1)๏ฟฝ1

0

1((1 โˆ’ ๐‘ก) + ๐‘–๐‘ก)2

๐‘‘๐‘ก

= (๐‘– โˆ’ 1) (โˆ’๐‘–)

Hence

โˆซ๐ถ1๐‘ง2๐‘‘๐‘ง = 1 + ๐‘–

49

Part (2)

y

x1

i

rฮธ (x0, y0)

(x1, y1)

Figure 3.2: Integration path

First integral Let ๐‘ง = ๐‘Ÿ๐‘’๐‘–๐œƒ then ๐‘‘๐‘ง๐‘‘๐œƒ = ๐‘Ÿ๐‘–๐‘’๐‘–๐œƒ. When ๐‘ง = 1 then ๐œƒ = 0. When ๐‘ง = ๐‘– then ๐œƒ = ๐œ‹

2 ,hence we can parameterize the contour integral using ๐œƒ and it becomes

๏ฟฝ๐ถ|๐‘ง|2 ๐‘‘๐‘ง = ๏ฟฝ

๐œ‹2

0๐‘Ÿ2 ๏ฟฝ๐‘Ÿ๐‘–๐‘’๐‘–๐œƒ๏ฟฝ ๐‘‘๐œƒ

= ๐‘–๐‘Ÿ3๏ฟฝ๐œ‹2

0๐‘’๐‘–๐œƒ๐‘‘๐œƒ

= ๐‘–๐‘Ÿ3 ๏ฟฝ๐‘’๐‘–๐œƒ

๐‘– ๏ฟฝ

๐œ‹2

0

= ๐‘Ÿ3 ๏ฟฝ๐‘’๐‘–๐œƒ๏ฟฝ๐œ‹2

0

= ๐‘Ÿ3 ๏ฟฝ๐‘’๐‘–๐œ‹2 โˆ’ ๐‘’0๏ฟฝ

= ๐‘Ÿ3 [๐‘– โˆ’ 1]

But ๐‘Ÿ = 1, therefore the above becomes

โˆซ๐ถ|๐‘ง|2 ๐‘‘๐‘ง = ๐‘– โˆ’ 1

second integral

Using the same parameterization above. But here the integrand now1๐‘ง2=

1๐‘Ÿ2๐‘’๐‘–2๐œƒ

Therefore

๏ฟฝ๐ถ

1๐‘ง2๐‘‘๐‘ง = ๏ฟฝ

๐œ‹2

0

1๐‘Ÿ2๐‘’๐‘–2๐œƒ

๏ฟฝ๐‘Ÿ๐‘–๐‘’๐‘–๐œƒ๏ฟฝ ๐‘‘๐œƒ

=๐‘–๐‘Ÿ ๏ฟฝ

๐œ‹2

0๐‘’โˆ’๐‘–๐œƒ๐‘‘๐œƒ

=๐‘–๐‘Ÿ ๏ฟฝ๐‘’โˆ’๐‘–๐œƒ

โˆ’๐‘– ๏ฟฝ

๐œ‹2

0

=โˆ’1๐‘Ÿ๏ฟฝ๐‘’โˆ’๐‘–๐œƒ๏ฟฝ

๐œ‹2

0

=โˆ’1๐‘Ÿ๏ฟฝ๐‘’โˆ’๐‘–

๐œ‹2 โˆ’ 1๏ฟฝ

=โˆ’1๐‘Ÿ(โˆ’๐‘– โˆ’ 1)

But ๐‘Ÿ = 1, hence

โˆซ๐ถ1๐‘ง2๐‘‘๐‘ง = 1 + ๐‘–

50

3.2.4 Problem 4

Use the Cauchy integral formula

๐‘“ (๐‘ง0) =12๐œ‹๐‘–โˆฎ

๐ถ

๐‘“ (๐‘ง)๐‘ง โˆ’ ๐‘ง0

๐‘‘๐‘ง

To evaluate

โˆฎ๐ถ

1(๐‘ง + 1) (๐‘ง + 2)

๐‘‘๐‘ง

Where ๐ถ is the circular contour |๐‘ง + 1| = ๐‘… with ๐‘… < 1. Note that if ๐‘… > 1 then a di๏ฟฝerentresult is found. Why canโ€™t the Cauchy integral formula above be used for ๐‘… > 1?

Solution

The disk |๐‘ง + 1| = ๐‘… is centered at ๐‘ง = โˆ’1 with ๐‘… < 1. The function

๐‘” (๐‘ง) =1

(๐‘ง + 1) (๐‘ง + 2)has pole at ๐‘ง = โˆ’1 and at ๐‘ง = โˆ’2.

R

x

y

โˆ’1โˆ’2

C

Figure 3.3: Showing location of pole

In the Cauchy integral formula, the function ๐‘“ (๐‘ง) is analytic on ๐ถ and inside ๐ถ. Hence,to use Cauchy integral formula, we need to convert ๐‘” (๐‘ง) = 1

(๐‘ง+1)(๐‘ง+2) to look like ๐‘“(๐‘ง)๐‘งโˆ’๐‘ง0

where๐‘“ (๐‘ง) is analytic inside ๐ถ. This is done as follows

1(๐‘ง + 1) (๐‘ง + 2)

=1

(๐‘ง+2)

๐‘ง โˆ’ (โˆ’1)

=๐‘“ (๐‘ง)

๐‘ง โˆ’ (โˆ’1)

Where now ๐‘“ (๐‘ง) = 1(๐‘ง+2) . This has pole at ๐‘ง = โˆ’2. Since this pole is outside ๐ถ then ๐‘“ (๐‘ง) is

analytic on and inside ๐ถ and can be used for the purpose of using Cauchy integral formula,which now can be written as

โˆฎ๐ถ

1(๐‘ง + 1) (๐‘ง + 2)

๐‘‘๐‘ง = โˆฎ๐ถ

1(๐‘ง+2)

๐‘ง โˆ’ (โˆ’1)๐‘‘๐‘ง

= โˆฎ๐ถ

๐‘“ (๐‘ง)๐‘ง โˆ’ (โˆ’1)

๐‘‘๐‘ง

= (2๐œ‹๐‘–) ๐‘“ (โˆ’1)

Therefore, we just need to evaluate ๐‘“ (โˆ’1) which is seen as 1. Hence

โˆฎ๐ถ

1(๐‘ง+1)(๐‘ง+2)๐‘‘๐‘ง = 2๐œ‹๐‘– (1)

To verify, we can solve this again using the residue theorem

โˆฎ๐ถ

๐‘” (๐‘ง) ๐‘‘๐‘ง = 2๐œ‹๐‘– ๏ฟฝsum of residues of ๐‘” (๐‘ง) inside ๐ถ๏ฟฝ

51

But ๐‘” (๐‘ง) = 1(๐‘ง+1)(๐‘ง+2) has only one pole inside ๐ถ, which is at ๐‘ง = โˆ’1. Therefore the above

becomes

โˆฎ๐ถ

1(๐‘ง + 1) (๐‘ง + 2)

= 2๐œ‹๐‘– ๏ฟฝresidue of ๐‘” (๐‘ง) at โˆ’ 1๏ฟฝ (2)

To find residue at โˆ’1, we can use one of the short cuts to do that. Where we write 1(๐‘ง+1)(๐‘ง+2) =

ฮฆ(๐‘ง)๐‘ง+1 where ฮฆ (๐‘ง) is analytic at ๐‘ง = โˆ’1 and ฮฆ (โˆ’1) โ‰  0. Therefore we see that ฮฆ (๐‘ง) = 1

๐‘ง+2 . Hence

residue of 1(๐‘ง+1)(๐‘ง+2) = ฮฆ (๐‘ง0) =

1(โˆ’1)+2 = 1. Equation (2) becomes

โˆฎ๐ถ

1(๐‘ง + 1) (๐‘ง + 2)

= 2๐œ‹๐‘–

Which is same result obtained in (1) by using Cauchy integral formula directly.

To answer last part, when ๐‘… > 1, then now both poles ๐‘ง = โˆ’1 and = โˆ’2, are inside ๐ถ.Therefore, we canโ€™t split 1

(๐‘ง+1)(๐‘ง+2) into one part that is analytic (the ๐‘“ (๐‘ง) in the above), in

order to obtain expression ๐‘“(๐‘ง)๐‘งโˆ’๐‘ง0

in order to apply Cauchy integral formula directly. Thereforewhen ๐‘… > 1 we should use

โˆฎ๐ถ

๐‘” (๐‘ง) ๐‘‘๐‘ง = 2๐œ‹๐‘– ๏ฟฝsum of residues of ๐‘” (๐‘ง) inside ๐ถ๏ฟฝ

3.2.5 Problem 5

Evaluate the integral

โˆฎ๐ถ

๐‘’๐‘ง2 ๏ฟฝ1๐‘ง2โˆ’1๐‘ง3 ๏ฟฝ

๐‘‘๐‘ง

Where he contour is the unit circle around origin (counter clockwise direction).

Solution

โˆฎ๐ถ

๐‘’๐‘ง2 ๏ฟฝ1๐‘ง2โˆ’1๐‘ง3 ๏ฟฝ

๐‘‘๐‘ง = โˆฎ๐ถ

๐‘’๐‘ง2 ๏ฟฝ๐‘ง โˆ’ 1๐‘ง3 ๏ฟฝ ๐‘‘๐‘ง

= โˆฎ๐ถ

๐‘“ (๐‘ง)(๐‘ง โˆ’ ๐‘ง0)

3๐‘‘๐‘ง

Where ๐‘ง0 = 0 and where

๐‘“ (๐‘ง) = ๐‘’๐‘ง2 (๐‘ง โˆ’ 1)

But ๐‘“ (๐‘ง) is analytic on ๐ถ and inside, since ๐‘’๐‘ง2 is analytic everywhere and ๐‘ง โˆ’ 1 has no poles.Hence we can use Cauchy integral formula for pole of higher order given by

โˆฎ๐ถ

๐‘“ (๐‘ง)(๐‘ง โˆ’ ๐‘ง0)

๐‘›+1๐‘‘๐‘ง =2๐œ‹๐‘–๐‘›!๐‘“(๐‘›) (๐‘ง0)

Where ๐‘› = 2 in this case. Therefore, since ๐‘ง0 = 0 the above reduces to

โˆฎ๐ถ

๐‘“ (๐‘ง)๐‘ง3

๐‘‘๐‘ง =2๐œ‹๐‘–2๐‘“โ€ฒโ€ฒ (0) (1)

Now we just need to find ๐‘“โ€ฒโ€ฒ (๐‘ง) and evaluate the result at ๐‘ง0 = 0

๐‘“โ€ฒ (๐‘ง) = 2๐‘ง๐‘’๐‘ง2 (๐‘ง โˆ’ 1) + ๐‘’๐‘ง2

๐‘“โ€ฒโ€ฒ (๐‘ง) = 2๐‘’๐‘ง2 (๐‘ง โˆ’ 1) + 2๐‘ง ๏ฟฝ2๐‘ง๐‘’๐‘ง2 (๐‘ง โˆ’ 1) + ๐‘’๐‘ง2๏ฟฝ + 2๐‘ง๐‘’๐‘ง2

Hence

๐‘“โ€ฒโ€ฒ (0) = โˆ’2

Therefore (1) becomes

โˆฎ๐ถ

๐‘’๐‘ง2 (๐‘ง โˆ’ 1)๐‘ง3

๐‘‘๐‘ง = โˆ’2๐œ‹๐‘– (2)

To verify, we will do the same integration by converting it to line integration using param-

52

eterization on ๐œƒ. Let ๐‘ง (๐œƒ) = ๐‘Ÿ๐‘’๐‘–๐œƒ, but ๐‘Ÿ = 1, therefore ๐‘ง (๐œƒ) = ๐‘’๐‘–๐œƒ, ๐‘‘๐‘ง = ๐‘–๐‘’๐‘–๐œƒ๐‘‘๐œƒ. Therefore theintegral becomes

โˆฎ๐ถ

๐‘’๐‘ง2 ๏ฟฝ๐‘ง โˆ’ 1๐‘ง3 ๏ฟฝ ๐‘‘๐‘ง = ๏ฟฝ

2๐œ‹

0๐‘’๐‘’2๐‘–๐œƒ ๏ฟฝ

๐‘’๐‘–๐œƒ โˆ’ 1๐‘’3๐‘–๐œƒ ๏ฟฝ ๐‘–๐‘’๐‘–๐œƒ๐‘‘๐œƒ

= ๐‘–๏ฟฝ2๐œ‹

0๐‘’๐‘’2๐‘–๐œƒ ๏ฟฝ

๐‘’๐‘–๐œƒ โˆ’ 1๐‘’2๐‘–๐œƒ ๏ฟฝ ๐‘‘๐œƒ

This is a hard integral to solve by hand. Using computer algebra software, it also gave โˆ’2๐œ‹๐‘–.This verified the result. Clearly using the Cauchy integral formula to solve this problemwas much simpler that using parameterization.

3.2.6 key solution to HW 2

53

54

55

56

3.3 HW 3

3.3.1 Problem 1

Part (a)

Use Cauchy-Riemann equations to determine if |๐‘ง| analytic function of the complex variable๐‘ง.

57

Solution

๐‘“ (๐‘ง) = |๐‘ง|

Let ๐‘ง = ๐‘ฅ + ๐‘–๐‘ฆ, then

๐‘“ (๐‘ง) = ๏ฟฝ๐‘ฅ2 + ๐‘ฆ2๏ฟฝ12

= ๐‘ข + ๐‘–๐‘ฃ

Hence

๐‘ข = ๏ฟฝ๐‘ฅ2 + ๐‘ฆ2

๐‘ฃ = 0

Cauchy-Riemann equations are

๐œ•๐‘ข๐œ•๐‘ฅ

=๐œ•๐‘ฃ๐œ•๐‘ฆ

(1)

โˆ’๐œ•๐‘ข๐œ•๐‘ฆ

=๐œ•๐‘ฃ๐œ•๐‘ฅ

(2)

First equation above gives ๐œ•๐‘ฃ๐œ•๐‘ฆ = 0 and

๐œ•๐‘ข๐œ•๐‘ฅ =

12

2๐‘ฅ

๏ฟฝ๐‘ฅ2+๐‘ฆ2, which shows that ๐œ•๐‘ฃ

๐œ•๐‘ฆ โ‰ ๐œ•๐‘ข๐œ•๐‘ฅ . Therefore

|๐‘ง| is not analytic.

Part (b)

Use Cauchy-Riemann equations to determine if Re (๐‘ง) analytic function of the complexvariable ๐‘ง.

Solution

๐‘“ (๐‘ง) = Re (๐‘ง)Let ๐‘ง = ๐‘ฅ + ๐‘–๐‘ฆ, then

๐‘“ (๐‘ง) = ๐‘ฅ= ๐‘ข + ๐‘–๐‘ฃ

Hence

๐‘ข = ๐‘ฅ๐‘ฃ = 0

Cauchy-Riemann equations are

๐œ•๐‘ข๐œ•๐‘ฅ

=๐œ•๐‘ฃ๐œ•๐‘ฆ

(1)

โˆ’๐œ•๐‘ข๐œ•๐‘ฆ

=๐œ•๐‘ฃ๐œ•๐‘ฅ

(2)

First equation above gives ๐œ•๐‘ฃ๐œ•๐‘ฆ = 0 and

๐œ•๐‘ข๐œ•๐‘ฅ = 1, which shows that ๐œ•๐‘ฃ

๐œ•๐‘ฆ โ‰ ๐œ•๐‘ข๐œ•๐‘ฅ . Therefore Re (๐‘ง) is

not analytic.

Part (c)

Use Cauchy-Riemann equations to determine if ๐‘’sin ๐‘ง analytic function of the complexvariable ๐‘ง.

Solution

๐‘“ (๐‘ง) = ๐‘’sin ๐‘ง is analytic since we can show that exp (๐‘ง) is analytic by applying Cauchy-Riemann (C-R), and also show that sin (๐‘ง) is analytic using C-R. Theory of analytic func-tions it says that the composition of analytic functions is also an analytic function, whichmeans ๐‘’sin ๐‘ง is analytic.

But this problems seems to ask to use C-R equations directly to show this. Therefore weneed to first determine the real and complex parts (๐‘ข, ๐‘ฃ) of the function ๐‘’sin ๐‘ง. Since

sin ๐‘ง = ๐‘ง โˆ’ ๐‘งโˆ’1

2๐‘–

58

Then

๐‘“ (๐‘ง) = ๐‘’sin ๐‘ง

= exp ๏ฟฝ๐‘ง โˆ’ ๐‘งโˆ’1

2๐‘– ๏ฟฝ

= exp ๏ฟฝ ๐‘ง2๐‘–๏ฟฝ exp ๏ฟฝ

โˆ’12๐‘–๐‘ง๏ฟฝ

But ๐‘ง = ๐‘ฅ + ๐‘–๐‘ฆ and the above expands to

exp (sin ๐‘ง) = exp ๏ฟฝ12๐‘–๏ฟฝ๐‘ฅ + ๐‘–๐‘ฆ๏ฟฝ๏ฟฝ exp

โŽ›โŽœโŽœโŽœโŽœโŽ

โˆ’12๐‘– ๏ฟฝ๐‘ฅ + ๐‘–๐‘ฆ๏ฟฝ

โŽžโŽŸโŽŸโŽŸโŽŸโŽ 

= exp ๏ฟฝโˆ’๐‘–2๐‘ฅ +

12๐‘ฆ๏ฟฝ exp

โŽ›โŽœโŽœโŽœโŽœโŽ๐‘–2

1๏ฟฝ๐‘ฅ + ๐‘–๐‘ฆ๏ฟฝ

โŽžโŽŸโŽŸโŽŸโŽŸโŽ 

= exp ๏ฟฝโˆ’๐‘–2๐‘ฅ +

12๐‘ฆ๏ฟฝ exp

โŽ›โŽœโŽœโŽœโŽœโŽ๐‘–2

๐‘ฅ โˆ’ ๐‘–๐‘ฆ๏ฟฝ๐‘ฅ + ๐‘–๐‘ฆ๏ฟฝ ๏ฟฝ๐‘ฅ โˆ’ ๐‘–๐‘ฆ๏ฟฝ

โŽžโŽŸโŽŸโŽŸโŽŸโŽ 

= exp ๏ฟฝโˆ’๐‘–2๐‘ฅ +

12๐‘ฆ๏ฟฝ exp ๏ฟฝ

๐‘–2๐‘ฅ โˆ’ ๐‘–๐‘ฆ๐‘ฅ2 + ๐‘ฆ2 ๏ฟฝ

= exp ๏ฟฝโˆ’๐‘–2๐‘ฅ +

12๐‘ฆ๏ฟฝ exp ๏ฟฝ

๐‘–2 ๏ฟฝ

๐‘ฅ๐‘ฅ2 + ๐‘ฆ2

โˆ’๐‘–๐‘ฆ

๐‘ฅ2 + ๐‘ฆ2 ๏ฟฝ๏ฟฝ

= exp ๏ฟฝโˆ’๐‘–2๐‘ฅ +

12๐‘ฆ๏ฟฝ exp ๏ฟฝ

๐‘–2

๐‘ฅ๐‘ฅ2 + ๐‘ฆ2

+12

๐‘ฆ๐‘ฅ2 + ๐‘ฆ2 ๏ฟฝ

= exp ๏ฟฝโˆ’๐‘–2๐‘ฅ๏ฟฝ exp ๏ฟฝ

12๐‘ฆ๏ฟฝ exp ๏ฟฝ

๐‘–2

๐‘ฅ๐‘ฅ2 + ๐‘ฆ2 ๏ฟฝ

exp ๏ฟฝ12

๐‘ฆ๐‘ฅ2 + ๐‘ฆ2 ๏ฟฝ

Collecting terms gives

exp (sin ๐‘ง) = exp ๏ฟฝ12๐‘ฆ +

12

๐‘ฆ๐‘ฅ2 + ๐‘ฆ2 ๏ฟฝ

exp ๏ฟฝ๐‘–2

๐‘ฅ๐‘ฅ2 + ๐‘ฆ2

โˆ’๐‘–2๐‘ฅ๏ฟฝ

= expโŽ›โŽœโŽœโŽœโŽœโŽ12๐‘ฆ ๏ฟฝ1 + ๏ฟฝ๐‘ฅ2 + ๐‘ฆ2๏ฟฝ๏ฟฝ

๐‘ฅ2 + ๐‘ฆ2

โŽžโŽŸโŽŸโŽŸโŽŸโŽ  exp

โŽ›โŽœโŽœโŽœโŽœโŽ๐‘–2

๐‘ฅ๐‘ฅ2 + ๐‘ฆ2

โˆ’๐‘–

2 ๏ฟฝ๐‘ฅ2 + ๐‘ฆ2๏ฟฝ๐‘ฅ ๏ฟฝ๐‘ฅ2 + ๐‘ฆ2๏ฟฝ

โŽžโŽŸโŽŸโŽŸโŽŸโŽ 

= expโŽ›โŽœโŽœโŽœโŽœโŽ12๐‘ฆ ๏ฟฝ1 + ๏ฟฝ๐‘ฅ2 + ๐‘ฆ2๏ฟฝ๏ฟฝ

๐‘ฅ2 + ๐‘ฆ2

โŽžโŽŸโŽŸโŽŸโŽŸโŽ  exp

โŽ›โŽœโŽœโŽœโŽœโŽ๐‘–12๐‘ฅ ๏ฟฝ1 โˆ’ ๏ฟฝ๐‘ฅ2 + ๐‘ฆ2๏ฟฝ๏ฟฝ

๐‘ฅ2 + ๐‘ฆ2

โŽžโŽŸโŽŸโŽŸโŽŸโŽ 

= expโŽ›โŽœโŽœโŽœโŽœโŽ12๐‘ฆ ๏ฟฝ1 + ๏ฟฝ๐‘ฅ2 + ๐‘ฆ2๏ฟฝ๏ฟฝ

๐‘ฅ2 + ๐‘ฆ2

โŽžโŽŸโŽŸโŽŸโŽŸโŽ 

โŽกโŽขโŽขโŽขโŽขโŽฃcos

โŽ›โŽœโŽœโŽœโŽœโŽ12๐‘ฅ ๏ฟฝ1 โˆ’ ๏ฟฝ๐‘ฅ2 + ๐‘ฆ2๏ฟฝ๏ฟฝ

๐‘ฅ2 + ๐‘ฆ2

โŽžโŽŸโŽŸโŽŸโŽŸโŽ  + ๐‘– sin

โŽ›โŽœโŽœโŽœโŽœโŽ12๐‘ฅ ๏ฟฝ1 โˆ’ ๏ฟฝ๐‘ฅ2 + ๐‘ฆ2๏ฟฝ๏ฟฝ

๐‘ฅ2 + ๐‘ฆ2

โŽžโŽŸโŽŸโŽŸโŽŸโŽ 

โŽคโŽฅโŽฅโŽฅโŽฅโŽฆ

= expโŽ›โŽœโŽœโŽœโŽœโŽ๐‘ฆ + ๐‘ฆ ๏ฟฝ๐‘ฅ2 + ๐‘ฆ2๏ฟฝ

2 ๏ฟฝ๐‘ฅ2 + ๐‘ฆ2๏ฟฝ

โŽžโŽŸโŽŸโŽŸโŽŸโŽ  cos

โŽ›โŽœโŽœโŽœโŽœโŽ๐‘ฅ โˆ’ ๐‘ฅ ๏ฟฝ๐‘ฅ2 + ๐‘ฆ2๏ฟฝ

2 ๏ฟฝ๐‘ฅ2 + ๐‘ฆ2๏ฟฝ

โŽžโŽŸโŽŸโŽŸโŽŸโŽ  + ๐‘– exp

โŽ›โŽœโŽœโŽœโŽœโŽ๐‘ฆ + ๐‘ฆ ๏ฟฝ๐‘ฅ2 + ๐‘ฆ2๏ฟฝ

2 ๏ฟฝ๐‘ฅ2 + ๐‘ฆ2๏ฟฝ

โŽžโŽŸโŽŸโŽŸโŽŸโŽ  sin

โŽ›โŽœโŽœโŽœโŽœโŽ12๐‘ฅ โˆ’ ๐‘ฅ ๏ฟฝ๐‘ฅ2 + ๐‘ฆ2๏ฟฝ

๏ฟฝ๐‘ฅ2 + ๐‘ฆ2๏ฟฝ

โŽžโŽŸโŽŸโŽŸโŽŸโŽ 

Therefore, since exp (sin ๐‘ง) = ๐‘ข + ๐‘–๐‘ฃ, then we see from above that

๐‘ข = expโŽ›โŽœโŽœโŽœโŽœโŽ12๐‘ฆ + ๐‘ฆ ๏ฟฝ๐‘ฅ2 + ๐‘ฆ2๏ฟฝ

๐‘ฅ2 + ๐‘ฆ2

โŽžโŽŸโŽŸโŽŸโŽŸโŽ  cos

โŽ›โŽœโŽœโŽœโŽœโŽ12๐‘ฅ โˆ’ ๐‘ฅ ๏ฟฝ๐‘ฅ2 + ๐‘ฆ2๏ฟฝ

๐‘ฅ2 + ๐‘ฆ2

โŽžโŽŸโŽŸโŽŸโŽŸโŽ 

๐‘ฃ = expโŽ›โŽœโŽœโŽœโŽœโŽ12๐‘ฆ + ๐‘ฆ ๏ฟฝ๐‘ฅ2 + ๐‘ฆ2๏ฟฝ

๐‘ฅ2 + ๐‘ฆ2

โŽžโŽŸโŽŸโŽŸโŽŸโŽ  sin

โŽ›โŽœโŽœโŽœโŽœโŽ12๐‘ฅ โˆ’ ๐‘ฅ ๏ฟฝ๐‘ฅ2 + ๐‘ฆ2๏ฟฝ

๐‘ฅ2 + ๐‘ฆ2

โŽžโŽŸโŽŸโŽŸโŽŸโŽ 

Now we need to check the Cauchy-Riemann equations on the above ๐‘ข, ๐‘ฃ functions wefound.

๐œ•๐‘ข๐œ•๐‘ฅ

=๐œ•๐‘ฃ๐œ•๐‘ฆ

(1)

โˆ’๐œ•๐‘ข๐œ•๐‘ฆ

=๐œ•๐‘ฃ๐œ•๐‘ฅ

(2)

59

Evaluating each partial derivative gives

๐œ•๐‘ข๐œ•๐‘ฅ

=๐‘‘๐‘‘๐‘ฅ

โŽ›โŽœโŽœโŽœโŽœโŽ12๐‘ฆ + ๐‘ฆ ๏ฟฝ๐‘ฅ2 + ๐‘ฆ2๏ฟฝ

๐‘ฅ2 + ๐‘ฆ2

โŽžโŽŸโŽŸโŽŸโŽŸโŽ  exp

โŽ›โŽœโŽœโŽœโŽœโŽ12๐‘ฆ + ๐‘ฆ ๏ฟฝ๐‘ฅ2 + ๐‘ฆ2๏ฟฝ

๐‘ฅ2 + ๐‘ฆ2

โŽžโŽŸโŽŸโŽŸโŽŸโŽ  cos

โŽ›โŽœโŽœโŽœโŽœโŽ12๐‘ฅ โˆ’ ๐‘ฅ ๏ฟฝ๐‘ฅ2 + ๐‘ฆ2๏ฟฝ

๐‘ฅ2 + ๐‘ฆ2

โŽžโŽŸโŽŸโŽŸโŽŸโŽ 

+ expโŽ›โŽœโŽœโŽœโŽœโŽ12๐‘ฆ + ๐‘ฆ ๏ฟฝ๐‘ฅ2 + ๐‘ฆ2๏ฟฝ

๐‘ฅ2 + ๐‘ฆ2

โŽžโŽŸโŽŸโŽŸโŽŸโŽ ๐‘‘๐‘‘๐‘ฅ

cosโŽ›โŽœโŽœโŽœโŽœโŽ12๐‘ฅ โˆ’ ๐‘ฅ ๏ฟฝ๐‘ฅ2 + ๐‘ฆ2๏ฟฝ

๐‘ฅ2 + ๐‘ฆ2

โŽžโŽŸโŽŸโŽŸโŽŸโŽ 

=122๐‘ฆ๐‘ฅ ๏ฟฝ๐‘ฅ2 + ๐‘ฆ2๏ฟฝ โˆ’ ๏ฟฝ๐‘ฆ + ๐‘ฆ ๏ฟฝ๐‘ฅ2 + ๐‘ฆ2๏ฟฝ๏ฟฝ 2๐‘ฅ

๏ฟฝ๐‘ฅ2 + ๐‘ฆ2๏ฟฝ2 exp

โŽ›โŽœโŽœโŽœโŽœโŽ12๐‘ฆ ๏ฟฝ๏ฟฝ๐‘ฅ2 + ๐‘ฆ2๏ฟฝ + 1๏ฟฝ

๐‘ฅ2 + ๐‘ฆ2

โŽžโŽŸโŽŸโŽŸโŽŸโŽ  cos

โŽ›โŽœโŽœโŽœโŽœโŽ12๐‘ฅ ๏ฟฝ1 โˆ’ ๏ฟฝ๐‘ฅ2 + ๐‘ฆ2๏ฟฝ๏ฟฝ

๐‘ฅ2 + ๐‘ฆ2

โŽžโŽŸโŽŸโŽŸโŽŸโŽ 

โˆ’ expโŽ›โŽœโŽœโŽœโŽœโŽ12๐‘ฆ + ๐‘ฆ ๏ฟฝ๐‘ฅ2 + ๐‘ฆ2๏ฟฝ

๐‘ฅ2 + ๐‘ฆ2

โŽžโŽŸโŽŸโŽŸโŽŸโŽ  sin

โŽ›โŽœโŽœโŽœโŽœโŽ12๐‘ฅ โˆ’ ๐‘ฅ ๏ฟฝ๐‘ฅ2 + ๐‘ฆ2๏ฟฝ

๐‘ฅ2 + ๐‘ฆ2

โŽžโŽŸโŽŸโŽŸโŽŸโŽ ๐‘‘๐‘‘๐‘ฅ

โŽ›โŽœโŽœโŽœโŽœโŽ๐‘ฅ โˆ’ ๐‘ฅ ๏ฟฝ๐‘ฅ2 + ๐‘ฆ2๏ฟฝ

2 ๏ฟฝ๐‘ฅ2 + ๐‘ฆ2๏ฟฝ

โŽžโŽŸโŽŸโŽŸโŽŸโŽ 

=โˆ’๐‘ฅ๐‘ฆ

๏ฟฝ๐‘ฅ2 + ๐‘ฆ2๏ฟฝ2 exp

โŽ›โŽœโŽœโŽœโŽœโŽ12๐‘ฆ ๏ฟฝ๏ฟฝ๐‘ฅ2 + ๐‘ฆ2๏ฟฝ + 1๏ฟฝ

๐‘ฅ2 + ๐‘ฆ2

โŽžโŽŸโŽŸโŽŸโŽŸโŽ  cos

โŽ›โŽœโŽœโŽœโŽœโŽ๐‘ฅ ๏ฟฝ1 โˆ’ ๏ฟฝ๐‘ฅ2 + ๐‘ฆ2๏ฟฝ๏ฟฝ

2 ๏ฟฝ๐‘ฅ2 + ๐‘ฆ2๏ฟฝ

โŽžโŽŸโŽŸโŽŸโŽŸโŽ 

โˆ’ expโŽ›โŽœโŽœโŽœโŽœโŽ๐‘ฆ + ๐‘ฆ ๏ฟฝ๐‘ฅ2 + ๐‘ฆ2๏ฟฝ

2 ๏ฟฝ๐‘ฅ2 + ๐‘ฆ2๏ฟฝ

โŽžโŽŸโŽŸโŽŸโŽŸโŽ  sin

โŽ›โŽœโŽœโŽœโŽœโŽ๐‘ฅ โˆ’ ๐‘ฅ ๏ฟฝ๐‘ฅ2 + ๐‘ฆ2๏ฟฝ

2 ๏ฟฝ๐‘ฅ2 + ๐‘ฆ2๏ฟฝ

โŽžโŽŸโŽŸโŽŸโŽŸโŽ 

โŽ›โŽœโŽœโŽœโŽœโŽœโŽœโŽ๏ฟฝ1 โˆ’ 3๐‘ฅ2 โˆ’ ๐‘ฆ2๏ฟฝ ๏ฟฝ๐‘ฅ2 + ๐‘ฆ2๏ฟฝ โˆ’ ๏ฟฝ๐‘ฅ โˆ’ ๐‘ฅ ๏ฟฝ๐‘ฅ2 + ๐‘ฆ2๏ฟฝ๏ฟฝ 2๐‘ฅ

2 ๏ฟฝ๐‘ฅ2 + ๐‘ฆ2๏ฟฝ2

โŽžโŽŸโŽŸโŽŸโŽŸโŽŸโŽŸโŽ 

=โˆ’๐‘ฅ๐‘ฆ

๏ฟฝ๐‘ฅ2 + ๐‘ฆ2๏ฟฝ2 exp

โŽ›โŽœโŽœโŽœโŽœโŽ12๐‘ฆ ๏ฟฝ๐‘ฅ2 + ๐‘ฆ2 + 1๏ฟฝ

๐‘ฅ2 + ๐‘ฆ2

โŽžโŽŸโŽŸโŽŸโŽŸโŽ  cos

โŽ›โŽœโŽœโŽœโŽœโŽ12๐‘ฅ ๏ฟฝ1 โˆ’ ๏ฟฝ๐‘ฅ2 + ๐‘ฆ2๏ฟฝ๏ฟฝ

๐‘ฅ2 + ๐‘ฆ2

โŽžโŽŸโŽŸโŽŸโŽŸโŽ 

โˆ’ expโŽ›โŽœโŽœโŽœโŽœโŽ12๐‘ฆ ๏ฟฝ๐‘ฅ2 + ๐‘ฆ2 + 1๏ฟฝ

๐‘ฅ2 + ๐‘ฆ2

โŽžโŽŸโŽŸโŽŸโŽŸโŽ  sin

โŽ›โŽœโŽœโŽœโŽœโŽ๐‘ฅ โˆ’ ๐‘ฅ ๏ฟฝ๐‘ฅ2 + ๐‘ฆ2๏ฟฝ

2 ๏ฟฝ๐‘ฅ2 + ๐‘ฆ2๏ฟฝ

โŽžโŽŸโŽŸโŽŸโŽŸโŽ 

โŽ›โŽœโŽœโŽœโŽœโŽœโŽœโŽ๏ฟฝโˆ’๐‘ฅ4 โˆ’ 2๐‘ฅ2๐‘ฆ2 โˆ’ ๐‘ฅ2 โˆ’ ๐‘ฆ4 + ๐‘ฆ2๏ฟฝ

2 ๏ฟฝ๐‘ฅ2 + ๐‘ฆ2๏ฟฝ2

โŽžโŽŸโŽŸโŽŸโŽŸโŽŸโŽŸโŽ 

The above can be simplified more to become

๐œ•๐‘ข๐œ•๐‘ฅ

=โˆ’1

2 ๏ฟฝ๐‘ฅ2 + ๐‘ฆ2๏ฟฝ2 exp

โŽ›โŽœโŽœโŽœโŽœโŽ๐‘ฆ ๏ฟฝ๐‘ฅ2 + ๐‘ฆ2 + 1๏ฟฝ

2 ๏ฟฝ๐‘ฅ2 + ๐‘ฆ2๏ฟฝ

โŽžโŽŸโŽŸโŽŸโŽŸโŽ 

โŽกโŽขโŽขโŽขโŽขโŽฃ2๐‘ฅ๐‘ฆ cos

๐‘ฅ โˆ’ ๐‘ฅ ๏ฟฝ๐‘ฅ2 + ๐‘ฆ2๏ฟฝ

2 ๏ฟฝ๐‘ฅ2 + ๐‘ฆ2๏ฟฝ+ ๏ฟฝโˆ’๐‘ฅ4 โˆ’ 2๐‘ฅ2๐‘ฆ2 โˆ’ ๐‘ฅ2 โˆ’ ๐‘ฆ4 + ๐‘ฆ2๏ฟฝ sin

๐‘ฅ โˆ’ ๐‘ฅ ๏ฟฝ๐‘ฅ2 + ๐‘ฆ2๏ฟฝ

2 ๏ฟฝ๐‘ฅ2 + ๐‘ฆ2๏ฟฝ

โŽคโŽฅโŽฅโŽฅโŽฅโŽฆ (3)

Now we evaluate ๐œ•๐‘ฃ๐œ•๐‘ฆ to see if it the same as above. Since ๐‘ฃ = exp ๏ฟฝ

12๐‘ฆ+๐‘ฆ๏ฟฝ๐‘ฅ2+๐‘ฆ2๏ฟฝ

๏ฟฝ๐‘ฅ2+๐‘ฆ2๏ฟฝ ๏ฟฝ sin ๏ฟฝ12๐‘ฅโˆ’๐‘ฅ๏ฟฝ๐‘ฅ2+๐‘ฆ2๏ฟฝ

๏ฟฝ๐‘ฅ2+๐‘ฆ2๏ฟฝ ๏ฟฝthen

๐œ•๐‘ฃ๐œ•๐‘ฆ

=๐‘‘๐‘‘๐‘ฆ

โŽ›โŽœโŽœโŽœโŽœโŽ๐‘ฆ + ๐‘ฆ ๏ฟฝ๐‘ฅ2 + ๐‘ฆ2๏ฟฝ

2 ๏ฟฝ๐‘ฅ2 + ๐‘ฆ2๏ฟฝ

โŽžโŽŸโŽŸโŽŸโŽŸโŽ  exp

โŽ›โŽœโŽœโŽœโŽœโŽ๐‘ฆ + ๐‘ฆ ๏ฟฝ๐‘ฅ2 + ๐‘ฆ2๏ฟฝ

2 ๏ฟฝ๐‘ฅ2 + ๐‘ฆ2๏ฟฝ

โŽžโŽŸโŽŸโŽŸโŽŸโŽ  sin

โŽ›โŽœโŽœโŽœโŽœโŽ๐‘ฅ โˆ’ ๐‘ฅ ๏ฟฝ๐‘ฅ2 + ๐‘ฆ2๏ฟฝ

2 ๏ฟฝ๐‘ฅ2 + ๐‘ฆ2๏ฟฝ

โŽžโŽŸโŽŸโŽŸโŽŸโŽ 

+ expโŽ›โŽœโŽœโŽœโŽœโŽ๐‘ฆ + ๐‘ฆ ๏ฟฝ๐‘ฅ2 + ๐‘ฆ2๏ฟฝ

2 ๏ฟฝ๐‘ฅ2 + ๐‘ฆ2๏ฟฝ

โŽžโŽŸโŽŸโŽŸโŽŸโŽ  cos

โŽ›โŽœโŽœโŽœโŽœโŽ๐‘ฅ โˆ’ ๐‘ฅ ๏ฟฝ๐‘ฅ2 + ๐‘ฆ2๏ฟฝ

2 ๏ฟฝ๐‘ฅ2 + ๐‘ฆ2๏ฟฝ

โŽžโŽŸโŽŸโŽŸโŽŸโŽ ๐‘‘๐‘‘๐‘ฆ

โŽ›โŽœโŽœโŽœโŽœโŽ๐‘ฅ โˆ’ ๐‘ฅ ๏ฟฝ๐‘ฅ2 + ๐‘ฆ2๏ฟฝ

2 ๏ฟฝ๐‘ฅ2 + ๐‘ฆ2๏ฟฝ

โŽžโŽŸโŽŸโŽŸโŽŸโŽ 

=

โŽ›โŽœโŽœโŽœโŽœโŽœโŽœโŽ12๏ฟฝ1 + ๐‘ฅ2 + 3๐‘ฆ2๏ฟฝ ๏ฟฝ๐‘ฅ2 + ๐‘ฆ2๏ฟฝ โˆ’ ๏ฟฝ๐‘ฆ + ๐‘ฆ ๏ฟฝ๐‘ฅ2 + ๐‘ฆ2๏ฟฝ๏ฟฝ 2๐‘ฆ

๏ฟฝ๐‘ฅ2 + ๐‘ฆ2๏ฟฝ2

โŽžโŽŸโŽŸโŽŸโŽŸโŽŸโŽŸโŽ  exp

โŽ›โŽœโŽœโŽœโŽœโŽ๐‘ฆ + ๐‘ฆ ๏ฟฝ๐‘ฅ2 + ๐‘ฆ2๏ฟฝ

2 ๏ฟฝ๐‘ฅ2 + ๐‘ฆ2๏ฟฝ

โŽžโŽŸโŽŸโŽŸโŽŸโŽ  sin

โŽ›โŽœโŽœโŽœโŽœโŽ๐‘ฅ โˆ’ ๐‘ฅ ๏ฟฝ๐‘ฅ2 + ๐‘ฆ2๏ฟฝ

2 ๏ฟฝ๐‘ฅ2 + ๐‘ฆ2๏ฟฝ

โŽžโŽŸโŽŸโŽŸโŽŸโŽ 

+ expโŽ›โŽœโŽœโŽœโŽœโŽ๐‘ฆ + ๐‘ฆ ๏ฟฝ๐‘ฅ2 + ๐‘ฆ2๏ฟฝ

2 ๏ฟฝ๐‘ฅ2 + ๐‘ฆ2๏ฟฝ

โŽžโŽŸโŽŸโŽŸโŽŸโŽ  cos

โŽ›โŽœโŽœโŽœโŽœโŽ๐‘ฅ โˆ’ ๐‘ฅ ๏ฟฝ๐‘ฅ2 + ๐‘ฆ2๏ฟฝ

2 ๏ฟฝ๐‘ฅ2 + ๐‘ฆ2๏ฟฝ

โŽžโŽŸโŽŸโŽŸโŽŸโŽ 

โŽ›โŽœโŽœโŽœโŽœโŽœโŽœโŽ12๏ฟฝโˆ’2๐‘ฅ๐‘ฆ๏ฟฝ ๏ฟฝ๐‘ฅ2 + ๐‘ฆ2๏ฟฝ โˆ’ ๏ฟฝ๐‘ฅ โˆ’ ๐‘ฅ ๏ฟฝ๐‘ฅ2 + ๐‘ฆ2๏ฟฝ๏ฟฝ ๏ฟฝ2๐‘ฆ๏ฟฝ

๏ฟฝ๐‘ฅ2 + ๐‘ฆ2๏ฟฝ2

โŽžโŽŸโŽŸโŽŸโŽŸโŽŸโŽŸโŽ 

=

โŽ›โŽœโŽœโŽœโŽœโŽœโŽœโŽ12๐‘ฅ4 + 2๐‘ฅ2๐‘ฆ2 + ๐‘ฅ2 + ๐‘ฆ4 โˆ’ ๐‘ฆ2

๏ฟฝ๐‘ฅ2 + ๐‘ฆ2๏ฟฝ2

โŽžโŽŸโŽŸโŽŸโŽŸโŽŸโŽŸโŽ  exp

โŽ›โŽœโŽœโŽœโŽœโŽ๐‘ฆ + ๐‘ฆ ๏ฟฝ๐‘ฅ2 + ๐‘ฆ2๏ฟฝ

2 ๏ฟฝ๐‘ฅ2 + ๐‘ฆ2๏ฟฝ

โŽžโŽŸโŽŸโŽŸโŽŸโŽ  sin

โŽ›โŽœโŽœโŽœโŽœโŽ12๐‘ฅ โˆ’ ๐‘ฅ ๏ฟฝ๐‘ฅ2 + ๐‘ฆ2๏ฟฝ

๐‘ฅ2 + ๐‘ฆ2

โŽžโŽŸโŽŸโŽŸโŽŸโŽ 

+ expโŽ›โŽœโŽœโŽœโŽœโŽ๐‘ฆ + ๐‘ฆ ๏ฟฝ๐‘ฅ2 + ๐‘ฆ2๏ฟฝ

2 ๏ฟฝ๐‘ฅ2 + ๐‘ฆ2๏ฟฝ

โŽžโŽŸโŽŸโŽŸโŽŸโŽ  cos

โŽ›โŽœโŽœโŽœโŽœโŽ๐‘ฅ โˆ’ ๐‘ฅ ๏ฟฝ๐‘ฅ2 + ๐‘ฆ2๏ฟฝ

2 ๏ฟฝ๐‘ฅ2 + ๐‘ฆ2๏ฟฝ

โŽžโŽŸโŽŸโŽŸโŽŸโŽ 

โŽ›โŽœโŽœโŽœโŽœโŽœโŽœโŽ12

โˆ’2๐‘ฅ๐‘ฆ

๏ฟฝ๐‘ฅ2 + ๐‘ฆ2๏ฟฝ2

โŽžโŽŸโŽŸโŽŸโŽŸโŽŸโŽŸโŽ 

=

โŽ›โŽœโŽœโŽœโŽœโŽœโŽœโŽ12๐‘ฅ4 + 2๐‘ฅ2๐‘ฆ2 + ๐‘ฅ2 + ๐‘ฆ4 โˆ’ ๐‘ฆ2

๏ฟฝ๐‘ฅ2 + ๐‘ฆ2๏ฟฝ2

โŽžโŽŸโŽŸโŽŸโŽŸโŽŸโŽŸโŽ  exp

โŽ›โŽœโŽœโŽœโŽœโŽ๐‘ฆ + ๐‘ฆ ๏ฟฝ๐‘ฅ2 + ๐‘ฆ2๏ฟฝ

2 ๏ฟฝ๐‘ฅ2 + ๐‘ฆ2๏ฟฝ

โŽžโŽŸโŽŸโŽŸโŽŸโŽ  sin

โŽ›โŽœโŽœโŽœโŽœโŽ12๐‘ฅ โˆ’ ๐‘ฅ ๏ฟฝ๐‘ฅ2 + ๐‘ฆ2๏ฟฝ

๐‘ฅ2 + ๐‘ฆ2

โŽžโŽŸโŽŸโŽŸโŽŸโŽ 

โˆ’ expโŽ›โŽœโŽœโŽœโŽœโŽ๐‘ฆ + ๐‘ฆ ๏ฟฝ๐‘ฅ2 + ๐‘ฆ2๏ฟฝ

2 ๏ฟฝ๐‘ฅ2 + ๐‘ฆ2๏ฟฝ

โŽžโŽŸโŽŸโŽŸโŽŸโŽ  cos

โŽ›โŽœโŽœโŽœโŽœโŽ๐‘ฅ โˆ’ ๐‘ฅ ๏ฟฝ๐‘ฅ2 + ๐‘ฆ2๏ฟฝ

2 ๏ฟฝ๐‘ฅ2 + ๐‘ฆ2๏ฟฝ

โŽžโŽŸโŽŸโŽŸโŽŸโŽ 

โŽ›โŽœโŽœโŽœโŽœโŽœโŽœโŽ

๐‘ฅ๐‘ฆ

๏ฟฝ๐‘ฅ2 + ๐‘ฆ2๏ฟฝ2

โŽžโŽŸโŽŸโŽŸโŽŸโŽŸโŽŸโŽ 

60

Simplifying the above more gives

๐œ•๐‘ฃ๐œ•๐‘ฆ

=โˆ’1

2 ๏ฟฝ๐‘ฅ2 + ๐‘ฆ2๏ฟฝ2 exp

โŽ›โŽœโŽœโŽœโŽœโŽ๐‘ฆ + ๐‘ฆ ๏ฟฝ๐‘ฅ2 + ๐‘ฆ2๏ฟฝ

2 ๏ฟฝ๐‘ฅ2 + ๐‘ฆ2๏ฟฝ

โŽžโŽŸโŽŸโŽŸโŽŸโŽ 

โŽกโŽขโŽขโŽขโŽขโŽฃ2๐‘ฅ๐‘ฆ cos

๐‘ฅ โˆ’ ๐‘ฅ ๏ฟฝ๐‘ฅ2 + ๐‘ฆ2๏ฟฝ

2 ๏ฟฝ๐‘ฅ2 + ๐‘ฆ2๏ฟฝ+ ๏ฟฝโˆ’๐‘ฅ4 โˆ’ 2๐‘ฅ2๐‘ฆ2 โˆ’ ๐‘ฅ2 โˆ’ ๐‘ฆ4 + ๐‘ฆ2๏ฟฝ sin

๐‘ฅ โˆ’ ๐‘ฅ ๏ฟฝ๐‘ฅ2 + ๐‘ฆ2๏ฟฝ

2 ๏ฟฝ๐‘ฅ2 + ๐‘ฆ2๏ฟฝ

โŽคโŽฅโŽฅโŽฅโŽฅโŽฆ (4)

Comparing (3) and (4) shows they are the same expressions. Therefore the first equationis verified.

๐œ•๐‘ข๐œ•๐‘ฅ

=๐œ•๐‘ฃ๐œ•๐‘ฆ

Now we verify the second equation โˆ’๐œ•๐‘ข๐œ•๐‘ฆ =๐œ•๐‘ฃ๐œ•๐‘ฅ . Since ๐‘ข = exp ๏ฟฝ

12๐‘ฆ+๐‘ฆ๏ฟฝ๐‘ฅ2+๐‘ฆ2๏ฟฝ

๏ฟฝ๐‘ฅ2+๐‘ฆ2๏ฟฝ ๏ฟฝ cos ๏ฟฝ12๐‘ฅโˆ’๐‘ฅ๏ฟฝ๐‘ฅ2+๐‘ฆ2๏ฟฝ

๏ฟฝ๐‘ฅ2+๐‘ฆ2๏ฟฝ ๏ฟฝthen

๐œ•๐‘ข๐œ•๐‘ฆ

=๐‘‘๐‘‘๐‘ฆ

โŽ›โŽœโŽœโŽœโŽœโŽ12๐‘ฆ + ๐‘ฆ ๏ฟฝ๐‘ฅ2 + ๐‘ฆ2๏ฟฝ

๏ฟฝ๐‘ฅ2 + ๐‘ฆ2๏ฟฝ

โŽžโŽŸโŽŸโŽŸโŽŸโŽ  exp

โŽ›โŽœโŽœโŽœโŽœโŽ๐‘ฆ + ๐‘ฆ ๏ฟฝ๐‘ฅ2 + ๐‘ฆ2๏ฟฝ

2 ๏ฟฝ๐‘ฅ2 + ๐‘ฆ2๏ฟฝ

โŽžโŽŸโŽŸโŽŸโŽŸโŽ  cos

โŽ›โŽœโŽœโŽœโŽœโŽ12๐‘ฅ โˆ’ ๐‘ฅ ๏ฟฝ๐‘ฅ2 + ๐‘ฆ2๏ฟฝ

๐‘ฅ2 + ๐‘ฆ2

โŽžโŽŸโŽŸโŽŸโŽŸโŽ 

โˆ’ expโŽ›โŽœโŽœโŽœโŽœโŽ๐‘ฆ + ๐‘ฆ ๏ฟฝ๐‘ฅ2 + ๐‘ฆ2๏ฟฝ

2 ๏ฟฝ๐‘ฅ2 + ๐‘ฆ2๏ฟฝ

โŽžโŽŸโŽŸโŽŸโŽŸโŽ  sin

โŽ›โŽœโŽœโŽœโŽœโŽ12๐‘ฅ โˆ’ ๐‘ฅ ๏ฟฝ๐‘ฅ2 + ๐‘ฆ2๏ฟฝ

๐‘ฅ2 + ๐‘ฆ2

โŽžโŽŸโŽŸโŽŸโŽŸโŽ ๐‘‘๐‘‘๐‘ฆ

โŽ›โŽœโŽœโŽœโŽœโŽ12๐‘ฅ โˆ’ ๐‘ฅ ๏ฟฝ๐‘ฅ2 + ๐‘ฆ2๏ฟฝ

๐‘ฅ2 + ๐‘ฆ2

โŽžโŽŸโŽŸโŽŸโŽŸโŽ 

=๏ฟฝ1 + ๐‘ฅ2 + 3๐‘ฆ2๏ฟฝ ๏ฟฝ๐‘ฅ2 + ๐‘ฆ2๏ฟฝ โˆ’ ๏ฟฝ๐‘ฆ + ๐‘ฆ ๏ฟฝ๐‘ฅ2 + ๐‘ฆ2๏ฟฝ๏ฟฝ 2๐‘ฆ

2 ๏ฟฝ๐‘ฅ2 + ๐‘ฆ2๏ฟฝ2 exp

โŽ›โŽœโŽœโŽœโŽœโŽ๐‘ฆ + ๐‘ฆ ๏ฟฝ๐‘ฅ2 + ๐‘ฆ2๏ฟฝ

2 ๏ฟฝ๐‘ฅ2 + ๐‘ฆ2๏ฟฝ

โŽžโŽŸโŽŸโŽŸโŽŸโŽ  cos

โŽ›โŽœโŽœโŽœโŽœโŽ๐‘ฅ โˆ’ ๐‘ฅ ๏ฟฝ๐‘ฅ2 + ๐‘ฆ2๏ฟฝ

2 ๏ฟฝ๐‘ฅ2 + ๐‘ฆ2๏ฟฝ

โŽžโŽŸโŽŸโŽŸโŽŸโŽ 

โˆ’ expโŽ›โŽœโŽœโŽœโŽœโŽ๐‘ฆ + ๐‘ฆ ๏ฟฝ๐‘ฅ2 + ๐‘ฆ2๏ฟฝ

2 ๏ฟฝ๐‘ฅ2 + ๐‘ฆ2๏ฟฝ

โŽžโŽŸโŽŸโŽŸโŽŸโŽ  sin

โŽ›โŽœโŽœโŽœโŽœโŽ12๐‘ฅ โˆ’ ๐‘ฅ ๏ฟฝ๐‘ฅ2 + ๐‘ฆ2๏ฟฝ

๐‘ฅ2 + ๐‘ฆ2

โŽžโŽŸโŽŸโŽŸโŽŸโŽ ๏ฟฝโˆ’2๐‘ฆ๏ฟฝ ๏ฟฝ๐‘ฅ2 + ๐‘ฆ2๏ฟฝ โˆ’ ๏ฟฝ๐‘ฅ โˆ’ ๐‘ฅ ๏ฟฝ๐‘ฅ2 + ๐‘ฆ2๏ฟฝ๏ฟฝ 2๐‘ฆ

2 ๏ฟฝ๐‘ฅ2 + ๐‘ฆ2๏ฟฝ2

=๏ฟฝ๐‘ฅ4 + 2๐‘ฅ2๐‘ฆ2 + ๐‘ฅ2 + ๐‘ฆ4 โˆ’ ๐‘ฆ2๏ฟฝ

2 ๏ฟฝ๐‘ฅ2 + ๐‘ฆ2๏ฟฝ2 exp

โŽ›โŽœโŽœโŽœโŽœโŽ๐‘ฆ + ๐‘ฆ ๏ฟฝ๐‘ฅ2 + ๐‘ฆ2๏ฟฝ

2 ๏ฟฝ๐‘ฅ2 + ๐‘ฆ2๏ฟฝ

โŽžโŽŸโŽŸโŽŸโŽŸโŽ  cos

โŽ›โŽœโŽœโŽœโŽœโŽ๐‘ฅ โˆ’ ๐‘ฅ ๏ฟฝ๐‘ฅ2 + ๐‘ฆ2๏ฟฝ

2 ๏ฟฝ๐‘ฅ2 + ๐‘ฆ2๏ฟฝ

โŽžโŽŸโŽŸโŽŸโŽŸโŽ 

โˆ’ expโŽ›โŽœโŽœโŽœโŽœโŽ๐‘ฆ + ๐‘ฆ ๏ฟฝ๐‘ฅ2 + ๐‘ฆ2๏ฟฝ

2 ๏ฟฝ๐‘ฅ2 + ๐‘ฆ2๏ฟฝ

โŽžโŽŸโŽŸโŽŸโŽŸโŽ  sin

โŽ›โŽœโŽœโŽœโŽœโŽ12๐‘ฅ โˆ’ ๐‘ฅ ๏ฟฝ๐‘ฅ2 + ๐‘ฆ2๏ฟฝ

๐‘ฅ2 + ๐‘ฆ2

โŽžโŽŸโŽŸโŽŸโŽŸโŽ ๏ฟฝโˆ’2๐‘ฆ๏ฟฝ ๏ฟฝ๐‘ฅ2 + ๐‘ฆ2๏ฟฝ โˆ’ 2๐‘ฆ๐‘ฅ + 2๐‘ฆ๐‘ฅ ๏ฟฝ๐‘ฅ2 + ๐‘ฆ2๏ฟฝ

2 ๏ฟฝ๐‘ฅ2 + ๐‘ฆ2๏ฟฝ2

=๏ฟฝ๐‘ฅ4 + 2๐‘ฅ2๐‘ฆ2 + ๐‘ฅ2 + ๐‘ฆ4 โˆ’ ๐‘ฆ2๏ฟฝ

2 ๏ฟฝ๐‘ฅ2 + ๐‘ฆ2๏ฟฝ2 exp

โŽ›โŽœโŽœโŽœโŽœโŽ๐‘ฆ + ๐‘ฆ ๏ฟฝ๐‘ฅ2 + ๐‘ฆ2๏ฟฝ

2 ๏ฟฝ๐‘ฅ2 + ๐‘ฆ2๏ฟฝ

โŽžโŽŸโŽŸโŽŸโŽŸโŽ  cos

โŽ›โŽœโŽœโŽœโŽœโŽ12๐‘ฅ โˆ’ ๐‘ฅ ๏ฟฝ๐‘ฅ2 + ๐‘ฆ2๏ฟฝ

๐‘ฅ2 + ๐‘ฆ2

โŽžโŽŸโŽŸโŽŸโŽŸโŽ 

+ expโŽ›โŽœโŽœโŽœโŽœโŽ๐‘ฆ + ๐‘ฆ ๏ฟฝ๐‘ฅ2 + ๐‘ฆ2๏ฟฝ

2 ๏ฟฝ๐‘ฅ2 + ๐‘ฆ2๏ฟฝ

โŽžโŽŸโŽŸโŽŸโŽŸโŽ  sin

โŽ›โŽœโŽœโŽœโŽœโŽ12๐‘ฅ โˆ’ ๐‘ฅ ๏ฟฝ๐‘ฅ2 + ๐‘ฆ2๏ฟฝ

๐‘ฅ2 + ๐‘ฆ2

โŽžโŽŸโŽŸโŽŸโŽŸโŽ 

๐‘ฆ๐‘ฅ

๏ฟฝ๐‘ฅ2 + ๐‘ฆ2๏ฟฝ2

The above can simplified more to give

๐œ•๐‘ข๐œ•๐‘ฆ

=1

2 ๏ฟฝ๐‘ฅ2 + ๐‘ฆ2๏ฟฝ2 exp

โŽ›โŽœโŽœโŽœโŽœโŽ๐‘ฆ + ๐‘ฆ ๏ฟฝ๐‘ฅ2 + ๐‘ฆ2๏ฟฝ

2 ๏ฟฝ๐‘ฅ2 + ๐‘ฆ2๏ฟฝ

โŽžโŽŸโŽŸโŽŸโŽŸโŽ 

โŽกโŽขโŽขโŽขโŽขโŽฃ๏ฟฝ๐‘ฅ

4 + 2๐‘ฅ2๐‘ฆ2 + ๐‘ฅ2 + ๐‘ฆ4 โˆ’ ๐‘ฆ2๏ฟฝ cos๐‘ฅ โˆ’ ๐‘ฅ ๏ฟฝ๐‘ฅ2 + ๐‘ฆ2๏ฟฝ

2 ๏ฟฝ๐‘ฅ2 + ๐‘ฆ2๏ฟฝ+ 2๐‘ฅ๐‘ฆ sin

๐‘ฅ โˆ’ ๐‘ฅ ๏ฟฝ๐‘ฅ2 + ๐‘ฆ2๏ฟฝ

2 ๏ฟฝ๐‘ฅ2 + ๐‘ฆ2๏ฟฝ

โŽคโŽฅโŽฅโŽฅโŽฅโŽฆ

Hence

โˆ’๐œ•๐‘ข๐œ•๐‘ฆ

=1

2 ๏ฟฝ๐‘ฅ2 + ๐‘ฆ2๏ฟฝ2 exp

โŽ›โŽœโŽœโŽœโŽœโŽ๐‘ฆ + ๐‘ฆ ๏ฟฝ๐‘ฅ2 + ๐‘ฆ2๏ฟฝ

2 ๏ฟฝ๐‘ฅ2 + ๐‘ฆ2๏ฟฝ

โŽžโŽŸโŽŸโŽŸโŽŸโŽ 

โŽกโŽขโŽขโŽขโŽขโŽฃโˆ’ ๏ฟฝ๐‘ฅ

4 + 2๐‘ฅ2๐‘ฆ2 + ๐‘ฅ2 + ๐‘ฆ4 โˆ’ ๐‘ฆ2๏ฟฝ cos๐‘ฅ โˆ’ ๐‘ฅ ๏ฟฝ๐‘ฅ2 + ๐‘ฆ2๏ฟฝ

2 ๏ฟฝ๐‘ฅ2 + ๐‘ฆ2๏ฟฝโˆ’ 2๐‘ฅ๐‘ฆ sin

๐‘ฅ โˆ’ ๐‘ฅ ๏ฟฝ๐‘ฅ2 + ๐‘ฆ2๏ฟฝ

2 ๏ฟฝ๐‘ฅ2 + ๐‘ฆ2๏ฟฝ

โŽคโŽฅโŽฅโŽฅโŽฅโŽฆ (5)

61

And since ๐‘ฃ = exp ๏ฟฝ12๐‘ฆ+๐‘ฆ๏ฟฝ๐‘ฅ2+๐‘ฆ2๏ฟฝ

๏ฟฝ๐‘ฅ2+๐‘ฆ2๏ฟฝ ๏ฟฝ sin ๏ฟฝ12๐‘ฅโˆ’๐‘ฅ๏ฟฝ๐‘ฅ2+๐‘ฆ2๏ฟฝ

๏ฟฝ๐‘ฅ2+๐‘ฆ2๏ฟฝ ๏ฟฝ then

๐œ•๐‘ฃ๐œ•๐‘ฅ

=๐‘‘๐‘‘๐‘ฅ

โŽ›โŽœโŽœโŽœโŽœโŽ12๐‘ฆ + ๐‘ฆ ๏ฟฝ๐‘ฅ2 + ๐‘ฆ2๏ฟฝ

๐‘ฅ2 + ๐‘ฆ2

โŽžโŽŸโŽŸโŽŸโŽŸโŽ  exp

โŽ›โŽœโŽœโŽœโŽœโŽ๐‘ฆ + ๐‘ฆ ๏ฟฝ๐‘ฅ2 + ๐‘ฆ2๏ฟฝ

2 ๏ฟฝ๐‘ฅ2 + ๐‘ฆ2๏ฟฝ

โŽžโŽŸโŽŸโŽŸโŽŸโŽ  sin

โŽ›โŽœโŽœโŽœโŽœโŽ12๐‘ฅ โˆ’ ๐‘ฅ ๏ฟฝ๐‘ฅ2 + ๐‘ฆ2๏ฟฝ

๐‘ฅ2 + ๐‘ฆ2

โŽžโŽŸโŽŸโŽŸโŽŸโŽ 

+ expโŽ›โŽœโŽœโŽœโŽœโŽ๐‘ฆ + ๐‘ฆ ๏ฟฝ๐‘ฅ2 + ๐‘ฆ2๏ฟฝ

2 ๏ฟฝ๐‘ฅ2 + ๐‘ฆ2๏ฟฝ

โŽžโŽŸโŽŸโŽŸโŽŸโŽ  cos

โŽ›โŽœโŽœโŽœโŽœโŽ12๐‘ฅ โˆ’ ๐‘ฅ ๏ฟฝ๐‘ฅ2 + ๐‘ฆ2๏ฟฝ

๐‘ฅ2 + ๐‘ฆ2

โŽžโŽŸโŽŸโŽŸโŽŸโŽ ๐‘‘๐‘‘๐‘ฅ

โŽ›โŽœโŽœโŽœโŽœโŽ12๐‘ฅ โˆ’ ๐‘ฅ ๏ฟฝ๐‘ฅ2 + ๐‘ฆ2๏ฟฝ

๐‘ฅ2 + ๐‘ฆ2

โŽžโŽŸโŽŸโŽŸโŽŸโŽ 

=12

โŽ›โŽœโŽœโŽœโŽœโŽœโŽœโŽ2๐‘ฅ๐‘ฆ ๏ฟฝ๐‘ฅ2 + ๐‘ฆ2๏ฟฝ โˆ’ ๏ฟฝ๐‘ฆ + ๐‘ฆ ๏ฟฝ๐‘ฅ2 + ๐‘ฆ2๏ฟฝ๏ฟฝ 2๐‘ฅ

๏ฟฝ๐‘ฅ2 + ๐‘ฆ2๏ฟฝ2

โŽžโŽŸโŽŸโŽŸโŽŸโŽŸโŽŸโŽ  exp

โŽ›โŽœโŽœโŽœโŽœโŽ๐‘ฆ + ๐‘ฆ ๏ฟฝ๐‘ฅ2 + ๐‘ฆ2๏ฟฝ

2 ๏ฟฝ๐‘ฅ2 + ๐‘ฆ2๏ฟฝ

โŽžโŽŸโŽŸโŽŸโŽŸโŽ  sin

โŽ›โŽœโŽœโŽœโŽœโŽ12๐‘ฅ โˆ’ ๐‘ฅ ๏ฟฝ๐‘ฅ2 + ๐‘ฆ2๏ฟฝ

๐‘ฅ2 + ๐‘ฆ2

โŽžโŽŸโŽŸโŽŸโŽŸโŽ 

+ expโŽ›โŽœโŽœโŽœโŽœโŽ๐‘ฆ + ๐‘ฆ ๏ฟฝ๐‘ฅ2 + ๐‘ฆ2๏ฟฝ

2 ๏ฟฝ๐‘ฅ2 + ๐‘ฆ2๏ฟฝ

โŽžโŽŸโŽŸโŽŸโŽŸโŽ  cos

โŽ›โŽœโŽœโŽœโŽœโŽ12๐‘ฅ โˆ’ ๐‘ฅ ๏ฟฝ๐‘ฅ2 + ๐‘ฆ2๏ฟฝ

๐‘ฅ2 + ๐‘ฆ2

โŽžโŽŸโŽŸโŽŸโŽŸโŽ 

โŽ›โŽœโŽœโŽœโŽœโŽœโŽœโŽ๏ฟฝ1 โˆ’ 3๐‘ฅ2 โˆ’ ๐‘ฆ2๏ฟฝ ๏ฟฝ๐‘ฅ2 + ๐‘ฆ2๏ฟฝ โˆ’ ๏ฟฝ๐‘ฅ โˆ’ ๐‘ฅ ๏ฟฝ๐‘ฅ2 + ๐‘ฆ2๏ฟฝ๏ฟฝ 2๐‘ฅ

2 ๏ฟฝ๐‘ฅ2 + ๐‘ฆ2๏ฟฝ2

โŽžโŽŸโŽŸโŽŸโŽŸโŽŸโŽŸโŽ 

=โˆ’๐‘ฅ๐‘ฆ

๏ฟฝ๐‘ฅ2 + ๐‘ฆ2๏ฟฝ2 exp

โŽ›โŽœโŽœโŽœโŽœโŽ12๐‘ฆ + ๐‘ฆ ๏ฟฝ๐‘ฅ2 + ๐‘ฆ2๏ฟฝ

๐‘ฅ2 + ๐‘ฆ2

โŽžโŽŸโŽŸโŽŸโŽŸโŽ  sin

โŽ›โŽœโŽœโŽœโŽœโŽ๐‘ฅ โˆ’ ๐‘ฅ ๏ฟฝ๐‘ฅ2 + ๐‘ฆ2๏ฟฝ

2 ๏ฟฝ๐‘ฅ2 + ๐‘ฆ2๏ฟฝ

โŽžโŽŸโŽŸโŽŸโŽŸโŽ 

+ expโŽ›โŽœโŽœโŽœโŽœโŽ๐‘ฆ + ๐‘ฆ ๏ฟฝ๐‘ฅ2 + ๐‘ฆ2๏ฟฝ

2 ๏ฟฝ๐‘ฅ2 + ๐‘ฆ2๏ฟฝ

โŽžโŽŸโŽŸโŽŸโŽŸโŽ  cos

โŽ›โŽœโŽœโŽœโŽœโŽ๐‘ฅ โˆ’ ๐‘ฅ ๏ฟฝ๐‘ฅ2 + ๐‘ฆ2๏ฟฝ

2 ๏ฟฝ๐‘ฅ2 + ๐‘ฆ2๏ฟฝ

โŽžโŽŸโŽŸโŽŸโŽŸโŽ 

โŽ›โŽœโŽœโŽœโŽœโŽœโŽœโŽโˆ’๐‘ฅ4 โˆ’ 2๐‘ฅ2๐‘ฆ2 โˆ’ ๐‘ฅ2 โˆ’ ๐‘ฆ4 + ๐‘ฆ2

2 ๏ฟฝ๐‘ฅ2 + ๐‘ฆ2๏ฟฝ2

โŽžโŽŸโŽŸโŽŸโŽŸโŽŸโŽŸโŽ 

The above can simplified more to give

๐œ•๐‘ฃ๐œ•๐‘ฅ

=1

2 ๏ฟฝ๐‘ฅ2 + ๐‘ฆ2๏ฟฝ2 exp

โŽ›โŽœโŽœโŽœโŽœโŽ๐‘ฆ + ๐‘ฆ ๏ฟฝ๐‘ฅ2 + ๐‘ฆ2๏ฟฝ

2 ๏ฟฝ๐‘ฅ2 + ๐‘ฆ2๏ฟฝ

โŽžโŽŸโŽŸโŽŸโŽŸโŽ 

โŽกโŽขโŽขโŽขโŽขโŽฃโˆ’ ๏ฟฝ๐‘ฅ

4 + 2๐‘ฅ2๐‘ฆ2 + ๐‘ฅ2 + ๐‘ฆ4 โˆ’ ๐‘ฆ2๏ฟฝ cos๐‘ฅ โˆ’ ๐‘ฅ ๏ฟฝ๐‘ฅ2 + ๐‘ฆ2๏ฟฝ

2 ๏ฟฝ๐‘ฅ2 + ๐‘ฆ2๏ฟฝโˆ’ 2๐‘ฅ๐‘ฆ sin

๐‘ฅ โˆ’ ๐‘ฅ ๏ฟฝ๐‘ฅ2 + ๐‘ฆ2๏ฟฝ

2 ๏ฟฝ๐‘ฅ2 + ๐‘ฆ2๏ฟฝ

โŽคโŽฅโŽฅโŽฅโŽฅโŽฆ (6)

Comparing (5,6) shows they are the same, i.e.

โˆ’๐œ•๐‘ข๐œ•๐‘ฆ

=๐œ•๐‘ฃ๐œ•๐‘ฅ

C-R equations are satisfied, and because it is clear that all partial derivatives ๐œ•๐‘ฃ๐œ•๐‘ฅ ,

๐œ•๐‘ฃ๐œ•๐‘ฆ ,

๐œ•๐‘ข๐œ•๐‘ฅ ,

๐œ•๐‘ข๐œ•๐‘ฆ

are continuous functions in ๐‘ฅ, ๐‘ฆ as they are made up of exponential and trigonometricfunctions which are continuous, then we conclude that ๐‘“ (๐‘ง) = ๐‘’sin ๐‘ง is analytic functioneverywhere.

3.3.2 Problem 2

Part (a)

Represent ๐‘ง+3๐‘งโˆ’3 by its Maclaurin series and give the region of validity for the representation.

Next expand this in powers of 1๐‘ง to find a Laurent series. What is the range of validity of

the Laurent series?

Solution

Maclaurin series is expansion of ๐‘“ (๐‘ง) around ๐‘ง = 0. Since ๐‘“ (๐‘ง) has simple pole at ๐‘ง = 3,then the region of validity will be a disk centered at ๐‘ง = 0 up to the nearest pole, which isat ๐‘ง = 3. Hence |๐‘ง| < 3 is the region.

๐‘“ (๐‘ง) =๐‘ง + 3๐‘ง โˆ’ 3

=๐‘ง + 3

โˆ’3 ๏ฟฝ1 โˆ’ ๐‘ง3๏ฟฝ

=๐‘ง + 3โˆ’3

โŽ›โŽœโŽœโŽœโŽœโŽ

11 โˆ’ ๐‘ง

3

โŽžโŽŸโŽŸโŽŸโŽŸโŽ 

62

Now we can expand using Binomial to obtain

๐‘“ (๐‘ง) =3 + ๐‘งโˆ’3 ๏ฟฝ1 +

๐‘ง3+ ๏ฟฝ

๐‘ง3๏ฟฝ2+ ๏ฟฝ

๐‘ง3๏ฟฝ3+โ‹ฏ๏ฟฝ

= ๏ฟฝโˆ’1 โˆ’๐‘ง3๏ฟฝ ๏ฟฝ1 +

๐‘ง3+ ๏ฟฝ

๐‘ง3๏ฟฝ2+ ๏ฟฝ

๐‘ง3๏ฟฝ3+โ‹ฏ๏ฟฝ

= ๏ฟฝโˆ’1 โˆ’๐‘ง3๏ฟฝ + ๏ฟฝโˆ’1 โˆ’

๐‘ง3๏ฟฝ ๏ฟฝ๐‘ง3๏ฟฝ + ๏ฟฝโˆ’1 โˆ’

๐‘ง3๏ฟฝ ๏ฟฝ๐‘ง3๏ฟฝ2+ ๏ฟฝโˆ’1 โˆ’

๐‘ง3๏ฟฝ ๏ฟฝ๐‘ง3๏ฟฝ3+โ‹ฏ

= โˆ’1 โˆ’๐‘ง3โˆ’๐‘ง3โˆ’ ๏ฟฝ

๐‘ง3๏ฟฝ2โˆ’ ๏ฟฝ

๐‘ง3๏ฟฝ2โˆ’ ๏ฟฝ

๐‘ง3๏ฟฝ3โˆ’ ๏ฟฝ

๐‘ง3๏ฟฝ3โˆ’ ๏ฟฝ

๐‘ง3๏ฟฝ4+โ‹ฏ

= โˆ’1 โˆ’23๐‘ง โˆ’

29๐‘ง2 โˆ’

227๐‘ง3 โˆ’

281๐‘ง4 โˆ’โ‹ฏ

Or

๐‘“ (๐‘ง) = โˆ’1 โˆ’โˆž๏ฟฝ๐‘›=1

23๐‘›๐‘ง๐‘›

To expand in negative powers of ๐‘ง, or in 1๐‘ง , then

๐‘“ (๐‘ง) =๐‘ง + 3

๐‘ง ๏ฟฝ1 โˆ’ 3๐‘ง๏ฟฝ

=๐‘ง + 3๐‘ง

โŽ›โŽœโŽœโŽœโŽœโŽœโŽ

11 โˆ’ 3

๐‘ง

โŽžโŽŸโŽŸโŽŸโŽŸโŽŸโŽ 

For ๏ฟฝ3๐‘ง ๏ฟฝ < 1 or |๐‘ง| < 3 the above becomes

๐‘“ (๐‘ง) =๐‘ง + 3๐‘ง

โŽ›โŽœโŽœโŽœโŽœโŽ1 +

3๐‘ง+ ๏ฟฝ

3๐‘ง๏ฟฝ

2

+ ๏ฟฝ3๐‘ง๏ฟฝ

3

+โ‹ฏโŽžโŽŸโŽŸโŽŸโŽŸโŽ 

= ๏ฟฝ1 +3๐‘ง๏ฟฝ

โŽ›โŽœโŽœโŽœโŽœโŽ1 +

3๐‘ง+ ๏ฟฝ

3๐‘ง๏ฟฝ

2

+ ๏ฟฝ3๐‘ง๏ฟฝ

3

+โ‹ฏโŽžโŽŸโŽŸโŽŸโŽŸโŽ 

= ๏ฟฝ1 +3๐‘ง๏ฟฝ+ ๏ฟฝ1 +

3๐‘ง๏ฟฝ3๐‘ง+ ๏ฟฝ1 +

3๐‘ง๏ฟฝ ๏ฟฝ

3๐‘ง ๏ฟฝ

2

+ ๏ฟฝ1 +3๐‘ง๏ฟฝ ๏ฟฝ

3๐‘ง ๏ฟฝ

3

+โ‹ฏ

= 1 +3๐‘ง+3๐‘ง+ ๏ฟฝ

3๐‘ง๏ฟฝ

2

+ ๏ฟฝ3๐‘ง๏ฟฝ

2

+ ๏ฟฝ3๐‘ง๏ฟฝ

3

+ ๏ฟฝ3๐‘ง๏ฟฝ

3

+ ๏ฟฝ3๐‘ง๏ฟฝ

4

+โ‹ฏ

= 1 +6๐‘ง+18๐‘ง2+54๐‘ง3+โ‹ฏ

This is valid for |๐‘ง| > 3. The residue is 6, which can be confirmed using

Residue (3) = lim๐‘งโ†’3

(๐‘ง โˆ’ 3) ๐‘“ (๐‘ง)

= lim๐‘งโ†’3

(๐‘ง โˆ’ 3)๐‘ง + 3๐‘ง โˆ’ 3

= lim๐‘งโ†’3

(๐‘ง + 3)

= 6

Summary

๐‘“ (๐‘ง) = ๐‘ง+3๐‘งโˆ’3 =

โŽงโŽชโŽชโŽจโŽชโŽชโŽฉโˆ’1 โˆ’ 2

3๐‘ง โˆ’29๐‘ง2 โˆ’ 2

27๐‘ง3 โˆ’ 2

81๐‘ง4 โˆ’โ‹ฏ |๐‘ง| < 3

1 + 6๐‘ง +

18๐‘ง2 +

54๐‘ง3 +โ‹ฏ |๐‘ง| > 3

Part (b)

Find Laurent series for ๐‘ง(๐‘ง+1)(๐‘งโˆ’3) in each of the following domains (i) |๐‘ง| < 1 (ii) 1 < |๐‘ง| < 3

(iii) |๐‘ง| > 3

Solution

The possible region are shown below. Since there is a pole at ๐‘ง = โˆ’1 and pole at ๐‘ง = 3, thenthere are three di๏ฟฝerent regions. They are named ๐ด,๐ต, ๐ถ in the following diagram

63

โˆ’1 3A

B

C

Region A contains no poles inside.Disk centered at zero up to thenearest pole at z = โˆ’1. Hence theseries expansion will contain onlyan analytical part and no principalpart.

Region B is annulus regionbetween region which is analytic upto the next pole at z = 3. Hencethe series expansion will containboth an analytical part and aprincipal part.

Region C is |z| > 3, Hence theseries expansion will contain only aprincipal part and no analyticalpart.

<z

=z

Figure 3.4: Laurent series regions

First the expression ๐‘ง(๐‘ง+1)(๐‘งโˆ’3) is expanded using partial fractions

๐‘ง(๐‘ง + 1) (๐‘ง โˆ’ 3)

=๐ด

(๐‘ง + 1)+

๐ต(๐‘ง โˆ’ 3)

(1)

Hence

๐‘ง = ๐ด (๐‘ง โˆ’ 3) + ๐ต (๐‘ง + 1)= ๐‘ง (๐ด + ๐ต) โˆ’ 3๐ด + ๐ต

The above gives two equations

๐ด + ๐ต = 10 = โˆ’3๐ด + ๐ต

First equation gives ๐ด = 1 โˆ’ ๐ต. Substituting in the second equation gives 0 = โˆ’3 (1 โˆ’ ๐ต) + ๐ตor 0 = โˆ’3 + 4๐ต, hence ๐ต = 3

4 , which implies ๐ด = 1 โˆ’ 34 =

14 , therefore (1) becomes

๐‘ง(๐‘ง + 1) (๐‘ง โˆ’ 3)

=14

1(๐‘ง + 1)

+34

1(๐‘ง โˆ’ 3)

Considering each term in turn. For 14

1(๐‘ง+1) , we can expand this as

14

1(๐‘ง + 1)

=14๏ฟฝ1 โˆ’ ๐‘ง + ๐‘ง2 โˆ’ ๐‘ง3 + ๐‘ง4 +โ‹ฏ๏ฟฝ |๐‘ง| < 1 (2a)

14

1(๐‘ง + 1)

=14๐‘ง

1

๏ฟฝ1 + 1๐‘ง๏ฟฝ=14๐‘ง

โŽ›โŽœโŽœโŽœโŽœโŽ1 โˆ’ ๏ฟฝ

1๐‘ง๏ฟฝ+ ๏ฟฝ

1๐‘ง๏ฟฝ

2

โˆ’ ๏ฟฝ1๐‘ง๏ฟฝ

3

+ ๏ฟฝ1๐‘ง๏ฟฝ

4

โˆ’โ‹ฏโŽžโŽŸโŽŸโŽŸโŽŸโŽ  |๐‘ง| > 1 (2b)

And for the term 34

1(๐‘งโˆ’3) , we can expand this as

34

1(๐‘ง โˆ’ 3)

= โˆ’14

1๏ฟฝ1 โˆ’ ๐‘ง

3๏ฟฝ= โˆ’

14 ๏ฟฝ1 +

๐‘ง3+ ๏ฟฝ

๐‘ง3๏ฟฝ2+ ๏ฟฝ

๐‘ง3๏ฟฝ3+ ๏ฟฝ

๐‘ง3๏ฟฝ4+โ‹ฏ๏ฟฝ |๐‘ง| < 3 (3a)

34

1(๐‘ง โˆ’ 3)

=34๐‘ง

1

๏ฟฝ1 โˆ’ 3๐‘ง๏ฟฝ=34๐‘ง

โŽ›โŽœโŽœโŽœโŽœโŽ1 + ๏ฟฝ

3๐‘ง๏ฟฝ+ ๏ฟฝ

3๐‘ง๏ฟฝ

2

+ ๏ฟฝ3๐‘ง๏ฟฝ

3

+โ‹ฏโŽžโŽŸโŽŸโŽŸโŽŸโŽ  |๐‘ง| > 3 (3b)

Now that we expanded all the terms in the two possible ways for each each, we nowconsider each region of interest, and look at the above 4 expansions, and simply pick foreach region the expansion which is valid in for that region of interest.

For (i), region ๐ด: In this region, we want |๐‘ง| < 1. From (2,3) we see that (2a) and (3a) are

64

valid expansions in |๐‘ง| < 1. Hence

๐‘ง(๐‘ง + 1) (๐‘ง โˆ’ 3)

=14๏ฟฝ1 โˆ’ ๐‘ง + ๐‘ง2 โˆ’ ๐‘ง3 + ๐‘ง4 +โ‹ฏ๏ฟฝ โˆ’

14 ๏ฟฝ1 +

๐‘ง3+ ๏ฟฝ

๐‘ง3๏ฟฝ2+ ๏ฟฝ

๐‘ง3๏ฟฝ3+ ๏ฟฝ

๐‘ง3๏ฟฝ4+โ‹ฏ๏ฟฝ

=14๏ฟฝ1 โˆ’ ๐‘ง + ๐‘ง2 โˆ’ ๐‘ง3 + ๐‘ง4 โˆ’โ‹ฏ๏ฟฝ โˆ’

14 ๏ฟฝ1 +

๐‘ง3+๐‘ง2

9+๐‘ง3

27+๐‘ง4

81+โ‹ฏ๏ฟฝ

= ๏ฟฝ14โˆ’14๐‘ง +

14๐‘ง2 โˆ’

14๐‘ง3 +

14๐‘ง4 โˆ’โ‹ฏ๏ฟฝ โˆ’ ๏ฟฝ

14+๐‘ง12+๐‘ง2

36+๐‘ง3

108+๐‘ง4

324+โ‹ฏ๏ฟฝ

= โˆ’14๐‘ง โˆ’

๐‘ง12+14๐‘ง2 โˆ’

๐‘ง2

36โˆ’14๐‘ง3 โˆ’

๐‘ง3

108+14๐‘ง4 โˆ’

๐‘ง4

324โˆ’โ‹ฏ

โˆ’13๐‘ง +

29๐‘ง2 โˆ’

727๐‘ง3 +

2081๐‘ง4 โˆ’โ‹ฏ

For (ii), region ๐ต: This is for 1 < |๐‘ง| < 3. From equations (2,3) we see that (2b) and (3a) arevalid in this region. Hence

๐‘ง(๐‘ง + 1) (๐‘ง โˆ’ 3)

=14๐‘ง

โŽ›โŽœโŽœโŽœโŽœโŽ1 โˆ’ ๏ฟฝ

1๐‘ง๏ฟฝ+ ๏ฟฝ

1๐‘ง๏ฟฝ

2

โˆ’ ๏ฟฝ1๐‘ง๏ฟฝ

3

+ ๏ฟฝ1๐‘ง๏ฟฝ

4

โˆ’โ‹ฏโŽžโŽŸโŽŸโŽŸโŽŸโŽ  โˆ’

14 ๏ฟฝ1 +

๐‘ง3+ ๏ฟฝ

๐‘ง3๏ฟฝ2+ ๏ฟฝ

๐‘ง3๏ฟฝ3+ ๏ฟฝ

๐‘ง3๏ฟฝ4+โ‹ฏ๏ฟฝ

=14๐‘ง ๏ฟฝ

1 โˆ’1๐‘ง+1๐‘ง2โˆ’1๐‘ง3+1๐‘ง4โˆ’โ‹ฏ๏ฟฝ โˆ’

14 ๏ฟฝ1 +

๐‘ง3+๐‘ง2

9+๐‘ง3

27+๐‘ง4

81+โ‹ฏ๏ฟฝ

= ๏ฟฝ14๐‘งโˆ’

14๐‘ง2

+14๐‘ง3

โˆ’14๐‘ง4

+14๐‘ง5

โˆ’โ‹ฏ๏ฟฝ โˆ’ ๏ฟฝ14+๐‘ง12+๐‘ง2

36+๐‘ง3

108+๐‘ง4

324+โ‹ฏ๏ฟฝ

=

principal part

๏ฟฝ๏ฟฝ๏ฟฝ๏ฟฝ๏ฟฝ๏ฟฝ๏ฟฝ๏ฟฝ๏ฟฝ๏ฟฝ๏ฟฝ๏ฟฝ๏ฟฝ๏ฟฝ๏ฟฝ๏ฟฝ๏ฟฝ๏ฟฝ๏ฟฝ๏ฟฝ๏ฟฝ๏ฟฝ๏ฟฝ๏ฟฝ๏ฟฝ๏ฟฝ๏ฟฝ๏ฟฝ๏ฟฝ๏ฟฝ๏ฟฝ๏ฟฝ๏ฟฝ๏ฟฝ๏ฟฝ๏ฟฝ๏ฟฝ๏ฟฝ๏ฟฝ๏ฟฝ๏ฟฝ๏ฟฝ๏ฟฝ๏ฟฝ๏ฟฝ๏ฟฝ๏ฟฝโ‹ฏ +

14๐‘ง5

โˆ’14๐‘ง4

+14๐‘ง3

โˆ’14๐‘ง2

+14๐‘งโˆ’

analytical part

๏ฟฝ๏ฟฝ๏ฟฝ๏ฟฝ๏ฟฝ๏ฟฝ๏ฟฝ๏ฟฝ๏ฟฝ๏ฟฝ๏ฟฝ๏ฟฝ๏ฟฝ๏ฟฝ๏ฟฝ๏ฟฝ๏ฟฝ๏ฟฝ๏ฟฝ๏ฟฝ๏ฟฝ๏ฟฝ๏ฟฝ๏ฟฝ๏ฟฝ๏ฟฝ๏ฟฝ๏ฟฝ๏ฟฝ๏ฟฝ๏ฟฝ๏ฟฝ๏ฟฝ๏ฟฝ๏ฟฝ๏ฟฝ๏ฟฝ๏ฟฝ๏ฟฝ๏ฟฝ๏ฟฝ14โˆ’๐‘ง12โˆ’๐‘ง2

36โˆ’๐‘ง3

108โˆ’๐‘ง4

324โˆ’โ‹ฏ

The residue is 14 by looking at the above. The value for the residue can be verified as

follows. Using

๐‘๐‘› =12๐œ‹๐‘–โˆฎ

๐ถ

๐‘“ (๐‘ง)(๐‘ง โˆ’ ๐‘ง0)

โˆ’๐‘›+1๐‘‘๐‘ง

Where in the above ๐‘ง0 is the location of the pole and ๐‘› is the coe๏ฟฝcient of the 1๐‘ง๐‘› is the

principal part. Since we want the residue, then ๐‘› = 1 and the above becomes

๐‘1 =12๐œ‹๐‘–โˆฎ

๐ถ

๐‘“ (๐‘ง) ๐‘‘๐‘ง

In the above, the contour ๐ถ is circle somewhere inside the annulus 1 < |๐‘ง| < 3. It does notmatter that the radius is, as long as it is located in this range. For example, choosing radius2 will work. The above then becomes

๐‘1 =12๐œ‹๐‘–โˆฎ

๐ถ

๐‘ง(๐‘ง + 1) (๐‘ง โˆ’ 3)

๐‘‘๐‘ง (5)

However, since ๐‘“ (๐‘ง) is analytic in this region, then โˆฎ๐ถ

๐‘“ (๐‘ง) ๐‘‘๐‘ง = 2๐œ‹๐‘–โˆ‘ (residues inside).

There is only one pole now inside ๐ถ, which is at ๐‘ง = โˆ’1. So all what we have to do is findthe residue at ๐‘ง = โˆ’1.

Residue (โˆ’1) = lim๐‘งโ†’โˆ’1

(๐‘ง + 1) ๐‘“ (๐‘ง)

= lim๐‘งโ†’โˆ’1

(๐‘ง + 1)๐‘ง

(๐‘ง + 1) (๐‘ง โˆ’ 3)

= lim๐‘งโ†’โˆ’1

๐‘ง(๐‘ง โˆ’ 3)

=โˆ’1

(โˆ’1 โˆ’ 3)

=14

65

Using this in (5) gives

๐‘1 =12๐œ‹๐‘– ๏ฟฝ

2๐œ‹๐‘–14๏ฟฝ

=14

Which agrees with what we found in (4) above.

For (iii), region ๐ถ: This is for |๐‘ง| > 3. From (2,3) we see that (2b) and (3b) are validexpansions in ๐‘ง > 3, Hence

๐‘ง(๐‘ง + 1) (๐‘ง โˆ’ 3)

=14๐‘ง

โŽ›โŽœโŽœโŽœโŽœโŽ1 โˆ’ ๏ฟฝ

1๐‘ง๏ฟฝ+ ๏ฟฝ

1๐‘ง๏ฟฝ

2

โˆ’ ๏ฟฝ1๐‘ง๏ฟฝ

3

+ ๏ฟฝ1๐‘ง๏ฟฝ

4

โˆ’โ‹ฏโŽžโŽŸโŽŸโŽŸโŽŸโŽ  +

34๐‘ง

โŽ›โŽœโŽœโŽœโŽœโŽ1 + ๏ฟฝ

3๐‘ง๏ฟฝ+ ๏ฟฝ

3๐‘ง๏ฟฝ

2

+ ๏ฟฝ3๐‘ง๏ฟฝ

3

+โ‹ฏโŽžโŽŸโŽŸโŽŸโŽŸโŽ 

= ๏ฟฝ14๐‘งโˆ’

14๐‘ง2

+14๐‘ง3

โˆ’14๐‘ง4

+14๐‘ง5

โˆ’โ‹ฏ๏ฟฝ +34๐‘ง ๏ฟฝ

1 +3๐‘ง+9๐‘ง2+27๐‘ง3+โ‹ฏ๏ฟฝ

= ๏ฟฝ14๐‘งโˆ’

14๐‘ง2

+14๐‘ง3

โˆ’14๐‘ง4

+14๐‘ง5

โˆ’โ‹ฏ๏ฟฝ + ๏ฟฝ34๐‘ง+

94๐‘ง2

+274๐‘ง3

+814๐‘ง4

+โ‹ฏ๏ฟฝ

= โ‹ฏ+20๐‘ง4+7๐‘ง3+2๐‘ง2+1๐‘ง

This is as expected contains only a principal part and no analytical part. The residue is 1.This above value for the residue can be verified as follows. Using

๐‘๐‘› =12๐œ‹๐‘–โˆฎ

๐ถ

๐‘“ (๐‘ง)(๐‘ง โˆ’ ๐‘ง0)

โˆ’๐‘›+1๐‘‘๐‘ง

Where in the above ๐‘ง0 is the location of the pole and ๐‘› is the coe๏ฟฝcient of the 1๐‘ง๐‘› is the

principal part. Since we want the residue, then ๐‘› = 1 and the above becomes

๐‘1 =12๐œ‹๐‘–โˆฎ

๐ถ

๐‘“ (๐‘ง) ๐‘‘๐‘ง

In the above, the contour ๐ถ is circle somewhere in |๐‘ง| > 3. It does not matter that the radiusis. The above integral then becomes

๐‘1 =12๐œ‹๐‘–โˆฎ

๐ถ

๐‘ง(๐‘ง + 1) (๐‘ง โˆ’ 3)

๐‘‘๐‘ง (7)

However, since ๐‘“ (๐‘ง) is analytic in |๐‘ง| > 3, then โˆฎ๐ถ

๐‘“ (๐‘ง) ๐‘‘๐‘ง = 2๐œ‹๐‘–โˆ‘ (residues inside). There

are now two poles inside ๐ถ, one at ๐‘ง = โˆ’1 and one at ๐‘ง = 3. So all what we have to do isfind the residues at each. We found earlier that Residue (โˆ’1) = 1

4 . Now

Residue (3) = lim๐‘งโ†’3

(๐‘ง โˆ’ 3) ๐‘“ (๐‘ง)

= lim๐‘งโ†’3

(๐‘ง โˆ’ 3)๐‘ง

(๐‘ง + 1) (๐‘ง โˆ’ 3)

= lim๐‘งโ†’3

๐‘ง(๐‘ง + 1)

=34

Therefore the sum of residues is 1. Using this result in (7) gives

๐‘1 =12๐œ‹๐‘– ๏ฟฝ

2๐œ‹๐‘– ๏ฟฝ14+34๏ฟฝ๏ฟฝ

= 1

Which agrees with what result from (6) above.

Summary of results

๐‘“ (๐‘ง) = ๐‘ง(๐‘ง+1)(๐‘งโˆ’3) =

โŽงโŽชโŽชโŽชโŽชโŽจโŽชโŽชโŽชโŽชโŽฉ

โˆ’13๐‘ง +29๐‘ง2 โˆ’ 7

27๐‘ง3 + 20

81๐‘ง4 โˆ’โ‹ฏ |๐‘ง| < 1

โ‹ฏ + 14๐‘ง5 โˆ’

14๐‘ง4

+ 14๐‘ง3 โˆ’

14๐‘ง2 +

14๐‘ง โˆ’

14 โˆ’

๐‘ง12 โˆ’

๐‘ง2

36 โˆ’๐‘ง3

108 โˆ’๐‘ง4

324 โˆ’โ‹ฏ 1 < |๐‘ง| < 3โ‹ฏ + 20

๐‘ง4+ 7

๐‘ง3 +2๐‘ง2 +

1๐‘ง |๐‘ง| > 3

66

3.3.3 Problem 3

Part (a)

Use residue theorem to evaluate โˆฎ๐ถ

๐‘’โˆ’2๐‘ง

๐‘ง2 ๐‘‘๐‘ง on contour ๐ถ which is circle |๐‘ง| = 1 in positive

sense.

Solution

For ๐‘“ (๐‘ง) which is analytic on and inside ๐ถ, the Cauchy integral formula says

โˆฎ๐ถ

๐‘“ (๐‘ง) ๐‘‘๐‘ง = 2๐œ‹๐‘–๏ฟฝ๐‘—

Residue ๏ฟฝ๐‘ง = ๐‘ง๐‘—๏ฟฝ (1)

Where the sum is over all residues located inside ๐ถ. for ๐‘“ (๐‘ง) = ๐‘’โˆ’2๐‘ง

๐‘ง2 there is a simple pole at๐‘ง = 0 of order 2. To find the residue, we use the formula for pole or order ๐‘š given by

Residue (๐‘ง0) = lim๐‘งโ†’๐‘ง0

๐‘‘๐‘šโˆ’1

๐‘‘๐‘ง๐‘šโˆ’1(๐‘ง โˆ’ ๐‘ง0)

๐‘š

(๐‘š โˆ’ 1)!๐‘“ (๐‘ง)

Hence for ๐‘š = 2 and ๐‘ง0 = 0 the above becomes

Residue (0) = lim๐‘งโ†’0

๐‘‘๐‘‘๐‘ง๐‘ง2๐‘“ (๐‘ง)

= lim๐‘งโ†’0

๐‘‘๐‘‘๐‘ง๐‘ง2๐‘’โˆ’2๐‘ง

๐‘ง2

= lim๐‘งโ†’0

๐‘‘๐‘‘๐‘ง๐‘’โˆ’2๐‘ง

= lim๐‘งโ†’0

๏ฟฝโˆ’2๐‘’โˆ’2๐‘ง๏ฟฝ

= โˆ’2

Therefore (1) becomes

โˆฎ๐ถ

๐‘’โˆ’2๐‘ง

๐‘ง2๐‘‘๐‘ง = 2๐œ‹๐‘– (โˆ’2)

= โˆ’4๐œ‹๐‘–

Part (b)

Use residue theorem to evaluate โˆฎ๐ถ

๐‘ง๐‘’1๐‘ง ๐‘‘๐‘ง on contour ๐ถ which is circle |๐‘ง| = 1 in positive

sense.

Solution

The singularity is at ๐‘ง = 0, but we can not use the simple pole residue finding method here,

since this is an essential singularity now due to the ๐‘’1๐‘ง term. To find the residue, we expand

๐‘“ (๐‘ง) around ๐‘ง = 0 in Laurent series and look for the coe๏ฟฝcient of 1๐‘ง term.

๐‘“ (๐‘ง) = ๐‘ง๐‘’1๐‘ง

= ๐‘ง ๏ฟฝ1 +1๐‘ง+121๐‘ง2+13!1๐‘ง3+โ‹ฏ๏ฟฝ

= ๐‘ง + 1 +121๐‘ง+13!1๐‘ง2+โ‹ฏ

Hence residue is 12 . Therefore

โˆฎ๐ถ

๐‘ง๐‘’1๐‘ง ๐‘‘๐‘ง = 2๐œ‹๐‘– ๏ฟฝ

12๏ฟฝ

= ๐œ‹๐‘–

67

Part (c)

Use residue theorem to evaluate โˆฎ๐ถ

๐‘ง+2๐‘ง2โˆ’ ๐‘ง

2๐‘‘๐‘ง on contour ๐ถ which is circle |๐‘ง| = 1 in positive

sense.

Solution

๐‘“ (๐‘ง) =๐‘ง + 2๐‘ง2 โˆ’ ๐‘ง

2

=๐‘ง + 2

๐‘ง ๏ฟฝ๐‘ง โˆ’ 12๏ฟฝ

Hence there is a simple pole at ๐‘ง = 0 and simple pole at ๐‘ง = 12

Residue (0) = lim๐‘งโ†’0

(๐‘ง) ๐‘“ (๐‘ง)

= lim๐‘งโ†’0

๐‘ง๐‘ง + 2

๐‘ง ๏ฟฝ๐‘ง โˆ’ 12๏ฟฝ

= lim๐‘งโ†’0

๐‘ง + 2

๏ฟฝ๐‘ง โˆ’ 12๏ฟฝ

=2โˆ’12

= โˆ’4

And

Residue ๏ฟฝ12๏ฟฝ= lim๐‘งโ†’ 1

2

๏ฟฝ๐‘ง โˆ’12๏ฟฝ๐‘“ (๐‘ง)

= lim๐‘งโ†’ 1

2

๏ฟฝ๐‘ง โˆ’12๏ฟฝ

๐‘ง + 2

๐‘ง ๏ฟฝ๐‘ง โˆ’ 12๏ฟฝ

= lim๐‘งโ†’ 1

2

๐‘ง + 2๐‘ง

=12 + 212

= 5

Therefore

โˆฎ๐ถ

๐‘ง + 2๐‘ง2 โˆ’ ๐‘ง

2

๐‘‘๐‘ง = 2๐œ‹๐‘– (5 โˆ’ 4)

= 2๐œ‹๐‘–

68

3.3.4 key solution to HW 3

69

70

71

72

3.4 HW 4

3.4.1 Problem 1

Using series expansion evaluate the integral ๐ผ = โˆซ1

0ln ๏ฟฝ1+๐‘ฅ1โˆ’๐‘ฅ

๏ฟฝ ๐‘‘๐‘ฅ๐‘ฅ

Solution

73

We first need to find the Taylor series for ln ๏ฟฝ1+๐‘ฅ1โˆ’๐‘ฅ๏ฟฝ expanded around ๐‘ฅ = 0. Since

ln ๏ฟฝ1 + ๐‘ฅ1 โˆ’ ๐‘ฅ๏ฟฝ

= ln ๏ฟฝ(1 + ๐‘ฅ) ๏ฟฝ1

1 โˆ’ ๐‘ฅ๏ฟฝ๏ฟฝ

= ln (1 + ๐‘ฅ) + ln ๏ฟฝ1

1 โˆ’ ๐‘ฅ๏ฟฝ

= ln (1 + ๐‘ฅ) โˆ’ ln (1 โˆ’ ๐‘ฅ) (1)

Looking at ln (1 + ๐‘ฅ), where now ๐‘“ (๐‘ฅ) = ln (1 + ๐‘ฅ), then we see that ๐‘“โ€ฒ (๐‘ฅ) = 11+๐‘ฅ , ๐‘“

โ€ฒโ€ฒ (๐‘ฅ) =โˆ’1

(1+๐‘ฅ)2, ๐‘“โ€ฒโ€ฒโ€ฒ (๐‘ฅ) = 2

(1+๐‘ฅ)3, ๐‘“(4) (๐‘ฅ) = โˆ’ 2โ‹…3

(1+๐‘ฅ)4,โ‹ฏ, therefore

ln (1 + ๐‘ฅ) = ๐‘“ (0) + ๐‘ฅ๐‘“โ€ฒ (0) + ๐‘ฅ2

2๐‘“โ€ฒโ€ฒ (0) +

๐‘ฅ3

3!๐‘“โ€ฒโ€ฒโ€ฒ (0) +

๐‘ฅ4

4!๐‘“(4) (0) +โ‹ฏ

= 0 + ๐‘ฅ โˆ’๐‘ฅ2

2+๐‘ฅ3

3โˆ’๐‘ฅ4

4+โ‹ฏ (2)

Similarly for ln (1 โˆ’ ๐‘ฅ), where now ๐‘“โ€ฒ (๐‘ฅ) = โˆ’11โˆ’๐‘ฅ , ๐‘“

โ€ฒโ€ฒ (๐‘ฅ) = โˆ’1(1โˆ’๐‘ฅ)2

, ๐‘“โ€ฒโ€ฒโ€ฒ (๐‘ฅ) = โˆ’2(1โˆ’๐‘ฅ)3

, ๐‘“(4) (๐‘ฅ) =

โˆ’ 2โ‹…3(1โˆ’๐‘ฅ)4

,โ‹ฏ, therefore

ln (1 โˆ’ ๐‘ฅ) = ๐‘“ (0) + ๐‘ฅ๐‘“โ€ฒ (0) + ๐‘ฅ2

2๐‘“โ€ฒโ€ฒ (0) +

๐‘ฅ3

3!๐‘“โ€ฒโ€ฒโ€ฒ (0) +

๐‘ฅ4

4!๐‘“(4) (0) +โ‹ฏ

= 0 โˆ’ ๐‘ฅ โˆ’๐‘ฅ2

2โˆ’๐‘ฅ3

3โˆ’๐‘ฅ4

4+โ‹ฏ (3)

Using (2,3) in (1) gives the series expansion for ln ๏ฟฝ1+๐‘ฅ1โˆ’๐‘ฅ๏ฟฝ as

ln ๏ฟฝ1 + ๐‘ฅ1 โˆ’ ๐‘ฅ๏ฟฝ

= ๏ฟฝ๐‘ฅ โˆ’๐‘ฅ2

2+๐‘ฅ3

3โˆ’๐‘ฅ4

4+โ‹ฏ๏ฟฝ โˆ’ ๏ฟฝโˆ’๐‘ฅ โˆ’

๐‘ฅ2

2โˆ’๐‘ฅ3

3โˆ’๐‘ฅ4

4+โ‹ฏ๏ฟฝ

= 2๐‘ฅ +23๐‘ฅ3 +

25๐‘ฅ5 +

27๐‘ฅ7 +โ‹ฏ (4)

Using (4) in the integral given results in

๐ผ = ๏ฟฝ1

0๏ฟฝ2๐‘ฅ +

23๐‘ฅ3 +

25๐‘ฅ5 +

27๐‘ฅ7 +โ‹ฏ๏ฟฝ

๐‘‘๐‘ฅ๐‘ฅ

= ๏ฟฝ1

0๏ฟฝ2 +

23๐‘ฅ2 +

25๐‘ฅ4 +

27๐‘ฅ6 +โ‹ฏ๏ฟฝ๐‘‘๐‘ฅ

= ๏ฟฝ2๐‘ฅ +23๐‘ฅ3

3+25๐‘ฅ5

5+27๐‘ฅ7

7+โ‹ฏ๏ฟฝ

1

0

Which simplifies to

๐ผ = 2 +2313+2515+2717+โ‹ฏ

= 2 +232+252+272+292+โ‹ฏ

= 2 ๏ฟฝ1 +132+152+172+192+โ‹ฏ๏ฟฝ

= 2โˆž๏ฟฝ๐‘›=0

1(2๐‘› + 1)2

(5)

The following are two methods to obtain closed form sum for โˆ‘โˆž๐‘›=0

1(2๐‘›+1)2

. The first method

is based on writingโˆž๏ฟฝ๐‘›=1

1๐‘›2

=โˆž๏ฟฝ๐‘›=1

1(2๐‘›)2

+โˆž๏ฟฝ๐‘›=0

1(2๐‘› + 1)2

(6)

Where the sum on the left is broken into odd and even terms on the right, as in

1 +122+132+142+152+โ‹ฏ = ๏ฟฝ

122+142+โ‹ฏ๏ฟฝ + ๏ฟฝ

112+132+152+โ‹ฏ๏ฟฝ

But, from lecture Sept. 12, 2018, we showed in class thatโˆž๏ฟฝ๐‘›=1

1๐‘›2

= ๐œ (2) =๐œ‹2

6(7)

74

(This is called the Basel problem, and the above closed form sum was first given by Eulerin 1734). Now using (7) into (6) results in

โˆž๏ฟฝ๐‘›=0

1(2๐‘› + 1)2

=โˆž๏ฟฝ๐‘›=1

1๐‘›2โˆ’

โˆž๏ฟฝ๐‘›=1

1(2๐‘›)2

=โˆž๏ฟฝ๐‘›=1

1๐‘›2โˆ’14

โˆž๏ฟฝ๐‘›=1

1๐‘›2

=34 ๏ฟฝ

โˆž๏ฟฝ๐‘›=1

1๐‘›2 ๏ฟฝ

=34 ๏ฟฝ๐œ‹2

6 ๏ฟฝ

=๐œ‹2

8Another way to obtained closed form sum for โˆ‘โˆž

๐‘›=01

(2๐‘›+1)2is to use Fourier series. Consid-

ering the Fourier series for the following periodic function

๐‘“ (๐‘ฅ) =

โŽงโŽชโŽชโŽจโŽชโŽชโŽฉโˆ’๐‘ฅ โˆ’๐œ‹ < ๐‘ฅ < 00 0 โ‰ค ๐‘ฅ โ‰ค ๐œ‹

Using

๐‘“ (๐‘ฅ) =๐ด02+

โˆž๏ฟฝ๐‘›=1

๐ด๐‘› cos (๐‘›๐‘ฅ) +โˆž๏ฟฝ๐‘›=1

๐ต๐‘› sin (๐‘›๐‘ฅ)

Therefore

๐ด0 =1๐œ‹ ๏ฟฝ

0

โˆ’๐œ‹โˆ’๐‘ฅ๐‘‘๐‘ฅ =

โˆ’1๐œ‹ ๏ฟฝ

๐‘ฅ2

2 ๏ฟฝ0

โˆ’๐œ‹=โˆ’12๐œ‹

๏ฟฝ๐‘ฅ2๏ฟฝ0

โˆ’๐œ‹=โˆ’12๐œ‹

๏ฟฝโˆ’๐œ‹2๏ฟฝ =12๐œ‹

And

๐ด๐‘› =โˆ’1๐œ‹ ๏ฟฝ

0

โˆ’๐œ‹๐‘ฅ cos (๐‘›๐‘ฅ) ๐‘‘๐‘ฅ = 1 + (โˆ’1)๐‘›+1

๐‘›2

๐ต๐‘› =โˆ’1๐œ‹ ๏ฟฝ

0

โˆ’๐œ‹๐‘ฅ sin (๐‘›๐‘ฅ) ๐‘‘๐‘ฅ =

(โˆ’1)๐‘›+1

๐‘›๐œ‹

Hence the Fourier series for ๐‘“ (๐‘ฅ) is

๐‘“ (๐‘ฅ) =๐œ‹4โˆ’1๐œ‹

โˆž๏ฟฝ๐‘›=1

1 + (โˆ’1)๐‘›+1

๐‘›2cos (๐‘›๐‘ฅ) โˆ’ 1

๐œ‹

โˆž๏ฟฝ๐‘›=0

(โˆ’1)๐‘›+1

๐‘›๐œ‹ (sin ๐‘›๐‘ฅ)

=๐œ‹4โˆ’1๐œ‹

โˆž๏ฟฝ๐‘›=1

1 + (โˆ’1)๐‘›+1

๐‘›2cos (๐‘›๐‘ฅ) โˆ’

โˆž๏ฟฝ๐‘›=0

(โˆ’1)๐‘›+1

๐‘›sin (๐‘›๐‘ฅ)

Evaluating the above at ๐‘ฅ = 0 then all the sin terms vanish and we obtain

0 =๐œ‹4โˆ’1๐œ‹

โˆž๏ฟฝ๐‘›=1

1 + (โˆ’1)๐‘›+1

๐‘›2

=๐œ‹4โˆ’2๐œ‹ ๏ฟฝ

1 +132+152+172+โ‹ฏ๏ฟฝ

=๐œ‹4โˆ’2๐œ‹

โˆž๏ฟฝ๐‘›=0

1(2๐‘› + 1)2

Therefore2๐œ‹

โˆž๏ฟฝ๐‘›=0

1(2๐‘› + 1)2

=๐œ‹4

โˆž๏ฟฝ๐‘›=0

1(2๐‘› + 1)2

=๐œ‹2

8

Now that we found closed form sum for โˆ‘โˆž๐‘›=0

1(2๐‘›+1)2

, we can find the value of the integral.

75

Since ๐ผ = 2โˆ‘โˆž๐‘›=0

1(2๐‘›+1)2

, then

๏ฟฝ1

0ln ๏ฟฝ

1 + ๐‘ฅ1 โˆ’ ๐‘ฅ๏ฟฝ

๐‘‘๐‘ฅ๐‘ฅ= 2 ๏ฟฝ

๐œ‹2

8 ๏ฟฝ

=๐œ‹2

4

3.4.2 Problem 2

Let ๐ผ (๐‘ฅ) = โˆซโˆž

0๐‘’๐‘ฅ๐‘“(๐‘ก)๐‘‘๐‘ก with ๐‘“ (๐‘ก) = ๐‘ก โˆ’ ๐‘’๐‘ก

๐‘ฅ , find a large ๐‘ฅ approximation for this integral.

Solution

๐ผ = ๏ฟฝโˆž

0exp ๏ฟฝ๐‘ฅ๐‘“ (๐‘ก)๏ฟฝ ๐‘‘๐‘ก

= ๏ฟฝโˆž

0exp ๏ฟฝ๐‘ฅ ๏ฟฝ๐‘ก โˆ’

๐‘’๐‘ก

๐‘ฅ ๏ฟฝ๏ฟฝ๐‘‘๐‘ก

= ๏ฟฝโˆž

0exp ๏ฟฝ๐‘ฅ๐‘ก โˆ’ ๐‘’๐‘ก๏ฟฝ ๐‘‘๐‘ก

= ๏ฟฝโˆž

0exp (๐น (๐‘ก)) ๐‘‘๐‘ก (1)

Where ๐น (๐‘ก) = ๐‘ฅ๐‘ก โˆ’ ๐‘’๐‘ก. We need to find saddle point where ๐น (๐‘ก) is maximum. Hence๐‘‘๐‘‘๐‘ก๐น (๐‘ก) = 0

๐‘ฅ โˆ’ ๐‘’๐‘ก = 0๐‘’๐‘ก = ๐‘ฅ๐‘ก0 = ln (๐‘ฅ)

Where ๐‘ก0 is location of ๐‘ก where ๐น (๐‘ก) is maximum. We called this in class ๐‘ก๐‘๐‘’๐‘Ž๐‘˜. We nowexpand ๐น (๐‘ก) around ๐‘ก0 using Taylor series

๐น (๐‘ก) = ๐น (๐‘ก0) + ๐นโ€ฒ (๐‘ก0) (๐‘ก โˆ’ ๐‘ก0) +12๐นโ€ฒโ€ฒ (๐‘ก0) (๐‘ก โˆ’ ๐‘ก0)

2 +โ‹ฏ (2)

But

๐น (๐‘ก0) = ๐‘ฅ ln (๐‘ฅ) โˆ’ ๐‘’ln ๐‘ฅ

= ๐‘ฅ ln ๐‘ฅ โˆ’ ๐‘ฅAnd ๐นโ€ฒ (๐‘ก) = ๐‘ฅ โˆ’ ๐‘’๐‘ก, hence as expected ๐นโ€ฒ (๐‘ก0) = 0. And ๐นโ€ฒโ€ฒ (๐‘ก) = โˆ’๐‘’๐‘ก, therefore ๐นโ€ฒโ€ฒ (๐‘ก0) = โˆ’๐‘’ln ๐‘ฅ =โˆ’๐‘ฅ. We see also that ๐นโ€ฒโ€ฒ (๐‘ก0) < 0, which means the saddle point was a maximum and not aminimum (since ๐‘ฅ is positive). Using these in (2) gives

๐น (๐‘ก) โ‰ˆ (๐‘ฅ ln ๐‘ฅ โˆ’ ๐‘ฅ) + 12(โˆ’๐‘ฅ) (๐‘ก โˆ’ ln ๐‘ฅ)2

= ๐‘ฅ ln ๐‘ฅ โˆ’ ๐‘ฅ โˆ’ 12๐‘ฅ (๐‘ก โˆ’ ln ๐‘ฅ)2

Substituting the above into (1) gives

๐ผ = ๏ฟฝโˆž

0exp ๏ฟฝ๐‘ฅ ln ๐‘ฅ โˆ’ ๐‘ฅ โˆ’ 1

2๐‘ฅ (๐‘ก โˆ’ ln ๐‘ฅ)2๏ฟฝ ๐‘‘๐‘ก

= ๏ฟฝโˆž

0exp (๐‘ฅ ln ๐‘ฅ) exp (โˆ’๐‘ฅ) exp ๏ฟฝโˆ’

12๐‘ฅ (๐‘ก โˆ’ ln ๐‘ฅ)2๏ฟฝ ๐‘‘๐‘ก

= exp (๐‘ฅ ln ๐‘ฅ) exp (โˆ’๐‘ฅ)๏ฟฝโˆž

0exp ๏ฟฝโˆ’

12๐‘ฅ (๐‘ก โˆ’ ln ๐‘ฅ)2๏ฟฝ ๐‘‘๐‘ก

= ๐‘ฅ๐‘ฅ๐‘’โˆ’๐‘ฅ๏ฟฝโˆž

0๐‘’โˆ’

12๐‘ฅ(๐‘กโˆ’ln ๐‘ฅ)2๐‘‘๐‘ก (3)

Now, since the peak value where ๐น (๐‘ก) occurs is on the positive real axis, because ๐‘ก0 = ln (๐‘ฅ),therefore ๐‘ฅ > 1 to have a maximum, and assuming a narrow peak, then all the contribution

to the integral comes from ๐‘ฅ close to the peak location, so we can change โˆซโˆž

0๐‘’โˆ’

12๐‘ฅ(๐‘กโˆ’ln ๐‘ฅ)2๐‘‘๐‘ก

76

to โˆซโˆž

โˆ’โˆž๐‘’โˆ’

12๐‘ฅ(๐‘กโˆ’ln ๐‘ฅ)2๐‘‘๐‘ก without a๏ฟฝecting the final result. Therefore (3) becomes

๐ผ = ๐‘ฅ๐‘ฅ๐‘’โˆ’๐‘ฅ๏ฟฝโˆž

โˆ’โˆž๐‘’โˆ’

12๐‘ฅ(๐‘กโˆ’ln ๐‘ฅ)2๐‘‘๐‘ก (4)

Now comparing โˆซโˆž

โˆ’โˆž๐‘’โˆ’

12๐‘ฅ(๐‘กโˆ’ln ๐‘ฅ)2๐‘‘๐‘ก to the Gaussian integral โˆซ

โˆž

โˆ’โˆž๐‘’โˆ’๐‘Ž(๐‘กโˆ’๐‘)

2๐‘‘๐‘ก = ๏ฟฝ

๐œ‹๐‘Ž , shows that

๐‘Ž = ๐‘ฅ2 for our case. Hence

๏ฟฝโˆž

โˆ’โˆž๐‘’โˆ’

12๐‘ฅ(๐‘กโˆ’ln ๐‘ฅ)2๐‘‘๐‘ก =

๏ฟฝ2๐œ‹๐‘ฅ

Therefore (4) becomes

๐ผ โ‰ˆ ๐‘ฅ๐‘ฅ๐‘’โˆ’๐‘ฅ๏ฟฝ2๐œ‹๐‘ฅ

For large ๐‘ฅ.

3.4.3 Problem 3

Evaluate the following integrals with aid of residue theorem ๐‘Ž โ‰ฅ 0. (a) โˆซโˆž

01

๐‘ฅ4+1๐‘‘๐‘ฅ (b)

โˆซโˆž0

cos(๐‘Ž๐‘ฅ)๐‘ฅ2+1 ๐‘‘๐‘ฅ

Part (a)

Since the integrand is even, then

๐ผ =12 ๏ฟฝ

โˆž

โˆ’โˆž

1๐‘ฅ4 + 1

๐‘‘๐‘ฅ

Now we consider the following contour

โˆ’R +R

R

CR

<z

=z

Figure 3.5: contour used for problem 3

Therefore

โˆฎ๐ถ

๐‘“ (๐‘ง) ๐‘‘๐‘ง = ๏ฟฝ lim๐‘…โ†’โˆž

๏ฟฝ0

โˆ’๐‘…๐‘“ (๐‘ฅ) ๐‘‘๐‘ฅ + lim

๏ฟฝ๏ฟฝโ†’โˆž๏ฟฝ

๏ฟฝ๏ฟฝ

0๐‘“ (๐‘ฅ) ๐‘‘๐‘ฅ๏ฟฝ + lim

๐‘…โ†’โˆž๏ฟฝ๐ถ๐‘…๐‘“ (๐‘ง) ๐‘‘๐‘ง

Using Cauchy principal value the integral above can be written as

โˆฎ๐ถ

๐‘“ (๐‘ง) ๐‘‘๐‘ง = lim๐‘…โ†’โˆž

๏ฟฝ๐‘…

โˆ’๐‘…๐‘“ (๐‘ฅ) ๐‘‘๐‘ฅ + lim

๐‘…โ†’โˆž๏ฟฝ๐ถ๐‘…๐‘“ (๐‘ง) ๐‘‘

= 2๐œ‹๐‘–๏ฟฝResidue

Whereโˆ‘Residue is sum of residues of 1๐‘ง4+1

for poles that are inside the contour ๐ถ. Thereforethe above becomes

lim๐‘…โ†’โˆž

๏ฟฝ๐‘…

โˆ’๐‘…๐‘“ (๐‘ฅ) ๐‘‘๐‘ฅ = 2๐œ‹๐‘–๏ฟฝResidueโˆ’ lim

๐‘…โ†’โˆž๏ฟฝ๐ถ๐‘…๐‘“ (๐‘ง) ๐‘‘๐‘ง

๏ฟฝโˆž

โˆ’โˆž

1๐‘ฅ4 + 1

๐‘‘๐‘ฅ = 2๐œ‹๐‘–๏ฟฝResidueโˆ’ lim๐‘…โ†’โˆž

๏ฟฝ๐ถ๐‘…

1๐‘ง4 + 1

๐‘‘๐‘ง (1)

77

Now we will show that lim๐‘…โ†’โˆžโˆซ๐ถ๐‘…1

๐‘ง4+1๐‘‘๐‘ง = 0. Since

๏ฟฝ๏ฟฝ๐ถ๐‘…

1๐‘ง4 + 1

๐‘‘๐‘ง๏ฟฝ โ‰ค ๐‘€๐ฟ

= ๏ฟฝ๐‘“ (๐‘ง)๏ฟฝmax

(๐œ‹๐‘…) (2)

But

๐‘“ (๐‘ง) =1

๏ฟฝ๐‘ง2 โˆ’ ๐‘–๏ฟฝ ๏ฟฝ๐‘ง2 + ๐‘–๏ฟฝ

Hence, and since ๐‘ง = ๐‘… ๐‘’๐‘–๐œƒ then

๏ฟฝ๐‘“ (๐‘ง)๏ฟฝmax

โ‰ค1

๏ฟฝ๐‘ง2 โˆ’ ๐‘–๏ฟฝmin

๏ฟฝ๐‘ง2 + ๐‘–๏ฟฝmin

But but inverse triangle inequality ๏ฟฝ๐‘ง2 โˆ’ ๐‘–๏ฟฝ โ‰ฅ |๐‘ง|2 + 1 and ๏ฟฝ๐‘ง2 + ๐‘–๏ฟฝ โ‰ฅ |๐‘ง|2 โˆ’ 1, and since |๐‘ง| = ๐‘…then the above becomes

๏ฟฝ๐‘“ (๐‘ง)๏ฟฝmax

โ‰ค1

๏ฟฝ๐‘…2 + 1๏ฟฝ ๏ฟฝ๐‘…2 โˆ’ 1๏ฟฝ

=1

๐‘…4 โˆ’ 1Therefore (2) becomes

๏ฟฝ๏ฟฝ๐ถ๐‘…

1๐‘ง4 + 1

๐‘‘๐‘ง๏ฟฝ โ‰ค๐œ‹๐‘…๐‘…4 โˆ’ 1

Then it is clear that as ๐‘… โ†’ โˆž the above goes to zero since lim๐‘…โ†’โˆž๐œ‹๐‘…๐‘…4โˆ’1

= lim๐‘…โ†’โˆž

๐œ‹๐‘…3

1โˆ’ 1๐‘…4

=

01 = 0. Then (1) now simplifies to

๏ฟฝโˆž

โˆ’โˆž

1๐‘ฅ4 + 1

๐‘‘๐‘ฅ = 2๐œ‹๐‘–๏ฟฝResidue (2A)

We just now need to find the residues of 1๐‘ง4+1

located in upper half plane. The zeros of

the denominator ๐‘ง4 + 1 = 0 are at ๐‘ง = โˆ’114 = ๏ฟฝ๐‘’๐‘–๐œ‹๏ฟฝ

14 , then the first zero is at ๐‘’๐‘–

๐œ‹4 , and the

second zero at ๐‘’๐‘–๏ฟฝ๐œ‹4 +

๐œ‹2 ๏ฟฝ = ๐‘’

๐‘–๏ฟฝ 34๐œ‹๏ฟฝ and the third zero at ๐‘’๐‘–๏ฟฝ 34๐œ‹+

๐œ‹2 ๏ฟฝ = ๐‘’

๐‘–๏ฟฝ 54๐œ‹๏ฟฝ and the fourth zero at

๐‘’๐‘–๏ฟฝ 54๐œ‹+

๐œ‹2 ๏ฟฝ = ๐‘’๐‘–

74๐œ‹. Hence poles are at

๐‘ง1 = ๐‘’๐‘–๐œ‹4

๐‘ง2 = ๐‘’๐‘– 34๐œ‹

๐‘ง3 = ๐‘’๐‘– 54๐œ‹

๐‘ง4 = ๐‘’๐‘– 74๐œ‹

Out of these only the first two are in upper half plane ๐‘ง1 and ๐‘ง1. Hence

Residue (๐‘ง1) = lim๐‘งโ†’๐‘ง1

(๐‘ง โˆ’ ๐‘ง1) ๐‘“ (๐‘ง)

= lim๐‘งโ†’๐‘ง1

(๐‘ง โˆ’ ๐‘ง1)1

๐‘ง4 โˆ’ 1Applying Lโ€™Hopitals

Residue (๐‘ง1) = lim๐‘งโ†’๐‘ง1

14๐‘ง3

=1

4 ๏ฟฝ๐‘’๐‘–๐œ‹4 ๏ฟฝ3

=1

4๐‘’๐‘–3๐œ‹4

78

Similarly for the other residue

Residue (๐‘ง2) = lim๐‘งโ†’๐‘ง2

(๐‘ง โˆ’ ๐‘ง2) ๐‘“ (๐‘ง)

= lim๐‘งโ†’๐‘ง1

(๐‘ง โˆ’ ๐‘ง2)1

๐‘ง4 โˆ’ 1Applying Lโ€™Hopitals

Residue (๐‘ง2) = lim๐‘งโ†’๐‘ง2

14๐‘ง3

=1

4 ๏ฟฝ๐‘’๐‘–34๐œ‹๏ฟฝ

3

=1

4๐‘’๐‘–9๐œ‹4

=1

4๐‘’๐‘–๐œ‹4

Hence (2A) becomes

๏ฟฝโˆž

โˆ’โˆž

1๐‘ฅ4 + 1

๐‘‘๐‘ฅ = 2๐œ‹๐‘–โŽ›โŽœโŽœโŽœโŽœโŽ1

4๐‘’๐‘–3๐œ‹4

+1

4๐‘’๐‘–๐œ‹4

โŽžโŽŸโŽŸโŽŸโŽŸโŽ 

= 2๐œ‹๐‘–โŽ›โŽœโŽœโŽœโŽœโŽโˆš24๐‘–

โŽžโŽŸโŽŸโŽŸโŽŸโŽ 

=12โˆš

2๐œ‹

But โˆซโˆž

01

๐‘ฅ4+1๐‘‘๐‘ฅ = 1

2โˆซโˆžโˆ’โˆž

1๐‘ฅ4+1

๐‘‘๐‘ฅ, therefore

๏ฟฝโˆž

0

1๐‘ฅ4 + 1

๐‘‘๐‘ฅ = โˆš24๐œ‹

=24โˆš2

๐œ‹

=๐œ‹2โˆš2

Part (b)

Since the integrand is even, then

๐ผ =12 ๏ฟฝ

โˆž

โˆ’โˆž

cos (๐‘Ž๐‘ฅ)๐‘ฅ2 + 1

๐‘‘๐‘ฅ

We will evaluate โˆซโˆž

โˆ’โˆž๐‘’๐‘–๐‘Ž๐‘ง

๐‘ฅ2+1๐‘‘๐‘ฅ and at the end take the real part of the answer. Consideringthe following contour

โˆ’R +R

R

CR

<z

=z

Figure 3.6: contour used for part b

79

Then

โˆฎ๐ถ

๐‘“ (๐‘ง) ๐‘‘๐‘ง = ๏ฟฝ lim๐‘…โ†’โˆž

๏ฟฝ0

โˆ’๐‘…๐‘“ (๐‘ฅ) ๐‘‘๐‘ฅ + lim

๏ฟฝ๏ฟฝโ†’โˆž๏ฟฝ

๏ฟฝ๏ฟฝ

0๐‘“ (๐‘ฅ) ๐‘‘๐‘ฅ๏ฟฝ + lim

๐‘…โ†’โˆž๏ฟฝ๐ถ๐‘…๐‘“ (๐‘ง) ๐‘‘๐‘ง

Using Cauchy principal value the integral above can be written as

โˆฎ๐ถ

๐‘“ (๐‘ง) ๐‘‘๐‘ง = lim๐‘…โ†’โˆž

๏ฟฝ๐‘…

โˆ’๐‘…๐‘“ (๐‘ฅ) ๐‘‘๐‘ฅ + lim

๐‘…โ†’โˆž๏ฟฝ๐ถ๐‘…๐‘“ (๐‘ง) ๐‘‘

= 2๐œ‹๐‘–๏ฟฝResidue

Whereโˆ‘Residue is sum of residues of ๐‘’๐‘–๐‘Ž๐‘ง

๐‘ฅ2+1 for poles that are inside the contour ๐ถ. Thereforethe above becomes

lim๐‘…โ†’โˆž

๏ฟฝ๐‘…

โˆ’๐‘…๐‘“ (๐‘ฅ) ๐‘‘๐‘ฅ = 2๐œ‹๐‘–๏ฟฝResidueโˆ’ lim

๐‘…โ†’โˆž๏ฟฝ๐ถ๐‘…๐‘“ (๐‘ง) ๐‘‘๐‘ง

๏ฟฝโˆž

โˆ’โˆž

๐‘’๐‘–๐‘Ž๐‘ฅ

๐‘ฅ2 + 1๐‘‘๐‘ฅ = 2๐œ‹๐‘–๏ฟฝResidueโˆ’ lim

๐‘…โ†’โˆž๏ฟฝ๐ถ๐‘…

๐‘’๐‘–๐‘Ž๐‘ง

๐‘ง2 + 1๐‘‘๐‘ง (1)

Now we will show that lim๐‘…โ†’โˆžโˆซ๐ถ๐‘…๐‘’๐‘–๐‘Ž๐‘ง

๐‘ง2+1๐‘‘๐‘ง = 0. Since

๏ฟฝ๏ฟฝ๐ถ๐‘…

๐‘’๐‘–๐‘Ž๐‘ง

๐‘ง2 + 1๐‘‘๐‘ง๏ฟฝ โ‰ค ๐‘€๐ฟ

= ๏ฟฝ๐‘“ (๐‘ง)๏ฟฝmax

(๐œ‹๐‘…) (2)

But

๐‘“ (๐‘ง) =๐‘’๐‘–๐‘Ž๐‘ง

(๐‘ง โˆ’ ๐‘–) (๐‘ง + ๐‘–)

=๐‘’๐‘–๐‘Ž๏ฟฝ๐‘ฅ+๐‘–๐‘ฆ๏ฟฝ

(๐‘ง โˆ’ ๐‘–) (๐‘ง + ๐‘–)

=๐‘’๐‘–๐‘Ž๐‘ฅโˆ’๐‘Ž๐‘ฆ

(๐‘ง โˆ’ ๐‘–) (๐‘ง + ๐‘–)

=๐‘’๐‘–๐‘Ž๐‘ฅ๐‘’โˆ’๐‘Ž๐‘ฆ

(๐‘ง โˆ’ ๐‘–) (๐‘ง + ๐‘–)Hence

๏ฟฝ๐‘“ (๐‘ง)๏ฟฝmax

=๏ฟฝ๐‘’๐‘–๐‘Ž๐‘ง๏ฟฝ

max|๐‘’โˆ’๐‘Ž๐‘ฆ|max

|๐‘ง โˆ’ ๐‘–|min |๐‘ง + ๐‘–|min

=|๐‘’โˆ’๐‘Ž๐‘ฆ|max

(๐‘… + 1) (๐‘… โˆ’ 1)

=|๐‘’โˆ’๐‘Ž๐‘ฆ|max๐‘…2 โˆ’ 1

Since ๐‘Ž > 0 and since in upper half ๐‘ฆ > 0 then |๐‘’โˆ’๐‘Ž๐‘ฆ|max = ๏ฟฝ๐‘’โˆ’๐‘Ž๐‘…๏ฟฝmax

= 1. Jordan inequalitywas not needed here, since there is no extra ๐‘ฅ in the numerator of the integrand in thisproblem. The above now reduces to

๏ฟฝ๐‘“ (๐‘ง)๏ฟฝmax

=1

๐‘…2 โˆ’ 1Equation (2) becomes

๏ฟฝ๏ฟฝ๐ถ๐‘…

๐‘’๐‘–๐‘Ž๐‘ง

๐‘ง2 + 1๐‘‘๐‘ง๏ฟฝ โ‰ค

๐œ‹๐‘…๐‘…2 โˆ’ 1

๐‘… โ†’ โˆž the above goes to zero since lim๐‘…โ†’โˆž๐œ‹๐‘…๐‘…2โˆ’1 = lim๐‘…โ†’โˆž

๐œ‹๐‘…2

1โˆ’ 1๐‘…2

= 01 = 0. Equation (1) now

simplifies to

๏ฟฝโˆž

โˆ’โˆž

๐‘’๐‘–๐‘Ž๐‘ฅ

๐‘ฅ4 + 1๐‘‘๐‘ฅ = 2๐œ‹๐‘–๏ฟฝResidue

We just now need to find the residues of 1๐‘ง2+1 that are located in upper half plane. The

80

zeros of the denominator ๐‘ง2 + 1 = 0 are at ๐‘ง = ยฑ๐‘–, hence poles are at

๐‘ง1 = ๐‘–๐‘ง2 = โˆ’๐‘–

Only ๐‘ง1 is in upper half plane. Therefore

Residue (๐‘ง1) = lim๐‘งโ†’๐‘ง1

(๐‘ง โˆ’ ๐‘ง1) ๐‘“ (๐‘ง)

= lim๐‘งโ†’๐‘ง1

(๐‘ง โˆ’ ๐‘ง1)๐‘’๐‘–๐‘Ž๐‘ง

(๐‘ง โˆ’ ๐‘ง1) (๐‘ง โˆ’ ๐‘ง2)

= lim๐‘งโ†’๐‘ง1

๐‘’๐‘–๐‘Ž๐‘ง

(๐‘ง โˆ’ ๐‘ง2)

=๐‘’๐‘–๐‘Ž(๐‘–)

(๐‘– + ๐‘–)

=๐‘’โˆ’๐‘Ž

2๐‘–Since โˆซ

โˆž

โˆ’โˆž๐‘’๐‘Ž๐‘ฅ

๐‘ฅ4+1๐‘‘๐‘ฅ = 2๐œ‹๐‘–โˆ‘Residue then

๏ฟฝโˆž

โˆ’โˆž

๐‘’๐‘–๐‘Ž๐‘ฅ

๐‘ฅ4 + 1๐‘‘๐‘ฅ = 2๐œ‹๐‘– ๏ฟฝ

๐‘’โˆ’๐‘Ž

2๐‘– ๏ฟฝ

= ๐œ‹๐‘’โˆ’๐‘Ž

Therefore

๏ฟฝโˆž

0

๐‘’๐‘–๐‘Ž๐‘ฅ

๐‘ฅ4 + 1๐‘‘๐‘ฅ =

12 ๏ฟฝ

โˆž

โˆ’โˆž

๐‘’๐‘Ž๐‘ฅ

๐‘ฅ4 + 1๐‘‘๐‘ฅ

=๐œ‹2๐‘’โˆ’๐‘Ž

But real part of the above is

๏ฟฝโˆž

0

cos (๐‘Ž๐‘ฅ)๐‘ฅ4 + 1

๐‘‘๐‘ฅ =๐œ‹2๐‘’โˆ’๐‘Ž

3.4.4 Problem 4

Using residues evaluate(a) โˆซ2๐œ‹

01

1+๐‘Ž cos๐œƒ๐‘‘๐œƒ for |๐‘Ž| < 1 (b) โˆซ๐œ‹

0(cos (๐œƒ))2๐‘› ๐‘‘๐œƒ for ๐‘› integer.

Part (a)

Using contour which is anti-clockwise over the unit circle

<z

=z

C

z = eiฮธ

ฮธ

Figure 3.7: contour used for problem 4

Let ๐‘ง = ๐‘’๐‘–๐œƒ, hence ๐‘‘๐‘ง = ๐‘‘๐œƒ๐‘–๐‘’๐‘–๐œƒ = ๐‘‘๐œƒ๐‘–๐‘ง. Using cos๐œƒ = ๐‘ง+๐‘งโˆ’1

2 then the integral can be written in

81

complex domain as

โˆฎ๐ถ

1๐‘–๐‘ง๐‘‘๐‘ง

1 + ๐‘Ž ๐‘ง+๐‘งโˆ’1

2

=2๐‘–โˆฎ๐ถ

1๐‘ง๐‘‘๐‘ง

2 + ๐‘Ž ๏ฟฝ๐‘ง + 1๐‘ง๏ฟฝ

=2๐‘–โˆฎ๐ถ

๐‘‘๐‘ง2๐‘ง + ๐‘Ž๐‘ง2 + ๐‘Ž

=2๐‘Ž๐‘–โˆฎ

๐ถ

๐‘‘๐‘ง๐‘ง2 + 2

๐‘Ž๐‘ง + 1

=2๐‘Ž๐‘–โˆฎ

๐ถ

๐‘‘๐‘ง(๐‘ง โˆ’ ๐‘ง1) (๐‘ง โˆ’ ๐‘ง2)

Where ๐‘ง1, ๐‘ง2 are roots of ๐‘ง2 +2๐‘Ž๐‘ง+ 1 = 0 which are found to be (using the quadratic formula)

as

๐‘ง1 =โˆ’1 โˆ’ โˆš1 โˆ’ ๐‘Ž2

๐‘Ž

๐‘ง2 =โˆ’1 + โˆš1 โˆ’ ๐‘Ž2

๐‘ŽSince |๐‘Ž| < 1 then only ๐‘ง2 will be inside the unit disk for all ๐‘Ž values. Therefore

2๐‘Ž๐‘–โˆฎ

๐ถ

๐‘‘๐‘ง(๐‘ง โˆ’ ๐‘ง1) (๐‘ง โˆ’ ๐‘ง2)

= ๏ฟฝ2๐‘Ž๐‘–๏ฟฝ

2๐œ‹๐‘–Residue (๐‘ง2)

=4๐‘Ž๐œ‹Residue (๐‘ง2) (1)

Now we will find the Residue (๐‘ง2) where in this case ๐‘“ (๐‘ง) = 1(๐‘งโˆ’๐‘ง1)(๐‘งโˆ’๐‘ง2)

. Hence

Residue (๐‘ง2) = lim๐‘งโ†’๐‘ง2

(๐‘ง โˆ’ ๐‘ง2) ๐‘“ (๐‘ง)

= lim๐‘งโ†’๐‘ง2

(๐‘ง โˆ’ ๐‘ง2)1

(๐‘ง โˆ’ ๐‘ง1) (๐‘ง โˆ’ ๐‘ง2)

= lim๐‘งโ†’๐‘ง2

1(๐‘ง โˆ’ ๐‘ง1)

=1

๏ฟฝโˆ’1+โˆš1โˆ’๐‘Ž2

๐‘Ž ๏ฟฝ โˆ’ ๏ฟฝโˆ’1โˆ’โˆš1โˆ’๐‘Ž2

๐‘Ž ๏ฟฝ

=๐‘Ž

2โˆš1 โˆ’ ๐‘Ž2Using the above result in (1) gives

๏ฟฝ2๐œ‹

0

11 + ๐‘Ž cos๐œƒ๐‘‘๐œƒ = ๏ฟฝ

4๐‘Ž๐œ‹๏ฟฝ

๐‘Ž2โˆš1 โˆ’ ๐‘Ž2

=2๐œ‹

โˆš1 โˆ’ ๐‘Ž2๐‘Ž โ‰  1

Using Maple, verified that the above result is correct.

Figure 3.8: Verification using Maple

Part (b)

Since integrand is even, then โˆซ๐œ‹

0(cos (๐œƒ))2๐‘› ๐‘‘๐œƒ = 1

2โˆซ2๐œ‹0

(cos (๐œƒ))2๐‘› ๐‘‘๐œƒ. Using same contour as

in part (a), and letting ๐‘ง = ๐‘’๐‘–๐œƒ, hence ๐‘‘๐‘ง = ๐‘‘๐œƒ๐‘–๐‘’๐‘–๐œƒ = ๐‘‘๐œƒ๐‘–๐‘ง and using cos๐œƒ = ๐‘ง+๐‘งโˆ’1

2 then the

82

integral can be written in complex domain as

๏ฟฝ2๐œ‹

0(cos (๐œƒ))2๐‘› ๐‘‘๐œƒ = โˆฎ

๐ถ

โŽ›โŽœโŽœโŽœโŽœโŽœโŽ๐‘ง + 1

๐‘ง2

โŽžโŽŸโŽŸโŽŸโŽŸโŽŸโŽ 

2๐‘›๐‘‘๐‘ง๐‘–๐‘ง

=1๐‘–โˆฎ๐ถ

๏ฟฝ๐‘ง + 1๐‘ง๏ฟฝ2๐‘›

22๐‘›๐‘‘๐‘ง๐‘ง

=14๐‘›๐‘–โˆฎ

๐ถ๏ฟฝ๐‘ง +

1๐‘ง๏ฟฝ

2๐‘› ๐‘‘๐‘ง๐‘ง

=14๐‘›๐‘–โˆฎ

๐ถ๏ฟฝ๐‘ง2 + 1๐‘ง ๏ฟฝ

2๐‘› ๐‘‘๐‘ง๐‘ง

=14๐‘›๐‘–โˆฎ

๐ถ

๏ฟฝ๐‘ง2 + 1๏ฟฝ2๐‘›

๐‘ง2๐‘›๐‘‘๐‘ง๐‘ง

=14๐‘›๐‘–โˆฎ

๐ถ

๏ฟฝ๐‘ง2 + 1๏ฟฝ2๐‘›

๐‘ง2๐‘›+1๐‘‘๐‘ง

Considering ๐‘“ (๐‘ง) =๏ฟฝ๐‘ง2+1๏ฟฝ

2๐‘›

๐‘ง2๐‘›+1, this has a pole at ๐‘ง = 0 of order ๐‘š = 2๐‘› + 1. Therefore

14๐‘›๐‘–โˆฎ

๐ถ

๏ฟฝ๐‘ง2 + 1๏ฟฝ2๐‘›

๐‘ง2๐‘›+1๐‘‘๐‘ง = ๏ฟฝ

14๐‘›๐‘–๏ฟฝ

2๐œ‹๐‘–Residue (๐‘ง = 0) (1)

So we now need to find residue of ๐‘“ (๐‘ง) at ๐‘ง = 0 but for pole of order ๐‘š = 2๐‘› + 1. Using theformula for finding residue for pole of order ๐‘š gives

Residue (๐‘ง0 = 0) = lim๐‘งโ†’๐‘ง0

๐‘‘๐‘šโˆ’1

๐‘‘๐‘ง๐‘šโˆ’1(๐‘ง โˆ’ ๐‘ง0)

๐‘š ๐‘“ (๐‘ง)(๐‘š โˆ’ 1)!

But ๐‘š = 2๐‘› + 1, and ๐‘ง0 = 0, hence the above becomes

Residue (0) = lim๐‘งโ†’0

๐‘‘2๐‘›

๐‘‘๐‘ง2๐‘›๐‘ง2๐‘›+1

(2๐‘›)!๏ฟฝ๐‘ง2 + 1๏ฟฝ

2๐‘›

๐‘ง2๐‘›+1

=1

(2๐‘›)!lim๐‘งโ†’0

๏ฟฝ๐‘‘2๐‘›

๐‘‘๐‘ง2๐‘›๏ฟฝ๐‘ง2 + 1๏ฟฝ

2๐‘›๏ฟฝ

Equation (1) becomes

๏ฟฝ2๐œ‹

0(cos (๐œƒ))2๐‘› ๐‘‘๐œƒ = ๏ฟฝ

14๐‘› ๏ฟฝ

2๐œ‹ ๏ฟฝ1

(2๐‘›)!lim๐‘งโ†’0

๏ฟฝ๐‘‘2๐‘›

๐‘‘๐‘ง2๐‘›๏ฟฝ๐‘ง2 + 1๏ฟฝ

2๐‘›๏ฟฝ๏ฟฝ

Therefore

๏ฟฝ๐œ‹

0(cos (๐œƒ))2๐‘› ๐‘‘๐œƒ = 1

2 ๏ฟฝ14๐‘› ๏ฟฝ

2๐œ‹ ๏ฟฝ1

(2๐‘›)!lim๐‘งโ†’0

๏ฟฝ๐‘‘2๐‘›

๐‘‘๐‘ง2๐‘›๏ฟฝ๐‘ง2 + 1๏ฟฝ

2๐‘›๏ฟฝ๏ฟฝ

=14๐‘›

๐œ‹(2๐‘›)!

lim๐‘งโ†’0

๏ฟฝ๐‘‘2๐‘›

๐‘‘๐‘ง2๐‘›๏ฟฝ๐‘ง2 + 1๏ฟฝ

2๐‘›๏ฟฝ

Will now try to obtained closed form solution. Trying for di๏ฟฝerent ๐‘› values in order to seethe pattern. From few lectures ago, we learned also that

ฮ“ ๏ฟฝ๐‘› +12๏ฟฝ=1 โ‹… 3 โ‹… 5 โ‹… โ‹ฏ โ‹… (2๐‘› โˆ’ 1)

2๐‘› โˆš๐œ‹

Now will generate a table to see the pattern

83

๐‘› 14๐‘›

๐œ‹(2๐‘›)! lim๐‘งโ†’0 ๏ฟฝ

๐‘‘2๐‘›

๐‘‘๐‘ง2๐‘›๏ฟฝ๐‘ง2 + 1๏ฟฝ

2๐‘›๏ฟฝ result of integral ฮ“ ๏ฟฝ๐‘› + 1

2๏ฟฝ

1 14๐œ‹2! lim๐‘งโ†’0

๐‘‘2

๐‘‘๐‘ง2๏ฟฝ๐‘ง2 + 1๏ฟฝ

2 ๐œ‹2 ฮ“ ๏ฟฝ1 + 1

2๏ฟฝ = โˆš๐œ‹

2

2 142๐œ‹4! lim๐‘งโ†’0

๐‘‘4

๐‘‘๐‘ง4๏ฟฝ๐‘ง2 + 1๏ฟฝ

4 3๐œ‹8 ฮ“ ๏ฟฝ2 + 1

2๏ฟฝ = 3โˆš๐œ‹

4

3 143๐œ‹6! lim๐‘งโ†’0

๐‘‘6

๐‘‘๐‘ง6๏ฟฝ๐‘ง2 + 1๏ฟฝ

6 5๐œ‹16 ฮ“ ๏ฟฝ3 + 1

2๏ฟฝ = 15โˆš๐œ‹

8

4 144๐œ‹8! lim๐‘งโ†’0

๐‘‘8

๐‘‘๐‘ง8๏ฟฝ๐‘ง2 + 1๏ฟฝ

8 35๐œ‹128 ฮ“ ๏ฟฝ4 + 1

2๏ฟฝ = 105โˆš๐œ‹

16

5 145

๐œ‹10! lim๐‘งโ†’0

๐‘‘10

๐‘‘๐‘ง10๏ฟฝ๐‘ง2 + 1๏ฟฝ

10 63๐œ‹256 ฮ“ ๏ฟฝ5 + 1

2๏ฟฝ = 945โˆš๐œ‹

32

โ‹ฎ โ‹ฎ โ‹ฎ โ‹ฎ

Based on the above, we see that ๐ผ =โˆš๐œ‹ฮ“๏ฟฝ๐‘›+

12 ๏ฟฝ

๐‘›! , which is verified as follows

๐‘› result of integral ฮ“ ๏ฟฝ๐‘› + 12๏ฟฝ

โˆš๐œ‹ฮ“๏ฟฝ๐‘›+12 ๏ฟฝ

๐‘›!

1 ๐œ‹2 ฮ“ ๏ฟฝ1 + 1

2๏ฟฝ = โˆš๐œ‹

2

โˆš๐œ‹๏ฟฝโˆš๐œ‹2 ๏ฟฝ

1 = 12๐œ‹

2 3๐œ‹8 ฮ“ ๏ฟฝ2 + 1

2๏ฟฝ = 3โˆš๐œ‹

4

โˆš๐œ‹๏ฟฝ3โˆš๐œ‹4 ๏ฟฝ

2! = 38๐œ‹

3 5๐œ‹16 ฮ“ ๏ฟฝ3 + 1

2๏ฟฝ = 15โˆš๐œ‹

8

โˆš๐œ‹๏ฟฝ15โˆš๐œ‹8 ๏ฟฝ

3! = 15๐œ‹(6)(8) =

15๐œ‹48 = 3

16๐œ‹

4 35๐œ‹128 ฮ“ ๏ฟฝ4 + 1

2๏ฟฝ = 105โˆš๐œ‹

16

โˆš๐œ‹๏ฟฝ105โˆš๐œ‹16 ๏ฟฝ

4! = โˆš๐œ‹๏ฟฝ105โˆš๐œ‹๏ฟฝ(24)(16) = 105๐œ‹

384 = 35128๐œ‹

5 63๐œ‹256 ฮ“ ๏ฟฝ5 + 1

2๏ฟฝ = 945โˆš๐œ‹

32

โˆš๐œ‹๏ฟฝ945โˆš๐œ‹32 ๏ฟฝ

5! = 945๐œ‹(120)(32) =

945๐œ‹3840 =

63256๐œ‹

โ‹ฎ โ‹ฎ โ‹ฎ โ‹ฎ

Therefore

๏ฟฝ๐œ‹

0(cos (๐œƒ))2๐‘› ๐‘‘๐œƒ =

โˆš๐œ‹ฮ“ ๏ฟฝ๐‘› +12๏ฟฝ

๐‘›!

Tried to do pole/zero cancellation on the integrand ofโˆฎ๐ถ

๏ฟฝ๐‘ง2+1๏ฟฝ2๐‘›

๐‘ง2๐‘›+1๐‘‘๐‘ง in order to find a simpler

method than the above but was not able to. The above result was verified using thecomputer

In[ ]:= AssumingElement[n, Integers] && n > 0, IntegrateCos[x]2 n, {x, 0, ฯ€};

TraditionalForm[%]

Out[ ]//TraditionalForm=

ฯ€ ฮ“n + 1

2

n !

Figure 3.9: Verification using Mathematica

84

3.4.5 key solution to HW 4

85

86

87

88

3.5 HW 5

3.5.1 Problem 1

Expand the following functions, which are periodic in 2๐œ‹๐ฟ , in Fourier series (i) ๐‘“ (๐‘ฅ) = 1 โˆ’ |๐‘ฅ|

๐ฟfor โˆ’๐ฟ

2 โ‰ค ๐‘ฅ โ‰ค ๐ฟ2 . (ii) ๐‘“ (๐‘ฅ) = ๐‘’

๐‘ฅ for โˆ’๐ฟ2 โ‰ค ๐‘ฅ โ‰ค ๐ฟ

2

Solution

89

Part 1

The following is a plot of the function ๐‘“ (๐‘ฅ) = 1 โˆ’ |๐‘ฅ|๐ฟ . In the plot below ๐ฟ = 1 was used for

illustration.

-0.4 -0.2 0.0 0.2 0.4

0.0

0.2

0.4

0.6

0.8

1.0

x

f(x)

Plot of function for one period

Figure 3.10: Function plot

L = 1;

f[x_] := 1 - Abs[x]/ L;

p = Plot[f[x], {x, -L/ 2, L/ 2},

AxesOrigin โ†’ {0, 0}, Frame โ†’ True,

FrameLabel โ†’ {{"f(x)", None}, {"x", "Plot of function for one period"}},

BaseStyle โ†’ 14,

GridLines โ†’ Automatic, GridLinesStyle โ†’ LightGray,

PlotStyle โ†’ Red]

Export["../images/p1_plot_1.pdf", p]

Figure 3.11: Code used

The Fourier series of ๐‘“ (๐‘ฅ) = is given by

๐‘“ (๐‘ฅ) =๐‘Ž02+

โˆž๏ฟฝ๐‘›=1

๐‘Ž๐‘› cos ๏ฟฝ2๐œ‹๐ฟ๐‘›๐‘ฅ๏ฟฝ + ๐‘๐‘› sin ๏ฟฝ

2๐œ‹๐ฟ๐‘›๐‘ฅ๏ฟฝ (1)

Where ๐ฟ is the period.

๐‘Ž0 =2๐ฟ ๏ฟฝ

๐ฟ2

โˆ’ ๐ฟ2

๐‘“ (๐‘ฅ) ๐‘‘๐‘ฅ

โˆซ๐ฟ2

โˆ’ ๐ฟ2๐‘“ (๐‘ฅ) ๐‘‘๐‘ฅ is the area under the curve. Looking at the plot above shows the area is made

up of the lower rectangle of area 12๐ฟ and a triangle whose area is ๏ฟฝ12๐ฟ๏ฟฝ ๏ฟฝ

12๏ฟฝ. Therefore the

total area is 12๐ฟ +

14๐ฟ =

34๐ฟ. Hence

๐‘Ž0 =2๐ฟ ๏ฟฝ

34๐ฟ๏ฟฝ

=32

And

๐‘Ž๐‘› =2๐ฟ ๏ฟฝ

๐ฟ2

โˆ’ ๐ฟ2

๐‘“ (๐‘ฅ) cos ๏ฟฝ2๐œ‹๐ฟ๐‘›๐‘ฅ๏ฟฝ ๐‘‘๐‘ฅ

90

Since ๐‘“ (๐‘ฅ) is an even function, the above simplifies to

๐‘Ž๐‘› =4๐ฟ ๏ฟฝ

๐ฟ2

0๐‘“ (๐‘ฅ) cos ๏ฟฝ

2๐œ‹๐ฟ๐‘›๐‘ฅ๏ฟฝ ๐‘‘๐‘ฅ

=4๐ฟ ๏ฟฝ

๐ฟ2

0๏ฟฝ1 โˆ’

๐‘ฅ๐ฟ๏ฟฝ cos ๏ฟฝ

2๐œ‹๐ฟ๐‘›๐‘ฅ๏ฟฝ ๐‘‘๐‘ฅ

=4๐ฟ

โŽ›โŽœโŽœโŽœโŽœโŽ๏ฟฝ

๐ฟ2

0cos ๏ฟฝ

2๐œ‹๐ฟ๐‘›๐‘ฅ๏ฟฝ ๐‘‘๐‘ฅ โˆ’

1๐ฟ ๏ฟฝ

๐ฟ2

0๐‘ฅ cos ๏ฟฝ

2๐œ‹๐ฟ๐‘›๐‘ฅ๏ฟฝ ๐‘‘๐‘ฅ

โŽžโŽŸโŽŸโŽŸโŽŸโŽ 

=4๐ฟ

โŽ›โŽœโŽœโŽœโŽœโŽœโŽœโŽ๏ฟฝsin ๏ฟฝ

2๐œ‹๐ฟ๐‘›๐‘ฅ๏ฟฝ๏ฟฝ

๐ฟ2

0โˆ’1๐ฟ ๏ฟฝ

๐ฟ2

0๐‘ฅ cos ๏ฟฝ

2๐œ‹๐ฟ๐‘›๐‘ฅ๏ฟฝ ๐‘‘๐‘ฅ

โŽžโŽŸโŽŸโŽŸโŽŸโŽŸโŽŸโŽ 

=4๐ฟ

โŽ›โŽœโŽœโŽœโŽœโŽ๏ฟฝsin ๏ฟฝ

2๐œ‹๐ฟ๐‘› ๏ฟฝ๐ฟ2๏ฟฝโˆ’ 0๏ฟฝ๏ฟฝ โˆ’

1๐ฟ ๏ฟฝ

๐ฟ2

0๐‘ฅ cos ๏ฟฝ

2๐œ‹๐ฟ๐‘›๐‘ฅ๏ฟฝ ๐‘‘๐‘ฅ

โŽžโŽŸโŽŸโŽŸโŽŸโŽ 

=4๐ฟ

โŽ›โŽœโŽœโŽœโŽœโŽœโŽ

0

๏ฟฝ๏ฟฝ๏ฟฝ๏ฟฝ๏ฟฝ๏ฟฝ๏ฟฝ๏ฟฝ๏ฟฝ[sin๐œ‹๐‘›] โˆ’ 1๐ฟ ๏ฟฝ

๐ฟ2

0๐‘ฅ cos ๏ฟฝ

2๐œ‹๐ฟ๐‘›๐‘ฅ๏ฟฝ ๐‘‘๐‘ฅ

โŽžโŽŸโŽŸโŽŸโŽŸโŽŸโŽ 

= โˆ’4๐ฟ2 ๏ฟฝ

๐ฟ2

0๐‘ฅ cos ๏ฟฝ

2๐œ‹๐ฟ๐‘›๐‘ฅ๏ฟฝ ๐‘‘๐‘ฅ

Using integration by parts: Let ๐‘ข = ๐‘ฅ, ๐‘‘๐‘ฃ = cos ๏ฟฝ2๐œ‹๐ฟ ๐‘›๐‘ฅ๏ฟฝ then ๐‘‘๐‘ข = 1, ๐‘ฃ =sin๏ฟฝ 2๐œ‹๐ฟ ๐‘›๐‘ฅ๏ฟฝ

2๐œ‹๐ฟ ๐‘›

= ๐ฟ2๐œ‹๐‘› sin ๏ฟฝ2๐œ‹๐ฟ ๐‘›๐‘ฅ๏ฟฝ.

The above integral becomes

๏ฟฝ๐ฟ2

0๐‘ฅ cos ๏ฟฝ

2๐œ‹๐ฟ๐‘›๐‘ฅ๏ฟฝ ๐‘‘๐‘ฅ = ๏ฟฝ

๐ฟ2๐œ‹๐‘›

๐‘ฅ sin ๏ฟฝ2๐œ‹๐ฟ๐‘›๐‘ฅ๏ฟฝ๏ฟฝ

๐ฟ2

0โˆ’

๐ฟ2๐œ‹๐‘› ๏ฟฝ

๐ฟ2

0sin ๏ฟฝ

2๐œ‹๐ฟ๐‘›๐‘ฅ๏ฟฝ ๐‘‘๐‘ฅ

= ๏ฟฝ๐ฟ2๐œ‹๐‘› ๏ฟฝ

๐ฟ2๏ฟฝ

sin ๏ฟฝ2๐œ‹๐ฟ๐‘›๐ฟ2๏ฟฝโˆ’ 0๏ฟฝ โˆ’

๐ฟ2๐œ‹๐‘›

โŽ›โŽœโŽœโŽœโŽœโŽœโŽœโŽœโŽœโŽโˆ’

cos ๏ฟฝ2๐œ‹๐ฟ ๐‘›๐‘ฅ๏ฟฝ2๐œ‹๐ฟ ๐‘›

โŽžโŽŸโŽŸโŽŸโŽŸโŽŸโŽŸโŽŸโŽŸโŽ 

๐ฟ2

0

=๐ฟ2

4๐œ‹๐‘›sin (๐‘›๐œ‹) + ๐ฟ2

4๐œ‹2๐‘›2 ๏ฟฝcos ๏ฟฝ

2๐œ‹๐ฟ๐‘› ๏ฟฝ๐ฟ2๏ฟฝ๏ฟฝ

โˆ’ 1๏ฟฝ

=๐ฟ2

4๐œ‹2๐‘›2(cos (๐œ‹๐‘›) โˆ’ 1)

Therefore

๐‘Ž๐‘› = โˆ’4๐ฟ2 ๏ฟฝ

๐ฟ2

4๐œ‹2๐‘›2(cos (๐œ‹๐‘›) โˆ’ 1)๏ฟฝ

=1

๐œ‹2๐‘›2(1 โˆ’ cos (๐œ‹๐‘›))

The above is zero for even ๐‘› and 2๐ฟ2

4๐œ‹2๐‘›2 for odd ๐‘›. Therefore the above simplifies to

๐‘Ž๐‘› =2

๐œ‹2๐‘›2๐‘› = 1, 3, 5,โ‹ฏ

Because ๐‘“ (๐‘ฅ) is an even function, then ๐‘๐‘› = 0 for all ๐‘›. The Fourier series from (1) nowbecomes

๐‘“ (๐‘ฅ) =34+

โˆž๏ฟฝ

๐‘›=1,3,5,โ‹ฏ

2๐œ‹2๐‘›2

cos ๏ฟฝ2๐œ‹๐ฟ๐‘›๐‘ฅ๏ฟฝ

To verify the above result, the Fourier series approximation given above was plotted forincreasing ๐‘› against the original ๐‘“ (๐‘ฅ) function in order to see how the approximationimproves as ๐‘› increases. Using ๐ฟ = 2, the result is given below.

The original function is in the red color. The plot shows that the convergence is fast (dueto the 1

๐‘›2 term). The convergence is uniform. After only 4 terms, the error between ๐‘“ (๐‘ฅ)and its Fourier series approximation becomes very small. As expected, the error is largestat the top and at the lower corners where the original function changes more rapidly andtherefore more terms would be needed in those regions compared to the straight edgesregions of the function ๐‘“ (๐‘ฅ) to get a better approximation.

91

x

f(x)

approximation for n=1

x

f(x)

approximation for n=3

x

f(x)

approximation for n=5

x

f(x)

approximation for n=7

Figure 3.12: Fourier series approximation, part 1

ClearAll[L, x, n, a]

L = 2;

a[n_] := 2/(Pi^2 n^2);

fApprox[x_, nTerms_] := 3/ 4 + Sum[a[n] Cos[2 Pi/ L n x], {n, 1, nTerms, 2}]

p = Table[

Plot[{f[x], fApprox[x, i]}, {x, -L/ 2, L/ 2},

Frame โ†’ True,

FrameLabel โ†’ {{"f(x)", None}, {"x", Row[{"approximation for n=", i}]}},

GridLines โ†’ Automatic, GridLinesStyle โ†’ LightGray,

PlotStyle โ†’ {Red, Blue},

ImageSize โ†’ 400,

BaseStyle โ†’ 16],

{i, 1, 7, 2}

];

p = Grid[Partition[p, 2]]

Export["../images/p1_plot_2.pdf", p]

Figure 3.13: Code used

Part 2

The following is a plot of the function ๐‘“ (๐‘ฅ) = ๐‘’๐‘ฅ. In this plot, ๐ฟ = 1 was used.

92

x

f(x)

Plot of function for one period

Figure 3.14: Function plot part 2

L = 1;

f[x_] := Exp[x];

p = Plot[f[x], {x, -L/ 2, L/ 2}, AxesOrigin โ†’ {0, 0},

Frame โ†’ True,

FrameLabel โ†’ {{"f(x)", None}, {"x", "Plot of function for one period"}},

BaseStyle โ†’ 14, GridLines โ†’ Automatic, GridLinesStyle โ†’ LightGray, PlotStyle โ†’ Red]

Export["../images/p1_plot_3.pdf", p]

Figure 3.15: Code used

The Fourier series of ๐‘“ (๐‘ฅ) = is given by

๐‘“ (๐‘ฅ) =๐‘Ž02+

โˆž๏ฟฝ๐‘›=1

๐‘Ž๐‘› cos ๏ฟฝ2๐œ‹๐ฟ๐‘›๐‘ฅ๏ฟฝ + ๐‘๐‘› sin ๏ฟฝ

2๐œ‹๐ฟ๐‘›๐‘ฅ๏ฟฝ (1A)

Where ๐ฟ is the period and

๐‘Ž0 =2๐ฟ ๏ฟฝ

๐ฟ2

โˆ’ ๐ฟ2

๐‘“ (๐‘ฅ) ๐‘‘๐‘ฅ

=2๐ฟ ๏ฟฝ

๐ฟ2

โˆ’ ๐ฟ2

๐‘’๐‘ฅ๐‘‘๐‘ฅ

=2๐ฟ[๐‘’๐‘ฅ]

๐ฟ2โˆ’ ๐ฟ2

=2๐ฟ ๏ฟฝ๐‘’๐ฟ2 โˆ’ ๐‘’โˆ’

๐ฟ2 ๏ฟฝ

=4๐ฟ

โŽกโŽขโŽขโŽขโŽขโŽขโŽฃ๐‘’๐ฟ2 โˆ’ ๐‘’โˆ’

๐ฟ2

2

โŽคโŽฅโŽฅโŽฅโŽฅโŽฅโŽฆ

=4๐ฟ

sinh ๏ฟฝ๐ฟ2๏ฟฝ

And

๐‘Ž๐‘› =2๐ฟ ๏ฟฝ

๐ฟ2

โˆ’ ๐ฟ2

๐‘“ (๐‘ฅ) cos ๏ฟฝ2๐œ‹๐ฟ๐‘›๐‘ฅ๏ฟฝ ๐‘‘๐‘ฅ

=2๐ฟ ๏ฟฝ

๐ฟ2

โˆ’ ๐ฟ2

๐‘’๐‘ฅ cos ๏ฟฝ2๐œ‹๐ฟ๐‘›๐‘ฅ๏ฟฝ ๐‘‘๐‘ฅ (1)

Integration by parts: Let ๐‘ข = cos ๏ฟฝ2๐œ‹๐ฟ ๐‘›๐‘ฅ๏ฟฝ , ๐‘‘๐‘ข = โˆ’2๐œ‹๐‘›๐ฟ sin ๏ฟฝ2๐œ‹๐ฟ ๐‘›๐‘ฅ๏ฟฝ and let ๐‘‘๐‘ฃ = ๐‘’๐‘ฅ, ๐‘ฃ = ๐‘’๐‘ฅ,

93

therefore

๐ผ = ๏ฟฝ๐ฟ2

โˆ’ ๐ฟ2

๐‘’๐‘ฅ cos ๏ฟฝ2๐œ‹๐ฟ๐‘›๐‘ฅ๏ฟฝ ๐‘‘๐‘ฅ

= ๏ฟฝ๐‘’๐‘ฅ cos ๏ฟฝ2๐œ‹๐ฟ๐‘›๐‘ฅ๏ฟฝ๏ฟฝ

๐ฟ2

โˆ’ ๐ฟ2

โˆ’๏ฟฝ๐ฟ2

โˆ’ ๐ฟ2

โˆ’2๐œ‹๐‘›๐ฟ

sin ๏ฟฝ2๐œ‹๐ฟ๐‘›๐‘ฅ๏ฟฝ ๐‘’๐‘ฅ๐‘‘๐‘ฅ

= ๏ฟฝ๐‘’๐ฟ2 cos ๏ฟฝ

2๐œ‹๐ฟ๐‘›๐ฟ2๏ฟฝโˆ’ ๐‘’โˆ’

๐ฟ2 cos ๏ฟฝ

2๐œ‹๐ฟ๐‘› ๏ฟฝโˆ’

๐ฟ2๏ฟฝ๏ฟฝ๏ฟฝ

+2๐œ‹๐‘›๐ฟ ๏ฟฝ

๐ฟ2

โˆ’ ๐ฟ2

sin ๏ฟฝ2๐œ‹๐ฟ๐‘›๐‘ฅ๏ฟฝ ๐‘’๐‘ฅ๐‘‘๐‘ฅ

= ๏ฟฝ๐‘’๐ฟ2 cos (๐œ‹๐‘›) โˆ’ ๐‘’โˆ’

๐ฟ2 cos (๐œ‹๐‘›)๏ฟฝ +

2๐œ‹๐‘›๐ฟ ๏ฟฝ

๐ฟ2

โˆ’ ๐ฟ2

sin ๏ฟฝ2๐œ‹๐ฟ๐‘›๐‘ฅ๏ฟฝ ๐‘’๐‘ฅ๐‘‘๐‘ฅ

= cos (๐œ‹๐‘›) ๏ฟฝ๐‘’๐ฟ2 โˆ’ ๐‘’โˆ’

๐ฟ2 ๏ฟฝ +

2๐œ‹๐‘›๐ฟ ๏ฟฝ

๐ฟ2

โˆ’ ๐ฟ2

sin ๏ฟฝ2๐œ‹๐ฟ๐‘›๐‘ฅ๏ฟฝ ๐‘’๐‘ฅ๐‘‘๐‘ฅ

= 2 cos (๐œ‹๐‘›) sinh ๏ฟฝ๐ฟ2๏ฟฝ+2๐œ‹๐‘›๐ฟ ๏ฟฝ

๐ฟ2

โˆ’ ๐ฟ2

sin ๏ฟฝ2๐œ‹๐ฟ๐‘›๐‘ฅ๏ฟฝ ๐‘’๐‘ฅ๐‘‘๐‘ฅ

Integration by parts again, let ๐‘ข = sin ๏ฟฝ2๐œ‹๐ฟ ๐‘›๐‘ฅ๏ฟฝ , ๐‘‘๐‘ข =2๐œ‹๐‘›๐ฟ cos ๏ฟฝ2๐œ‹๐ฟ ๐‘›๐‘ฅ๏ฟฝ and ๐‘‘๐‘ฃ = ๐‘’

๐‘ฅ, ๐‘ฃ = ๐‘’๐‘ฅ. Theabove becomes

๐ผ = 2 cos (๐œ‹๐‘›) sinh ๏ฟฝ๐ฟ2๏ฟฝ+2๐œ‹๐‘›๐ฟ

โŽ›โŽœโŽœโŽœโŽœโŽœโŽœโŽ๏ฟฝ๐‘’

๐‘ฅ sin ๏ฟฝ2๐œ‹๐ฟ๐‘›๐‘ฅ๏ฟฝ๏ฟฝ

๐ฟ2

โˆ’ ๐ฟ2

โˆ’๏ฟฝ๐ฟ2

โˆ’ ๐ฟ2

2๐œ‹๐‘›๐ฟ

cos ๏ฟฝ2๐œ‹๐ฟ๐‘›๐‘ฅ๏ฟฝ ๐‘’๐‘ฅ๐‘‘๐‘ฅ

โŽžโŽŸโŽŸโŽŸโŽŸโŽŸโŽŸโŽ 

The term ๏ฟฝ๐‘’๐‘ฅ sin ๏ฟฝ2๐œ‹๐ฟ ๐‘›๐‘ฅ๏ฟฝ๏ฟฝ๐ฟ2

โˆ’ ๐ฟ2

goes to zero since it gives sin (๐‘›๐œ‹) and ๐‘› is integer. The above

simplifies to

๐ผ = 2 cos (๐œ‹๐‘›) sinh ๏ฟฝ๐ฟ2๏ฟฝ+2๐œ‹๐‘›๐ฟ

โŽ›โŽœโŽœโŽœโŽœโŽโˆ’2๐œ‹๐‘›๐ฟ ๏ฟฝ

๐ฟ2

โˆ’ ๐ฟ2

cos ๏ฟฝ2๐œ‹๐ฟ๐‘›๐‘ฅ๏ฟฝ ๐‘’๐‘ฅ๐‘‘๐‘ฅ

โŽžโŽŸโŽŸโŽŸโŽŸโŽ 

= 2 cos (๐œ‹๐‘›) sinh ๏ฟฝ๐ฟ2๏ฟฝโˆ’4๐œ‹2๐‘›2

๐ฟ2 ๏ฟฝ๐ฟ2

โˆ’ ๐ฟ2

cos ๏ฟฝ2๐œ‹๐ฟ๐‘›๐‘ฅ๏ฟฝ ๐‘’๐‘ฅ๐‘‘๐‘ฅ

Since โˆซ๐ฟ2

โˆ’ ๐ฟ2

cos ๏ฟฝ2๐œ‹๐ฟ ๐‘›๐‘ฅ๏ฟฝ ๐‘’๐‘ฅ๐‘‘๐‘ฅ = ๐ผ the above reduces to

๐ผ = 2 cos (๐œ‹๐‘›) sinh ๏ฟฝ๐ฟ2๏ฟฝโˆ’4๐œ‹2๐‘›2

๐ฟ2๐ผ

๐ผ ๏ฟฝ1 +4๐œ‹2๐‘›2

๐ฟ2 ๏ฟฝ = 2 cos (๐œ‹๐‘›) sinh ๏ฟฝ๐ฟ2๏ฟฝ

๐ผ =2 cos (๐œ‹๐‘›) sinh ๏ฟฝ๐ฟ2๏ฟฝ

1 + 4๐œ‹2๐‘›2

๐ฟ2

Using the above in (1) gives

๐‘Ž๐‘› =2๐ฟ

2 cos (๐œ‹๐‘›) sinh ๏ฟฝ๐ฟ2๏ฟฝ

1 + 4๐œ‹2๐‘›2

๐ฟ2

=2๐ฟ2

๐ฟ

2 cos (๐œ‹๐‘›) sinh ๏ฟฝ๐ฟ2๏ฟฝ

๐ฟ2 + 4๐œ‹2๐‘›2

=4๐ฟ

๐ฟ2 + 4๐œ‹2๐‘›2cos (๐œ‹๐‘›) sinh ๏ฟฝ

๐ฟ2๏ฟฝ

94

Next, ๐‘๐‘› is found:

๐‘๐‘› =2๐ฟ ๏ฟฝ

๐ฟ2

โˆ’ ๐ฟ2

๐‘“ (๐‘ฅ) sin ๏ฟฝ2๐œ‹๐ฟ๐‘›๐‘ฅ๏ฟฝ ๐‘‘๐‘ฅ

=2๐ฟ ๏ฟฝ

๐ฟ2

โˆ’ ๐ฟ2

๐‘’๐‘ฅ sin ๏ฟฝ2๐œ‹๐ฟ๐‘›๐‘ฅ๏ฟฝ ๐‘‘๐‘ฅ (2)

Integration by parts: Let ๐‘ข = sin ๏ฟฝ2๐œ‹๐ฟ ๐‘›๐‘ฅ๏ฟฝ , ๐‘‘๐‘ข =2๐œ‹๐‘›๐ฟ sin ๏ฟฝ2๐œ‹๐ฟ ๐‘›๐‘ฅ๏ฟฝ and let ๐‘‘๐‘ฃ = ๐‘’๐‘ฅ, ๐‘ฃ = ๐‘’๐‘ฅ, therefore

๐ผ = ๏ฟฝ๐ฟ2

โˆ’ ๐ฟ2

๐‘’๐‘ฅ sin ๏ฟฝ2๐œ‹๐ฟ๐‘›๐‘ฅ๏ฟฝ ๐‘‘๐‘ฅ

= ๏ฟฝ๐‘’๐‘ฅ sin ๏ฟฝ2๐œ‹๐ฟ๐‘›๐‘ฅ๏ฟฝ๏ฟฝ

๐ฟ2

โˆ’ ๐ฟ2

โˆ’๏ฟฝ๐ฟ2

โˆ’ ๐ฟ2

2๐œ‹๐‘›๐ฟ

cos ๏ฟฝ2๐œ‹๐ฟ๐‘›๐‘ฅ๏ฟฝ ๐‘’๐‘ฅ๐‘‘๐‘ฅ

But ๏ฟฝ๐‘’๐‘ฅ sin ๏ฟฝ2๐œ‹๐ฟ ๐‘›๐‘ฅ๏ฟฝ๏ฟฝ๐ฟ2

โˆ’ ๐ฟ2

goes to zero as sin (๐œ‹๐‘›) = 0 for integer ๐‘› and the above simplifies to

๐ผ = โˆ’2๐œ‹๐‘›๐ฟ ๏ฟฝ

๐ฟ2

โˆ’ ๐ฟ2

cos ๏ฟฝ2๐œ‹๐ฟ๐‘›๐‘ฅ๏ฟฝ ๐‘’๐‘ฅ๐‘‘๐‘ฅ

Integration by parts again: let ๐‘ข = cos ๏ฟฝ2๐œ‹๐ฟ ๐‘›๐‘ฅ๏ฟฝ , ๐‘‘๐‘ข = โˆ’2๐œ‹๐‘›๐ฟ sin ๏ฟฝ2๐œ‹๐ฟ ๐‘›๐‘ฅ๏ฟฝ and ๐‘‘๐‘ฃ = ๐‘’

๐‘ฅ, ๐‘ฃ = ๐‘’๐‘ฅ. Theabove becomes

๐ผ = โˆ’2๐œ‹๐‘›๐ฟ

โŽ›โŽœโŽœโŽœโŽœโŽœโŽœโŽ๏ฟฝ๐‘’

๐‘ฅ cos ๏ฟฝ2๐œ‹๐ฟ๐‘›๐‘ฅ๏ฟฝ๏ฟฝ

๐ฟ2

โˆ’ ๐ฟ2

โˆ’๏ฟฝ๐ฟ2

โˆ’ ๐ฟ2

โˆ’2๐œ‹๐‘›๐ฟ

sin ๏ฟฝ2๐œ‹๐ฟ๐‘›๐‘ฅ๏ฟฝ ๐‘’๐‘ฅ๐‘‘๐‘ฅ

โŽžโŽŸโŽŸโŽŸโŽŸโŽŸโŽŸโŽ 

= โˆ’2๐œ‹๐‘›๐ฟ

โŽ›โŽœโŽœโŽœโŽœโŽ2 cos (๐œ‹๐‘›) sinh ๏ฟฝ

๐ฟ2๏ฟฝ+2๐œ‹๐‘›๐ฟ ๏ฟฝ

๐ฟ2

โˆ’ ๐ฟ2

sin ๏ฟฝ2๐œ‹๐ฟ๐‘›๐‘ฅ๏ฟฝ ๐‘’๐‘ฅ๐‘‘๐‘ฅ

โŽžโŽŸโŽŸโŽŸโŽŸโŽ 

But โˆซ๐ฟ2

โˆ’ ๐ฟ2

sin ๏ฟฝ2๐œ‹๐ฟ ๐‘›๐‘ฅ๏ฟฝ ๐‘’๐‘ฅ๐‘‘๐‘ฅ = ๐ผ and the above reduces to

๐ผ = โˆ’2๐œ‹๐‘›๐ฟ ๏ฟฝ2 cos (๐œ‹๐‘›) sinh ๏ฟฝ

๐ฟ2๏ฟฝ+2๐œ‹๐‘›๐ฟ๐ผ๏ฟฝ

๐ผ = โˆ’4๐œ‹๐‘›๐ฟ

cos (๐œ‹๐‘›) sinh ๏ฟฝ๐ฟ2๏ฟฝโˆ’4๐œ‹2๐‘›2

๐ฟ2๐ผ

๐ผ ๏ฟฝ1 +4๐œ‹2๐‘›2

๐ฟ2 ๏ฟฝ = โˆ’4๐œ‹๐‘›๐ฟ

cos (๐œ‹๐‘›) sinh ๏ฟฝ๐ฟ2๏ฟฝ

๐ผ =โˆ’4๐œ‹๐‘›๐ฟ cos (๐œ‹๐‘›) sinh ๏ฟฝ๐ฟ2๏ฟฝ

1 + 4๐œ‹2๐‘›2

๐ฟ2

=โˆ’4๐œ‹๐‘›๐ฟ cos (๐œ‹๐‘›) sinh ๏ฟฝ๐ฟ2๏ฟฝ

๐ฟ2 + 4๐œ‹2๐‘›2Using the above in (2) gives

๐‘๐‘› =2๐ฟ

โˆ’4๐œ‹๐‘›๐ฟ cos (๐œ‹๐‘›) sinh ๏ฟฝ๐ฟ2๏ฟฝ

๐ฟ2 + 4๐œ‹2๐‘›2

=โˆ’8๐œ‹๐‘›

๐ฟ2 + 4๐œ‹2๐‘›2cos (๐œ‹๐‘›) sinh ๏ฟฝ

๐ฟ2๏ฟฝ

Therefore, from (1A) the Fourier series is

๐‘“ (๐‘ฅ) =2๐ฟ

sinh ๏ฟฝ๐ฟ2๏ฟฝ+

โˆž๏ฟฝ๐‘›=1

4๐ฟ๐ฟ2 + 4๐œ‹2๐‘›2

cos (๐œ‹๐‘›) sinh ๏ฟฝ๐ฟ2๏ฟฝ

cos ๏ฟฝ2๐œ‹๐ฟ๐‘›๐‘ฅ๏ฟฝ โˆ’

8๐œ‹๐‘›๐ฟ2 + 4๐œ‹2๐‘›2

cos (๐œ‹๐‘›) sinh ๏ฟฝ๐ฟ2๏ฟฝ

sin ๏ฟฝ2๐œ‹๐ฟ๐‘›๐‘ฅ๏ฟฝ

(3)

To verify the result, the above was plotted for increasing ๐‘› against the original ๐‘“ (๐‘ฅ) functionto see how the approximation improves as ๐‘› increases. Using ๐ฟ = 2, the result is displayedbelow. The original function is in the red color.

95

Compared to part (1), more terms are needed here to get good approximation. Since theoriginal function is piecewise continuous when extending over multiple periods, the conver-gence is no longer a uniform convergence. At the point of discontinuity, the approximationconverges to the average value of the original function at that point. At about 20 termsthe approximation started to give good results. Due to Gibbs phenomena, at the points ofdiscontinuities, the error is largest. Here is a plot showing one period

x

f(x)

approximation for n=1

x

f(x)

approximation for n=3

x

f(x)

approximation for n=5

x

f(x)

approximation for n=7

x

f(x)

approximation for n=9

x

f(x)

approximation for n=11

x

f(x)

approximation for n=13

x

f(x)

approximation for n=15

x

f(x)

approximation for n=17

x

f(x)

approximation for n=19

Figure 3.16: Fourier series approximation, showing one period

96

ClearAll[L, x, n, a]

L = 2;

f[x_] := Exp[x];

a[n_] := 4 L/(L^2 + 4 Pi^2 n^2) Cos[Pi n] Sinh[L/ 2] ;

b[n_] := -8 Pi n/(L^2 + 4 Pi^2 n^2) Cos[Pi n] Sinh[L/ 2] ;

fApprox[x_, nTerms_] := 2/ L Sinh[L/ 2] + Sum[a[n] Cos[2 Pi/ L n x] + b[n] Sin[2 Pi/ L n x], {n, 1, nTerms, 1}]

p = Table[

Plot[{f[x], fApprox[x, i]}, {x, -L/ 2, L/ 2},

Frame โ†’ True,

FrameLabel โ†’ {{"f(x)", None}, {"x", Row[{"approximation for n=", i}]}},

GridLines โ†’ Automatic, GridLinesStyle โ†’ LightGray,

PlotStyle โ†’ {Red, Blue}, ImageSize โ†’ 400, BaseStyle โ†’ 16],

{i, 1, 20, 2}

];

p = Grid[Partition[p, 2]]

Export["../images/p1_plot_4.pdf", p]

Figure 3.17: Code used

In the following plot, 3 periods are shown to make it easier to see the e๏ฟฝect of discontinuitiesand the Gibbs phenomena

97

x

f(x)

approximation for n=1

x

f(x)

approximation for n=2

x

f(x)

approximation for n=3

xf(x)

approximation for n=4

x

f(x)

approximation for n=5

x

f(x)

approximation for n=6

x

f(x)

approximation for n=7

x

f(x)

approximation for n=8

x

f(x)

approximation for n=9

x

f(x)

approximation for n=10

Figure 3.18: Fourier series approximation, showing 3 periods

98

ClearAll[L, x, n, a]

L = 2;

f[x_] := Piecewise[{

{Exp[x + L], -3/ 2 L < x < -L/ 2},

{Exp[x], -L/ 2 < x < L/ 2},

{Exp[x - L], L/ 2 < x < 3/ 2 L}}

];

a[n_] := 4 L/(L^2 + 4 Pi^2 n^2) Cos[Pi n] Sinh[L/ 2] ;

b[n_] := -8 Pi n/(L^2 + 4 Pi^2 n^2) Cos[Pi n] Sinh[L/ 2] ;

fApprox[x_, nTerms_] := 2/ L Sinh[L/ 2] + Sum[a[n] Cos[2 Pi/ L n x] + b[n] Sin[2 Pi/ L n x], {n, 1, nTerms, 1}];

p = Table[

Plot[{f[x], fApprox[x, i]}, {x, -3/ 2 L, 3/ 2 L},

Frame โ†’ True, FrameLabel โ†’ {{"f(x)", None}, {"x", Row[{"approximation for n=", i}]}},

GridLines โ†’ Automatic, GridLinesStyle โ†’ LightGray,

PlotStyle โ†’ {Red, Blue},

ImageSize โ†’ 400, BaseStyle โ†’ 16],

{i, 1, 10, 1}

];

p = Grid[Partition[p, 2]]

Export["../images/p1_plot_5.pdf", p]

Figure 3.19: Code used

3.5.2 Problem 2

Find the general solution of

1. 2๐‘ฅ3๐‘ฆโ€ฒ = 1 + ๏ฟฝ1 + 4๐‘ฅ2๐‘ฆ

2. ๐‘’๐‘ฅ sin ๐‘ฆ โˆ’ 2๐‘ฆ sin ๐‘ฅ + ๏ฟฝ๐‘ฆ2 + ๐‘’๐‘ฅ cos ๐‘ฆ + 2 cos ๐‘ฆ๏ฟฝ ๐‘ฆโ€ฒ = 0

3. ๐‘ฆโ€ฒ + ๐‘ฆ cos ๐‘ฅ = 12 sin ๐‘ฅ

Solution

part 1

This ODE is not separable and it is also not exact (It was checked for exactness and failedthe test). The ODE is next checked to see if it is isobaric. An ODE ๐‘ฆโ€ฒ = ๐‘“ ๏ฟฝ๐‘ฅ, ๐‘ฆ๏ฟฝ is isobaric(which is a generalization of a homogeneous ODE) if the substitution

๐‘ฆ (๐‘ฅ) = ๐‘ฃ (๐‘ฅ) ๐‘ฅ๐‘š

Changes the ODE to be a separable one in ๐‘ฃ (๐‘ฅ). To determine if it isobaric, a weight ๐‘š isassigned to ๐‘ฆ and to ๐‘‘๐‘ฆ, and a weight of 1 is assigned to ๐‘ฅ and to ๐‘‘๐‘ฅ, then if an ๐‘š could befound such that each term in the ODE will have the same weight, then the ODE is isobaricand it can be made separable using the above substitution. Writing the above ODE as

2๐‘ฅ3๐‘‘๐‘ฆ = ๏ฟฝ1 + ๏ฟฝ1 + 4๐‘ฅ2๐‘ฆ๏ฟฝ ๐‘‘๐‘ฅ

๏ฟฝ2๐‘ฅ3๐‘‘๐‘ฆ โˆ’โž๐‘‘๐‘ฅโˆ’๏ฟฝ๏ฟฝ๏ฟฝ๏ฟฝ๏ฟฝ๏ฟฝ๏ฟฝ๏ฟฝ๏ฟฝ๏ฟฝ๏ฟฝ๏ฟฝ๏ฟฝ๏ฟฝ๏ฟฝ๏ฟฝ1 + 4๐‘ฅ

2๐‘ฆ๐‘‘๐‘ฅ = 0

Adding the weights of the first term above gives 2๐‘ฅ3๐‘‘๐‘ฆ โ†’ 3 + ๐‘š. The next term weight is๐‘‘๐‘ฅ โ†’ 1. The next term weight is ๏ฟฝ1 + 4๐‘ฅ2๐‘ฆ๐‘‘๐‘ฅ โ†’

12(2 + ๐‘š) + 1 = 2 + ๐‘š

2 . Therefore the weightsof each term are

{3 + ๐‘š, 1, 2 +๐‘š2}

Each term weight can be made the same by selecting ๐‘š = โˆ’2. This value makes each termhave weight 1 and the above becomes

{1, 1, 1}

Therefore the ODE is isobaric. Using this value of ๐‘š the substitution ๐‘ฆ = ๐‘ฃ๐‘ฅ2 is now used

to make the original ODE separable๐‘‘๐‘ฆ๐‘‘๐‘ฅ

=1๐‘ฅ2๐‘‘๐‘ฃ๐‘‘๐‘ฅโˆ’ 2

๐‘ฃ๐‘ฅ3

99

The original ODE now becomes (where each ๐‘ฆ is replaced by ๐‘ฃ๐‘ฅ2 ) separable as follows

2๐‘ฅ3 ๏ฟฝ1๐‘ฅ2๐‘‘๐‘ฃ๐‘‘๐‘ฅโˆ’ 2

๐‘ฃ๐‘ฅ3 ๏ฟฝ

= 1 +๏ฟฝ1 + 4๐‘ฅ2

๐‘ฃ๐‘ฅ2

2๐‘ฅ๐‘‘๐‘ฃ๐‘‘๐‘ฅโˆ’ 4๐‘ฃ = 1 + โˆš1 + 4๐‘ฃ

2๐‘ฅ๐‘‘๐‘ฃ๐‘‘๐‘ฅ

= 1 + โˆš1 + 4๐‘ฃ + 4๐‘ฃ

Solving this ODE for ๐‘ฃ (๐‘ฅ)๐‘‘๐‘ฃ

1 + โˆš1 + 4๐‘ฃ + 4๐‘ฃ=12๐‘ฅ๐‘‘๐‘ฅ

Integrating both sides gives

๏ฟฝ๐‘‘๐‘ฃ

1 + โˆš1 + 4๐‘ฃ + 4๐‘ฃ=12

ln |๐‘ฅ| + ๐‘ (2)

The integral above is solved by substitution. Let โˆš1 + 4๐‘ฃ = ๐‘ข, hence ๐‘‘๐‘ข๐‘‘๐‘ฃ =

12

4

โˆš1+4๐‘ฃ= 2

๐‘ข or

๐‘‘๐‘ฃ = 12๐‘ข๐‘‘๐‘ข. Squaring both sides of โˆš1 + 4๐‘ฃ = ๐‘ข (and assuming 1 + 4๐‘ฃ > 0) gives 1 + 4๐‘ฃ = ๐‘ข2

or ๐‘ฃ = ๐‘ข2โˆ’14 . Therefore the LHS integral in (2) becomes

๏ฟฝ1

1 + โˆš1 + 4๐‘ฃ + 4๐‘ฃ๐‘‘๐‘ฃ =

12 ๏ฟฝ

๐‘ข

1 + ๐‘ข + 4 ๏ฟฝ๐‘ข2โˆ’14๏ฟฝ๐‘‘๐‘ข

=12 ๏ฟฝ

๐‘ข๐‘ข + ๐‘ข2

๐‘‘๐‘ข

=12 ๏ฟฝ

11 + ๐‘ข

๐‘‘๐‘ข

=12

ln |1 + ๐‘ข|

Using this result in (2) gives the following (the absolute values are removed because theconstant of integration absorbs the sign).

12

ln (1 + ๐‘ข) = 12

ln ๐‘ฅ + ๐‘

ln (1 + ๐‘ข) = ln ๐‘ฅ + 2๐‘Let 2๐‘ = ๐ถ0 be a new constant. The above becomes

ln (1 + ๐‘ข) = ln ๐‘ฅ + ๐ถ0๐‘’ln(1+๐‘ข) = ๐‘’ln ๐‘ฅ+๐ถ0

1 + ๐‘ข = ๐‘’๐ถ0๐‘ฅ1 + ๐‘ข = ๐ถ๐‘ฅ

Where ๐ถ = ๐‘’๐ถ0 is a new constant. Therefore the solution is

๐‘ข (๐‘ฅ) = ๐ถ๐‘ฅ โˆ’ 1

Since ๐‘ข (๐‘ฅ) = โˆš1 + 4๐‘ฃ then the above becomes

โˆš1 + 4๐‘ฃ = ๐ถ๐‘ฅ โˆ’ 1

1 + 4๐‘ฃ = (๐ถ๐‘ฅ โˆ’ 1)2

๐‘ฃ (๐‘ฅ) =(๐ถ๐‘ฅ โˆ’ 1)2 โˆ’ 1

4But ๐‘ฆ = ๐‘ฃ

๐‘ฅ2 therefore the above gives the final solution as

๐‘ฆ (๐‘ฅ) =(๐ถ๐‘ฅ โˆ’ 1)2 โˆ’ 1

4๐‘ฅ2Where ๐ถ is the constant of integration.

Part 2

๐‘’๐‘ฅ sin ๐‘ฆ โˆ’ 2๐‘ฆ sin ๐‘ฅ + ๏ฟฝ๐‘ฆ2 + ๐‘’๐‘ฅ cos ๐‘ฆ + 2 cos ๐‘ฅ๏ฟฝ ๐‘ฆโ€ฒ = 0

100

The first step is to write the ODE in standard form to check if it is an exact ODE

๐‘€(๐‘ฅ, ๐‘ฆ)๐‘‘๐‘ฅ + ๐‘(๐‘ฅ, ๐‘ฆ)๐‘‘๐‘ฆ = 0

Hence

๐‘€(๐‘ฅ, ๐‘ฆ) = ๐‘’๐‘ฅ sin ๐‘ฆ โˆ’ 2 ๐‘ฆ sin ๐‘ฅ๐‘(๐‘ฅ, ๐‘ฆ) = ๐‘ฆ2 + ๐‘’๐‘ฅ cos ๐‘ฆ + 2 cos ๐‘ฅ

Next, the ODE is determined if it is exact or not. The ODE is exact if the followingcondition is satisfied

๐œ•๐‘€๐œ•๐‘ฆ

=๐œ•๐‘๐œ•๐‘ฅ

Applying the above on the given ODE results in

๐œ•๐‘€๐œ•๐‘ฆ

= ๐‘’๐‘ฅ cos ๐‘ฆ โˆ’ 2 sin ๐‘ฅ

๐œ•๐‘๐œ•๐‘ฅ

= ๐‘’๐‘ฅ cos ๐‘ฆ โˆ’ 2 sin ๐‘ฅ

Because ๐œ•๐‘€๐œ•๐‘ฆ = ๐œ•๐‘

๐œ•๐‘ฅ , then the ODE is exact. The following equations are used to solve for

the function ๐œ™ ๏ฟฝ๐‘ฅ, ๐‘ฆ๏ฟฝ

๐œ•๐œ™๐œ•๐‘ฅ

= ๐‘€ = ๐‘’๐‘ฅ sin ๐‘ฆ โˆ’ 2 ๐‘ฆ sin ๐‘ฅ (3)

๐œ•๐œ™๐œ•๐‘ฆ

= ๐‘ = ๐‘ฆ2 + ๐‘’๐‘ฅ cos ๐‘ฆ + 2 cos ๐‘ฅ (4)

Integrating (3) w.r.t ๐‘ฅ gives

๏ฟฝ๐œ•๐œ™๐œ•๐‘ฅ๐‘‘๐‘ฅ = ๏ฟฝ๐‘’๐‘ฅ sin ๐‘ฆ โˆ’ 2 ๐‘ฆ sin ๐‘ฅ๐‘‘๐‘ฅ

๐œ™ ๏ฟฝ๐‘ฅ, ๐‘ฆ๏ฟฝ = ๐‘’๐‘ฅ sin ๐‘ฆ + 2 ๐‘ฆ cos ๐‘ฅ + ๐‘“(๐‘ฆ) (5)

Where ๐‘“(๐‘ฆ) is used as the constant of integration because ๐œ™ ๏ฟฝ๐‘ฅ, ๐‘ฆ๏ฟฝ is a function of both ๐‘ฅand ๐‘ฆ. Taking derivative of (5) w.r.t ๐‘ฆ gives

๐œ•๐œ™๐œ•๐‘ฆ

= ๐‘’๐‘ฅ cos ๐‘ฆ + 2 cos ๐‘ฅ + ๐‘“โ€ฒ(๐‘ฆ) (6)

But (4) says that๐œ•๐œ™๐œ•๐‘ฆ = ๐‘ฆ

2 + ๐‘’๐‘ฅ cos ๐‘ฆ + 2 cos ๐‘ฅ. Therefore by equating (4) and (6) then ๐‘“โ€ฒ ๏ฟฝ๐‘ฆ๏ฟฝcan be solved for:

๐‘ฆ2 + ๐‘’๐‘ฅ cos ๐‘ฆ + 2 cos ๐‘ฅ = ๐‘’๐‘ฅ cos ๐‘ฆ + 2 cos ๐‘ฅ + ๐‘“โ€ฒ(๐‘ฆ) (7)

Solving the above for ๐‘“โ€ฒ(๐‘ฆ) gives

๐‘“โ€ฒ(๐‘ฆ) = ๐‘ฆ2

Integrating w.r.t ๐‘ฆ gives ๐‘“ ๏ฟฝ๐‘ฆ๏ฟฝ

๏ฟฝ๐‘“โ€ฒ๐‘‘๐‘ฆ = ๏ฟฝ๐‘ฆ2๐‘‘๐‘ฆ

๐‘“(๐‘ฆ) =13๐‘ฆ3 + ๐ถ1

Where ๐ถ1 is constant of integration. Substituting the value of ๐‘“(๐‘ฆ) back into (5) gives ๐œ™ ๏ฟฝ๐‘ฅ, ๐‘ฆ๏ฟฝ

๐œ™ = ๐‘’๐‘ฅ sin ๐‘ฆ + 2 ๐‘ฆ cos ๐‘ฅ + 13๐‘ฆ3 + ๐ถ1

But since ๐œ™ itself is a constant function, say ๐œ™ = ๐ถ0 where ๐ถ0 is new constant, then bycombining ๐ถ1 and ๐ถ0 constants into a new constant ๐ถ1, the above gives the solution

๐ถ1 = ๐‘’๐‘ฅ sin ๐‘ฆ (๐‘ฅ) + 2 ๐‘ฆ (๐‘ฅ) cos ๐‘ฅ + 13๐‘ฆ3 (๐‘ฅ)

The above is left in implicit form for simplicity.

Part 3

๐‘ฆโ€ฒ + ๐‘ฆ cos ๐‘ฅ = 12

sin (2๐‘ฅ)

101

This ODE is linear in ๐‘ฆ. It is solved using an integrating factor ๐œ‡ = ๐‘’โˆซ cos ๐‘ฅ๐‘‘๐‘ฅ = ๐‘’sin ๐‘ฅ.Multiplying both sides of the ODE by ๐œ‡ makes the left side an exact di๏ฟฝerential

๐‘‘ ๏ฟฝ๐‘ฆ๐œ‡๏ฟฝ =12๐œ‡ sin (2) ๐‘ฅ๐‘‘๐‘ฅ

Integrating both sides gives

๐‘ฆ๐œ‡ =12 ๏ฟฝ

๐œ‡ sin (2๐‘ฅ) ๐‘‘๐‘ฅ + ๐ถ

๐‘ฆ๐‘’sin ๐‘ฅ =12 ๏ฟฝ

๐‘’sin ๐‘ฅ sin (2๐‘ฅ) ๐‘‘๐‘ฅ + ๐ถ (1)

The above integral can be solved as follows. Since sin (2๐‘ฅ) = 2 sin ๐‘ฅ cos ๐‘ฅ therefore then

๐ผ =12 ๏ฟฝ

๐‘’sin ๐‘ฅ sin (2๐‘ฅ) ๐‘‘๐‘ฅ = ๏ฟฝ๐‘’sin ๐‘ฅ sin ๐‘ฅ cos ๐‘ฅ๐‘‘๐‘ฅ

Using the substitution ๐‘ง = sin ๐‘ฅ, then ๐‘‘๐‘ง = ๐‘‘๐‘ฅ cos ๐‘ฅ and the above becomes

๐ผ = ๏ฟฝ๐‘’๐‘ง๐‘ง๐‘‘๐‘ง

Integrating the above by parts: โˆซ๐‘ข๐‘‘๐‘ฃ = ๐‘ข๐‘ฃ โˆ’ โˆซ๐‘ฃ๐‘‘๐‘ข. Let ๐‘ข = ๐‘ง, ๐‘‘๐‘ฃ = ๐‘’๐‘ง โ†’ ๐‘‘๐‘ข = 1, ๐‘ฃ = ๐‘’๐‘ง, andthe above becomes

๐ผ = ๐‘ง๐‘’๐‘ง โˆ’๏ฟฝ๐‘’๐‘ง๐‘‘๐‘ง

= ๐‘ง๐‘’๐‘ง โˆ’ ๐‘’๐‘ง

= ๐‘’๐‘ง (๐‘ง โˆ’ 1)

Since ๐‘ง = sin ๐‘ฅ the above reduces to๐ผ = ๐‘’sin ๐‘ฅ (sin (๐‘ฅ) โˆ’ 1)

Substituting this back in (1) results in

๐‘ฆ๐‘’sin ๐‘ฅ = ๐‘’sin ๐‘ฅ (sin (๐‘ฅ) โˆ’ 1) + ๐ถTherefore the final solution is

๐‘ฆ (๐‘ฅ) = sin (๐‘ฅ) โˆ’ 1 + ๐ถ๐‘’โˆ’ sin ๐‘ฅ

Where ๐ถ is the constant of integration.

3.5.3 Problem 3

Find general solution of

1. ๐‘ฆโ€ฒโ€ฒโ€ฒ โˆ’ 4๐‘ฆโ€ฒโ€ฒ โˆ’ 4๐‘ฆโ€ฒ + 16 = 8 sin ๐‘ฅ

2. ๐‘Ž2๐‘ฆโ€ฒ2 = ๏ฟฝ1 + ๐‘ฆโ€ฒ2๏ฟฝ3

Solution

Part 1

๐‘ฆโ€ฒโ€ฒโ€ฒ โˆ’ 4๐‘ฆโ€ฒโ€ฒ โˆ’ 4๐‘ฆโ€ฒ = 8 sin ๐‘ฅ โˆ’ 16This is linear nonhomogeneous ODE with constant coe๏ฟฝcients. Solving first the homo-geneous ODE ๐‘ฆโ€ฒโ€ฒโ€ฒ โˆ’ 4๐‘ฆโ€ฒโ€ฒ โˆ’ 4๐‘ฆโ€ฒ = 0. Since the term ๐‘ฆ is missing from the ODE then thesubstitution ๐‘ฆโ€ฒ = ๐‘ข reduces the ODE to a second order ODE

๐‘ขโ€ฒโ€ฒ โˆ’ 4๐‘ขโ€ฒ โˆ’ 4๐‘ข = 0 (1)

Let ๐‘ข = ๐‘’๐œ†๐‘ฅ. Substituting this into the above and simplifying gives the characteristic equation

๐œ†2 โˆ’ 4๐œ† โˆ’ 4 = 0

102

The Roots are ๐œ† = โˆ’ ๐‘2๐‘Ž ยฑ

12๐‘Žโˆš๐‘

2 โˆ’ 4๐‘Ž๐‘ or

๐œ† =42ยฑ12๏ฟฝ

16 โˆ’ 4 (โˆ’4)

= 2 ยฑ12โˆš

32

= 2 ยฑ 2โˆš2

= 2 ๏ฟฝ1 ยฑ โˆš2๏ฟฝ

Hence the solution to (1) is given by linear combinations of ๐‘’๐œ†1๐‘ฅ, ๐‘’๐œ†2๐‘ฅ as

๐‘ขโ„Ž (๐‘ฅ) = ๐‘1๐‘’2๏ฟฝ1+โˆš2๏ฟฝ๐‘ฅ + ๐‘2๐‘’

2๏ฟฝ1โˆ’โˆš2๏ฟฝ๐‘ฅ

But since ๐‘ฆโ€ฒ = ๐‘ข, then ๐‘ฆ is found by integrating the above

๐‘ฆโ„Ž = ๏ฟฝ๐‘1๐‘’2๏ฟฝ1+โˆš2๏ฟฝ๐‘ฅ + ๐‘2๐‘’

2๏ฟฝ1โˆ’โˆš2๏ฟฝ๐‘ฅ๐‘‘๐‘ฅ

= ๐‘1๐‘’2๏ฟฝ1+โˆš2๏ฟฝ๐‘ฅ

2 ๏ฟฝ1 + โˆš2๏ฟฝ+ ๐‘2

๐‘’๏ฟฝ2โˆ’2โˆš2๏ฟฝ๐‘ฅ

2 ๏ฟฝ1 โˆ’ โˆš2๏ฟฝ+ ๐ถ3

To simplify the above, let ๐‘12๏ฟฝ1+โˆš2๏ฟฝ

= ๐ถ1,๐‘2

2๏ฟฝ1โˆ’โˆš2๏ฟฝ= ๐ถ2, where ๐ถ1, ๐ถ2 are new constants. The

above simplifies to

๐‘ฆโ„Ž = ๐ถ1๐‘’2๏ฟฝ1+โˆš2๏ฟฝ๐‘ฅ + ๐ถ2๐‘’

๏ฟฝ2โˆ’2โˆš2๏ฟฝ๐‘ฅ + ๐ถ3The above solution is homogeneous solution to the original ODE. Next, the particularsolution is found. Since the RHS of the original ODE is sin ๐‘ฅ โˆ’ 16 then choosing ๐‘ฆ๐‘ to havethe form

๐‘ฆ๐‘ = ๐ด sin ๐‘ฅ + ๐ต cos ๐‘ฅ + ๐‘˜๐‘ฅTherefore

๐‘ฆโ€ฒ๐‘ = ๐‘˜ + ๐ด cos ๐‘ฅ โˆ’ ๐ต sin ๐‘ฅ๐‘ฆโ€ฒโ€ฒ๐‘ = โˆ’๐ด sin ๐‘ฅ โˆ’ ๐ต cos ๐‘ฅ๐‘ฆโ€ฒโ€ฒโ€ฒ๐‘ = โˆ’๐ด cos ๐‘ฅ + ๐ต sin ๐‘ฅ

Substituting these back into the original ODE ๐‘ฆโ€ฒโ€ฒโ€ฒ โˆ’ 4๐‘ฆโ€ฒโ€ฒ โˆ’ 4๐‘ฆโ€ฒ = 8 sin ๐‘ฅ โˆ’ 16 gives(โˆ’๐ด cos ๐‘ฅ + ๐ต sin ๐‘ฅ) โˆ’ 4 (โˆ’๐ด sin ๐‘ฅ โˆ’ ๐ต cos ๐‘ฅ) โˆ’ 4 (๐‘˜ + ๐ด cos ๐‘ฅ โˆ’ ๐ต sin ๐‘ฅ) = 8 sin ๐‘ฅ โˆ’ 16โˆ’๐ด cos ๐‘ฅ + ๐ต sin ๐‘ฅ + 4๐ด sin ๐‘ฅ + 4๐ต cos ๐‘ฅ โˆ’ 4๐ด cos ๐‘ฅ + 4๐ต sin ๐‘ฅ โˆ’ 4๐‘˜ = 8 sin ๐‘ฅ โˆ’ 16

cos ๐‘ฅ (โˆ’๐ด + 4๐ต โˆ’ 4๐ด) + sin ๐‘ฅ (๐ต + 4๐ด + 4๐ต) โˆ’ 4๐‘˜ = 8 sin ๐‘ฅ โˆ’ 16cos ๐‘ฅ (โˆ’5๐ด + 4๐ต) + sin ๐‘ฅ (5๐ต + 4๐ด) โˆ’ 4๐‘˜ = 8 sin ๐‘ฅ โˆ’ 16

Comparing coe๏ฟฝcients gives the following equations to solve for the unknowns ๐ด,๐ต, ๐‘˜

โˆ’4๐‘˜ = โˆ’16โˆ’5๐ด + 4๐ต = 05๐ต + 4๐ด = 8

The second equation gives ๐ต = 54๐ด. Using this in the third equation gives 5 ๏ฟฝ54๐ด๏ฟฝ + 4๐ด = 8,

solving gives ๐ด = 3241 . Hence ๐ต = 5

4๏ฟฝ3241๏ฟฝ = 40

41 . The first equation gives ๐‘˜ = 4. Therefore theparticular solution is

๐‘ฆ๐‘ = ๐ด sin ๐‘ฅ + ๐ต cos ๐‘ฅ + ๐‘˜๐‘ฅ

=3241

sin ๐‘ฅ + 4041

cos ๐‘ฅ + 4๐‘ฅ

Now that ๐‘ฆโ„Ž and ๐‘ฆ๐‘ are found, the general solution is found as

๐‘ฆ = ๐‘ฆโ„Ž + ๐‘ฆ๐‘

= ๐ถ1๐‘’2๏ฟฝ1+โˆš2๏ฟฝ๐‘ฅ + ๐ถ2๐‘’

๏ฟฝ2โˆ’2โˆš2๏ฟฝ๐‘ฅ + ๐ถ3 +3241

sin ๐‘ฅ + 4041

cos ๐‘ฅ + 4๐‘ฅ

Where ๐ถ1, ๐ถ2 are the two constants of integration.

103

Part 2

๐‘Ž2๐‘ฆโ€ฒ2 = ๏ฟฝ1 + ๐‘ฆโ€ฒ2๏ฟฝ3

Let ๐‘ฆโ€ฒ = ๐ด, the above becomes

๐‘Ž2๐ด2 = ๏ฟฝ1 + ๐ด2๏ฟฝ3

= 1 + 3๐ด2 +(3) (2)2!

๐ด4 +(3) (2) (1)

3!๐ด6

= 1 + 3๐ด2 + 3๐ด4 + ๐ด6

Hence the polynomial is

๐ด6 + 3๐ด4 + ๐ด2 ๏ฟฝ3 โˆ’ ๐‘Ž2๏ฟฝ + 1 = 0

Let ๐ด2 = ๐ต and the above becomes

๐ต3 + 3๐ต2 + ๐ต ๏ฟฝ3 โˆ’ ๐‘Ž2๏ฟฝ + 1 = 0

With the help of the computer, the cubic roots of the above are

๐ต1 =3

๏ฟฝ๏ฟฝ14๐‘Ž4 โˆ’

127๐‘Ž6 โˆ’

12๐‘Ž2 +

13

๐‘Ž2

3

๏ฟฝ๏ฟฝ14๐‘Ž4 โˆ’ 1

27๐‘Ž6 โˆ’ 1

2๐‘Ž2โˆ’ 1

๐ต2 =12๐‘–โˆš3

โŽ›โŽœโŽœโŽœโŽœโŽœโŽœโŽœโŽœโŽœโŽœโŽœโŽ

3

๏ฟฝ๏ฟฝ14๐‘Ž4 โˆ’

127๐‘Ž6 โˆ’

12๐‘Ž2 โˆ’

13

๐‘Ž2

3

๏ฟฝ๏ฟฝ14๐‘Ž4 โˆ’ 1

27๐‘Ž6 โˆ’ 1

2๐‘Ž2

โŽžโŽŸโŽŸโŽŸโŽŸโŽŸโŽŸโŽŸโŽŸโŽŸโŽŸโŽŸโŽ โˆ’12

3

๏ฟฝ๏ฟฝ14๐‘Ž4 โˆ’

127๐‘Ž6 โˆ’

12๐‘Ž2 โˆ’

16

๐‘Ž2

3

๏ฟฝ๏ฟฝ14๐‘Ž4 โˆ’ 1

27๐‘Ž6 โˆ’ 1

2๐‘Ž2โˆ’ 1

๐ต3 = โˆ’12

3

๏ฟฝ๏ฟฝ14๐‘Ž4 โˆ’

127๐‘Ž6 โˆ’

12๐‘Ž2 โˆ’

12๐‘–โˆš3

โŽ›โŽœโŽœโŽœโŽœโŽœโŽœโŽœโŽœโŽœโŽœโŽœโŽ

3

๏ฟฝ๏ฟฝ14๐‘Ž4 โˆ’

127๐‘Ž6 โˆ’

12๐‘Ž2 โˆ’

13

๐‘Ž2

3

๏ฟฝ๏ฟฝ14๐‘Ž4 โˆ’ 1

27๐‘Ž6 โˆ’ 1

2๐‘Ž2

โŽžโŽŸโŽŸโŽŸโŽŸโŽŸโŽŸโŽŸโŽŸโŽŸโŽŸโŽŸโŽ โˆ’16

๐‘Ž2

3

๏ฟฝ๏ฟฝ14๐‘Ž4 โˆ’ 1

27๐‘Ž6 โˆ’ 1

2๐‘Ž2โˆ’ 1

Therefore ๐ด1 = ยฑโˆš๐ต1, ๐ด2 = ยฑโˆš๐ต2, ๐ด3 = ยฑโˆš๐ต3 or, since ๐‘ฆโ€ฒ (๐‘ฅ) = ๐ด , then there are 6 solutions,each is a solution for one root.

๐‘‘๐‘ฆ1๐‘‘๐‘ฅ

= +โˆš๐ต1๐‘‘๐‘ฆ2๐‘‘๐‘ฅ

= โˆ’โˆš๐ต1๐‘‘๐‘ฆ3๐‘‘๐‘ฅ

= +โˆš๐ต2๐‘‘๐‘ฆ4๐‘‘๐‘ฅ

= โˆ’โˆš๐ต2๐‘‘๐‘ฆ5๐‘‘๐‘ฅ

= +๏ฟฝ๐ต3๐‘‘๐‘ฆ6๐‘‘๐‘ฅ

= โˆ’๏ฟฝ๐ต3

But the roots ยฑ๐ต๐‘– are constants. Therefore each of the above can be solved by directintegration. The final solution which gives the solutions

๐‘ฆ1 = โˆš๐ต1๐‘ฅ + ๐ถ1๐‘ฆ2 = โˆ’โˆš๐ต1๐‘ฅ + ๐ถ2๐‘ฆ3 = โˆš๐ต2๐‘ฅ + ๐ถ3๐‘ฆ4 = โˆ’โˆš๐ต2๐‘ฅ + ๐ถ4๐‘ฆ5 = ๏ฟฝ๐ต3๐‘ฅ + ๐ถ5๐‘ฆ6 = โˆ’๏ฟฝ๐ต3๐‘ฅ + ๐ถ6

Where the constants ๐ต๐‘– are given above.

104

3.5.4 key solution to HW 5

105

106

107

108

109

110

3.6 HW 6

3.6.1 Problem 1

Consider the equation ๐‘ฅ๐‘ฆโ€ฒโ€ฒ + (๐‘ โˆ’ ๐‘ฅ) ๐‘ฆโ€ฒ โˆ’ ๐‘Ž๐‘ฆ = 0. Identify a regular singular point and findtwo series solutions around this point. Test the solutions for convergence.

Solution

111

Writing the ODE as

๐‘ฆโ€ฒโ€ฒ + ๐ด (๐‘ฅ) ๐‘ฆโ€ฒ + ๐ต (๐‘ฅ) ๐‘ฆ = 0

Where

๐ด (๐‘ฅ) =(๐‘ โˆ’ ๐‘ฅ)๐‘ฅ

๐ต (๐‘ฅ) =โˆ’๐‘Ž๐‘ฅ

The above shows that ๐‘ฅ0 = 0 is a singularity point for both ๐ด (๐‘ฅ) and ๐ต (๐‘ฅ). Examining ๐ด (๐‘ฅ)and ๐ต (๐‘ฅ) to determine what type of singular point it is

lim๐‘ฅโ†’๐‘ฅ0

(๐‘ฅ โˆ’ ๐‘ฅ0) ๐ด (๐‘ฅ) = lim๐‘ฅโ†’0

๐‘ฅ(๐‘ โˆ’ ๐‘ฅ)๐‘ฅ

= lim๐‘ฅโ†’0

(๐‘ โˆ’ ๐‘ฅ) = ๐‘

Because the limit exists, then ๐‘ฅ0 = 0 is regular singular point for ๐ด (๐‘ฅ) .

lim๐‘ฅโ†’๐‘ฅ0

(๐‘ฅ โˆ’ ๐‘ฅ0)2 ๐ต (๐‘ฅ) = lim

๐‘ฅโ†’0๐‘ฅ2 ๏ฟฝ

โˆ’๐‘Ž๐‘ฅ๏ฟฝ = lim

๐‘ฅโ†’0(โˆ’๐‘Ž๐‘ฅ) = 0

Because the limit exists, then ๐‘ฅ0 = 0 is also regular singular point for ๐ต (๐‘ฅ).

Therefore ๐‘ฅ0 = 0 is a regular singular point for the ODE.

Assuming the solution is Frobenius series gives

๐‘ฆ (๐‘ฅ) = ๐‘ฅ๐‘Ÿโˆž๏ฟฝ๐‘›=0

๐ถ๐‘› (๐‘ฅ โˆ’ ๐‘ฅ0)๐‘› ๐ถ0 โ‰  0

= ๐‘ฅ๐‘Ÿโˆž๏ฟฝ๐‘›=0

๐ถ๐‘›๐‘ฅ๐‘›

=โˆž๏ฟฝ๐‘›=0

๐ถ๐‘›๐‘ฅ๐‘›+๐‘Ÿ

Therefore

๐‘ฆโ€ฒ =โˆž๏ฟฝ๐‘›=0

(๐‘› + ๐‘Ÿ) ๐ถ๐‘›๐‘ฅ๐‘›+๐‘Ÿโˆ’1

๐‘ฆโ€ฒโ€ฒ =โˆž๏ฟฝ๐‘›=0

(๐‘› + ๐‘Ÿ) (๐‘› + ๐‘Ÿ โˆ’ 1) ๐ถ๐‘›๐‘ฅ๐‘›+๐‘Ÿโˆ’2

Substituting the above in the original ODE ๐‘ฅ๐‘ฆโ€ฒโ€ฒ + (๐‘ โˆ’ ๐‘ฅ) ๐‘ฆโ€ฒ โˆ’ ๐‘Ž๐‘ฆ = 0 gives

๐‘ฅโˆž๏ฟฝ๐‘›=0

(๐‘› + ๐‘Ÿ) (๐‘› + ๐‘Ÿ โˆ’ 1) ๐ถ๐‘›๐‘ฅ๐‘›+๐‘Ÿโˆ’2 + (๐‘ โˆ’ ๐‘ฅ)โˆž๏ฟฝ๐‘›=0

(๐‘› + ๐‘Ÿ) ๐ถ๐‘›๐‘ฅ๐‘›+๐‘Ÿโˆ’1 โˆ’ ๐‘Žโˆž๏ฟฝ๐‘›=0

๐ถ๐‘›๐‘ฅ๐‘›+๐‘Ÿ = 0

โˆž๏ฟฝ๐‘›=0

(๐‘› + ๐‘Ÿ) (๐‘› + ๐‘Ÿ โˆ’ 1) ๐‘๐‘›๐‘ฅ๐‘›+๐‘Ÿโˆ’1 + ๐‘โˆž๏ฟฝ๐‘›=0

(๐‘› + ๐‘Ÿ) ๐ถ๐‘›๐‘ฅ๐‘›+๐‘Ÿโˆ’1 โˆ’ ๐‘ฅโˆž๏ฟฝ๐‘›=0

(๐‘› + ๐‘Ÿ) ๐ถ๐‘›๐‘ฅ๐‘›+๐‘Ÿโˆ’1 โˆ’โˆž๏ฟฝ๐‘›=0

๐‘Ž๐ถ๐‘›๐‘ฅ๐‘›+๐‘Ÿ = 0

โˆž๏ฟฝ๐‘›=0

(๐‘› + ๐‘Ÿ) (๐‘› + ๐‘Ÿ โˆ’ 1) ๐ถ๐‘›๐‘ฅ๐‘›+๐‘Ÿโˆ’1 +โˆž๏ฟฝ๐‘›=0

๐‘ (๐‘› + ๐‘Ÿ) ๐ถ๐‘›๐‘ฅ๐‘›+๐‘Ÿโˆ’1 โˆ’โˆž๏ฟฝ๐‘›=0

(๐‘› + ๐‘Ÿ) ๐ถ๐‘›๐‘ฅ๐‘›+๐‘Ÿ โˆ’โˆž๏ฟฝ๐‘›=0

๐‘Ž๐ถ๐‘›๐‘ฅ๐‘›+๐‘Ÿ = 0

โˆž๏ฟฝ๐‘›=0

((๐‘› + ๐‘Ÿ) (๐‘› + ๐‘Ÿ โˆ’ 1) + ๐‘ (๐‘› + ๐‘Ÿ)) ๐ถ๐‘›๐‘ฅ๐‘›+๐‘Ÿโˆ’1 โˆ’โˆž๏ฟฝ๐‘›=0

((๐‘› + ๐‘Ÿ) + ๐‘Ž) ๐ถ๐‘›๐‘ฅ๐‘›+๐‘Ÿ = 0

Since all powers of ๐‘ฅ have to be the same, adjusting indices and exponents gives (wherein the second sum above, the outside index ๐‘› is increased by 1 and ๐‘› inside the sum isdecreased by 1)

โˆž๏ฟฝ๐‘›=0

((๐‘› + ๐‘Ÿ) (๐‘› + ๐‘Ÿ โˆ’ 1) + ๐‘ (๐‘› + ๐‘Ÿ)) ๐ถ๐‘›๐‘ฅ๐‘›+๐‘Ÿโˆ’1 โˆ’โˆž๏ฟฝ๐‘›=1

((๐‘› โˆ’ 1 + ๐‘Ÿ) + ๐‘Ž) ๐ถ๐‘›โˆ’1๐‘ฅ๐‘›+๐‘Ÿโˆ’1 = 0 (1)

Setting ๐‘› = 0 gives the indicial equation, which only comes from the first sum above as thesecond sum starts from ๐‘› = 1.

((๐‘Ÿ) (๐‘Ÿ โˆ’ 1) + ๐‘๐‘Ÿ) ๐ถ0 = 0

Since ๐ถ0 โ‰  0 then(๐‘Ÿ) (๐‘Ÿ โˆ’ 1) + ๐‘๐‘Ÿ = 0

๐‘Ÿ2 โˆ’ ๐‘Ÿ + ๐‘๐‘Ÿ = 0๐‘Ÿ (๐‘Ÿ + ๐‘ โˆ’ 1) = 0

112

The roots are

๐‘Ÿ1 = 1 โˆ’ ๐‘๐‘Ÿ2 = 0

Assuming that ๐‘Ÿ2 โˆ’ ๐‘Ÿ1 is not an integer, in other words, assuming 1 โˆ’ ๐‘ is not an integer(problem did not say), then In this case, two linearly independent solutions can be con-structed directly. The first is associated with ๐‘Ÿ1 = 1 โˆ’ ๐‘ and the second is associated with๐‘Ÿ2 = 0. These solutions are

๐‘ฆ1 (๐‘ฅ) =โˆž๏ฟฝ๐‘›=0

๐ถ๐‘›๐‘ฅ๐‘›+1โˆ’๐‘ ๐ถ0 โ‰  0

๐‘ฆ2 (๐‘ฅ) =โˆž๏ฟฝ๐‘›=0

๐ท๐‘›๐‘ฅ๐‘› ๐ท0 โ‰  0

The coe๏ฟฝcients are not the same in each solution. For the first one ๐ถ๐‘› is used and for thesecond ๐ท๐‘› is used.

The solution ๐‘ฆ1 (๐‘ฅ) associated with ๐‘Ÿ1 = 1 โˆ’ ๐‘ is now found. From (1), and replacing ๐‘Ÿ by1 โˆ’ ๐‘ givesโˆž๏ฟฝ๐‘›=0

((๐‘› + 1 โˆ’ ๐‘) (๐‘› + 1 โˆ’ ๐‘ โˆ’ 1) + ๐‘ (๐‘› + 1 โˆ’ ๐‘)) ๐ถ๐‘›๐‘ฅ๐‘›+1โˆ’๐‘โˆ’1 โˆ’โˆž๏ฟฝ๐‘›=1

((๐‘› โˆ’ 1 + 1 โˆ’ ๐‘) + ๐‘Ž) ๐ถ๐‘›โˆ’1๐‘ฅ๐‘›+1โˆ’๐‘โˆ’1 = 0

โˆž๏ฟฝ๐‘›=0

((๐‘› + 1 โˆ’ ๐‘) (๐‘› โˆ’ ๐‘) + ๐‘ (๐‘› + 1 โˆ’ ๐‘)) ๐ถ๐‘›๐‘ฅ๐‘›โˆ’๐‘ โˆ’โˆž๏ฟฝ๐‘›=1

((๐‘› โˆ’ ๐‘) + ๐‘Ž) ๐ถ๐‘›โˆ’1๐‘ฅ๐‘›โˆ’๐‘ = 0

โˆž๏ฟฝ๐‘›=0

๐‘› (๐‘› โˆ’ ๐‘ + 1) ๐ถ๐‘›๐‘ฅ๐‘›โˆ’๐‘ โˆ’โˆž๏ฟฝ๐‘›=1

((๐‘› โˆ’ ๐‘) + ๐‘Ž) ๐ถ๐‘›โˆ’1๐‘ฅ๐‘›โˆ’๐‘ = 0

For ๐‘› > 0 the above gives the recursive relation (๐‘› = 0 is not used, since it was used to find๐‘Ÿ). For ๐‘› > 0 the last equation above gives

๐‘› (๐‘› โˆ’ ๐‘ + 1) ๐ถ๐‘› โˆ’ ((๐‘› โˆ’ ๐‘) + ๐‘Ž) ๐ถ๐‘›โˆ’1 = 0

๐ถ๐‘› =((๐‘› โˆ’ ๐‘) + ๐‘Ž)๐‘› (๐‘› โˆ’ ๐‘ + 1)

๐ถ๐‘›โˆ’1

Few terms are generated to see the pattern. For ๐‘› = 1

๐ถ1 =(1 โˆ’ ๐‘ + ๐‘Ž)1 (1 โˆ’ ๐‘ + 1)

๐ถ0 =(1 โˆ’ ๐‘ + ๐‘Ž)(2 โˆ’ ๐‘)

๐ถ0

For ๐‘› = 2

๐ถ2 =(2 โˆ’ ๐‘ + ๐‘Ž)2 (2 โˆ’ ๐‘ + 1)

๐ถ1

=(2 โˆ’ ๐‘ + ๐‘Ž)2 (3 โˆ’ ๐‘)

(1 โˆ’ ๐‘ + ๐‘Ž)(2 โˆ’ ๐‘)

๐ถ0

For ๐‘› = 3

๐ถ3 =(3 โˆ’ ๐‘ + ๐‘Ž)3 (3 โˆ’ ๐‘ + 1)

๐ถ2

=(3 โˆ’ ๐‘ + ๐‘Ž)3 (4 โˆ’ ๐‘)

(2 โˆ’ ๐‘ + ๐‘Ž)2 (3 โˆ’ ๐‘)

(1 โˆ’ ๐‘ + ๐‘Ž)(2 โˆ’ ๐‘)

๐ถ0

And so on. The pattern for general term is

๐ถ๐‘› =((๐‘› โˆ’ ๐‘) + ๐‘Ž)๐‘› (๐‘› โˆ’ ๐‘ + 1)

โ‹… โ‹… โ‹…(3 โˆ’ ๐‘ + ๐‘Ž)3 (3 โˆ’ ๐‘ + 1)

(2 โˆ’ ๐‘ + ๐‘Ž)2 (2 โˆ’ ๐‘ + 1)

(1 โˆ’ ๐‘ + ๐‘Ž)1 (1 โˆ’ ๐‘ + 1)

๐ถ0

=๐‘›๏ฟฝ๐‘š=1

((๐‘š โˆ’ ๐‘) + ๐‘Ž)๐‘š (๐‘› โˆ’ ๐‘ + 1)

Therefore the solution associated with ๐‘Ÿ1 = 1 โˆ’ ๐‘ is

๐‘ฆ1 (๐‘ฅ) =โˆž๏ฟฝ๐‘›=0

๐ถ๐‘›๐‘ฅ๐‘›+๐‘Ÿ

=โˆž๏ฟฝ๐‘›=0

๐ถ๐‘›๐‘ฅ๐‘›+1โˆ’๐‘

= ๐ถ0๐‘ฅ1โˆ’๐‘ + ๐ถ1๐‘ฅ2โˆ’๐‘ + ๐ถ2๐‘ฅ3โˆ’๐‘ +โ‹ฏ

113

Using results found above, and looking at few terms gives the first solution as

๐‘ฆ1 (๐‘ฅ) = ๐ถ0๐‘ฅ1โˆ’๐‘ ๏ฟฝ1 +(1 โˆ’ ๐‘ + ๐‘Ž)(2 โˆ’ ๐‘)

๐‘ฅ +12(2 โˆ’ ๐‘ + ๐‘Ž)(3 โˆ’ ๐‘)

(1 โˆ’ ๐‘ + ๐‘Ž)(2 โˆ’ ๐‘)

๐‘ฅ2 +16(3 โˆ’ ๐‘ + ๐‘Ž)(4 โˆ’ ๐‘)

(2 โˆ’ ๐‘ + ๐‘Ž)(3 โˆ’ ๐‘)

(1 โˆ’ ๐‘ + ๐‘Ž)(2 โˆ’ ๐‘)

๐‘ฅ3 +โ‹ฏ๏ฟฝ

The second solution associated with ๐‘Ÿ2 = 0 is now found. As above, using (1) but with ๐ท๐‘›instead of ๐ถ๐‘› for coe๏ฟฝcients and replacing ๐‘Ÿ by zero gives

โˆž๏ฟฝ๐‘›=0

(๐‘› (๐‘› โˆ’ 1) + ๐‘๐‘›)๐ท๐‘›๐‘ฅ๐‘›โˆ’1 โˆ’โˆž๏ฟฝ๐‘›=1

((๐‘› โˆ’ 1) + ๐‘Ž)๐ท๐‘›โˆ’1๐‘ฅ๐‘›โˆ’1 = 0

For ๐‘› > 0 the above gives the recursive relation for the second solution

(๐‘› (๐‘› โˆ’ 1) + ๐‘๐‘›)๐ท๐‘› โˆ’ ((๐‘› โˆ’ 1) + ๐‘Ž)๐ท๐‘›โˆ’1 = 0

๐ท๐‘› =๐‘› โˆ’ 1 + ๐‘Ž

๐‘› (๐‘› โˆ’ 1) + ๐‘๐‘›๐ท๐‘›โˆ’1

=๐‘› โˆ’ 1 + ๐‘Ž๐‘๐‘› โˆ’ ๐‘› + ๐‘›2

๐ท๐‘›โˆ’1

Few terms are now generated to see the pattern. For ๐‘› = 1

๐ท1 =๐‘Ž๐‘๐ท0

For ๐‘› = 2

๐ท2 =1 + ๐‘Ž

2๐‘ โˆ’ 2 + 4๐ท1

=1 + ๐‘Ž2 (๐‘ + 1)

๐‘Ž๐‘๐ท0

For ๐‘› = 3

๐ท3 =3 โˆ’ 1 + ๐‘Ž3๐‘ โˆ’ 3 + 9

๐ท2

=2 + ๐‘Ž3 (๐‘ + 2)

1 + ๐‘Ž2 (๐‘ + 1)

๐‘Ž๐‘๐ท0

And so on. Hence the solution ๐‘ฆ2 (๐‘ฅ) is

๐‘ฆ2 (๐‘ฅ) =โˆž๏ฟฝ๐‘›=0

๐ท๐‘›๐‘ฅ๐‘›

= ๐ท0 + ๐ท1๐‘ฅ + ๐ท2๐‘ฅ2 +โ‹ฏ

Using result found above gives the second solution as

๐‘ฆ2 (๐‘ฅ) = ๐ท0 ๏ฟฝ1 +๐‘Ž๐‘๐‘ฅ +

12(1 + ๐‘Ž) ๐‘Ž๐‘ (๐‘ + 1)

๐‘ฅ2 +16๐‘Ž (1 + ๐‘Ž) (2 + ๐‘Ž)๐‘ (๐‘ + 2) (๐‘ + 1)

๐‘ฅ3 +โ‹ฏ๏ฟฝ

The final solution is therefore the sum of the two solutions

๐‘ฆ (๐‘ฅ) = ๐ถ0๐‘ฅ1โˆ’๐‘ ๏ฟฝ1 +(1 โˆ’ ๐‘ + ๐‘Ž)(2 โˆ’ ๐‘)

๐‘ฅ +12(2 โˆ’ ๐‘ + ๐‘Ž)(3 โˆ’ ๐‘)

(1 โˆ’ ๐‘ + ๐‘Ž)(2 โˆ’ ๐‘)

๐‘ฅ2 +16(3 โˆ’ ๐‘ + ๐‘Ž)(4 โˆ’ ๐‘)

(2 โˆ’ ๐‘ + ๐‘Ž)(3 โˆ’ ๐‘)

(1 โˆ’ ๐‘ + ๐‘Ž)(2 โˆ’ ๐‘)

๐‘ฅ3 +โ‹ฏ๏ฟฝ

(2)

+ ๐ท0 ๏ฟฝ1 +๐‘Ž๐‘๐‘ฅ +

12(1 + ๐‘Ž) ๐‘Ž๐‘ (๐‘ + 1)

๐‘ฅ2 +16๐‘Ž (1 + ๐‘Ž) (2 + ๐‘Ž)๐‘ (๐‘ + 2) (๐‘ + 1)

๐‘ฅ3 +โ‹ฏ๏ฟฝ

Where ๐ถ0, ๐ท0 are the two constant of integration.

Testing for convergence. For ๐‘ฆ1 (๐‘ฅ) solution, the general term from above was

๐ถ๐‘›๐‘ฅ๐‘› =((๐‘› โˆ’ ๐‘) + ๐‘Ž)๐‘› (๐‘› โˆ’ ๐‘ + 1)

๐ถ๐‘›โˆ’1๐‘ฅ๐‘›

114

Hence by ratio test

๐ฟ = lim๐‘›โ†’โˆž

๏ฟฝ๐ถ๐‘›๐‘ฅ๐‘›

๐ถ๐‘›โˆ’1๐‘ฅ๐‘›โˆ’1๏ฟฝ

= lim๐‘›โ†’โˆž

๏ฟฝ๏ฟฝ

((๐‘›โˆ’๐‘)+๐‘Ž)๐‘›(๐‘›โˆ’๐‘+1)๐ถ๐‘›โˆ’1๐‘ฅ

๐‘›

๐ถ๐‘›โˆ’1๐‘ฅ๐‘›โˆ’1๏ฟฝ๏ฟฝ

= lim๐‘›โ†’โˆž

๏ฟฝ((๐‘› โˆ’ ๐‘) + ๐‘Ž) ๐‘ฅ(๐‘› (๐‘› โˆ’ ๐‘ + 1))

๏ฟฝ

= |๐‘ฅ| lim๐‘›โ†’โˆž

๏ฟฝ๐‘› โˆ’ ๐‘ + ๐‘Ž๐‘›2 โˆ’ ๐‘›๐‘ + ๐‘›

๏ฟฝ

= |๐‘ฅ| lim๐‘›โ†’โˆž

๏ฟฝ๏ฟฝ

1๐‘› โˆ’

๐‘๐‘›2 +

๐‘Ž๐‘›2

1 โˆ’ ๐‘๐‘› +

1๐‘›

๏ฟฝ๏ฟฝ

= |๐‘ฅ| ๏ฟฝ01๏ฟฝ

= 0

Therefore the series ๐‘ฆ1 (๐‘ฅ) converges for all ๐‘ฅ.

Testing for convergence. For ๐‘ฆ2 (๐‘ฅ) solution, the general term is

๐ท๐‘›๐‘ฅ๐‘› =๐‘› โˆ’ 1 + ๐‘Ž๐‘๐‘› โˆ’ ๐‘› + ๐‘›2

๐ท๐‘›โˆ’1๐‘ฅ๐‘›

Hence by ratio test

๐ฟ = lim๐‘›โ†’โˆž

๏ฟฝ๐ท๐‘›๐‘ฅ๐‘›

๐ท๐‘›โˆ’1๐‘ฅ๐‘›โˆ’1๏ฟฝ

= lim๐‘›โ†’โˆž

๏ฟฝ๏ฟฝ

๐‘›โˆ’1+๐‘Ž๐‘๐‘›โˆ’๐‘›+๐‘›2๐ท๐‘›โˆ’1๐‘ฅ๐‘›

๐ท๐‘›โˆ’1๐‘ฅ๐‘›โˆ’1๏ฟฝ๏ฟฝ

= lim๐‘›โ†’โˆž

๏ฟฝ๐‘› โˆ’ 1 + ๐‘Ž๐‘๐‘› โˆ’ ๐‘› + ๐‘›2

๐‘ฅ๏ฟฝ

= |๐‘ฅ| lim๐‘›โ†’โˆž

๏ฟฝ๐‘› โˆ’ 1 + ๐‘Ž๐‘๐‘› โˆ’ ๐‘› + ๐‘›2

๏ฟฝ

= |๐‘ฅ| lim๐‘›โ†’โˆž

๏ฟฝ๏ฟฝ

1๐‘› โˆ’

1๐‘›2 +

๐‘Ž๐‘›2

๐‘๐‘› โˆ’

1๐‘› + 1

๏ฟฝ๏ฟฝ

= |๐‘ฅ| ๏ฟฝ01๏ฟฝ

= 0

Therefore the series ๐‘ฆ2 (๐‘ฅ) also converges for all ๐‘ฅ. This means the solution ๐‘ฆ (๐‘ฅ) = ๐‘ฆ1 (๐‘ฅ) +๐‘ฆ2 (๐‘ฅ) found in (2) above also converges for all ๐‘ฅ.

3.6.2 Problem 2

The Sturm Liouville equation can be expressed as

๐ฟ [๐‘ข (๐‘ฅ)] = ๐œ†๐œŒ (๐‘ฅ) ๐‘ข (๐‘ฅ)

Where ๐ฟ is given as in class. Show ๐ฟ is Hermitian on the domain ๐‘Ž โ‰ค ๐‘ฅ โ‰ค ๐‘ with boundaryconditions ๐‘ข (๐‘Ž) = ๐‘ข (๐‘) = 0. Find the orthogonality condition.

Solution

๐ฟ = โˆ’ ๏ฟฝ๐‘๐‘‘2

๐‘‘๐‘ฅ2+ ๐‘โ€ฒ

๐‘‘๐‘‘๐‘ฅโˆ’ ๐‘ž๏ฟฝ

The operator ๐ฟ is Hermitian if

๏ฟฝ๐‘

๐‘Ž๏ฟฝ๏ฟฝ๐ฟ [๐‘ข] ๐‘‘๐‘ฅ = ๏ฟฝ

๐‘

๐‘Ž๏ฟฝ๏ฟฝ๐ฟ [๐‘ฃ] ๐‘‘๐‘ฅ

Where in the above ๐‘ข, ๐‘ฃ are any two functions defined over the domain that satisfy the

115

boundary conditions given. Starting from the left integral to show it will result in the right

integral. Replacing ๐ฟ [๐‘ข] by โˆ’ ๏ฟฝ๐‘ ๐‘‘2

๐‘‘๐‘ฅ2 + ๐‘โ€ฒ ๐‘‘๐‘‘๐‘ฅ โˆ’ ๐‘ž๏ฟฝ ๐‘ข in the LHS of the above gives

โˆ’๏ฟฝ๐‘

๐‘Ž๏ฟฝ๏ฟฝ ๏ฟฝ๐‘

๐‘‘2

๐‘‘๐‘ฅ2+ ๐‘โ€ฒ

๐‘‘๐‘‘๐‘ฅโˆ’ ๐‘ž๏ฟฝ ๐‘ข ๐‘‘๐‘ฅ = โˆ’๏ฟฝ

๐‘

๐‘Ž๏ฟฝ๏ฟฝ ๏ฟฝ๐‘

๐‘‘2๐‘ข๐‘‘๐‘ฅ2

+ ๐‘โ€ฒ๐‘‘๐‘ข๐‘‘๐‘ฅ

โˆ’ ๐‘ž๐‘ข๏ฟฝ ๐‘‘๐‘ฅ

= โˆ’๏ฟฝ๐‘

๐‘Ž๏ฟฝ๏ฟฝ๐‘๐‘‘2๐‘ข๐‘‘๐‘ฅ2

+ ๏ฟฝ๏ฟฝ๐‘โ€ฒ๐‘‘๐‘ข๐‘‘๐‘ฅ

โˆ’ ๐‘ž๏ฟฝ๏ฟฝ๐‘ข ๐‘‘๐‘ฅ

= โˆ’

๐ผ1๏ฟฝ๏ฟฝ๏ฟฝ๏ฟฝ๏ฟฝ๏ฟฝ๏ฟฝ๏ฟฝ๏ฟฝ๏ฟฝ๏ฟฝ๏ฟฝ๏ฟฝ๏ฟฝ๏ฟฝ๏ฟฝ

๐‘

๐‘Ž๐‘๏ฟฝ๏ฟฝ๐‘‘2๐‘ข๐‘‘๐‘ฅ2

๐‘‘๐‘ฅ โˆ’๏ฟฝ๐‘

๐‘Ž๏ฟฝ๏ฟฝ๐‘โ€ฒ

๐‘‘๐‘ข๐‘‘๐‘ฅ๐‘‘๐‘ฅ +๏ฟฝ

๐‘

๐‘Ž๐‘ž๏ฟฝ๏ฟฝ๐‘ข ๐‘‘๐‘ฅ (1)

Looking at the first integral above, which is ๐ผ1 = โˆซ๐‘

๐‘Ž๏ฟฝ๐‘๏ฟฝ๏ฟฝ๏ฟฝ ๏ฟฝ๐‘‘

2๐‘ข๐‘‘๐‘ฅ2๏ฟฝ ๐‘‘๐‘ฅ. The idea is to integrate

this twice to move the second derivative from ๐‘ข to ๏ฟฝ๏ฟฝ. Applying โˆซ๐ด๐‘‘๐ต = ๐ด๐ตโˆ’โˆซ๐ต๐‘‘๐ด, where

๐ด โ‰ก ๐‘๏ฟฝ๏ฟฝ

๐‘‘๐ต โ‰ก๐‘‘2๐‘ข๐‘‘๐‘ฅ2

Hence

๐‘‘๐ด = ๐‘๐‘‘๏ฟฝ๏ฟฝ๐‘‘๐‘ฅ+ ๐‘โ€ฒ๏ฟฝ๏ฟฝ

๐ต =๐‘‘๐‘ข๐‘‘๐‘ฅ

Therefore the integral ๐ผ1 in (1) becomes

๐ผ1 = ๏ฟฝ๐‘

๐‘Ž๐‘๏ฟฝ๏ฟฝ

๐‘‘2

๐‘‘๐‘ฅ2๐‘ข

= ๏ฟฝ๐‘๏ฟฝ๏ฟฝ๐‘‘๐‘ข๐‘‘๐‘ฅ ๏ฟฝ

๐‘

๐‘Žโˆ’๏ฟฝ

๐‘

๐‘Ž

๐‘‘๐‘ข๐‘‘๐‘ฅ ๏ฟฝ

๐‘๐‘‘๏ฟฝ๏ฟฝ๐‘‘๐‘ฅ+ ๐‘โ€ฒ๏ฟฝ๏ฟฝ๏ฟฝ ๐‘‘๐‘ฅ

But ๏ฟฝ๏ฟฝ (๐‘Ž) = 0 and ๏ฟฝ๏ฟฝ (๐‘) = 0, hence the boundary terms above vanish and simplifies to

๐ผ1 = โˆ’๏ฟฝ๐‘

๐‘Ž๐‘๐‘‘๐‘ข๐‘‘๐‘ฅ๐‘‘๏ฟฝ๏ฟฝ๐‘‘๐‘ฅ+ ๐‘โ€ฒ๏ฟฝ๏ฟฝ

๐‘‘๐‘ข๐‘‘๐‘ฅ๐‘‘๐‘ฅ

= โˆ’๏ฟฝ๐‘

๐‘Ž๐‘๐‘‘๐‘ข๐‘‘๐‘ฅ๐‘‘๏ฟฝ๏ฟฝ๐‘‘๐‘ฅ๐‘‘๐‘ฅ โˆ’๏ฟฝ

๐‘

๐‘Ž๐‘โ€ฒ๏ฟฝ๏ฟฝ

๐‘‘๐‘ข๐‘‘๐‘ฅ๐‘‘๐‘ฅ (2)

Before integrating by parts a second time, putting the result of ๐ผ1 back into (1) first simplifiesthe result. Substituting (2) into (1) gives

๏ฟฝ๐‘

๐‘Ž๏ฟฝ๏ฟฝ๐ฟ [๐‘ข] ๐‘‘๐‘ฅ = โˆ’๐ผ1 โˆ’๏ฟฝ

๐‘

๐‘Ž๏ฟฝ๏ฟฝ๐‘โ€ฒ

๐‘‘๐‘ข๐‘‘๐‘ฅ๐‘‘๐‘ฅ +๏ฟฝ

๐‘

๐‘Ž๐‘ž๏ฟฝ๏ฟฝ๐‘ข ๐‘‘๐‘ฅ

= โˆ’

๐ผ1๏ฟฝ๏ฟฝ๏ฟฝ๏ฟฝ๏ฟฝ๏ฟฝ๏ฟฝ๏ฟฝ๏ฟฝ๏ฟฝ๏ฟฝ๏ฟฝ๏ฟฝ๏ฟฝ๏ฟฝ๏ฟฝ๏ฟฝ๏ฟฝ๏ฟฝ๏ฟฝ๏ฟฝ๏ฟฝ๏ฟฝ๏ฟฝ๏ฟฝ๏ฟฝ๏ฟฝ๏ฟฝ๏ฟฝ๏ฟฝ๏ฟฝ๏ฟฝ๏ฟฝ๏ฟฝ๏ฟฝ๏ฟฝ๏ฟฝ๏ฟฝ๏ฟฝ๏ฟฝ๏ฟฝ๏ฟฝ๏ฟฝ๏ฟฝ๏ฟฝ๏ฟฝโˆ’๏ฟฝ

๐‘

๐‘Ž๐‘๐‘‘๐‘ข๐‘‘๐‘ฅ๐‘‘๏ฟฝ๏ฟฝ๐‘‘๐‘ฅ๐‘‘๐‘ฅ โˆ’๏ฟฝ

๐‘

๐‘Ž๐‘โ€ฒ๏ฟฝ๏ฟฝ

๐‘‘๐‘ข๐‘‘๐‘ฅ๐‘‘๐‘ฅ๏ฟฝ โˆ’๏ฟฝ

๐‘

๐‘Ž๏ฟฝ๏ฟฝ๐‘โ€ฒ

๐‘‘๐‘ข๐‘‘๐‘ฅ๐‘‘๐‘ฅ +๏ฟฝ

๐‘

๐‘Ž๐‘ž๏ฟฝ๏ฟฝ๐‘ข ๐‘‘๐‘ฅ

= ๏ฟฝ๐‘

๐‘Ž๐‘๐‘‘๐‘ข๐‘‘๐‘ฅ๐‘‘๏ฟฝ๏ฟฝ๐‘‘๐‘ฅ๐‘‘๐‘ฅ +๏ฟฝ

๐‘

๐‘Ž๐‘โ€ฒ๏ฟฝ๏ฟฝ

๐‘‘๐‘ข๐‘‘๐‘ฅ๐‘‘๐‘ฅ โˆ’๏ฟฝ

๐‘

๐‘Ž๏ฟฝ๏ฟฝ๐‘โ€ฒ

๐‘‘๐‘ข๐‘‘๐‘ฅ๐‘‘๐‘ฅ +๏ฟฝ

๐‘

๐‘Ž๐‘ž๏ฟฝ๏ฟฝ๐‘ข ๐‘‘๐‘ฅ

The second and third terms above cancel and the result becomes

๏ฟฝ๐‘

๐‘Ž๏ฟฝ๏ฟฝ๐ฟ [๐‘ข] ๐‘‘๐‘ฅ =

๐ผ2๏ฟฝ๏ฟฝ๏ฟฝ๏ฟฝ๏ฟฝ๏ฟฝ๏ฟฝ๏ฟฝ๏ฟฝ๏ฟฝ๏ฟฝ๏ฟฝ๏ฟฝ๏ฟฝ๏ฟฝ๏ฟฝ๏ฟฝ๏ฟฝ

๐‘

๐‘Ž๐‘๐‘‘๐‘ข๐‘‘๐‘ฅ๐‘‘๏ฟฝ๏ฟฝ๐‘‘๐‘ฅ๐‘‘๐‘ฅ +๏ฟฝ

๐‘

๐‘Ž๐‘ž๏ฟฝ๏ฟฝ๐‘ข ๐‘‘๐‘ฅ (3)

Now integration by parts is applied on the first integral above. Let ๐ผ2 = โˆซ๐‘๐‘Ž

๐‘‘๐‘ข๐‘‘๐‘ฅ๏ฟฝ๐‘๐‘‘๏ฟฝ๏ฟฝ๐‘‘๐‘ฅ๏ฟฝ ๐‘‘๐‘ฅ.

Applying โˆซ๐ด๐‘‘๐ต = ๐ด๐ต โˆ’ โˆซ๐ต๐‘‘๐ด, where

๐ด โ‰ก ๐‘๐‘‘๏ฟฝ๏ฟฝ๐‘‘๐‘ฅ

๐‘‘๐ต โ‰ก๐‘‘๐‘ข๐‘‘๐‘ฅ

116

Hence

๐‘‘๐ด = ๐‘๐‘‘2๏ฟฝ๏ฟฝ๐‘‘๐‘ฅ2

+ ๐‘โ€ฒ๐‘‘๏ฟฝ๏ฟฝ๐‘‘๐‘ฅ

๐ต = ๐‘ข

Therefore the integral ๐ผ2 becomes

๐ผ2 = ๏ฟฝ๐‘๐‘‘๏ฟฝ๏ฟฝ๐‘‘๐‘ฅ๐‘ข๏ฟฝ๐‘

๐‘Žโˆ’๏ฟฝ

๐‘

๐‘Ž๐‘ข ๏ฟฝ๐‘

๐‘‘2๏ฟฝ๏ฟฝ๐‘‘๐‘ฅ2

+ ๐‘โ€ฒ๐‘‘๏ฟฝ๏ฟฝ๐‘‘๐‘ฅ๏ฟฝ

๐‘‘๐‘ฅ

But ๐‘ข (๐‘Ž) = 0, ๐‘ข (๐‘) = 0, hence the boundary term vanishes and the above simplifies to

๐ผ2 = โˆ’๏ฟฝ๐‘

๐‘Ž๐‘ข ๏ฟฝ๐‘

๐‘‘2๏ฟฝ๏ฟฝ๐‘‘๐‘ฅ2

+ ๐‘โ€ฒ๐‘‘๏ฟฝ๏ฟฝ๐‘‘๐‘ฅ๏ฟฝ

๐‘‘๐‘ฅ

Substituting the above back into (3) gives

๏ฟฝ๐‘

๐‘Ž๏ฟฝ๏ฟฝ๐ฟ [๐‘ข] ๐‘‘๐‘ฅ = โˆ’๏ฟฝ

๐‘

๐‘Ž๐‘ข ๏ฟฝ๐‘

๐‘‘2๏ฟฝ๏ฟฝ๐‘‘๐‘ฅ2

+ ๐‘โ€ฒ๐‘‘๏ฟฝ๏ฟฝ๐‘‘๐‘ฅ๏ฟฝ

๐‘‘๐‘ฅ +๏ฟฝ๐‘

๐‘Ž๐‘ž๏ฟฝ๏ฟฝ๐‘ข ๐‘‘๐‘ฅ

= โˆ’๏ฟฝ๐‘

๐‘Ž๐‘ข ๏ฟฝ๐‘

๐‘‘2๏ฟฝ๏ฟฝ๐‘‘๐‘ฅ2

+ ๐‘โ€ฒ๐‘‘๏ฟฝ๏ฟฝ๐‘‘๐‘ฅโˆ’ ๐‘ž๏ฟฝ๏ฟฝ๏ฟฝ ๐‘‘๐‘ฅ

But โˆ’ ๏ฟฝ๐‘๐‘‘2๏ฟฝ๏ฟฝ๐‘‘๐‘ฅ2 + ๐‘

โ€ฒ ๐‘‘๏ฟฝ๏ฟฝ๐‘‘๐‘ฅ โˆ’ ๐‘ž๏ฟฝ๏ฟฝ๏ฟฝ = ๐ฟ [๏ฟฝ๏ฟฝ] by definition, and the above becomes

๏ฟฝ๐‘

๐‘Ž๏ฟฝ๏ฟฝ๐ฟ [๐‘ข] ๐‘‘๐‘ฅ = ๏ฟฝ

๐‘

๐‘Ž๐‘ข๐ฟ [๏ฟฝ๏ฟฝ] ๐‘‘๐‘ฅ

But โˆซ๐‘

๐‘Ž๐‘ข๐ฟ [๏ฟฝ๏ฟฝ] ๐‘‘๐‘ฅ = โˆซ

๐‘

๐‘Ž๏ฟฝ๏ฟฝ๐ฟ [๐‘ฃ] ๐‘‘๐‘ฅ, and the above becomes

๏ฟฝ๐‘

๐‘Ž๏ฟฝ๏ฟฝ๐ฟ [๐‘ข] ๐‘‘๐‘ฅ = ๏ฟฝ

๐‘

๐‘Ž๏ฟฝ๏ฟฝ (๐ฟ [๐‘ฃ]) ๐‘‘๐‘ฅ

Therefore ๐ฟ is Hermitian.

3.6.3 Problem 3

1. For the equation ๐‘ฆโ€ฒโ€ฒ + 1โˆ’๐›ผ2

4๐‘ฅ2 ๐‘ฆ = 0 show that two solutions are ๐‘ฆ1 (๐‘ฅ) = ๐‘Ž0๐‘ฅ1+๐›ผ2 and

๐‘ฆ2 (๐‘ฅ) = ๐‘Ž0๐‘ฅ1โˆ’๐›ผ2

2. For ๐›ผ = 0, the two solutions are not independent. Find a second solution ๐‘ฆ20 bysolving ๐‘Šโ€ฒ = 0 (๐‘Š is the Wronskian).

3. Show that the second solution found in (2) is a limiting case of the two solutionsfrom part (1). That is

๐‘ฆ20 = lim๐›ผโ†’0

๐‘ฆ1 โˆ’ ๐‘ฆ2๐›ผ

Solution

Part 1

The point ๐‘ฅ0 = 0 is a regular singular point. This is shown as follows.

lim๐‘ฅโ†’๐‘ฅ0

(๐‘ฅ โˆ’ ๐‘ฅ0)2 1 โˆ’ ๐›ผ2

4๐‘ฅ2= lim๐‘ฅโ†’0

๐‘ฅ21 โˆ’ ๐›ผ2

4๐‘ฅ2

= lim๐‘ฅโ†’0

1 โˆ’ ๐›ผ2

4

=1 โˆ’ ๐›ผ2

4Since the limit exist, then ๐‘ฅ0 = 0 is a regular singular point. Assuming the solution is aFrobenius series given by

๐‘ฆ (๐‘ฅ) =โˆž๏ฟฝ๐‘›=0

๐‘๐‘›๐‘ฅ๐‘›+๐‘Ÿ ๐‘0 โ‰  0

117

Therefore

๐‘ฆโ€ฒ (๐‘ฅ) =โˆž๏ฟฝ๐‘›=0

(๐‘› + ๐‘Ÿ) ๐‘๐‘›๐‘ฅ๐‘›+๐‘Ÿโˆ’1

๐‘ฆโ€ฒโ€ฒ (๐‘ฅ) =โˆž๏ฟฝ๐‘›=0

(๐‘› + ๐‘Ÿ) (๐‘› + ๐‘Ÿ โˆ’ 1) ๐‘๐‘›๐‘ฅ๐‘›+๐‘Ÿโˆ’2

Substituting the above 2 expressions back into the original ODE gives

4๐‘ฅ2 ๏ฟฝโˆž๏ฟฝ๐‘›=0

(๐‘› + ๐‘Ÿ) (๐‘› + ๐‘Ÿ โˆ’ 1) ๐‘๐‘›๐‘ฅ๐‘›+๐‘Ÿโˆ’2๏ฟฝ + ๏ฟฝ1 โˆ’ ๐›ผ2๏ฟฝ ๏ฟฝโˆž๏ฟฝ๐‘›=0

๐‘๐‘›๐‘ฅ๐‘›+๐‘Ÿ๏ฟฝ = 0

โˆž๏ฟฝ๐‘›=0

4 (๐‘› + ๐‘Ÿ) (๐‘› + ๐‘Ÿ โˆ’ 1) ๐‘๐‘›๐‘ฅ๐‘›+๐‘Ÿ + ๏ฟฝ1 โˆ’ ๐›ผ2๏ฟฝ ๏ฟฝโˆž๏ฟฝ๐‘›=0

๐‘๐‘›๐‘ฅ๐‘›+๐‘Ÿ๏ฟฝ = 0 (1)

Looking at ๐‘› = 0 first, in order to obtain the indicial equation gives

4 (๐‘Ÿ) (๐‘Ÿ โˆ’ 1) ๐‘0 + ๏ฟฝ1 โˆ’ ๐›ผ2๏ฟฝ ๐‘0 = 0

๐‘0 ๏ฟฝ4๐‘Ÿ2 โˆ’ 4๐‘Ÿ + ๏ฟฝ1 โˆ’ ๐›ผ2๏ฟฝ๏ฟฝ = 0

But ๐‘0 โ‰  0, therefore

๐‘Ÿ2 โˆ’ ๐‘Ÿ +๏ฟฝ1 โˆ’ ๐›ผ2๏ฟฝ

4= 0

The roots are ๐‘Ÿ = โˆ’๐‘2๐‘Ž ยฑ

12๐‘Žโˆš๐‘

2 โˆ’ 4๐‘Ž๐‘, but ๐‘Ž = 1, ๐‘ = โˆ’1, ๐‘ =๏ฟฝ1โˆ’๐›ผ2๏ฟฝ

4 , hence the roots are

๐‘Ÿ =12ยฑ12๏ฟฝ

1 โˆ’ ๏ฟฝ1 โˆ’ ๐›ผ2๏ฟฝ

=12ยฑ12โˆš๐›ผ2

=12ยฑ12๐›ผ

Hence ๐‘Ÿ1 =12(1 + ๐›ผ) and ๐‘Ÿ2 =

12(1 โˆ’ ๐›ผ). Each one of these roots gives a solution. The

di๏ฟฝerence is

๐‘Ÿ2 โˆ’ ๐‘Ÿ1 =12(1 + ๐›ผ) โˆ’

12(1 โˆ’ ๐›ผ)

= ๐›ผ

Therefore, to use the same solution form ๐‘ฆ1 (๐‘ฅ) = โˆ‘โˆž๐‘›=0 ๐‘๐‘›๐‘ฅ

๐‘›+๐‘Ÿ1 and ๐‘ฆ2 (๐‘ฅ) = โˆ‘โˆž๐‘›=0 ๐‘‘๐‘›๐‘ฅ

๐‘›+๐‘Ÿ2 foreach, it is assumed that ๐›ผ is not an integer. In this case, the recursive relation for ๐‘ฆ1 (๐‘ฅ) is

found from (1) by using ๐‘Ÿ = 12(1 + ๐›ผ) which results in

โˆž๏ฟฝ๐‘›=0

4 ๏ฟฝ๐‘› +12(1 + ๐›ผ)๏ฟฝ ๏ฟฝ๐‘› +

12(1 + ๐›ผ) โˆ’ 1๏ฟฝ ๐‘๐‘›๐‘ฅ

๐‘›+ 12 (1+๐›ผ) + ๏ฟฝ1 โˆ’ ๐›ผ2๏ฟฝ ๏ฟฝ

โˆž๏ฟฝ๐‘›=0

๐‘๐‘›๐‘ฅ๐‘›+ 1

2 (1+๐›ผ)๏ฟฝ = 0

For ๐‘› > 0 the above becomes

4 ๏ฟฝ๐‘› +12(1 + ๐›ผ)๏ฟฝ ๏ฟฝ๐‘› +

12(1 + ๐›ผ) โˆ’ 1๏ฟฝ ๐‘๐‘› + ๏ฟฝ1 โˆ’ ๐›ผ2๏ฟฝ ๐‘๐‘› = 0

๏ฟฝ4 ๏ฟฝ๐‘› +12(1 + ๐›ผ)๏ฟฝ ๏ฟฝ๐‘› +

12(1 + ๐›ผ) โˆ’ 1๏ฟฝ + ๏ฟฝ1 โˆ’ ๐›ผ2๏ฟฝ๏ฟฝ ๐‘๐‘› = 0

4๐‘› (๐‘› + ๐›ผ) ๐‘๐‘› = 0

The above can be true for all ๐‘› > 0 only when ๐‘๐‘› = 0 for ๐‘› > 0. Therefore the solution isonly the term with ๐‘0

๐‘ฆ1 (๐‘ฅ) =โˆž๏ฟฝ๐‘›=0

๐‘๐‘›๐‘ฅ๐‘›+๐‘Ÿ1 = ๐‘0๐‘ฅ๐‘Ÿ1 = ๐‘0๐‘ฅ12 (1+๐›ผ)

To find the second solution ๐‘ฆ2 (๐‘ฅ), the above is repeated but with

๐‘ฆ2 (๐‘ฅ) =โˆž๏ฟฝ๐‘›=0

๐‘‘๐‘›๐‘ฅ๐‘›+๐‘Ÿ2

Where the constants are not the same and by replacing ๐‘Ÿ in (1) by ๐‘Ÿ2 =12(1 โˆ’ ๐›ผ). This

results inโˆž๏ฟฝ๐‘›=0

4 ๏ฟฝ๐‘› +12(1 โˆ’ ๐›ผ)๏ฟฝ ๏ฟฝ๐‘› +

12(1 โˆ’ ๐›ผ) โˆ’ 1๏ฟฝ ๐‘‘๐‘›๐‘ฅ

๐‘›+ 12 (1โˆ’๐›ผ) + ๏ฟฝ1 โˆ’ ๐›ผ2๏ฟฝ ๏ฟฝ

โˆž๏ฟฝ๐‘›=0

๐‘‘๐‘›๐‘ฅ๐‘›+ 1

2 (1โˆ’๐›ผ)๏ฟฝ = 0

118

For ๐‘› > 0

๏ฟฝ4 ๏ฟฝ๐‘› +12(1 โˆ’ ๐›ผ)๏ฟฝ ๏ฟฝ๐‘› +

12(1 โˆ’ ๐›ผ) โˆ’ 1๏ฟฝ + ๏ฟฝ1 โˆ’ ๐›ผ2๏ฟฝ๏ฟฝ ๐‘‘๐‘› = 0

4๐‘› (๐‘› โˆ’ ๐›ผ) ๐‘‘๐‘› = 0

The above is true for all ๐‘› > 0 only when ๐‘๐‘› = 0 for ๐‘› > 0. Therefore the solution is just theterm with ๐‘‘0

๐‘ฆ2 (๐‘ฅ) =โˆž๏ฟฝ๐‘›=0

๐‘‘๐‘›๐‘ฅ๐‘›+๐‘Ÿ2 = ๐‘‘0๐‘ฅ๐‘Ÿ2 = ๐‘‘0๐‘ฅ12 (1โˆ’๐›ผ)

Therefore the two solutions are

๐‘ฆ1 (๐‘ฅ) = ๐‘0๐‘ฅ12 (1+๐›ผ)

๐‘ฆ2 (๐‘ฅ) = ๐‘‘0๐‘ฅ12 (1โˆ’๐›ผ)

Part 2

When ๐›ผ = 0 then the ODE becomes

4๐‘ฅ2๐‘ฆโ€ฒโ€ฒ + ๐‘ฆ = 0

And the two solutions found in part (1) simplify to

๐‘ฆ1 (๐‘ฅ) = ๐‘0โˆš๐‘ฅ๐‘ฆ2 (๐‘ฅ) = ๐‘‘0โˆš๐‘ฅ

Therefore the two solutions are not linearly independent. Let ๐‘ฆ20 (๐‘ฅ) be the second solution.The Wronskian is

๐‘Š(๐‘ฅ) = ๏ฟฝ๐‘ฆ1 ๐‘ฆ20๐‘ฆโ€ฒ1 ๐‘ฆโ€ฒ20

๏ฟฝ = ๐‘ฆ1๐‘ฆโ€ฒ20 โˆ’ ๐‘ฆ20๐‘ฆโ€ฒ1 (1)

Using Abelโ€™s theorem which says that for ODE of form ๐‘ฆโ€ฒโ€ฒ+๐‘ (๐‘ฅ) ๐‘ฆโ€ฒ+๐‘ž (๐‘ฅ) ๐‘ฆ = 0, theWronskianis ๐‘Š(๐‘ฅ) = ๐ถ๐‘’โˆ’โˆซ๐‘(๐‘ฅ)๐‘‘๐‘ฅ. Applying this to the given ODE above and since ๐‘ (๐‘ฅ) = 0 then theabove becomes

๐‘Š(๐‘ฅ) = ๐ถ

Where ๐ถ is constant. For ๐‘ฆ20 to be linearly independent from ๐‘ฆ1 ๐‘Š(๐‘ฅ) โ‰  0. Using ๐‘Š(๐‘ฅ) = ๐ถin (1) results in the following equation (here it is also assumed that ๐‘ฆ1 โ‰  0, or ๐‘ฅ โ‰  0, becausethe equation is divided by ๐‘ฆ1)

๐‘ฆ1๐‘ฆโ€ฒ20 โˆ’ ๐‘ฆ20๐‘ฆโ€ฒ1 = ๐ถ

๐‘ฆโ€ฒ20 โˆ’ ๐‘ฆ20๐‘ฆโ€ฒ1๐‘ฆ1=๐ถ๐‘ฆ1

Since ๐‘ฆ1 = โˆš๐‘ฅ and ๐‘ฆโ€ฒ1 =121

โˆš๐‘ฅthe above simplifies to

๐‘ฆโ€ฒ20 โˆ’ ๐‘ฆ20

121

โˆš๐‘ฅ

โˆš๐‘ฅ=

๐ถ

โˆš๐‘ฅ

๐‘ฆโ€ฒ20 โˆ’ ๐‘ฆ2012๐‘ฅ

=๐ถ

โˆš๐‘ฅ(2)

But the above is linear first order ODE of the form ๐‘Œโ€ฒ + ๐‘๐‘Œ = ๐‘ž, therefore the standardintegrating factor to use is ๐ผ = ๐‘’โˆซ๐‘(๐‘ฅ)๐‘‘๐‘ฅ which results in

๐ผ = ๐‘’โˆซโˆ’12๐‘ฅ ๐‘‘๐‘ฅ

= ๐‘’โˆ’12 โˆซ

1๐‘ฅ๐‘‘๐‘ฅ

= ๐‘’โˆ’12 ln ๐‘ฅ

=1

โˆš๐‘ฅ

119

Multiplying both sides of (2) by this integrating factor, makes the left side of (2) an exactdi๏ฟฝerential

๐‘‘๐‘‘๐‘ฅ ๏ฟฝ

๐‘ฆ201

โˆš๐‘ฅ๏ฟฝ =

๐ถ๐‘ฅ

Integrating both sides gives

๐‘ฆ201

โˆš๐‘ฅ= ๐ถ๏ฟฝ

1๐‘ฅ๐‘‘๐‘ฅ + ๐ถ1

๐‘ฆ201

โˆš๐‘ฅ= 2๐ถ ln ๐‘ฅ + ๐ถ1

๐‘ฆ20 = 2๐ถ ln ๐‘ฅโˆš๐‘ฅ + ๐ถ1โˆš๐‘ฅOr

๐‘ฆ20 = ๐ถ1 ln ๐‘ฅโˆš๐‘ฅ + ๐ถ2โˆš๐‘ฅ (3)

The above is the second solution. Therefore the final solution is

๐‘ฆ (๐‘ฅ) = ๐ถ0๐‘ฆ1 (๐‘ฅ) + ๐ถ3๐‘ฆ20 (๐‘ฅ)

Substituting ๐‘ฆ1 = โˆš๐‘ฅ and ๐‘ฆ20 found above and combining the common term โˆš๐‘ฅ andrenaming constants gives

๐‘ฆ (๐‘ฅ) = ๐ถ1โˆš๐‘ฅ + ๐ถ2 ln ๐‘ฅโˆš๐‘ฅAnother method to find the second solution

This method is called the reduction of order method. It does not require finding๐‘Š(๐‘ฅ) first.Let the second solution be

๐‘ฆ20 = ๐‘Œ = ๐‘ฃ (๐‘ฅ) ๐‘ฆ1 (๐‘ฅ) (4)

Where ๐‘ฃ (๐‘ฅ) is unknown function to be determined, and ๐‘ฆ1 (๐‘ฅ) = โˆš๐‘ฅ which is the first solutionthat is already known. Therefore

๐‘Œโ€ฒ = ๐‘ฃโ€ฒ๐‘ฆ1 + ๐‘ฃ๐‘ฆโ€ฒ1๐‘Œโ€ฒโ€ฒ = ๐‘ฃโ€ฒโ€ฒ๐‘ฆ1 + ๐‘ฃโ€ฒ๐‘ฆโ€ฒ1 + ๐‘ฃโ€ฒ๐‘ฆโ€ฒ1 + ๐‘ฃ๐‘ฆโ€ฒโ€ฒ1

= ๐‘ฃโ€ฒโ€ฒ๐‘ฆ1 + 2๐‘ฃโ€ฒ๐‘ฆโ€ฒ1 + ๐‘ฃ๐‘ฆโ€ฒโ€ฒ1Since ๐‘Œ is a solution to the ODE 4๐‘ฅ2๐‘ฆโ€ฒโ€ฒ + ๐‘ฆ = 0, then substituting the above equations backinto the ODE 4๐‘ฅ2๐‘ฆโ€ฒโ€ฒ + ๐‘ฆ = 0 gives

4๐‘ฅ2 ๏ฟฝ๐‘ฃโ€ฒโ€ฒ๐‘ฆ1 + 2๐‘ฃโ€ฒ๐‘ฆโ€ฒ1 + ๐‘ฃ๐‘ฆโ€ฒโ€ฒ1 ๏ฟฝ + ๐‘ฃ๐‘ฆ1 = 0

๐‘ฃโ€ฒโ€ฒ ๏ฟฝ4๐‘ฅ2๐‘ฆ1๏ฟฝ + ๐‘ฃโ€ฒ ๏ฟฝ8๐‘ฅ2๐‘ฆโ€ฒ1๏ฟฝ + ๐‘ฃ

โŽ›โŽœโŽœโŽœโŽœโŽœโŽœโŽ

0

๏ฟฝ๏ฟฝ๏ฟฝ๏ฟฝ๏ฟฝ๏ฟฝ๏ฟฝ๏ฟฝ๏ฟฝ๏ฟฝ๏ฟฝ๏ฟฝ๏ฟฝ4๐‘ฅ2๐‘ฆโ€ฒโ€ฒ1 + ๐‘ฆ1

โŽžโŽŸโŽŸโŽŸโŽŸโŽŸโŽŸโŽ  = 0

But 4๐‘ฅ2๐‘ฆโ€ฒโ€ฒ1 + ๐‘ฆ1 = 0 because ๐‘ฆ1 is a solution. The above simplifies to

๐‘ฃโ€ฒโ€ฒ ๏ฟฝ4๐‘ฅ2๐‘ฆ1๏ฟฝ + ๐‘ฃโ€ฒ ๏ฟฝ8๐‘ฅ2๐‘ฆโ€ฒ1๏ฟฝ = 0

But ๐‘ฆ1 = ๐‘ฅ12 , hence ๐‘ฆโ€ฒ1 =

12๐‘ฅ

โˆ’12 and the above simplifies to

๐‘ฃโ€ฒโ€ฒ ๏ฟฝ4๐‘ฅ2๐‘ฅ12 ๏ฟฝ + ๐‘ฃโ€ฒ ๏ฟฝ4๐‘ฅ2๐‘ฅ

โˆ’12 ๏ฟฝ = 0

๐‘ฅ52๐‘ฃโ€ฒโ€ฒ + ๐‘ฃโ€ฒ๐‘ฅ

32 = 0

๐‘ฅ๐‘ฃโ€ฒโ€ฒ + ๐‘ฃโ€ฒ = 0

๐‘ฃโ€ฒโ€ฒ +1๐‘ฅ๐‘ฃโ€ฒ = 0

This ODE is now easy to solve because the ๐‘ฃ (๐‘ฅ) term is missing. Let ๐‘ค = ๐‘ฃโ€ฒ and the above

first order ODE ๐‘คโ€ฒ+ 1๐‘ฅ๐‘ค = 0. This is linear in ๐‘ค. Hence using integrating factor ๐ผ = ๐‘’โˆซ

1๐‘ฅ๐‘‘๐‘ง = ๐‘ฅ,

this ODE becomes๐‘‘๐‘ฅ(๐‘ค๐‘ฅ) = 0

๐‘ค๐‘ฅ = ๐ถ

๐‘ค =๐ถ๐‘ฅ

120

Where ๐ถ is constant of integration. Since ๐‘ฃโ€ฒ = ๐‘ค, then ๐‘ฃโ€ฒ = ๐ถ1๐‘ฅ . Now ๐‘ฃ (๐‘ฅ) is found by

integrating both sides

๐‘ฃ = ๐ถ1 ln ๐‘ฅ + ๐ถ2Therefore the second solution from (4) becomes

๐‘ฆ20 = ๐ถ1 ln ๐‘ฅ๐‘ฆ1 + ๐ถ2๐‘ฆ1= ๐ถ1โˆš๐‘ฅ ln ๐‘ฅ + ๐ถ2โˆš๐‘ฅ (5)

Comparing the above to (3), shows it is the same solution. Both methods can be used, butreduction of order method is a more common method and it does not require finding theWronskian first, although it is not hard to find by using Abelโ€™s theorem.

Part 3

The solutions we found in part (1) are

๐‘ฆ1 (๐‘ฅ) = ๐ถ1๐‘ฅ12 (1+๐›ผ)

๐‘ฆ2 (๐‘ฅ) = ๐ถ2๐‘ฅ12 (1โˆ’๐›ผ)

Therefore

lim๐›ผโ†’0

๐‘ฆ1 โˆ’ ๐‘ฆ2๐›ผ

= lim๐›ผโ†’0

๐ถ1๐‘ฅ12 (1+๐›ผ) โˆ’ ๐ถ2๐‘ฅ

12 (1โˆ’๐›ผ)

๐›ผApplying Lโ€™Hopitalโ€™s

lim๐›ผโ†’0

๐‘ฆ1 โˆ’ ๐‘ฆ2๐›ผ

= lim๐›ผโ†’0

๐ถ1๐‘‘๐‘‘๐›ผ๏ฟฝ๐‘ฅ

12 (1+๐›ผ)๏ฟฝ โˆ’ ๐ถ2

๐‘‘๐‘‘๐›ผ๏ฟฝ๐‘ฅ

12 (1โˆ’๐›ผ)๏ฟฝ

1(1)

But๐‘‘๐‘‘๐›ผ

๏ฟฝ๐‘ฅ12 (1+๐›ผ)๏ฟฝ =

๐‘‘๐‘‘๐›ผ๐‘’12 (1+๐›ผ) ln ๐‘ฅ

=๐‘‘๐‘‘๐›ผ๐‘’๏ฟฝ 12 ln ๐‘ฅ+๐›ผ ln ๐‘ฅ๏ฟฝ

= ln ๐‘ฅ๐‘’๏ฟฝ12 ln ๐‘ฅ+๐›ผ ln ๐‘ฅ๏ฟฝ

And๐‘‘๐‘‘๐›ผ

๏ฟฝ๐‘ฅ12 (1โˆ’๐›ผ)๏ฟฝ =

๐‘‘๐‘‘๐›ผ๐‘’12 (1โˆ’๐›ผ) ln ๐‘ฅ

=๐‘‘๐‘‘๐›ผ๐‘’๏ฟฝ 12 ln ๐‘ฅโˆ’๐›ผ ln ๐‘ฅ๏ฟฝ

= โˆ’ ln ๐‘ฅ๐‘’๏ฟฝ12 ln ๐‘ฅโˆ’๐›ผ ln ๐‘ฅ๏ฟฝ

Therefore (1) becomes

lim๐›ผโ†’0

๐‘ฆ1 โˆ’ ๐‘ฆ2๐›ผ

= lim๐›ผโ†’0

๐ถ1 ln ๐‘ฅ๐‘’๏ฟฝ12 ln ๐‘ฅ+๐›ผ ln ๐‘ฅ๏ฟฝ

+ ๐ถ2 ln ๐‘ฅ๐‘’๏ฟฝ12 ln ๐‘ฅโˆ’๐›ผ ln ๐‘ฅ๏ฟฝ

= ln ๐‘ฅ ๏ฟฝlim๐›ผโ†’0

๐ถ1๐‘’๏ฟฝ 12 ln ๐‘ฅ+๐›ผ ln ๐‘ฅ๏ฟฝ

+ ๐ถ2๐‘’๏ฟฝ 12 ln ๐‘ฅโˆ’๐›ผ ln ๐‘ฅ๏ฟฝ

๏ฟฝ

= ln ๐‘ฅ ๏ฟฝ๐ถ1๐‘’12 ln ๐‘ฅ + ๐ถ2๐‘’

12 ln ๐‘ฅ๏ฟฝ

= ln ๐‘ฅ ๏ฟฝ๐ถ1โˆš๐‘ฅ + ๐ถ2โˆš๐‘ฅ๏ฟฝ= ๐ถโˆš๐‘ฅ ln ๐‘ฅ

The above is the same as (3) found in part (2). Hence

๐‘ฆ20 (๐‘ฅ) = lim๐›ผโ†’0

๐‘ฆ1 โˆ’ ๐‘ฆ2๐›ผ

Which is what the problem asked to show.

121

3.6.4 key solution to HW 6

122

123

124

125

126

3.7 HW 7

3.7.1 Problem 1

Figure 3.20: Problem statement

Solution

๐‘ฆโ€ฒโ€ฒ โˆ’ 2๐‘ฅ๐‘ฆโ€ฒ + 2๐‘›๐‘ฆ = 0 โˆ’ โˆž < ๐‘ฅ < โˆž

Part 1

๐‘” (๐‘ฅ, ๐‘ก) =โˆž๏ฟฝ๐‘›=0

๐ป๐‘› (๐‘ฅ)๐‘ก๐‘›

๐‘›!Di๏ฟฝerentiating w.r.t ๐‘ฅ, and assuming term by term di๏ฟฝerentiation is allowed, gives

๐œ•๐‘” (๐‘ฅ, ๐‘ก)๐œ•๐‘ฅ

=โˆž๏ฟฝ๐‘›=0

๐ปโ€ฒ๐‘› (๐‘ฅ)

๐‘ก๐‘›

๐‘›!Using ๐ปโ€ฒ

๐‘› (๐‘ฅ) = 2๐‘›๐ป๐‘›โˆ’1 (๐‘ฅ) in the above results in

๐œ•๐‘” (๐‘ฅ, ๐‘ก)๐œ•๐‘ฅ

=โˆž๏ฟฝ๐‘›=0

2๐‘›๐ป๐‘›โˆ’1 (๐‘ฅ)๐‘ก๐‘›

๐‘›!But for ๐‘› = 0, the first term is zero, so the sum can start from 1 and give the same result

๐œ•๐‘” (๐‘ฅ, ๐‘ก)๐œ•๐‘ฅ

=โˆž๏ฟฝ๐‘›=1

2๐‘›๐ป๐‘›โˆ’1 (๐‘ฅ)๐‘ก๐‘›

๐‘›!Now, decreasing the summation index by 1 and increasing the ๐‘› inside the sum by 1 gives

๐œ•๐‘” (๐‘ฅ, ๐‘ก)๐œ•๐‘ฅ

=โˆž๏ฟฝ๐‘›=0

2 (๐‘› + 1)๐ป๐‘› (๐‘ฅ)๐‘ก๐‘›+1

(๐‘› + 1)!

=โˆž๏ฟฝ๐‘›=0

2 (๐‘› + 1)๐ป๐‘› (๐‘ฅ)๐‘ก๐‘›+1

(๐‘› + 1) ๐‘›!

=โˆž๏ฟฝ๐‘›=0

2๐ป๐‘› (๐‘ฅ)๐‘ก๐‘›+1

๐‘›!

=โˆž๏ฟฝ๐‘›=0

2๐‘ก ๏ฟฝ๐ป๐‘› (๐‘ฅ)๐‘ก๐‘›

๐‘›!๏ฟฝ

= 2๐‘กโˆž๏ฟฝ๐‘›=0

๐ป๐‘› (๐‘ฅ)๐‘ก๐‘›

๐‘›!

127

But โˆ‘โˆž๐‘›=0๐ป๐‘› (๐‘ฅ)

๐‘ก๐‘›

๐‘›! = ๐‘” (๐‘ฅ, ๐‘ก) and the above reduces to

๐œ•๐‘” (๐‘ฅ, ๐‘ก)๐œ•๐‘ฅ

= 2๐‘ก๐‘” (๐‘ฅ, ๐‘ก)

The problem says it is supposed to be a first order di๏ฟฝerential equation and not a first orderpartial di๏ฟฝerential equation. Therefore, by assuming ๐‘ฅ to be a fixed parameter instead ofanother independent variable, the above can now be written as

๐‘‘๐‘‘๐‘ฅ๐‘” (๐‘ฅ, ๐‘ก) โˆ’ 2๐‘ก๐‘” (๐‘ฅ, ๐‘ก) = 0

Part 2

From the solution found in part (1)๐‘‘๐‘‘๐‘ฅ๐‘” (๐‘ฅ, ๐‘ก)๐‘” (๐‘ฅ, ๐‘ก)

= 2๐‘ก

๐‘‘๐‘” (๐‘ฅ, ๐‘ก)๐‘” (๐‘ฅ, ๐‘ก)

= 2๐‘ก๐‘‘๐‘ฅ

Integrating both sides gives

๏ฟฝ๐‘‘๐‘” (๐‘ฅ, ๐‘ก)๐‘” (๐‘ฅ, ๐‘ก)

= ๏ฟฝ2๐‘ก๐‘‘๐‘ฅ

ln ๏ฟฝ๐‘” (๐‘ฅ, ๐‘ก)๏ฟฝ = 2๐‘ก๐‘ฅ + ๐ถ๐‘” (๐‘ฅ, ๐‘ก) = ๐‘’2๐‘ก๐‘ฅ+๐ถ

๐‘” (๐‘ฅ, ๐‘ก) = ๐ถ1๐‘’2๐‘ก๐‘ฅ

Where ๐ถ1 = ๐‘’๐ถ a new constant. Let ๐‘” (0, ๐‘ก) = ๐‘”0 then the above shows that ๐ถ1 = ๐‘”0 and theabove can now be written as

๐‘” (๐‘ฅ, ๐‘ก) = ๐‘” (0, ๐‘ก) ๐‘’2๐‘ก๐‘ฅ

Part 3

Using the given definition of ๐‘” (๐‘ฅ, ๐‘ก) = โˆ‘โˆž๐‘›=0๐ป๐‘› (๐‘ฅ)

๐‘ก๐‘›

๐‘›! and when ๐‘ฅ = 0 then

๐‘” (0, ๐‘ก) =โˆž๏ฟฝ๐‘›=0

๐ป๐‘› (0)๐‘ก๐‘›

๐‘›!

= ๐ป0 (0) + ๐ป1 (0) +โˆž๏ฟฝ๐‘›=2

๐ป๐‘› (0)๐‘ก๐‘›

๐‘›!But ๐ป0 (๐‘ฅ) = 1, hence ๐ป0 (0) = 1 and ๐ป1 (๐‘ฅ) = 2๐‘ฅ, hence ๐ป1 (0) = 0 and the above becomes

๐‘” (0, ๐‘ก) = 1 +โˆž๏ฟฝ๐‘›=2

๐ป๐‘› (0)๐‘ก๐‘›

๐‘›!For the remaining series, it can be written as sum of even and odd terms

๐‘” (0, ๐‘ก) = 1 +โˆž๏ฟฝ

๐‘›=2,4,6,โ‹ฏ๐ป๐‘› (0)

๐‘ก๐‘›

๐‘›!+

โˆž๏ฟฝ

๐‘›=3,5,7,โ‹ฏ๐ป๐‘› (0)

๐‘ก๐‘›

๐‘›!

Or, equivalently

๐‘” (0, ๐‘ก) = 1 +โˆž๏ฟฝ

๐‘›=1,2,3,โ‹ฏ๐ป2๐‘› (0)

๐‘ก2๐‘›

(2๐‘›)!+

โˆž๏ฟฝ

๐‘›=1,2,3,โ‹ฏ๐ป2๐‘›+1 (0)

๐‘ก2๐‘›+1

(2๐‘› + 1)!

But using the hint given that ๐ป2๐‘›+1 (0) = 0 and ๐ป2๐‘› (0) =(โˆ’1)๐‘›(2๐‘›)!

๐‘›! the above simplifies to

๐‘” (0, ๐‘ก) = 1 +โˆž๏ฟฝ

๐‘›=1,2,3,โ‹ฏ

(โˆ’1)๐‘› (2๐‘›)!๐‘›!

๐‘ก2๐‘›

(2๐‘›)!

= 1 +โˆž๏ฟฝ

๐‘›=1,2,3,โ‹ฏ(โˆ’1)๐‘›

๐‘ก2๐‘›

๐‘›!

But since (โˆ’1)๐‘› ๐‘ก2๐‘›

๐‘›! = 1 when ๐‘› = 0, then the above sum can be made to start as zero and itsimplifies to

๐‘” (0, ๐‘ก) =โˆž๏ฟฝ๐‘›=0

(โˆ’1)๐‘›๐‘ก2๐‘›

๐‘›!

128

Therefore the solution ๐‘” (๐‘ฅ, ๐‘ก) = ๐‘” (0, ๐‘ก) ๐‘’๐‘ก๐‘ฅ found in part (2) becomes

๐‘” (๐‘ฅ, ๐‘ก) = ๏ฟฝโˆž๏ฟฝ๐‘›=0

(โˆ’1)๐‘›๐‘ก2๐‘›

๐‘›! ๏ฟฝ๐‘’2๐‘ก๐‘ฅ (1)

Now the sum โˆ‘โˆž๐‘›=0 (โˆ’1)

๐‘› ๐‘ก2๐‘›

๐‘›! = 1 โˆ’ ๐‘ก2 + ๐‘ก4

2! โˆ’๐‘ก6

3! +โ‹ฏ and comparing this sum to standard series

of ๐‘’๐‘ง = 1 + ๐‘ง + ๐‘ง2

2! +๐‘ง3

3! +โ‹ฏ, then this shows that when ๐‘ง = โˆ’๐‘ก2 and series for ๐‘’โˆ’๐‘ก2 becomes

๐‘’โˆ’๐‘ก2 = 1 + ๏ฟฝโˆ’๐‘ก2๏ฟฝ +๏ฟฝโˆ’๐‘ก2๏ฟฝ

2

2!+๏ฟฝโˆ’๐‘ก2๏ฟฝ

3

3!+๏ฟฝโˆ’๐‘ก2๏ฟฝ

4

4!โ‹ฏ

= 1 โˆ’ ๐‘ก2 +๐‘ก4

2!โˆ’๐‘ก6

3!+๐‘ก8

4!โ‹ฏ

Henceโˆž๏ฟฝ๐‘›=0

(โˆ’1)๐‘›๐‘ก2๐‘›

๐‘›!= ๐‘’โˆ’๐‘ก2

Substituting this into (1) gives

๐‘” (๐‘ฅ, ๐‘ก) = ๐‘’โˆ’๐‘ก2๐‘’2๐‘ก๐‘ฅ

= ๐‘’2๐‘ก๐‘ฅโˆ’๐‘ก2

Part 4

Since ๐‘” (๐‘ฅ, ๐‘ก) = ๐‘’2๐‘ก๐‘ฅโˆ’๐‘ก2 from part (3), then

๐œ•๐œ•๐‘ก๐‘” (๐‘ฅ, ๐‘ก) = (2๐‘ฅ โˆ’ 2๐‘ก) ๐‘’2๐‘ก๐‘ฅโˆ’๐‘ก2

= (2๐‘ฅ โˆ’ 2๐‘ก) ๐‘” (๐‘ฅ, ๐‘ก)

But ๐‘” (๐‘ฅ, ๐‘ก) = โˆ‘โˆž๐‘›=0๐ป๐‘› (๐‘ฅ)

๐‘ก๐‘›

๐‘›! , therefore the above can be written as

๐œ•๐œ•๐‘ก๐‘” (๐‘ฅ, ๐‘ก) = (2๐‘ฅ โˆ’ 2๐‘ก)

โˆž๏ฟฝ๐‘›=0

๐ป๐‘› (๐‘ฅ)๐‘ก๐‘›

๐‘›!

= 2๐‘ฅโˆž๏ฟฝ๐‘›=0

๐ป๐‘› (๐‘ฅ)๐‘ก๐‘›

๐‘›!โˆ’ 2๐‘ก

โˆž๏ฟฝ๐‘›=0

๐ป๐‘› (๐‘ฅ)๐‘ก๐‘›

๐‘›!

= 2๐‘ฅโˆž๏ฟฝ๐‘›=0

๐ป๐‘› (๐‘ฅ)๐‘ก๐‘›

๐‘›!โˆ’ 2

โˆž๏ฟฝ๐‘›=0

๐ป๐‘› (๐‘ฅ)๐‘ก๐‘›+1

๐‘›!

= 2๐‘ฅโˆž๏ฟฝ๐‘›=0

๐ป๐‘› (๐‘ฅ)๐‘ก๐‘›

๐‘›!โˆ’ 2

โˆž๏ฟฝ๐‘›=1

๐ป๐‘›โˆ’1 (๐‘ฅ)๐‘ก๐‘›

(๐‘› โˆ’ 1)!

= 2๐‘ฅโˆž๏ฟฝ๐‘›=0

๐ป๐‘› (๐‘ฅ)๐‘ก๐‘›

๐‘›!โˆ’ 2

โˆž๏ฟฝ๐‘›=1

๐‘›๐ป๐‘›โˆ’1 (๐‘ฅ)๐‘ก๐‘›

๐‘› (๐‘› โˆ’ 1)!

= 2๐‘ฅโˆž๏ฟฝ๐‘›=0

๐ป๐‘› (๐‘ฅ)๐‘ก๐‘›

๐‘›!โˆ’ 2

โˆž๏ฟฝ๐‘›=1

๐‘›๐ป๐‘›โˆ’1 (๐‘ฅ)๐‘ก๐‘›

๐‘›!(1)

On the other hand,๐œ•๐œ•๐‘ก๐‘” (๐‘ฅ, ๐‘ก) =

๐œ•๐œ•๐‘ก

โˆž๏ฟฝ๐‘›=0

๐ป๐‘› (๐‘ฅ)๐‘ก๐‘›

๐‘›!

=โˆž๏ฟฝ๐‘›=0

๐‘›๐ป๐‘› (๐‘ฅ)๐‘ก๐‘›โˆ’1

๐‘›!Since at ๐‘› = 0 the sum is zero, then it can be started from ๐‘› = 1 without changing the result

๐œ•๐œ•๐‘ก๐‘” (๐‘ฅ, ๐‘ก) =

โˆž๏ฟฝ๐‘›=1

๐‘›๐ป๐‘› (๐‘ฅ)๐‘ก๐‘›โˆ’1

๐‘›!

=โˆž๏ฟฝ๐‘›=0

(๐‘› + 1)๐ป๐‘›+1 (๐‘ฅ)๐‘ก๐‘›

(๐‘› + 1)!

=โˆž๏ฟฝ๐‘›=0

(๐‘› + 1)๐ป๐‘›+1 (๐‘ฅ)๐‘ก๐‘›

(๐‘› + 1) ๐‘›!

=โˆž๏ฟฝ๐‘›=0

๐ป๐‘›+1 (๐‘ฅ)๐‘ก๐‘›

๐‘›!(2)

129

Equating (1) and (2) givesโˆž๏ฟฝ๐‘›=0

๐ป๐‘›+1 (๐‘ฅ)๐‘ก๐‘›

๐‘›!= 2๐‘ฅ

โˆž๏ฟฝ๐‘›=0

๐ป๐‘› (๐‘ฅ)๐‘ก๐‘›

๐‘›!โˆ’ 2

โˆž๏ฟฝ๐‘›=1

๐‘›๐ป๐‘›โˆ’1 (๐‘ฅ)๐‘ก๐‘›

๐‘›!

But โˆ‘โˆž๐‘›=1 ๐‘›๐ป๐‘›โˆ’1 (๐‘ฅ)

๐‘ก๐‘›

๐‘›! = โˆ‘โˆž๐‘›=0 ๐‘›๐ป๐‘›โˆ’1 (๐‘ฅ)

๐‘ก๐‘›

๐‘›! because at ๐‘› = 0 it is zero, so it does not a๏ฟฝect theresult to start the sum from zero, and now the above can be written as

โˆž๏ฟฝ๐‘›=0

๐ป๐‘›+1 (๐‘ฅ)๐‘ก๐‘›

๐‘›!= 2๐‘ฅ

โˆž๏ฟฝ๐‘›=0

๐ป๐‘› (๐‘ฅ)๐‘ก๐‘›

๐‘›!โˆ’ 2

โˆž๏ฟฝ๐‘›=0

๐‘›๐ป๐‘›โˆ’1 (๐‘ฅ)๐‘ก๐‘›

๐‘›!Now since all the sums start from ๐‘› = 0 then the above means the same as

๐ป๐‘›+1 (๐‘ฅ)๐‘ก๐‘›

๐‘›!= 2๐‘ฅ๐ป๐‘› (๐‘ฅ)

๐‘ก๐‘›

๐‘›!โˆ’ 2๐‘›๐ป๐‘›โˆ’1 (๐‘ฅ)

๐‘ก๐‘›

๐‘›!Canceling ๐‘ก๐‘›

๐‘›! from each term gives

๐ป๐‘›+1 (๐‘ฅ) = 2๐‘ฅ๐ป๐‘› (๐‘ฅ) โˆ’ 2๐‘›๐ป๐‘›โˆ’1 (๐‘ฅ)

Which is the result required to show.

Part 5

The problem is asking to show that

๏ฟฝโˆž

โˆ’โˆž๐‘’โˆ’๐‘ฅ2๐ป๐‘š (๐‘ฅ)๐ป๐‘› (๐‘ฅ) ๐‘‘๐‘ฅ =

โŽงโŽชโŽชโŽจโŽชโŽชโŽฉ

0 ๐‘› โ‰  ๐‘š2๐‘›๐‘›!โˆš๐œ‹ ๐‘› = ๐‘š

The first part below will show the case for ๐‘› โ‰  ๐‘š and the second part part will show thecase for ๐‘› = ๐‘š

case ๐‘› โ‰  ๐‘š This is shown by using the di๏ฟฝerential equation directly. I found this methodeasier and more direct. Before starting, the ODE ๐‘ฆโ€ฒโ€ฒ โˆ’ 2๐‘ฅ๐‘ฆโ€ฒ + 2๐‘›๐‘ฆ = 0 is rewritten as

๐‘’๐‘ฅ2๐‘‘๐‘‘๐‘ฅ๏ฟฝ๐‘’โˆ’๐‘ฅ2๐‘ฆโ€ฒ๏ฟฝ + 2๐‘›๐‘ฆ = 0 (1)

The above form is exactly the same as the original ODE as can be seen by expandingit. Now, Let ๐ป๐‘› (๐‘ฅ) be one solution to (1) and let ๐ป๐‘š (๐‘ฅ) be another solution to (1) whichresults in the following two ODEโ€™s

๐‘’๐‘ฅ2๐‘‘๐‘‘๐‘ฅ๏ฟฝ๐‘’โˆ’๐‘ฅ2๐ปโ€ฒ

๐‘›๏ฟฝ + 2๐‘›๐ป๐‘› = 0 (1A)

๐‘’๐‘ฅ2๐‘‘๐‘‘๐‘ฅ๏ฟฝ๐‘’โˆ’๐‘ฅ2๐ปโ€ฒ

๐‘š๏ฟฝ + 2๐‘š๐ป๐‘š = 0 (2A)

Multiplying (1A) by ๐ป๐‘š and (2A) by ๐ป๐‘› and subtracting gives

๐ป๐‘š ๏ฟฝ๐‘’๐‘ฅ2 ๐‘‘๐‘‘๐‘ฅ๏ฟฝ๐‘’โˆ’๐‘ฅ2๐ปโ€ฒ

๐‘›๏ฟฝ + 2๐‘›๐ป๐‘›๏ฟฝ โˆ’ ๐ป๐‘› ๏ฟฝ๐‘’๐‘ฅ2 ๐‘‘๐‘‘๐‘ฅ๏ฟฝ๐‘’โˆ’๐‘ฅ2๐ปโ€ฒ

๐‘š๏ฟฝ + 2๐‘š๐ป๐‘š๏ฟฝ = 0

๏ฟฝ๐ป๐‘š๐‘’๐‘ฅ2 ๐‘‘๐‘‘๐‘ฅ๏ฟฝ๐‘’โˆ’๐‘ฅ2๐ปโ€ฒ

๐‘›๏ฟฝ + 2๐‘›๐ป๐‘›๐ป๐‘š๏ฟฝ โˆ’ ๏ฟฝ๐ป๐‘›๐‘’๐‘ฅ2 ๐‘‘๐‘‘๐‘ฅ๏ฟฝ๐‘’โˆ’๐‘ฅ2๐ปโ€ฒ

๐‘š๏ฟฝ + 2๐‘š๐ป๐‘›๐ป๐‘š๏ฟฝ = 0

๐ป๐‘š๐‘’๐‘ฅ2 ๐‘‘๐‘‘๐‘ฅ๏ฟฝ๐‘’โˆ’๐‘ฅ2๐ปโ€ฒ

๐‘›๏ฟฝ โˆ’ ๐ป๐‘›๐‘’๐‘ฅ2 ๐‘‘๐‘‘๐‘ฅ๏ฟฝ๐‘’โˆ’๐‘ฅ2๐ปโ€ฒ

๐‘š๏ฟฝ + 2 (๐‘› โˆ’ ๐‘š)๐ป๐‘›๐ป๐‘š = 0

๐ป๐‘š๐‘‘๐‘‘๐‘ฅ๏ฟฝ๐‘’โˆ’๐‘ฅ2๐ปโ€ฒ

๐‘›๏ฟฝ โˆ’ ๐ป๐‘›๐‘‘๐‘‘๐‘ฅ๏ฟฝ๐‘’โˆ’๐‘ฅ2๐ปโ€ฒ

๐‘š๏ฟฝ + 2 (๐‘› โˆ’ ๐‘š)๐ป๐‘›๐ป๐‘š๐‘’โˆ’๐‘ฅ2 = 0 (3)

But

๐ป๐‘š๐‘‘๐‘‘๐‘ฅ๏ฟฝ๐‘’โˆ’๐‘ฅ2๐ปโ€ฒ

๐‘›๏ฟฝ =๐‘‘๐‘‘๐‘ฅ๏ฟฝ๐‘’โˆ’๐‘ฅ2๐ปโ€ฒ

๐‘›๐ป๐‘š๏ฟฝ โˆ’ ๐‘’โˆ’๐‘ฅ2๐ปโ€ฒ

๐‘›๐ปโ€ฒ๐‘š

And

๐ป๐‘›๐‘‘๐‘‘๐‘ฅ๏ฟฝ๐‘’โˆ’๐‘ฅ2๐ปโ€ฒ

๐‘š๏ฟฝ =๐‘‘๐‘‘๐‘ฅ๏ฟฝ๐‘’โˆ’๐‘ฅ2๐ปโ€ฒ

๐‘š๐ป๐‘›๏ฟฝ โˆ’ ๐‘’โˆ’๐‘ฅ2๐ปโ€ฒ

๐‘š๐ปโ€ฒ๐‘›

Therefore

๐ป๐‘š๐‘‘๐‘‘๐‘ฅ๏ฟฝ๐‘’โˆ’๐‘ฅ2๐ปโ€ฒ

๐‘›๏ฟฝ โˆ’ ๐ป๐‘›๐‘‘๐‘‘๐‘ฅ๏ฟฝ๐‘’โˆ’๐‘ฅ2๐ปโ€ฒ

๐‘š๏ฟฝ = ๏ฟฝ๐‘‘๐‘‘๐‘ฅ๏ฟฝ๐‘’โˆ’๐‘ฅ2๐ปโ€ฒ

๐‘›๐ป๐‘š๏ฟฝ โˆ’ ๐‘’โˆ’๐‘ฅ2๐ปโ€ฒ

๐‘›๐ปโ€ฒ๐‘š๏ฟฝ โˆ’ ๏ฟฝ

๐‘‘๐‘‘๐‘ฅ๏ฟฝ๐‘’โˆ’๐‘ฅ2๐ปโ€ฒ

๐‘š๐ป๐‘›๏ฟฝ โˆ’ ๐‘’โˆ’๐‘ฅ2๐ปโ€ฒ

๐‘š๐ปโ€ฒ๐‘›๏ฟฝ

=๐‘‘๐‘‘๐‘ฅ๏ฟฝ๐‘’โˆ’๐‘ฅ2๐ปโ€ฒ

๐‘›๐ป๐‘š๏ฟฝ โˆ’๐‘‘๐‘‘๐‘ฅ๏ฟฝ๐‘’โˆ’๐‘ฅ2๐ปโ€ฒ

๐‘š๐ป๐‘›๏ฟฝ

=๐‘‘๐‘‘๐‘ฅ๏ฟฝ๐‘’โˆ’๐‘ฅ2 (๐ปโ€ฒ

๐‘›๐ป๐‘š โˆ’ ๐ปโ€ฒ๐‘š๐ป๐‘›)๏ฟฝ

130

Substituting the above relation back into (3) gives๐‘‘๐‘‘๐‘ฅ๏ฟฝ๐‘’โˆ’๐‘ฅ2 (๐ปโ€ฒ

๐‘›๐ป๐‘š โˆ’ ๐ปโ€ฒ๐‘š๐ป๐‘›)๏ฟฝ + 2 (๐‘› โˆ’ ๐‘š)๐ป๐‘›๐ป๐‘š๐‘’โˆ’๐‘ฅ

2 = 0

Integrating gives

๏ฟฝโˆž

โˆ’โˆž

๐‘‘๐‘‘๐‘ฅ๏ฟฝ๐‘’โˆ’๐‘ฅ2 (๐ปโ€ฒ

๐‘›๐ป๐‘š โˆ’ ๐ปโ€ฒ๐‘š๐ป๐‘›)๏ฟฝ ๐‘‘๐‘ฅ +๏ฟฝ

โˆž

โˆ’โˆž2 (๐‘› โˆ’ ๐‘š)๐ป๐‘›๐ป๐‘š๐‘’โˆ’๐‘ฅ

2๐‘‘๐‘ฅ = 0

๏ฟฝโˆž

โˆ’โˆž๐‘‘ ๏ฟฝ๐‘’โˆ’๐‘ฅ2 (๐ปโ€ฒ

๐‘›๐ป๐‘š โˆ’ ๐ปโ€ฒ๐‘š๐ป๐‘›)๏ฟฝ + 2 (๐‘› โˆ’ ๐‘š)๏ฟฝ

โˆž

โˆ’โˆž๐ป๐‘›๐ป๐‘š๐‘’โˆ’๐‘ฅ

2๐‘‘๐‘ฅ = 0

๏ฟฝ๐‘’โˆ’๐‘ฅ2 (๐ปโ€ฒ๐‘›๐ป๐‘š โˆ’ ๐ปโ€ฒ

๐‘š๐ป๐‘›)๏ฟฝโˆž

โˆ’โˆž+ 2 (๐‘› โˆ’ ๐‘š)๏ฟฝ

โˆž

โˆ’โˆž๐ป๐‘›๐ป๐‘š๐‘’โˆ’๐‘ฅ

2๐‘‘๐‘ฅ = 0

But lim๐‘ฅโ†’ยฑโˆž ๐‘’โˆ’๐‘ฅ2 โ†’ 0 so the first term above vanishes and the above becomes

2 (๐‘› โˆ’ ๐‘š)๏ฟฝโˆž

โˆ’โˆž๐ป๐‘›๐ป๐‘š๐‘’โˆ’๐‘ฅ

2๐‘‘๐‘ฅ = 0

Since this is the case where ๐‘› โ‰  ๐‘š then the above shows that

๏ฟฝโˆž

โˆ’โˆž๐ป๐‘›๐ป๐‘š๐‘’โˆ’๐‘ฅ

2๐‘‘๐‘ฅ = 0 ๐‘› โ‰  ๐‘š

Now the case ๐‘› = ๐‘š is proofed. When ๐ป๐‘› = ๐ป๐‘š then the integral becomes โˆซโˆž

โˆ’โˆž๐ป๐‘›๐ป๐‘›๐‘’โˆ’๐‘ฅ

2๐‘‘๐‘ฅ.Using the known Rodrigues formula for Hermite polynomials, given by

๐ป๐‘› (๐‘ฅ) = (โˆ’1)๐‘› ๐‘’๐‘ฅ2

๐‘‘๐‘›

๐‘‘๐‘ฅ๐‘›๐‘’โˆ’๐‘ฅ2

Then applying the above the above to one of the ๐ป๐‘› (๐‘ฅ) in the integral โˆซโˆž

โˆ’โˆž๐ป๐‘›๐ป๐‘›๐‘’โˆ’๐‘ฅ

2๐‘‘๐‘ฅ, gives

๏ฟฝโˆž

โˆ’โˆž๐ป๐‘›๐ป๐‘›๐‘’โˆ’๐‘ฅ

2๐‘‘๐‘ฅ = ๏ฟฝโˆž

โˆ’โˆž๏ฟฝ(โˆ’1)๐‘› ๐‘’๐‘ฅ

2 ๐‘‘๐‘›

๐‘‘๐‘ฅ๐‘›๐‘’โˆ’๐‘ฅ2๏ฟฝ๐ป๐‘›๐‘’โˆ’๐‘ฅ

2๐‘‘๐‘ฅ

= (โˆ’1)๐‘›๏ฟฝโˆž

โˆ’โˆž๏ฟฝ๐‘‘๐‘›

๐‘‘๐‘ฅ๐‘›๐‘’โˆ’๐‘ฅ2๏ฟฝ๐ป๐‘›๐‘‘๐‘ฅ

Now integration by parts is carried out. โˆซ๐‘ข๐‘‘๐‘ฃ = ๐‘ข๐‘ฃ โˆ’ โˆซ๐‘ฃ๐‘‘๐‘ข. Let ๐‘ข = ๐ป๐‘› and let ๐‘‘๐‘ฃ = ๐‘‘๐‘›

๐‘‘๐‘ฅ๐‘› ๐‘’โˆ’๐‘ฅ2,

therefore ๐‘‘๐‘ข = ๐ปโ€ฒ๐‘› (๐‘ฅ) = 2๐‘›๐ป๐‘›โˆ’1 (๐‘ฅ) and ๐‘ฃ =

๐‘‘๐‘›โˆ’1

๐‘‘๐‘ฅ๐‘›โˆ’1๐‘’โˆ’๐‘ฅ2, therefore

๏ฟฝโˆž

โˆ’โˆž๐ป๐‘›๐ป๐‘›๐‘’โˆ’๐‘ฅ

2๐‘‘๐‘ฅ = (โˆ’1)๐‘›โŽ›โŽœโŽœโŽœโŽœโŽ๏ฟฝ๐ป๐‘› (๐‘ฅ)

๐‘‘๐‘›โˆ’1

๐‘‘๐‘ฅ๐‘›โˆ’1๐‘’โˆ’๐‘ฅ2๏ฟฝ

โˆž

โˆ’โˆžโˆ’๏ฟฝ

โˆž

โˆ’โˆž๏ฟฝ๐‘‘๐‘›โˆ’1

๐‘‘๐‘ฅ๐‘›โˆ’1๐‘’โˆ’๐‘ฅ2๏ฟฝ 2๐‘›๐ป๐‘›โˆ’1 (๐‘ฅ) ๐‘‘๐‘ฅ

โŽžโŽŸโŽŸโŽŸโŽŸโŽ 

But ๏ฟฝ๐ป๐‘› (๐‘ฅ)๐‘‘๐‘›โˆ’1

๐‘‘๐‘ฅ๐‘›โˆ’1๐‘’โˆ’๐‘ฅ2๏ฟฝ

โˆž

โˆ’โˆžโ†’ 0 as ๐‘ฅ โ†’ ยฑโˆž because each derivative of ๐‘‘๐‘›โˆ’1

๐‘‘๐‘ฅ๐‘›โˆ’1๐‘’โˆ’๐‘ฅ2 produces a

term with ๐‘’โˆ’๐‘ฅ2 which vanishes at both ends of the real line. Hence the above integral nowbecomes

๏ฟฝโˆž

โˆ’โˆž๐ป๐‘›๐ป๐‘›๐‘’โˆ’๐‘ฅ

2๐‘‘๐‘ฅ = (โˆ’1)๐‘› ๏ฟฝโˆ’2๐‘›๏ฟฝโˆž

โˆ’โˆž๏ฟฝ๐‘‘๐‘›โˆ’1

๐‘‘๐‘ฅ๐‘›โˆ’1๐‘’โˆ’๐‘ฅ2๏ฟฝ๐ป๐‘›โˆ’1 (๐‘ฅ) ๐‘‘๐‘ฅ๏ฟฝ

Now the process is repeated, doing one more integration by parts. This results in

๏ฟฝโˆž

โˆ’โˆž๐ป๐‘›๐ป๐‘›๐‘’โˆ’๐‘ฅ

2๐‘‘๐‘ฅ = (โˆ’1)๐‘› ๏ฟฝโˆ’2๐‘› ๏ฟฝโˆ’2 (๐‘› โˆ’ 1)๏ฟฝโˆž

โˆ’โˆž๏ฟฝ๐‘‘๐‘›โˆ’2

๐‘‘๐‘ฅ๐‘›โˆ’2๐‘’โˆ’๐‘ฅ2๏ฟฝ๐ป๐‘›โˆ’2 (๐‘ฅ) ๐‘‘๐‘ฅ๏ฟฝ๏ฟฝ

And again

๏ฟฝโˆž

โˆ’โˆž๐ป๐‘›๐ป๐‘›๐‘’โˆ’๐‘ฅ

2๐‘‘๐‘ฅ = (โˆ’1)๐‘› ๏ฟฝโˆ’2๐‘› ๏ฟฝโˆ’2 (๐‘› โˆ’ 1) ๏ฟฝโˆ’2 (๐‘› โˆ’ 2)๏ฟฝโˆž

โˆ’โˆž๏ฟฝ๐‘‘๐‘›โˆ’3

๐‘‘๐‘ฅ๐‘›โˆ’3๐‘’โˆ’๐‘ฅ2๏ฟฝ๐ป๐‘›โˆ’3 (๐‘ฅ) ๐‘‘๐‘ฅ๏ฟฝ๏ฟฝ๏ฟฝ

This process continues ๐‘› times. After ๐‘› integrations by parts, the above becomes

๏ฟฝโˆž

โˆ’โˆž๐ป๐‘›๐ป๐‘›๐‘’โˆ’๐‘ฅ

2๐‘‘๐‘ฅ = (โˆ’1)๐‘› ๏ฟฝโˆ’2๐‘› ๏ฟฝโˆ’2 (๐‘› โˆ’ 1) ๏ฟฝโˆ’2 (๐‘› โˆ’ 2) ๏ฟฝโ‹ฏ๏ฟฝ๏ฟฝโˆž

โˆ’โˆž๐‘’โˆ’๐‘ฅ2๐ป0 (๐‘ฅ) ๐‘‘๐‘ฅ๏ฟฝ๏ฟฝ๏ฟฝ๏ฟฝ๏ฟฝ

= (โˆ’1)๐‘› (โˆ’2)๐‘› ๐‘›!๏ฟฝโˆž

โˆ’โˆž๐‘’โˆ’๐‘ฅ2๐ป0 (๐‘ฅ) ๐‘‘๐‘ฅ

= 2๐‘›๐‘›!๏ฟฝโˆž

โˆ’โˆž๐‘’โˆ’๐‘ฅ2๐ป0 (๐‘ฅ) ๐‘‘๐‘ฅ

But ๐ป0 (๐‘ฅ) = 1, therefore the above becomes

๏ฟฝโˆž

โˆ’โˆž๐ป๐‘›๐ป๐‘›๐‘’โˆ’๐‘ฅ

2๐‘‘๐‘ฅ = 2๐‘›๐‘›!๏ฟฝโˆž

โˆ’โˆž๐‘’โˆ’๐‘ฅ2๐‘‘๐‘ฅ

131

But

๏ฟฝโˆž

โˆ’โˆž๐‘’โˆ’๐‘ฅ2 = 2๏ฟฝ

โˆž

0๐‘’โˆ’๐‘ฅ2

= 2โˆš๐œ‹2

= โˆš๐œ‹

Therefore

๏ฟฝโˆž

โˆ’โˆž๐ป๐‘›๐ป๐‘›๐‘’โˆ’๐‘ฅ

2๐‘‘๐‘ฅ = 2๐‘›๐‘›!โˆš๐œ‹

This completes the case for ๐‘› = ๐‘š. Hence

๏ฟฝโˆž

โˆ’โˆž๐‘’โˆ’๐‘ฅ2๐ป๐‘š (๐‘ฅ)๐ป๐‘› (๐‘ฅ) ๐‘‘๐‘ฅ =

โŽงโŽชโŽชโŽจโŽชโŽชโŽฉ

0 ๐‘› โ‰  ๐‘š2๐‘›๐‘›!โˆš๐œ‹ ๐‘› = ๐‘š

Which is what the problem asked to show.

3.7.2 Problem 2

Figure 3.21: Problem statement

Solution

Part (a)

๐‘ฆโ€ฒโ€ฒ (๐‘Ÿ) +1๐‘Ÿ๐‘ฆโ€ฒ (๐‘Ÿ) โˆ’

๐‘›2

๐‘Ÿ2๐‘ฆ (๐‘Ÿ) = 0 0 < ๐‘Ÿ < โˆž

Or

๐‘Ÿ2๐‘ฆโ€ฒโ€ฒ (๐‘Ÿ) + ๐‘Ÿ๐‘ฆโ€ฒ (๐‘Ÿ) โˆ’ ๐‘›2๐‘ฆ (๐‘Ÿ) = 0

case ๐‘› = 0

The ode becomes ๐‘Ÿ2๐‘ฆโ€ฒโ€ฒ (๐‘Ÿ) + ๐‘Ÿ๐‘ฆโ€ฒ (๐‘Ÿ) = 0. Let ๐‘ง = ๐‘ฆโ€ฒ and it becomes ๐‘Ÿ2๐‘งโ€ฒ (๐‘Ÿ) + ๐‘Ÿ๐‘ง (๐‘Ÿ) = 0 or

๐‘งโ€ฒ (๐‘Ÿ) + 1๐‘Ÿ ๐‘ง (๐‘Ÿ) = 0. This is linear in ๐‘ง (๐‘Ÿ). Integrating factor is ๐ผ = ๐‘’โˆซ

1๐‘Ÿ ๐‘‘๐‘Ÿ = ๐‘Ÿ. Multiplying the

ode by ๐ผ it becomes exact di๏ฟฝerential ๐‘‘๐‘‘๐‘Ÿ(๐‘ง๐‘Ÿ) = 0 or ๐‘‘ (๐‘ง๐‘Ÿ) = 0, hence ๐‘ง = ๐‘1

๐‘Ÿ where ๐‘1 isconstant of integration. Therefore

๐‘ฆโ€ฒ (๐‘Ÿ) =๐‘1๐‘Ÿ

Integrating again gives

๐‘ฆ (๐‘Ÿ) =๐‘1

ln ๐‘Ÿ + ๐‘2Since lim๐‘Ÿโ†’0 the solution is bounded, then ๐‘1 must be zero. Therefore 0 = ๐‘2 and thisimplies ๐‘2 = 0 also. Therefore when ๐‘› = 0 the solution is

๐‘ฆ (๐‘Ÿ) = 0

132

Case ๐‘› โ‰  0

Since powers of ๐‘Ÿ is the same as order of derivative in each term, this is an Euler ODE. Itis solved by assuming ๐‘ฆ = ๐‘Ÿ๐›ผ. Hence ๐‘ฆโ€ฒ = ๐›ผ๐‘Ÿ๐›ผโˆ’1, ๐‘ฆโ€ฒโ€ฒ = ๐›ผ (๐›ผ โˆ’ 1) ๐‘Ÿ๐›ผโˆ’2. Substituting these intothe above ODE gives

๐‘Ÿ2๐›ผ (๐›ผ โˆ’ 1) ๐‘Ÿ๐›ผโˆ’2 + ๐‘Ÿ๐›ผ๐‘Ÿ๐›ผโˆ’1 โˆ’ ๐‘›2๐‘Ÿ๐›ผ = 0๐›ผ (๐›ผ โˆ’ 1) ๐‘Ÿ๐›ผ + ๐›ผ๐‘Ÿ๐›ผ โˆ’ ๐‘›2๐‘Ÿ๐›ผ = 0

๐‘Ÿ๐›ผ ๏ฟฝ๐›ผ (๐›ผ โˆ’ 1) + ๐›ผ โˆ’ ๐‘›2๏ฟฝ = 0

Assuming non-trivial solution ๐‘Ÿ๐›ผ โ‰  0, then the indicial equation is

๐›ผ (๐›ผ โˆ’ 1) + ๐›ผ โˆ’ ๐‘›2 = 0๐›ผ2 = ๐‘›2

๐›ผ = ยฑ๐‘›

Hence one solution is

๐‘ฆ1 (๐‘Ÿ) = ๐‘Ÿ๐‘›

And second solution is

๐‘ฆ2 (๐‘Ÿ) = ๐‘Ÿโˆ’๐‘›

And the general solution is linear combination of these solutions

๐‘ฆ (๐‘Ÿ) = ๐‘1๐‘Ÿ๐‘› + ๐‘2๐‘Ÿโˆ’๐‘›

The above shows that lim๐‘Ÿโ†’0 ๐‘ฆ1 (๐‘Ÿ) = 0 and lim๐‘Ÿโ†’โˆž ๐‘ฆ2 (๐‘Ÿ) = 0.

Part (b)

Short version of the solution

To simplify the notations, ๐‘Ÿ0 is used instead of ๐‘Ÿโ€ฒ in all the following.

๐‘ฆโ€ฒโ€ฒ (๐‘Ÿ) +1๐‘Ÿ๐‘ฆโ€ฒ (๐‘Ÿ) โˆ’

๐‘›2

๐‘Ÿ2๐‘ฆ (๐‘Ÿ) =

1๐‘Ÿ๐›ฟ (๐‘Ÿ โˆ’ ๐‘Ÿ0) 0 < ๐‘Ÿ < โˆž

Multiplying both sides by ๐‘Ÿ the above becomes

๐‘Ÿ๐‘ฆโ€ฒโ€ฒ (๐‘Ÿ) + ๐‘ฆโ€ฒ (๐‘Ÿ) โˆ’๐‘›2

๐‘Ÿ๐‘ฆ (๐‘Ÿ) = ๐›ฟ (๐‘Ÿ โˆ’ ๐‘Ÿ0) (1)

But the two solutions2 to the homogeneous ODE ๐‘Ÿ๐‘ฆโ€ฒโ€ฒ (๐‘Ÿ) + ๐‘ฆโ€ฒ (๐‘Ÿ) โˆ’ ๐‘›2

๐‘Ÿ ๐‘ฆ (๐‘Ÿ) = 0 were found inpart (a). These are

๐‘ฆ1 (๐‘Ÿ) = ๐‘Ÿ๐‘› (1A)

๐‘ฆ2 (๐‘Ÿ) = ๐‘Ÿโˆ’๐‘›

The Green function is the solution to

๐‘Ÿ๐บ (๐‘Ÿ, ๐‘Ÿ0) + ๐บ (๐‘Ÿ, ๐‘Ÿ0) โˆ’๐‘›2

๐‘Ÿ๐บ (๐‘Ÿ, ๐‘Ÿ0) = ๐›ฟ (๐‘Ÿ โˆ’ ๐‘Ÿ0) (1B)

lim๐‘Ÿโ†’0

๐บ (๐‘Ÿ, ๐‘Ÿ0) = 0

lim๐‘Ÿโ†’โˆž

๐บ (๐‘Ÿ, ๐‘Ÿ0) = 0

Which is given by (Using class notes, Lecture December 5, 2018) as

๐บ (๐‘Ÿ, ๐‘Ÿ0) =1๐ถ

โŽงโŽชโŽชโŽจโŽชโŽชโŽฉ๐‘ฆ1 (๐‘Ÿ) ๐‘ฆ2 (๐‘Ÿ0) 0 < ๐‘Ÿ < ๐‘Ÿ0๐‘ฆ1 (๐‘Ÿ0) ๐‘ฆ2 (๐‘Ÿ) ๐‘Ÿ0 < ๐‘Ÿ < โˆž

(2)

Note, I used +1๐ถ and not โˆ’1

๐ถ as in class notes, since I am using ๐ฟ = โˆ’ ๏ฟฝ๏ฟฝ๐‘๐‘ฆโ€ฒ๏ฟฝโ€ฒโˆ’ ๐‘ž๐‘ฆ๏ฟฝ as the

operator and not ๐ฟ = + ๏ฟฝ๏ฟฝ๐‘๐‘ฆโ€ฒ๏ฟฝโ€ฒ+ ๐‘ž๐‘ฆ๏ฟฝ. Now ๐ถ is given by

๐ถ = ๐‘ (๐‘Ÿ0) ๏ฟฝ๐‘ฆ1 (๐‘Ÿ0) ๐‘ฆโ€ฒ2 (๐‘Ÿ0) โˆ’ ๐‘ฆโ€ฒ1 (๐‘Ÿ0) ๐‘ฆ2 (๐‘Ÿ0)๏ฟฝ

2All the following is for ๐‘› โ‰  0, since for ๐‘› = 0, only trivial solution exist

133

Where from (1A) we see that

๐‘ฆ1 (๐‘Ÿ0) = ๐‘Ÿ๐‘›0๐‘ฆโ€ฒ2 (๐‘Ÿ0) = โˆ’๐‘›๐‘Ÿโˆ’๐‘›โˆ’10

๐‘ฆโ€ฒ1 (๐‘Ÿ0) = ๐‘›๐‘Ÿ๐‘›โˆ’10

๐‘ฆ2 (๐‘Ÿ0) = ๐‘Ÿโˆ’๐‘›0Therefore ๐ถ becomes

๐ถ = ๐‘ (๐‘Ÿ0) ๏ฟฝโˆ’๐‘›๐‘Ÿโˆ’๐‘›โˆ’10 ๐‘Ÿ๐‘›0 โˆ’ ๐‘›๐‘Ÿ๐‘›โˆ’10 ๐‘Ÿโˆ’๐‘›0 ๏ฟฝ

= 2๐‘›๐‘Ÿโˆ’10 ๐‘ (๐‘Ÿ0)

We just need now to find ๐‘ (๐‘Ÿ0). This comes from Sturm Liouville form. We need to convertthe ODE ๐‘Ÿ2๐‘ฆโ€ฒโ€ฒ (๐‘Ÿ) + ๐‘Ÿ๐‘ฆโ€ฒ (๐‘Ÿ) โˆ’ ๐‘›2๐‘ฆ (๐‘Ÿ) = 0 to Sturm Liouville. Writing this ODE as ๐‘Ž๐‘ฆโ€ฒโ€ฒ + ๐‘๐‘ฆโ€ฒ +(๐‘ + ๐œ†) ๐‘ฆ = 0 where ๐‘Ž = ๐‘Ÿ2, ๐‘ = ๐‘Ÿ, ๐‘ = 0, ๐œ† = โˆ’๐‘›2, therefore

๐‘ = ๐‘’โˆซ๐‘๐‘Ž๐‘‘๐‘Ÿ = ๐‘’

โˆซ ๐‘Ÿ๐‘Ÿ2๐‘‘๐‘Ÿ = ๐‘Ÿ

๐‘ž = โˆ’๐‘๐‘๐‘Ž= 0

๐œŒ =๐‘๐‘Ž=๐‘Ÿ๐‘Ÿ2=1๐‘Ÿ

Hence the SL form is ๏ฟฝ๐‘๐‘ฆโ€ฒ๏ฟฝโ€ฒโˆ’ ๐‘ž๐‘ฆ + ๐œ†๐œŒ๐‘ฆ = 0. Hence the SL form is ๏ฟฝ๐‘๐‘ฆโ€ฒ๏ฟฝ

โ€ฒโˆ’ ๐‘ž๐‘ฆ + ๐œ†๐œŒ๐‘ฆ = 0 or

๏ฟฝ๐‘Ÿ๐‘ฆโ€ฒ๏ฟฝโ€ฒโˆ’1๐‘Ÿ๐‘›2๐‘ฆ = 0 (2A)

Hence the operator is ๐ฟ ๏ฟฝ๐‘ฆ๏ฟฝ = โˆ’ ๏ฟฝ ๐‘‘๐‘‘๐‘Ÿ ๏ฟฝ๐‘Ÿ๐‘‘๐‘‘๐‘Ÿ๏ฟฝ๏ฟฝ ๏ฟฝ๐‘ฆ๏ฟฝ and in standard form it becomes ๐ฟ ๏ฟฝ๐‘ฆ๏ฟฝ+ 1

๐‘Ÿ๐‘›2๐‘ฆ = 0.

The above shows that ๐‘ (๐‘Ÿ0) = ๐‘Ÿ0. Therefore

๐ถ = 2๐‘›

Hence Green function is now found from (2) as, for ๐‘› โ‰  0

๐บ (๐‘Ÿ, ๐‘Ÿ0) =12๐‘›

โŽงโŽชโŽชโŽจโŽชโŽชโŽฉ๐‘Ÿ๐‘›๐‘Ÿโˆ’๐‘›0 0 < ๐‘Ÿ < ๐‘Ÿ0๐‘Ÿ๐‘›0๐‘Ÿโˆ’๐‘› ๐‘Ÿ0 < ๐‘Ÿ < โˆž

Since ๐‘“ (๐‘Ÿ) in the original ODE is zero, there is nothing to convolve with. i.e. ๐‘ฆ (๐‘Ÿ) =โˆซโˆž0๐บ (๐‘Ÿ, ๐‘Ÿ0) ๐‘“ (๐‘Ÿ0) ๐‘‘๐‘Ÿ0 here is not needed since there is no ๐‘“ (๐‘Ÿ). Therefore the above is the

final solution.

Extended solution

This solution shows derivation of (2) above. It can be considered as an appendix. TheGreen function is the solution to

๐‘Ÿ๐บ (๐‘Ÿ, ๐‘Ÿ0) + ๐บ (๐‘Ÿ, ๐‘Ÿ0) โˆ’๐‘›2

๐‘Ÿ๐บ (๐‘Ÿ, ๐‘Ÿ0) = ๐›ฟ (๐‘Ÿ โˆ’ ๐‘Ÿ0) (1B)

lim๐‘Ÿโ†’0

๐บ (๐‘Ÿ, ๐‘Ÿ0) = 0

lim๐‘Ÿโ†’โˆž

๐บ (๐‘Ÿ, ๐‘Ÿ0) = 0

In (1B), ๐‘Ÿ0 is the location of the impulse and ๐‘Ÿ is the location of the observed response dueto this impulse. The solution to the above ODE is now broken to two regions

๐บ (๐‘Ÿ, ๐‘Ÿ0) =

โŽงโŽชโŽชโŽจโŽชโŽชโŽฉ๐ด1๐‘ฆ1 (๐‘Ÿ) + ๐ด2๐‘ฆ2 (๐‘Ÿ) 0 < ๐‘Ÿ < ๐‘Ÿ0๐ต1๐‘ฆ1 (๐‘Ÿ) + ๐ต1๐‘ฆ2 (๐‘Ÿ) ๐‘Ÿ0 < ๐‘Ÿ < โˆž

(2)

Where ๐‘ฆ1 (๐‘Ÿ) , ๐‘ฆ2 (๐‘Ÿ) are the solution to ๐‘Ÿ๐‘ฆโ€ฒโ€ฒ (๐‘Ÿ) + ๐‘ฆโ€ฒ (๐‘Ÿ) โˆ’ ๐‘›2

๐‘Ÿ ๐‘ฆ (๐‘Ÿ) = 0 and these were found inpart (a) to be ๐‘ฆ1 (๐‘Ÿ) = ๐‘Ÿ๐‘›, ๐‘ฆ2 (๐‘Ÿ) = ๐‘Ÿโˆ’๐‘› and ๐ด1, ๐ด2, ๐ต1, ๐ต2 needs to be determined. Hence (2)becomes

๐บ (๐‘Ÿ, ๐‘Ÿ0) =

โŽงโŽชโŽชโŽจโŽชโŽชโŽฉ๐ด1๐‘Ÿ๐‘› + ๐ด2๐‘Ÿโˆ’๐‘› 0 < ๐‘Ÿ < ๐‘Ÿ0๐ต1๐‘Ÿ๐‘› + ๐ต2๐‘Ÿโˆ’๐‘› ๐‘Ÿ0 < ๐‘Ÿ < โˆž

(3)

The left boundary condition lim๐‘Ÿโ†’0 ๐บ (๐‘Ÿ, ๐‘Ÿ0) = 0 implies ๐ด2 = 0 and the right boundarycondition lim๐‘Ÿโ†’โˆž ๐บ (๐‘Ÿ, ๐‘Ÿ0) = 0 implies ๐ต1 = 0. This is needed to keep the solution bounded.

134

Hence (3) simplifies to

๐บ (๐‘Ÿ, ๐‘Ÿ0) =

โŽงโŽชโŽชโŽจโŽชโŽชโŽฉ๐ด1๐‘Ÿ๐‘› 0 < ๐‘Ÿ < ๐‘Ÿ0๐ต2๐‘Ÿโˆ’๐‘› ๐‘Ÿ0 < ๐‘Ÿ < โˆž

(4)

To determine the remaining two constants ๐ด1, ๐ต2, two additional conditions are needed.The first is that ๐บ (๐‘Ÿ, ๐‘Ÿ0) is continuous at ๐‘Ÿ = ๐‘Ÿ0 which implies

๐ด1๐‘Ÿ๐‘›0 = ๐ต2๐‘Ÿโˆ’๐‘›0 (5)

The second condition is the jump in the derivative of ๐บ (๐‘Ÿ, ๐‘Ÿ0) given by๐‘‘๐‘‘๐‘Ÿ๐บ (๐‘Ÿ, ๐‘Ÿ0)๏ฟฝ

๐‘Ÿ>๐‘Ÿ0โˆ’๐‘‘๐‘‘๐‘Ÿ๐บ (๐‘Ÿ, ๐‘Ÿ0)๏ฟฝ

๐‘Ÿ<๐‘Ÿ0=

โˆ’1๐‘ (๐‘Ÿ0)

Where ๐‘ (๐‘Ÿ0) comes from the Sturm Liouville form of the homogeneous ODE. This wasfound above as ๐‘ (๐‘Ÿ0) = ๐‘Ÿ0. Hence the above condition becomes

๐‘‘๐‘‘๐‘Ÿ๐บ (๐‘Ÿ, ๐‘Ÿ0)๏ฟฝ

๐‘Ÿ>๐‘Ÿ0โˆ’๐‘‘๐‘‘๐‘Ÿ๐บ (๐‘Ÿ, ๐‘Ÿ0)๏ฟฝ

๐‘Ÿ<๐‘Ÿ0=โˆ’1๐‘Ÿ0

Equation (4) shows that ๐‘‘๐‘‘๐‘Ÿ๐บ (๐‘Ÿ, ๐‘Ÿ0)๏ฟฝ๐‘Ÿ>๐‘Ÿ0

= โˆ’๐‘›๐ต2๐‘Ÿโˆ’๐‘›โˆ’10 and that ๐‘‘๐‘‘๐‘Ÿ๐บ (๐‘Ÿ, ๐‘Ÿ0)๏ฟฝ๐‘Ÿ<๐‘Ÿ0

= ๐‘›๐ด1๐‘Ÿ๐‘›โˆ’10 . Using

these in the above gives the second equation needed

โˆ’๐‘›๐ต2๐‘Ÿโˆ’๐‘›โˆ’10 โˆ’ ๐‘›๐ด1๐‘Ÿ๐‘›โˆ’10 =โˆ’1๐‘Ÿ0

(6)

Solving (5,6) for ๐ด1, ๐ต1: From (5) ๐ด1 = ๐ต2๐‘Ÿโˆ’2๐‘›0 . Substituting this in (6) gives

โˆ’๐‘›๐ต2๐‘Ÿโˆ’๐‘›โˆ’10 โˆ’ ๐‘› ๏ฟฝ๐ต2๐‘Ÿโˆ’2๐‘›0 ๏ฟฝ ๐‘Ÿ๐‘›โˆ’1 =โˆ’1๐‘Ÿ0

โˆ’๐‘›๐ต2๐‘Ÿโˆ’๐‘›โˆ’1 โˆ’ ๐‘›๐ต2๐‘Ÿโˆ’๐‘›โˆ’1 =โˆ’1๐‘Ÿ0

โˆ’2๐‘›๐ต2๐‘Ÿโˆ’๐‘›โˆ’10 = โˆ’๐‘Ÿโˆ’10

๐ต2 =โˆ’๐‘Ÿโˆ’10

โˆ’2๐‘›๐‘Ÿโˆ’๐‘›โˆ’10

=12๐‘›๐‘Ÿ๐‘›0

But since ๐ด1 = ๐ต2๐‘Ÿโˆ’2๐‘›0 , then

๐ด1 =12๐‘›๐‘Ÿ๐‘›0๐‘Ÿโˆ’2๐‘›0

=12๐‘›๐‘Ÿโˆ’๐‘›0

Therefore the solution (4), which is the Green function, becomes, for ๐‘› โ‰  0

๐บ (๐‘Ÿ, ๐‘Ÿ0) =

โŽงโŽชโŽชโŽจโŽชโŽชโŽฉ

12๐‘›๐‘Ÿ

โˆ’๐‘›0 ๐‘Ÿ๐‘› 0 < ๐‘Ÿ < ๐‘Ÿ0

12๐‘›๐‘Ÿ

๐‘›0๐‘Ÿโˆ’๐‘› ๐‘Ÿ0 < ๐‘Ÿ < โˆž

(7)

135

3.7.3 key solution to HW 7

136

137

138

139

top related